Ortho Flashcards

1
Q

A 68 year old man presents to the plastics team with severe burns to his hands. He is not distressed by the burns. He has bilateral charcot joints. On examination; there is loss of pain and temperature sensation of the upper limbs.

A.	Osteomyelitis
B.	Potts disease of the spine
C.	Scheuermanns disease
D.	Transverse myelitis
E.	Tabes dorsalis
F.	Subacute degeneration of the cord
G.	Brown-Sequard syndrome
H.	Syringomyelia
I.	Epidural haematoma
A

Syringomyelia

This patient has syringomyelia which selectively affects the spinotholamic tracts. Syringomyelia is a disorder in which a cystic cavity forms within the spinal cord. The commonest variant is the Arnold- Chiari malformation in which the cavity connects with a congenital malformation affecting the cerebellum. Acquired forms of the condition may occur as a result of previous meningitis, surgery or tumours. Many neurological manifestations have been reported, although the classical variety spares the dorsal columns and medial lemniscus and affecting only the spinothalamic tract with loss of pain and temperature sensation. The bilateral distribution of this patients symptoms would therefore favor syringomyelia over SCID or Brown Sequard syndrome. Osteomyelitis would tend to present with back pain and fever in addition to any neurological signs. Epidural haematoma large enough to produce neurological impairment will usually have motor symptoms in addition to any selective sensory loss, and the history is usually shorter.

Spinal disorders

Dorsal column lesion : Loss vibration and proprioception, Tabes dorsalis, SACD

Spinothalamic tract lesion
Loss of pain, sensation and temperature

Central cord lesion
Flaccid paralysis of the upper limbs

Osteomyelitis	
Normally progressive
Staph aureus in IVDU, normally cervical region affected
Fungal infections in immunocompromised
Thoracic region affected in TB

Infarction spinal cord
Dorsal column signs (loss of proprioception and fine discrimination)

Cord compression	
UMN signs
Malignancy
Haematoma
Fracture

Brown-sequard syndrome
Hemisection of the spinal cord
Ipsilateral paralysis
Ipsilateral loss of proprioception and fine discrimination
Contralateral loss of pain and temperature

Dermatomes
C2 to C4 The C2 dermatome covers the occiput and the top part of the neck. C3 covers the lower part of the neck to the clavicle. C4 covers the area just below the clavicle.
C5 to T1 Situated in the arms. C5 covers the lateral arm at and above the elbow. C6 covers the forearm and the radial (thumb) side of the hand. C7 is the middle finger, C8 is the medial aspect of the hand, and T1 covers the medial side of the forearm.
T2 to T12 The thoracic covers the axillary and chest region. T3 to T12 covers the chest and back to the hip girdle. The nipples are situated in the middle of T4. T10 is situated at the umbilicus. T12 ends just above the hip girdle.
L1 to L5 The cutaneous dermatome representing the hip girdle and groin area is innervated by L1 spinal cord. L2 and 3 cover the front part of the thighs. L4 and L5 cover medial and lateral aspects of the lower leg.
S1 to S5 S1 covers the heel and the middle back of the leg. S2 covers the back of the thighs. S3 cover the medial side of the buttocks and S4-5 covers the perineal region. S5 is of course the lowest dermatome and represents the skin immediately at and adjacent to the anus.

Myotomes

Upper limb
Elbow flexors/Biceps	C5
Wrist extensors	C6
Elbow extensors/Triceps	C7
Long finger flexors	C8
Small finger abductors	T1
Lower limb
Hip flexors (psoas)	L1 and L2
Knee extensors (quadriceps)	L3
Ankle dorsiflexors (tibialis anterior)	L4 and L5
Toe extensors (hallucis longus)	L 5
Ankle plantar flexors (gastrocnemius)	S1

The anal sphincter is innervated by S2,3,4

How well did you know this?
1
Not at all
2
3
4
5
Perfectly
2
Q

A 24 year old man presents with localised spinal pain over 2 months which is worsened on movement. He is known to be an IVDU. He has no history suggestive of tuberculosis. The pain is now excruciating at rest and not improving with analgesia. He has a temperature of 39 oC.

 A.	Osteomyelitis
B.	Potts disease of the spine
C.	Scheuermanns disease
D.	Transverse myelitis
E.	Tabes dorsalis
F.	Subacute degeneration of the cord
G.	Brown-Sequard syndrome
H.	Syringomyelia
I.	Epidural haematoma
A

Osteomyelitis

In an IVDU with back pain and pyrexia have a high suspicion for osteomylelitis. The most likely organism is staph aureus and the cervical spine is the most common region affected. TB tends to affect the thoracic spine and in other causes of osteomyelitis the lumbar spine is affected.

Spinal disorders

Dorsal column lesion : Loss vibration and proprioception, Tabes dorsalis, SACD

Spinothalamic tract lesion
Loss of pain, sensation and temperature

Central cord lesion
Flaccid paralysis of the upper limbs

Osteomyelitis	
Normally progressive
Staph aureus in IVDU, normally cervical region affected
Fungal infections in immunocompromised
Thoracic region affected in TB

Infarction spinal cord
Dorsal column signs (loss of proprioception and fine discrimination)

Cord compression	
UMN signs
Malignancy
Haematoma
Fracture

Brown-sequard syndrome
Hemisection of the spinal cord
Ipsilateral paralysis
Ipsilateral loss of proprioception and fine discrimination
Contralateral loss of pain and temperature

Dermatomes
C2 to C4 The C2 dermatome covers the occiput and the top part of the neck. C3 covers the lower part of the neck to the clavicle. C4 covers the area just below the clavicle.
C5 to T1 Situated in the arms. C5 covers the lateral arm at and above the elbow. C6 covers the forearm and the radial (thumb) side of the hand. C7 is the middle finger, C8 is the medial aspect of the hand, and T1 covers the medial side of the forearm.
T2 to T12 The thoracic covers the axillary and chest region. T3 to T12 covers the chest and back to the hip girdle. The nipples are situated in the middle of T4. T10 is situated at the umbilicus. T12 ends just above the hip girdle.
L1 to L5 The cutaneous dermatome representing the hip girdle and groin area is innervated by L1 spinal cord. L2 and 3 cover the front part of the thighs. L4 and L5 cover medial and lateral aspects of the lower leg.
S1 to S5 S1 covers the heel and the middle back of the leg. S2 covers the back of the thighs. S3 cover the medial side of the buttocks and S4-5 covers the perineal region. S5 is of course the lowest dermatome and represents the skin immediately at and adjacent to the anus.

Myotomes

Upper limb
Elbow flexors/Biceps	C5
Wrist extensors	C6
Elbow extensors/Triceps	C7
Long finger flexors	C8
Small finger abductors	T1
Lower limb
Hip flexors (psoas)	L1 and L2
Knee extensors (quadriceps)	L3
Ankle dorsiflexors (tibialis anterior)	L4 and L5
Toe extensors (hallucis longus)	L 5
Ankle plantar flexors (gastrocnemius)	S1

The anal sphincter is innervated by S2,3,4

How well did you know this?
1
Not at all
2
3
4
5
Perfectly
3
Q

A 22 year man is shot in the back, in the lumbar region. He has increased tone and hyper-reflexia of his right leg. He cannot feel his left leg.

 A.	Osteomyelitis
B.	Potts disease of the spine
C.	Scheuermanns disease
D.	Transverse myelitis
E.	Tabes dorsalis
F.	Subacute degeneration of the cord
G.	Brown-Sequard syndrome
H.	Syringomyelia
I.	Epidural haematoma
A

Brown-Sequard syndrome

Brown -Sequard syndrome is caused by hemisection of the spinal cord. It may result from stab injuries or lateral vertebral fractures. It results in ipsilateral paralysis (pyramidal tract) , and also loss of proprioception and fine discrimination (dorsal columns). Pain and temperature sensation are lost on the contra-lateral side. This is because the fibres of the spinothalamic tract have decussated below the level of the cord transection.

Spinal disorders

Dorsal column lesion : Loss vibration and proprioception, Tabes dorsalis, SACD

Spinothalamic tract lesion
Loss of pain, sensation and temperature

Central cord lesion
Flaccid paralysis of the upper limbs

Osteomyelitis	
Normally progressive
Staph aureus in IVDU, normally cervical region affected
Fungal infections in immunocompromised
Thoracic region affected in TB

Infarction spinal cord
Dorsal column signs (loss of proprioception and fine discrimination)

Cord compression	
UMN signs
Malignancy
Haematoma
Fracture

Brown-sequard syndrome
Hemisection of the spinal cord
Ipsilateral paralysis
Ipsilateral loss of proprioception and fine discrimination
Contralateral loss of pain and temperature

Dermatomes
C2 to C4 The C2 dermatome covers the occiput and the top part of the neck. C3 covers the lower part of the neck to the clavicle. C4 covers the area just below the clavicle.
C5 to T1 Situated in the arms. C5 covers the lateral arm at and above the elbow. C6 covers the forearm and the radial (thumb) side of the hand. C7 is the middle finger, C8 is the medial aspect of the hand, and T1 covers the medial side of the forearm.
T2 to T12 The thoracic covers the axillary and chest region. T3 to T12 covers the chest and back to the hip girdle. The nipples are situated in the middle of T4. T10 is situated at the umbilicus. T12 ends just above the hip girdle.
L1 to L5 The cutaneous dermatome representing the hip girdle and groin area is innervated by L1 spinal cord. L2 and 3 cover the front part of the thighs. L4 and L5 cover medial and lateral aspects of the lower leg.
S1 to S5 S1 covers the heel and the middle back of the leg. S2 covers the back of the thighs. S3 cover the medial side of the buttocks and S4-5 covers the perineal region. S5 is of course the lowest dermatome and represents the skin immediately at and adjacent to the anus.

Myotomes

Upper limb
Elbow flexors/Biceps	C5
Wrist extensors	C6
Elbow extensors/Triceps	C7
Long finger flexors	C8
Small finger abductors	T1
Lower limb
Hip flexors (psoas)	L1 and L2
Knee extensors (quadriceps)	L3
Ankle dorsiflexors (tibialis anterior)	L4 and L5
Toe extensors (hallucis longus)	L 5
Ankle plantar flexors (gastrocnemius)	S1

The anal sphincter is innervated by S2,3,4

How well did you know this?
1
Not at all
2
3
4
5
Perfectly
4
Q

A 24 year old man is brought to the emergency department have suffered a crush injury to his forearm. Assessment demonstrates that the arm is tender, red and swollen. There is clinical evidence of an ulnar fracture and the patient cannot move their fingers. Which is the most appropriate course of action?

	Application of an external fixation device
	Closed reduction
	Debridement
	Discharge and review in fracture clinic
	Fasciotomy
A

Fasciotomy

The combination of a crush injury, limb swelling and inability to move digits should raise suspicion of a compartment syndrome that will require a fasciotomy. Paralysis is a very late sign.

Compartment syndrome

This is a particular complication that may occur following fractures (or following ischaemia re-perfusion injury in vascular patients). It is characterised by raised pressure within a closed anatomical space.
The raised pressure within the compartment will eventually compromise tissue perfusion resulting in necrosis. The two main fractures carrying this complication include supracondylar fractures and tibial shaft injuries.

Symptoms and signs
Pain, especially on movement (even passive)
Parasthesiae
Pallor may be present
Arterial pulsation may still be felt as the necrosis occurs as a result of microvascular compromise
Paralysis of the muscle group may occur

Diagnosis
Is made by measurement of intracompartmental pressure measurements. Pressures in excess of 20mmHg are abnormal and >40mmHg is diagnostic.

Treatment
This is essentially prompt and extensive fasciotomies
In the lower limb the deep muscles may be inadequately decompressed by the inexperienced operator when smaller incisions are performed
Myoglobinuria may occur following fasciotomy and result in renal failure and for this reason these patients require aggressive IV fluids
Where muscle groups are frankly necrotic at fasciotomy they should be debrided and amputation may have to be considered
Death of muscle groups may occur within 4-6 hours

How well did you know this?
1
Not at all
2
3
4
5
Perfectly
5
Q

A 19 year old sportswoman presents with knee pain which is worse on walking down the stairs and when sitting still. On examination there is wasting of the quadriceps and pseudolocking of the knee.

 A.	Chondromalacia patellae
B.	Dislocated patella
C.	Undisplaced fracture patella
D.	Displaced patella fracture
E.	Avulsion fracture of the tibial tubercle
F.	Quadriceps tendon rupture
G.	Osgood Schlatters disease
A

Chondromalacia patellae

A teenage girl with knee pain on walking down the stairs is characteristic for chondromalacia patellae (anterior knee pain). Most cases are managed with physiotherapy.

Ruptured anterior cruciate ligament
Sport injury
Mechanism: high twisting force applied to a bent knee
Typically presents with: loud crack, pain and RAPID joint swelling (haemoarthrosis)
Poor healing
Management: intense physiotherapy or surgery

Ruptured posterior cruciate ligament
Mechanism: hyperextension injuries
Tibia lies back on the femur
Paradoxical anterior draw test

Rupture of medial collateral ligament
Mechanism: leg forced into valgus via force outside the leg
Knee unstable when put into valgus position

Menisceal tear
Rotational sporting injuries
Delayed knee swelling
Joint locking (Patient may develop skills to “unlock” the knee
Recurrent episodes of pain and effusions are common, often following minor trauma

Chondromalacia patellae
Teenage girls, following an injury to knee e.g. Dislocation patella
Typical history of pain on going downstairs or at rest
Tenderness, quadriceps wasting

Dislocation of the patella
Most commonly occurs as a traumatic primary event, either through direct trauma or through severe contraction of quadriceps with knee stretched in valgus and external rotation
Genu valgum, tibial torsion and high riding patella are risk factors
Skyline x-ray views of patella are required, although displaced patella may be clinically obvious
An osteochondral fracture is present in 5%
The condition has a 20% recurrence rate

Fractured patella
2 types:
i. Direct blow to patella causing undisplaced fragments
ii. Avulsion fracture

Tibial plateau fracture
Occur in the elderly (or following significant trauma in young)
Mechanism: knee forced into valgus or varus, but the knee fractures before the ligaments rupture
Varus injury affects medial plateau and if valgus injury, lateral plateau depressed fracture occurs

Schatzker Classification system for tibial plateau fractures
Type Anatomical description Features
1 Vertical split of lateral condyle Fracture through dense bone, usually in the young. It may be virtually undisplaced, or the condylar fragment may be pushed inferiorly and tilted
2 Vertical split of the lateral condyle combined with an adjacent load bearing part of the condyle The wedge fragment (which may be of variable size), is displaced laterally; the joint is widened. Untreated, a valgus deformity may develop
3 Depression of the articular surface with intact condylar rim The split does not extend to the edge of the plateau. Depressed fragments may be firmly embedded in subchondral bone, the joint is stable
4 Fragment of the medial tibial condyle Two injuries are seen in this category; (1) a depressed fracture of osteoporotic bone in the elderly. (2) a high energy fracture resulting in a condylar split that runs from the intercondylar eminence to the medial cortex. Associated ligamentous injury may be severe
5 Fracture of both condyles Both condyles fractured but the column of the metaphysis remains in continuity with the tibial shaft
6 Combined condylar and subcondylar fractures High energy fracture with marked comminution

How well did you know this?
1
Not at all
2
3
4
5
Perfectly
6
Q

A tall 18 year old male athlete is admitted to the emergency room after being hit in the knee by a hockey stick. On examination his knee is tense and swollen. X-ray shows no fractures.

A.	Chondromalacia patellae
B.	Dislocated patella
C.	Undisplaced fracture patella
D.	Displaced patella fracture
E.	Avulsion fracture of the tibial tubercle
F.	Quadriceps tendon rupture
G.	Osgood Schlatters disease
A

Dislocated patella

A patella dislocation is a common cause of haemarthrosis and many will spontaneously reduce when the leg is straightened. In the chronic setting physiotherapy is used to strengthen the quadriceps muscles.

Ruptured anterior cruciate ligament
Sport injury
Mechanism: high twisting force applied to a bent knee
Typically presents with: loud crack, pain and RAPID joint swelling (haemoarthrosis)
Poor healing
Management: intense physiotherapy or surgery

Ruptured posterior cruciate ligament
Mechanism: hyperextension injuries
Tibia lies back on the femur
Paradoxical anterior draw test

Rupture of medial collateral ligament
Mechanism: leg forced into valgus via force outside the leg
Knee unstable when put into valgus position

Menisceal tear
Rotational sporting injuries
Delayed knee swelling
Joint locking (Patient may develop skills to “unlock” the knee
Recurrent episodes of pain and effusions are common, often following minor trauma

Chondromalacia patellae
Teenage girls, following an injury to knee e.g. Dislocation patella
Typical history of pain on going downstairs or at rest
Tenderness, quadriceps wasting

Dislocation of the patella
Most commonly occurs as a traumatic primary event, either through direct trauma or through severe contraction of quadriceps with knee stretched in valgus and external rotation
Genu valgum, tibial torsion and high riding patella are risk factors
Skyline x-ray views of patella are required, although displaced patella may be clinically obvious
An osteochondral fracture is present in 5%
The condition has a 20% recurrence rate

Fractured patella
2 types:
i. Direct blow to patella causing undisplaced fragments
ii. Avulsion fracture

Tibial plateau fracture
Occur in the elderly (or following significant trauma in young)
Mechanism: knee forced into valgus or varus, but the knee fractures before the ligaments rupture
Varus injury affects medial plateau and if valgus injury, lateral plateau depressed fracture occurs

Schatzker Classification system for tibial plateau fractures
Type Anatomical description Features
1 Vertical split of lateral condyle Fracture through dense bone, usually in the young. It may be virtually undisplaced, or the condylar fragment may be pushed inferiorly and tilted
2 Vertical split of the lateral condyle combined with an adjacent load bearing part of the condyle The wedge fragment (which may be of variable size), is displaced laterally; the joint is widened. Untreated, a valgus deformity may develop
3 Depression of the articular surface with intact condylar rim The split does not extend to the edge of the plateau. Depressed fragments may be firmly embedded in subchondral bone, the joint is stable
4 Fragment of the medial tibial condyle Two injuries are seen in this category; (1) a depressed fracture of osteoporotic bone in the elderly. (2) a high energy fracture resulting in a condylar split that runs from the intercondylar eminence to the medial cortex. Associated ligamentous injury may be severe
5 Fracture of both condyles Both condyles fractured but the column of the metaphysis remains in continuity with the tibial shaft
6 Combined condylar and subcondylar fractures High energy fracture with marked comminution

How well did you know this?
1
Not at all
2
3
4
5
Perfectly
7
Q

An athletic 15 year old boy presents with knee pain of 3 weeks duration. It is worst during activity and settles with rest. On examination there is tenderness overlying the tibial tuberosity and an associated swelling at this site.

A.	Chondromalacia patellae
B.	Dislocated patella
C.	Undisplaced fracture patella
D.	Displaced patella fracture
E.	Avulsion fracture of the tibial tubercle
F.	Quadriceps tendon rupture
G.	Osgood Schlatters disease
A

Osgood Schlatters disease

Athletic boys and girls may develop this condition in their teenage years. It is caused by multiple micro fractures at the point of insertion of the tendon into the tibial tuberosity. Most cases settle with physiotherapy and rest.

Ruptured anterior cruciate ligament
Sport injury
Mechanism: high twisting force applied to a bent knee
Typically presents with: loud crack, pain and RAPID joint swelling (haemoarthrosis)
Poor healing
Management: intense physiotherapy or surgery

Ruptured posterior cruciate ligament
Mechanism: hyperextension injuries
Tibia lies back on the femur
Paradoxical anterior draw test

Rupture of medial collateral ligament
Mechanism: leg forced into valgus via force outside the leg
Knee unstable when put into valgus position

Menisceal tear
Rotational sporting injuries
Delayed knee swelling
Joint locking (Patient may develop skills to “unlock” the knee
Recurrent episodes of pain and effusions are common, often following minor trauma

Chondromalacia patellae
Teenage girls, following an injury to knee e.g. Dislocation patella
Typical history of pain on going downstairs or at rest
Tenderness, quadriceps wasting

Dislocation of the patella
Most commonly occurs as a traumatic primary event, either through direct trauma or through severe contraction of quadriceps with knee stretched in valgus and external rotation
Genu valgum, tibial torsion and high riding patella are risk factors
Skyline x-ray views of patella are required, although displaced patella may be clinically obvious
An osteochondral fracture is present in 5%
The condition has a 20% recurrence rate

Fractured patella
2 types:
i. Direct blow to patella causing undisplaced fragments
ii. Avulsion fracture

Tibial plateau fracture
Occur in the elderly (or following significant trauma in young)
Mechanism: knee forced into valgus or varus, but the knee fractures before the ligaments rupture
Varus injury affects medial plateau and if valgus injury, lateral plateau depressed fracture occurs

Schatzker Classification system for tibial plateau fractures
Type Anatomical description Features
1 Vertical split of lateral condyle Fracture through dense bone, usually in the young. It may be virtually undisplaced, or the condylar fragment may be pushed inferiorly and tilted
2 Vertical split of the lateral condyle combined with an adjacent load bearing part of the condyle The wedge fragment (which may be of variable size), is displaced laterally; the joint is widened. Untreated, a valgus deformity may develop
3 Depression of the articular surface with intact condylar rim The split does not extend to the edge of the plateau. Depressed fragments may be firmly embedded in subchondral bone, the joint is stable
4 Fragment of the medial tibial condyle Two injuries are seen in this category; (1) a depressed fracture of osteoporotic bone in the elderly. (2) a high energy fracture resulting in a condylar split that runs from the intercondylar eminence to the medial cortex. Associated ligamentous injury may be severe
5 Fracture of both condyles Both condyles fractured but the column of the metaphysis remains in continuity with the tibial shaft
6 Combined condylar and subcondylar fractures High energy fracture with marked comminution

How well did you know this?
1
Not at all
2
3
4
5
Perfectly
8
Q

An 8 year old boy presents with symptoms of right knee pain. The pain has been present on most occasions for the past three months and the pain typically lasts for several hours at a time. On examination; he walks with an antalgic gait and has apparent right leg shortening. What is the most likely diagnosis?

	Perthes Disease
	Osteosarcoma of the femur
	Osteoarthritis of the hip
	Transient synovitis of the hip
	Torn medial meniscus
A

Perches Disease

There are many causes of the irritable hip in the 10-14 year age group. Many of these may cause both hip pain or knee pain. Transient synovitis of the hip the commonest disorder but does not typically last for 3 months. An osteosarcoma would not usually present with apparent limb shortening unless pathological fracture had occurred. A slipped upper femoral epiphysis can cause a similar presentation although it typically presents later and with different patient characteristics.

Perthes disease
Idiopathic avascular necrosis of the femoral epiphysis of the femoral head
Impaired blood supply to femoral head, causing bone infarction. New vessels develop and ossification occurs. The bone either heals or a subchondral fracture occurs.

Clinical features
Males 4x's greater than females
Age between 2-12 years (the younger the age of onset, the better the prognosis)
Limp
Hip pain
Bilateral in 20%

Diagnosis
Plain x-ray, Technetium bone scan or magnetic resonance imaging if normal x-ray and symptoms persist.

Catterall staging
Stage Features
Stage 1 Clinical and histological features only
Stage 2 Sclerosis with or without cystic changes and preservation of the articular surface
Stage 3 Loss of structural integrity of the femoral head
Stage 4 Loss of acetabular integrity

Management
To keep the femoral head within the acetabulum: cast, braces
If less than 6 years: observation
Older: surgical management with moderate results
Operate on severe deformities

Prognosis
Most cases will resolve with conservative management. Early diagnosis improves outcomes.

How well did you know this?
1
Not at all
2
3
4
5
Perfectly
9
Q

Which of the following types of growth plate fractures may have similar radiological appearances?

	Salter Harris types 1 and 5
	Salter Harris types 4 and 5
	Salter Harris types 3 and 5
	Salter Harris types 1 and 2
	Salter Harris types 1 and 3
A

Salter Harris types 1 and 5

Mnemonic: SALTER

S (Type 1): Straight through the growth plate
A (Type 2): Above - through growth plate and Above involving the metaphysis
L (Type 3): Lower -through growth plate and beLow involving the epiphysis
T (Type 4):Through - Through both metaphysis, epiphysis and growth plate
E (Type 5): Everything - Crush / compression injury
R (Type 5): Ruined
As recommended by one of our users

Salter Harris injury types 1 and 5 (transverse fracture through growth plate Vs. Compression fracture) may mimic each other radiologically. Type 5 injuries have the worst outcomes. Radiological signs of type 5 injuries are subtle and may include narrowing of the growth plate.

Epiphyseal fractures

Fractures involving the growth plate in children are classified using the Salter - Harris system.
There are 5 main types.

Salter Harris Classification
Type Description
Type 1 Transverse fracture through the growth plate
Type 2 Fracture through the growth plate to the metaphysis (commonest type)
Type 3 Fracture through the growth plate and the epiphysis with metaphysis spared
Type 4 Fracture involving the growth plate, metaphysis and epiphysis
Type 5 Compression fracture of the growth plate (worst outcome)

Management
Non displaced type 1 injuries can generally be managed conservatively. Unstable or more extensive injuries will usually require surgical reduction and/ or fixation, as proper alignment is crucial.

How well did you know this?
1
Not at all
2
3
4
5
Perfectly
10
Q

A 20 year old woman trips over a step, injuring her ankle. Examination reveals tenderness over the lateral malleolus and an x-ray demonstrates an undisplaced fracture distal to the syndesmosis.

A. Surgical fixation
B. Below knee amputation
C. Application of below knee plaster
D. Application of ankle boot
E. Application of external fixation device
F. Application of compression dressing and physiotherapy
G. Immediate reduction and application of backslab

A

Application of ankle boot

This is a Weber A fracture. It is a stable ankle injury and can therefore be managed conservatively. Whilst this patient could also be treated in a below knee plaster, most clinicians would nowadays treat this injury in an ankle boot. Patients should be advised to mobilise in the ankle boot, as pain allows, and can wean themselves out of the boot as the symptoms improve.

An ankle fracture relates to a fracture around the tibio-talar joint. It generally refers to a fracture involving the lateral, and/or medial and/or posterior malleolus. Pilon and Tillaux fractures are also considered to be ankle fractures, but are not covered here.
Ankle fractures are common. They effect men and women in equal numbers, but men have a higher rate as young adults (sports and contact injuries), and women a higher rate post-menopausal (fragility type fracture).

Osseous anatomy
The ankle (or mortise) joint consists of the distal tibia (tibial plafond and posterior malleolus), the distal fibula (lateral malleolus), and the talus. The main movement at the ankle joint is plantar and dorsiflexion.

Ligamentous anatomy
Medial side: Deltoid ligament. This is divided into superficial and deep portions. It is the primary restraint to valgus tilting of the talus.

Lateral side: Lateral ligament complex consisting from anterior to posterior of the anterior talofibular ligament (ATFL), calcaneofibular ligament (CFL), and the posterior talofibular ligament (PTFL). Together they resist valgus stress to the ankle, and are a restraint to anterior translation of the talus within the mortise joint.

Syndesmosis: The syndesmosis is a ligament complex between the distal tibia and fibula, holding the two bones together. It is fundamental to the integrity of the ankle joint, and its disruption leads to instability. It consists of (from anterior to posterior) the anterior-inferior tibiofibular ligament (AITFL), the transverse tibiofibular ligament (TTFL), the interosseous membrane, and the posterior-inferior tibiofibular ligament (PITFL).

Presentation and initial management
Patients will present following a traumatic event with a painful, swollen ankle, and reluctance/inability to weight bear. The Ottawa rules can be applied to differentiate between an ankle fracture and sprain, but can be unreliable.
In high energy injuries, management should follow ATLS principles to identify more significant injuries first. Neurovascular status of the foot should be documented, and open injuries should be excluded. If an open injury is identified, it should be managed in line with BOAST 4 principles1. If an obvious deformity exists, it should be reduced as soon as possible with appropriate analgesia or conscious sedation. Radiographs of clearly deformed or dislocated joints are not necessary, and removing the pressure on the surrounding soft tissues from the underlying bony deformity is the priority. If the fracture pattern is not clinically obvious then plain radiographs are appropriate and will guide the subsequent manipulation during plaster-of-paris below knee backslab application.

Imaging
AP, lateral and mortise views (20o internal rotation) are essential to evaluate fracture displacement and syndesmotic injury. Decreased tibiofibular overlap, medial joint clear space and lateral talar shift all indicate a syndesmotic injury. (In subtle cases of shift, imaging the uninjured ankle can be helpful as a proportion of the population have little or no tibiotalar overlap 2.)

Where there is suspicion of syndesmosis involvement in the absence of radiographic evidence, stress radiographs can be diagnostic.
Complex fracture patterns (and increasingly posterior malleolar fractures) are best defined using CT.

Classification
The most commonly used classifications are Lauge-Hansen and Danis-Weber.

Lauge-Hansen
Comprises two parts: first part is the foot position, and the second part is the force applied. Useful for understanding the forces involved and therefore predict the ligamentous or bony injury. Results in four injury patterns:
Supination - Adduction (SA) - 10-20%
Supination - External rotation (SER) - 40-75%
Pronation - Abduction (PA) - 5-20%
Pronation - External rotation (PER) - 5-20%

Not often used in clinical practice but good for understanding the principles of ankle fracture.

Danis-Weber
Commonly used. Based on the level of the fibula fracture in relation to the syndesmosis. The more proximal, the greater the risk of syndesmotic injury and therefore fracture instability.
A - fracture below the level of the syndesmosis
B - fracture at the level of the syndesmosis / level of the tibial plafond
C - fracture above the level of the syndesmosis. This includes Maisonneuve fractures (proximal fibula fracture), which can be associated with ankle instability. Beware the high fibula fracture - it may be an ankle fracture!

The Weber classification is based purely on the the lateral side. All injuries can include a medial or posterior bony or ligamentous injury which also dictates fracture stability (bimalleolar and trimalleolar fractures are more unstable).

Treatment
When deciding upon treatment for an ankle fracture, one must consider both the fracture and the patient. Diabetic patients and smokers are at greater risk of post-operative complication, especially wound problems and infection. Likewise, the long term outcome of post-traumatic arthritis from a malunited ankle fracture is extremely important for a young patient, but not as relevant in the elderly. Therefore, normal surgical decision processes apply as with all fractures.

Defining stability of an ankle fracture underpins the treatment decision.
Weber A - Unimalleolar Weber A Weber fractures by definition are stable and therefore can be mobilised fully weight bearing in an ankle boot.

Weber C - Fractures tend to include syndesmotic disruption and are usually bimalleolar (either bony or ligamentous). They are therefore unstable and usually require operative fixation. In addition to the fracture fixation, the syndesmosis usually requires reconstruction/augmentation with screws to restore the joint integrity and function.

Weber B - B fractures vary greatly. They can be part of a trimalleolar injury and therefore extremely unstable, requiring fixation. Alternatively, a uni-malleolar Weber B fracture can be a stable injury, and therefore mobilised immediately in an ankle boot. Defining the stability can be challenging, and often involves stress radiographs, or a trial of mobilisation and repeat radiographs. Defining stability is the subject of much ongoing research. However, treating undisplaced ankle fractures in a below knee plaster, non-weight bearing for six weeks is still widely practised, and a safe approach.

When operative fixation is appropriate, it is usually via open reduction and internal fixation using plates and screws. It must be carried out when soft tissue swelling has settled in order to minimise the risk of wound problems. This can often take a week to settle.

The use of fibula nails is expanding, but is not yet mainstream. Ankle fractures can also be treated with external fixation, or with a hind foot nail in patients who need fixation but where soft tissue or bone quality is poor.

Post operative management
Ankle fractures generally take 6 weeks to unite enough to prevent secondary displacement. This is therefore an appropriate time period to keep a cast on in a conservatively managed patient. Weight bearing post-operatively depends on the quality of the fixation and bone quality, and preference varies between surgeons, ranging from aggressive early mobilisation to a period of non-weight bearing. Return to activities takes approximately three months, and often requires assistance of a physiotherapist to improve range-of-movement and muscle strengthening.

How well did you know this?
1
Not at all
2
3
4
5
Perfectly
11
Q

A 30 year old man injures his ankle playing football. On examination he has tenderness over both medial and lateral malleoli. X-ray demonstrates a bimalleolar fracture with a displaced distal fibula fracture, at the level of the syndesmosis and fracture of the medial malleolus with talar shift. The ankle has been provisionally reduced and splinted in the emergency department.

A. Surgical fixation
B. Below knee amputation
C. Application of below knee plaster
D. Application of ankle boot
E. Application of external fixation device
F. Application of compression dressing and physiotherapy
G. Immediate reduction and application of backslab

A

Surgical fixation

This is an unstable fracture pattern with a Weber B fracture of the distal fibula and a fracture of the medial malleolus. Talar shift indicates loss of ankle mortice congruity. This injury should therefore be treated with surgical fixation.

An ankle fracture relates to a fracture around the tibio-talar joint. It generally refers to a fracture involving the lateral, and/or medial and/or posterior malleolus. Pilon and Tillaux fractures are also considered to be ankle fractures, but are not covered here.
Ankle fractures are common. They effect men and women in equal numbers, but men have a higher rate as young adults (sports and contact injuries), and women a higher rate post-menopausal (fragility type fracture).

Osseous anatomy
The ankle (or mortise) joint consists of the distal tibia (tibial plafond and posterior malleolus), the distal fibula (lateral malleolus), and the talus. The main movement at the ankle joint is plantar and dorsiflexion.

Ligamentous anatomy
Medial side: Deltoid ligament. This is divided into superficial and deep portions. It is the primary restraint to valgus tilting of the talus.

Lateral side: Lateral ligament complex consisting from anterior to posterior of the anterior talofibular ligament (ATFL), calcaneofibular ligament (CFL), and the posterior talofibular ligament (PTFL). Together they resist valgus stress to the ankle, and are a restraint to anterior translation of the talus within the mortise joint.

Syndesmosis: The syndesmosis is a ligament complex between the distal tibia and fibula, holding the two bones together. It is fundamental to the integrity of the ankle joint, and its disruption leads to instability. It consists of (from anterior to posterior) the anterior-inferior tibiofibular ligament (AITFL), the transverse tibiofibular ligament (TTFL), the interosseous membrane, and the posterior-inferior tibiofibular ligament (PITFL).

Presentation and initial management
Patients will present following a traumatic event with a painful, swollen ankle, and reluctance/inability to weight bear. The Ottawa rules can be applied to differentiate between an ankle fracture and sprain, but can be unreliable.
In high energy injuries, management should follow ATLS principles to identify more significant injuries first. Neurovascular status of the foot should be documented, and open injuries should be excluded. If an open injury is identified, it should be managed in line with BOAST 4 principles1. If an obvious deformity exists, it should be reduced as soon as possible with appropriate analgesia or conscious sedation. Radiographs of clearly deformed or dislocated joints are not necessary, and removing the pressure on the surrounding soft tissues from the underlying bony deformity is the priority. If the fracture pattern is not clinically obvious then plain radiographs are appropriate and will guide the subsequent manipulation during plaster-of-paris below knee backslab application.

Imaging
AP, lateral and mortise views (20o internal rotation) are essential to evaluate fracture displacement and syndesmotic injury. Decreased tibiofibular overlap, medial joint clear space and lateral talar shift all indicate a syndesmotic injury. (In subtle cases of shift, imaging the uninjured ankle can be helpful as a proportion of the population have little or no tibiotalar overlap 2.)

Where there is suspicion of syndesmosis involvement in the absence of radiographic evidence, stress radiographs can be diagnostic.
Complex fracture patterns (and increasingly posterior malleolar fractures) are best defined using CT.

Classification
The most commonly used classifications are Lauge-Hansen and Danis-Weber.

Lauge-Hansen
Comprises two parts: first part is the foot position, and the second part is the force applied. Useful for understanding the forces involved and therefore predict the ligamentous or bony injury. Results in four injury patterns:
Supination - Adduction (SA) - 10-20%
Supination - External rotation (SER) - 40-75%
Pronation - Abduction (PA) - 5-20%
Pronation - External rotation (PER) - 5-20%

Not often used in clinical practice but good for understanding the principles of ankle fracture.

Danis-Weber
Commonly used. Based on the level of the fibula fracture in relation to the syndesmosis. The more proximal, the greater the risk of syndesmotic injury and therefore fracture instability.
A - fracture below the level of the syndesmosis
B - fracture at the level of the syndesmosis / level of the tibial plafond
C - fracture above the level of the syndesmosis. This includes Maisonneuve fractures (proximal fibula fracture), which can be associated with ankle instability. Beware the high fibula fracture - it may be an ankle fracture!

The Weber classification is based purely on the the lateral side. All injuries can include a medial or posterior bony or ligamentous injury which also dictates fracture stability (bimalleolar and trimalleolar fractures are more unstable).

Treatment
When deciding upon treatment for an ankle fracture, one must consider both the fracture and the patient. Diabetic patients and smokers are at greater risk of post-operative complication, especially wound problems and infection. Likewise, the long term outcome of post-traumatic arthritis from a malunited ankle fracture is extremely important for a young patient, but not as relevant in the elderly. Therefore, normal surgical decision processes apply as with all fractures.

Defining stability of an ankle fracture underpins the treatment decision.
Weber A - Unimalleolar Weber A Weber fractures by definition are stable and therefore can be mobilised fully weight bearing in an ankle boot.

Weber C - Fractures tend to include syndesmotic disruption and are usually bimalleolar (either bony or ligamentous). They are therefore unstable and usually require operative fixation. In addition to the fracture fixation, the syndesmosis usually requires reconstruction/augmentation with screws to restore the joint integrity and function.

Weber B - B fractures vary greatly. They can be part of a trimalleolar injury and therefore extremely unstable, requiring fixation. Alternatively, a uni-malleolar Weber B fracture can be a stable injury, and therefore mobilised immediately in an ankle boot. Defining the stability can be challenging, and often involves stress radiographs, or a trial of mobilisation and repeat radiographs. Defining stability is the subject of much ongoing research. However, treating undisplaced ankle fractures in a below knee plaster, non-weight bearing for six weeks is still widely practised, and a safe approach.

When operative fixation is appropriate, it is usually via open reduction and internal fixation using plates and screws. It must be carried out when soft tissue swelling has settled in order to minimise the risk of wound problems. This can often take a week to settle.

The use of fibula nails is expanding, but is not yet mainstream. Ankle fractures can also be treated with external fixation, or with a hind foot nail in patients who need fixation but where soft tissue or bone quality is poor.

Post operative management
Ankle fractures generally take 6 weeks to unite enough to prevent secondary displacement. This is therefore an appropriate time period to keep a cast on in a conservatively managed patient. Weight bearing post-operatively depends on the quality of the fixation and bone quality, and preference varies between surgeons, ranging from aggressive early mobilisation to a period of non-weight bearing. Return to activities takes approximately three months, and often requires assistance of a physiotherapist to improve range-of-movement and muscle strengthening.

How well did you know this?
1
Not at all
2
3
4
5
Perfectly
12
Q

A 50 year old female slips on wet floor injuring her ankle. On examination, she has tenderness over the lateral and medial malleolus. X-rays demonstrate an undisplaced fracture of the distal fiibula at the level of the syndesmosis and a congruent ankle mortice.

A. Surgical fixation
B. Below knee amputation
C. Application of below knee plaster
D. Application of ankle boot
E. Application of external fixation device
F. Application of compression dressing and physiotherapy
G. Immediate reduction and application of backslab

A

Application of below knee plaster

This is a Weber B fracture and therefore potentially unstable. Medial malleolar tenderness indicates deltoid ligament injury. As the fracture is currently undisplaced and the ankle mortice is congruent, the injury can be initially managed conservatively in a below knee plaster but the patient should be monitored in the outpatient clinic for fracture displacement in the first few weeks.

An ankle fracture relates to a fracture around the tibio-talar joint. It generally refers to a fracture involving the lateral, and/or medial and/or posterior malleolus. Pilon and Tillaux fractures are also considered to be ankle fractures, but are not covered here.
Ankle fractures are common. They effect men and women in equal numbers, but men have a higher rate as young adults (sports and contact injuries), and women a higher rate post-menopausal (fragility type fracture).

Osseous anatomy
The ankle (or mortise) joint consists of the distal tibia (tibial plafond and posterior malleolus), the distal fibula (lateral malleolus), and the talus. The main movement at the ankle joint is plantar and dorsiflexion.

Ligamentous anatomy
Medial side: Deltoid ligament. This is divided into superficial and deep portions. It is the primary restraint to valgus tilting of the talus.

Lateral side: Lateral ligament complex consisting from anterior to posterior of the anterior talofibular ligament (ATFL), calcaneofibular ligament (CFL), and the posterior talofibular ligament (PTFL). Together they resist valgus stress to the ankle, and are a restraint to anterior translation of the talus within the mortise joint.

Syndesmosis: The syndesmosis is a ligament complex between the distal tibia and fibula, holding the two bones together. It is fundamental to the integrity of the ankle joint, and its disruption leads to instability. It consists of (from anterior to posterior) the anterior-inferior tibiofibular ligament (AITFL), the transverse tibiofibular ligament (TTFL), the interosseous membrane, and the posterior-inferior tibiofibular ligament (PITFL).

Presentation and initial management
Patients will present following a traumatic event with a painful, swollen ankle, and reluctance/inability to weight bear. The Ottawa rules can be applied to differentiate between an ankle fracture and sprain, but can be unreliable.
In high energy injuries, management should follow ATLS principles to identify more significant injuries first. Neurovascular status of the foot should be documented, and open injuries should be excluded. If an open injury is identified, it should be managed in line with BOAST 4 principles1. If an obvious deformity exists, it should be reduced as soon as possible with appropriate analgesia or conscious sedation. Radiographs of clearly deformed or dislocated joints are not necessary, and removing the pressure on the surrounding soft tissues from the underlying bony deformity is the priority. If the fracture pattern is not clinically obvious then plain radiographs are appropriate and will guide the subsequent manipulation during plaster-of-paris below knee backslab application.

Imaging
AP, lateral and mortise views (20o internal rotation) are essential to evaluate fracture displacement and syndesmotic injury. Decreased tibiofibular overlap, medial joint clear space and lateral talar shift all indicate a syndesmotic injury. (In subtle cases of shift, imaging the uninjured ankle can be helpful as a proportion of the population have little or no tibiotalar overlap 2.)

Where there is suspicion of syndesmosis involvement in the absence of radiographic evidence, stress radiographs can be diagnostic.
Complex fracture patterns (and increasingly posterior malleolar fractures) are best defined using CT.

Classification
The most commonly used classifications are Lauge-Hansen and Danis-Weber.

Lauge-Hansen
Comprises two parts: first part is the foot position, and the second part is the force applied. Useful for understanding the forces involved and therefore predict the ligamentous or bony injury. Results in four injury patterns:
Supination - Adduction (SA) - 10-20%
Supination - External rotation (SER) - 40-75%
Pronation - Abduction (PA) - 5-20%
Pronation - External rotation (PER) - 5-20%

Not often used in clinical practice but good for understanding the principles of ankle fracture.

Danis-Weber
Commonly used. Based on the level of the fibula fracture in relation to the syndesmosis. The more proximal, the greater the risk of syndesmotic injury and therefore fracture instability.
A - fracture below the level of the syndesmosis
B - fracture at the level of the syndesmosis / level of the tibial plafond
C - fracture above the level of the syndesmosis. This includes Maisonneuve fractures (proximal fibula fracture), which can be associated with ankle instability. Beware the high fibula fracture - it may be an ankle fracture!

The Weber classification is based purely on the the lateral side. All injuries can include a medial or posterior bony or ligamentous injury which also dictates fracture stability (bimalleolar and trimalleolar fractures are more unstable).

Treatment
When deciding upon treatment for an ankle fracture, one must consider both the fracture and the patient. Diabetic patients and smokers are at greater risk of post-operative complication, especially wound problems and infection. Likewise, the long term outcome of post-traumatic arthritis from a malunited ankle fracture is extremely important for a young patient, but not as relevant in the elderly. Therefore, normal surgical decision processes apply as with all fractures.

Defining stability of an ankle fracture underpins the treatment decision.
Weber A - Unimalleolar Weber A Weber fractures by definition are stable and therefore can be mobilised fully weight bearing in an ankle boot.

Weber C - Fractures tend to include syndesmotic disruption and are usually bimalleolar (either bony or ligamentous). They are therefore unstable and usually require operative fixation. In addition to the fracture fixation, the syndesmosis usually requires reconstruction/augmentation with screws to restore the joint integrity and function.

Weber B - B fractures vary greatly. They can be part of a trimalleolar injury and therefore extremely unstable, requiring fixation. Alternatively, a uni-malleolar Weber B fracture can be a stable injury, and therefore mobilised immediately in an ankle boot. Defining the stability can be challenging, and often involves stress radiographs, or a trial of mobilisation and repeat radiographs. Defining stability is the subject of much ongoing research. However, treating undisplaced ankle fractures in a below knee plaster, non-weight bearing for six weeks is still widely practised, and a safe approach.

When operative fixation is appropriate, it is usually via open reduction and internal fixation using plates and screws. It must be carried out when soft tissue swelling has settled in order to minimise the risk of wound problems. This can often take a week to settle.

The use of fibula nails is expanding, but is not yet mainstream. Ankle fractures can also be treated with external fixation, or with a hind foot nail in patients who need fixation but where soft tissue or bone quality is poor.

Post operative management
Ankle fractures generally take 6 weeks to unite enough to prevent secondary displacement. This is therefore an appropriate time period to keep a cast on in a conservatively managed patient. Weight bearing post-operatively depends on the quality of the fixation and bone quality, and preference varies between surgeons, ranging from aggressive early mobilisation to a period of non-weight bearing. Return to activities takes approximately three months, and often requires assistance of a physiotherapist to improve range-of-movement and muscle strengthening.

How well did you know this?
1
Not at all
2
3
4
5
Perfectly
13
Q

A 30 year old woman presents with pain and swelling of the left shoulder. There is a large radiolucent lesion in the head of the humerus extending to the subchondral plate.

 A.	Osteosarcoma
B.	Osteomalacia
C.	Osteoporosis
D.	Metastatic carcinoma
E.	Osteoblastoma
F.	Giant cell tumour
G.	Ewing's sarcoma
A

Giant cell tumour

Giant cell tumours on x-ray have a ‘soap bubble’ appearance. They present as pain or pathological fractures. They commonly metastasize to the lungs.

A pathological fracture occurs in abnormal bone due to insignificant injury
Causes	
Metastatic tumours	
Breast
Lung
Thyroid
Renal
Prostate
Bone disease	
Osteogenesis imperfecta
Osteoporosis
Metabolic bone disease
Paget's disease
Local benign conditions	
Chronic osteomyelitis
Solitary bone cyst
Primary malignant tumours	
Chondrosarcoma
Osteosarcoma
Ewing's tumour
How well did you know this?
1
Not at all
2
3
4
5
Perfectly
14
Q

A 72 year old woman has a lumbar vertebral crush fracture. She has hypocalcaemia and a low urinary calcium.

A.	Osteosarcoma
B.	Osteomalacia
C.	Osteoporosis
D.	Metastatic carcinoma
E.	Osteoblastoma
F.	Giant cell tumour
G.	Ewing's sarcoma
A

Osteomalacia

Hypocalcemia and low urinary calcium are biochemical features of osteomalacia. Unfortunately surgeons do need to look at some blood results!

A pathological fracture occurs in abnormal bone due to insignificant injury
Causes	
Metastatic tumours	
Breast
Lung
Thyroid
Renal
Prostate
Bone disease	
Osteogenesis imperfecta
Osteoporosis
Metabolic bone disease
Paget's disease
Local benign conditions	
Chronic osteomyelitis
Solitary bone cyst
Primary malignant tumours	
Chondrosarcoma
Osteosarcoma
Ewing's tumour
How well did you know this?
1
Not at all
2
3
4
5
Perfectly
15
Q

A 16 year old boy presents with severe groin pain after kicking a football. Imaging confirms a pelvic fracture. A previous pelvic x-ray performed 2 weeks ago shows a lytic lesion with ‘onion type’ periosteal reaction.

 A.	Osteosarcoma
B.	Osteomalacia
C.	Osteoporosis
D.	Metastatic carcinoma
E.	Osteoblastoma
F.	Giant cell tumour
G.	Ewing's sarcoma
A

Ewing’s sarcoma

A Ewings sarcoma is most common in males between 10-20 years. It can occur in girls. A lytic lesion with a lamellated or onion type periosteal reaction is a classical finding on x-rays. Most patients present with metastatic disease with a 5 year prognosis between 5-10%.

A pathological fracture occurs in abnormal bone due to insignificant injury
Causes	
Metastatic tumours	
Breast
Lung
Thyroid
Renal
Prostate
Bone disease	
Osteogenesis imperfecta
Osteoporosis
Metabolic bone disease
Paget's disease
Local benign conditions	
Chronic osteomyelitis
Solitary bone cyst
Primary malignant tumours	
Chondrosarcoma
Osteosarcoma
Ewing's tumour
How well did you know this?
1
Not at all
2
3
4
5
Perfectly
16
Q

An 8 year old boy falls onto an outstretched hand and is brought to the emergency department. He is examined by a doctor and a bony injury is cleared clinically. He re-presents a week later with pain in his hand. What is the most likely underlying injury?

	Fracture of the distal radius
	Fracture of the scaphoid
	Dislocation of the lunate
	Rupture of flexor pollicis longus tendon
	Bennett's fracture
A

Scaphoid fractures in children are rare, will usually involve the distal pole and are easily missed. The initial clinical examination (and sometimes x-rays) may be normal and repeated clinical examination and imaging is advised for this reason. Whilst the other injuries may be sustained from a fall onto an outstretched hand they are less likely to be overlooked on clinical examination. In the case of a Bennetts fracture, the injury mechanism is less compatible with this type of injury.

Scaphoid fractures

Scaphoid fractures are the commonest carpal fractures.
Surface of scaphoid is covered by articular cartilage with small area available for blood vessels (fracture risks blood supply)
Forms floor of anatomical snuffbox
Risk of fracture associated with fall onto outstretched hand (tubercle, waist, or proximal third)
Ulnar deviation AP needed for visualization of scaphoid
Immobilization of scaphoid fractures difficult

Management
Non-displaced fractures - Casts or splints
- Percutaneous scaphoid fixation
Displaced fracture Surgical fixation, usually with a screw

Complications
Non union of scaphoid
Avascular necrosis of the scaphoid
Scapholunate disruption and wrist collapse
Degenerative changes of the adjacent joint

How well did you know this?
1
Not at all
2
3
4
5
Perfectly
17
Q

A 23 year old rugby player falls directly onto his shoulder. There is pain and swelling of the shoulder joint. The clavicle is prominent and there appears to be a step deformity.

A.	Glenohumeral dislocation
B.	Acromioclavicular dislocation
C.	Sternoclavicular dislocation
D.	Biceps tendon tear
E.	Supraspinatus tear
F.	Fracture of the surgical neck of the humerus
G.	Infra spinatus tear
A

Acromioclavicular dislocation

Acromioclavicular joint (ACJ) dislocation normally occurs secondary to direct injury to the superior aspect of the acromion. Loss of shoulder contour and prominent clavicle are key features. Note; rotator cuff tears rarely occur in the second decade.

Shoulder fractures and dislocations
Fractures
Proximal humerus
Background
Third most common fragility fracture in the elderly.
Results from low energy fall in predominantly elderly females, or from high energy trauma in young males.
Can be associated with nerve injury (commonly axillary), and fracture-dislocation of the humeral head. Detailed neurological assessment is essential for all upper limb injuries.

Anatomy
Osteology
Consists of articular head, greater tuberosity, lesser tuberosity, metaphysis and diaphysis. Between the articular head and the tuberosities is the anatomical neck (previous physis). Between the tuberosities and the metaphysis is the surgical neck.
The supraspinatus, infraspinatus and teres minor muscles attach to the greater tuberosity. The subscapularis muscle attaches to the lesser tuberosity.

Vascular Supply
Humeral head is supplied by the anterior and posterior humeral circumflex arteries. Anatomical neck fractures are at greatest risk of osteonecrosis.

Imaging
Imaging aims to both delineate the fracture pattern, and confirm/exlude the presence of an associated dislocation.
Radiographs - True anteroposterior (AP), axillary lateral and/or scapula Y view.
CT - indicated to better define intra-articular involvement and to aid pre-operative planning. MRI is not useful for fracture imaging.

Classification
Description of the fracture is often more useful than classification. Particular attention should be paid to humeral alignment, fracture displacement, and greater tuberosity position (rotator cuff will pull the GT supero-posterioly, which can cause impingement problems with malunion).
- Neer Classification: Most commonly used. Describes fracture as 2,3,or 4 part depending upon the number main fragments. Also comments on the degree of displacement. Fragments:
-greater tuberosity
-lesser tuberosity
- articular surface
- shaft
Displacement: >1cm or angulation >45 degrees.

Treatment
The vast majority of proximal humeral fractures are minimally displaced, and therefore can be managed conservatively. This involves immobilisation in a polysling, and progressive mobilisation. Pendular exercise can commence at 14 days, and active abduction from 4-6 weeks.

Irreducible fracture dislocation is an indication for operative management. Other indications include large displacement, younger patient, head splitting (intra-articular fractures). However, the recent PROFHER trial (1) has suggested no benefit to operative intervention on patient outcome (it must be applied cautiously as majority of patients were elderly with extraarticular fractures). Options available for surgical management include:

ORIF Most commonly used. Plate and screw fixation. Can reconstruct complex fractures.
Intramedullary nail Suitable for extra-articular configuration, predominantly surgical neck +/- GT fractures.

Hemiarthroplasty Used for un-reconstructable fractures in the older patient who has good glenoid quality.
Total shoulder arthroplasty Unconstructable fractures where high functioning shoulder is required (hemiarthroplasty will cause glenoid erosion)
Reverse shoulder arthroplasty Total shoulder arthroplasty that provides better functional outcome than conventional total shoulder replacement.

Scapula
Background
Uncommon fractures usually associated with high energy trauma. Most commonly involve scapula body or spine (50%), glenoid fossa and glenoid neck. Important to exclude associated life threatening injury.

Imaging
Plain radiographs should include true anteroposterior (AP), axillary lateral and/or scapula Y view. CT scanning is useful for defining intra-articular involvement, displacement and for three dimensional reconstruction.

Classification
Based on the location of the fracture (coracoid, acromion, glenoid neck, glenoid fossa, scapula body). Beware of ipsilateral glenoid neck and clavicle fracture -floating shoulder - where limb is effectively dissociated from axial skeleton.

Treatment
The vast majority of scapula fractures are amenable to conservative management, consisting of sling immobilisation for two weeks followed by early rehabilitation. Floating shoulder will usually require fixation, and consideration of surgery should also be given to intra-articular and displaced/angulated glenoid fractures.

Dislocations

Types
Dislocations around the shoulder joint include glenohumeral dislocation, acromioclavicular joint disruption and sternoclavicular dislocation. Only glenohumeral dislocation will be covered here.

Glenohumeral dislocation
Diagnosis, classification and management are covered here.

Background
Shoulder dislocation is commonly seen in A&E. It has a high recurrence rate that is as high as 80% in teenagers. Initial management requires emergent reduction to prevent lasting chondral damage.

Early assessment and management
Usually a traumatic cause (multi-directional instability in frequent dislocations requires discussion with orthopaedics and is not covered here). Careful history, examination and documentation of neurovascular status of the limb, in particular the axillary nerve (regimental badge sensation). This should be re-assessed post manipulation. Early radiographs to confirm direction of dislocation.

Initial management consists of emergent closed reduction under under entanox and analgesia, but often requires conscious sedation. Arm should then be immobilised in a polysling, and XR to confirm relocation.

Imaging - True anteroposterior (AP), axillary lateral and/or scapula Y view. Reduced humeral head should lie between acromion and coracoid on lateral/scapula view.

Types
Direction Features Cause Examination Reduction techniques
Anterior Most Common >90% Usually traumatic - anterior force on arm when shoulder is abducted, eternally rotated Loss of shoulder contour - sulcus sign. Humeral head can be felt anteriorly.
Hippocratic.
Milch.
Stimson.

Kocher not advised due to complication of fracture
Posterior 50% missed in A&E 50% traumatic, but classically post seizure or electrocution Shoulder locked in internal rotation. XR may show lightbulb appearance. Gentle lateral traction to adducted arm.
Inferior Rare Associated with pectorals and rotator cuff tears, and glenoid fracture As for primary injury Management of primary injury

Superior Rare Associated with acrominon/clavicle fracture As for primary injury Management of primary injury

Associated injuries
Bankart lesion - avulsion of the anterior glenoid labrum with an anterior shoulder dislocation (reverse Bankart if poster labrum in posterior dislocation).
Hill Sachs defect - chondral impaction on posteriosuperior humeral head from contact with gleonoid rim. Can be large enough to lock shoulder, requiring open reduction. (Reverse Hill Sachs in posterior dislocation).
Rotator cuff tear - increases with age.
Greater or lesser tuberosity fracture - increases with age.
Humeral neck fracture - shoulder fracture dislocation. More common in high energy trauma and elderly. Should be discussed with orthopaedics prior to any attempted reduction.

Rotator Cuff Disease

Rotator cuff disease is a spectrum of conditions that ranges from subacromial impingement to rotator cuff tears and eventually to rotator cuff arthropathy (arthritis).

Anatomy
The rotator cuff is a group of four muscles that are important in shoulder movements, and maintenance of glenohumeral stability.
Muscle Scapular attachment Humeral attachment Action Innervation
Supraspinatus Supraspinatus fossa Superior facet of greater tuberosity Initiation of abduction of humerus Suprascapular nerve
Infraspinatus Infraspinatus fossa Posterior facet of greater tuberosity External rotation of humerus Suprascapular nerve
Teres Minor Lateral border Inferior facet of greater tuberosity External rotation of humerus Axillary Nerve
Subscapularis Subscapular fossa Lesser tuberosity Internal rotation of humerus Upper and lower subscapular nerve

The inferior rotator cuff muscles (infraspinatus, teres minor, and subscapularis) balance the superior pull of the deltoid. Injury/tear results in upward migration of the humeral head on the glenoid (can be seen on AP radiograph).
Likewise, the anterior muscles (subscapularis) are balanced with the posterior muscles (infraspinatus, teres minor).

Subacromial Impingement

The most common cause of shoulder pain, which results from impingement of the superior cuff on the undersurface of the acromion, and an inflammatory bursitis.
Associated with certain types of acromial morphology (Bigliani classification).
Presents as insidious pain which is exacerbated by overhead activities.

Rotator Cuff Tear

Often presents as an acute event on the background of chronic subacromial impingement in the older patient, but can present as an avulsion injury in younger patients.
Majority of tears are to the superior cuff (supraspinatus, infraspinatus, teres minor), though a tear to subscapularis is associated with subcoracoid impingement.
Tears present as pain and weakness when using the muscles in question.

Rotator Cuff Arthropathy

Defined as shoulder arthritis in the setting of rotator cuff dysfunction. Results from superior migration due to the loss of rotator cuff function and integrity. Unopposed deltoid pulls the humeral head superiorly.
Associated with massive chronic cuff tears.

Imaging

Plain radiographs
AP of the shoulder may show superior migration of the humerus with a cuff tear, and features of arthritis with arthropathy. Other causes of pain may also be identified (e.g. calcific tendonitis/fracture)
Outlet view is useful for defining the acromial morphology

USS
Allows dynamic imaging of the cuff, and is inexpensive. However, it is very user dependent.

MRI
Best imaging modality for cuff pathology.
Also allows imaging of the rest of the shoulder. When intra-articular pathology is suspected, can be combined with an arthrogram for improved sensitivity and specificity.

Treatment

Subacromial impingement
Physiotherapy, oral anti-inflammatory medication
Subacromial steroid injection can settle inflammation
Arthroscopic subacromial decompression by shaving away the undersurface of the acromion, more space is created for the rotator cuff. Cuff integrity is assessed also at time of surgery, and can be repaired if necessary.

Rotator cuff tear
When considering repair of a cuff tear, the age and activity of the patient, the nature of the tear (degenerative vs. acute traumatic), and the size and retraction of the tear should be considered when making a surgical plan.
Mild tears or tears in the elderly can be managed conservatively, as outlined above.
Moderate tears can be repaired arthroscopically. Massive or retracted tears will often require an open repair (occasionally with a tendon transfer). Subacromial decompression is performed at the same time to reduce impingement, symptoms and recurrence.

Calcific tendonitis
Calcific tendonitis involves calcific deposits within tendons anywhere in the body, but most commonly in the rotator cuff (specifically the supraspinatus tendon). When present in the shoulder, it is associated with subacromial impingement and pain.

Pathology
More common in women aged 30-60 years.
Association with diabetes and hypothyroidism

There are three stages of calcification
Formative phase characterized by calcific deposits
Resting phase deposit is stable, but presents with impingement problems
Resorptive phase phagocytic resorption. Most painful stage.

Presentation
Similar in presentation to subacromial impingement, with pain especially with over head activities. Atraumatic in nature.

Imaging
Plain radiographs show calcification of the rotator cuff, usually within 1.5cm of its insertion on the humerus. Supraspinatus outlet views can show level of impingment. Further imaging is rarely needed.

Treatment
Non-operative NSAIDS, steroid injection (controversial, but practiced) and physiotherapy. Approximately 75% will resolve by 6 months with conservative management.
Ultrasound guided or surgical needle barbotage can break down deposits and resolve symptoms. Occasionally surgical excision is required.

Adhesive capsulitis (Frozen Shoulder)
Pain and loss of movement of shoulder joint, which involves fibroplastic proliferation of capsular tissue, causing soft tissue scarring and contracture. Patients present with a painful and decreased arc of motion.
Associated with prolonged immobilization, previous surgery, thyroid disorders (AI) and diabetes
Classically three stages which can take up to two years to resolve:
Stage one the freezing and painful stage
Stage two the frozen and stiff stage
Stage three the thawing stage, where shoulder movement slowly improves

Imaging
Plain radiographs to exclude other causes of a painful shoulder
MRI arthrogram may show capsular contracture, and again may be used to exclude cuff pathology. However, often not performed as diagnosis is largely clinical.

Treatment
Non-operative NSAIDS, steroid injection and physiotherapy. Patience is required as condition can take up to 2 years to improve.
Operative MUA or arthroscopic adhesiolysis (release of adhesions) can expedite recovery, followed by intensive physiotherapy.

Glenohumeral Arthritis
Background
May be osteoarthritis (primary or secondary to cuff tear or trauma), rheumatoid arthritis, or as part of a spondyloarthropathy. Majority of those with RA will develop symptoms.
More common in the elderly
Presents like any other arthritis - pain at night and with movement

Imaging
AP and axillary radiographs will show features of arthritis.
CT/MRI is often useful to classify the shape of the glenoid and extent of bone loss when considering arthroplasty. MRI also essential to asses integrity of rotator cuff if considering shoulder replacement.

Treatment
Like all orthopaedics, start with simple measures:
NSAIDS, management of RA, physiotherapy, steroid injection.
Hemiarthroplasty can sometimes be considered if glenoid is in excellent condition or if patient has large comorbidity.
Arthroscopic debridement is useful if patient has isolated ACJ arthritis, but is rarely used for glenohumeral arthritis.
Total shoulder replacement is shown to produce superior outcome when compared to hemiarthroplasty in terms of pain relief, function and implant survival.
Total shoulder replacement can be anatomical (ball on humerus, with cup on glenoid), or reverse geometry (ball on glenoid, with cup on humerus). Anatomical TSR requires an in tact rotator cuff, so often reverse is preferable when the cuff if questionable in integrity.

How well did you know this?
1
Not at all
2
3
4
5
Perfectly
18
Q

A 22 year old man falls over and presents to casualty. A shoulder x-ray is performed, the radiologist comments that a Hill-Sachs lesion is present.

A.	Glenohumeral dislocation
B.	Acromioclavicular dislocation
C.	Sternoclavicular dislocation
D.	Biceps tendon tear
E.	Supraspinatus tear
F.	Fracture of the surgical neck of the humerus
G.	Infra spinatus tear
A

Glenohumeral dislocation

A Hill-Sachs lesion occurs when the cartilage surface of the humerus is in contact with the rim of the glenoid. About 50% of anterior glenohumeral dislocations are associated with this lesion.

Shoulder fractures and dislocations
Fractures
Proximal humerus
Background
Third most common fragility fracture in the elderly.
Results from low energy fall in predominantly elderly females, or from high energy trauma in young males.
Can be associated with nerve injury (commonly axillary), and fracture-dislocation of the humeral head. Detailed neurological assessment is essential for all upper limb injuries.

Anatomy
Osteology
Consists of articular head, greater tuberosity, lesser tuberosity, metaphysis and diaphysis. Between the articular head and the tuberosities is the anatomical neck (previous physis). Between the tuberosities and the metaphysis is the surgical neck.
The supraspinatus, infraspinatus and teres minor muscles attach to the greater tuberosity. The subscapularis muscle attaches to the lesser tuberosity.

Vascular Supply
Humeral head is supplied by the anterior and posterior humeral circumflex arteries. Anatomical neck fractures are at greatest risk of osteonecrosis.

Imaging
Imaging aims to both delineate the fracture pattern, and confirm/exlude the presence of an associated dislocation.
Radiographs - True anteroposterior (AP), axillary lateral and/or scapula Y view.
CT - indicated to better define intra-articular involvement and to aid pre-operative planning. MRI is not useful for fracture imaging.

Classification
Description of the fracture is often more useful than classification. Particular attention should be paid to humeral alignment, fracture displacement, and greater tuberosity position (rotator cuff will pull the GT supero-posterioly, which can cause impingement problems with malunion).
- Neer Classification: Most commonly used. Describes fracture as 2,3,or 4 part depending upon the number main fragments. Also comments on the degree of displacement. Fragments:
-greater tuberosity
-lesser tuberosity
- articular surface
- shaft
Displacement: >1cm or angulation >45 degrees.

Treatment
The vast majority of proximal humeral fractures are minimally displaced, and therefore can be managed conservatively. This involves immobilisation in a polysling, and progressive mobilisation. Pendular exercise can commence at 14 days, and active abduction from 4-6 weeks.

Irreducible fracture dislocation is an indication for operative management. Other indications include large displacement, younger patient, head splitting (intra-articular fractures). However, the recent PROFHER trial (1) has suggested no benefit to operative intervention on patient outcome (it must be applied cautiously as majority of patients were elderly with extraarticular fractures). Options available for surgical management include:

ORIF Most commonly used. Plate and screw fixation. Can reconstruct complex fractures.
Intramedullary nail Suitable for extra-articular configuration, predominantly surgical neck +/- GT fractures.

Hemiarthroplasty Used for un-reconstructable fractures in the older patient who has good glenoid quality.
Total shoulder arthroplasty Unconstructable fractures where high functioning shoulder is required (hemiarthroplasty will cause glenoid erosion)
Reverse shoulder arthroplasty Total shoulder arthroplasty that provides better functional outcome than conventional total shoulder replacement.

Scapula
Background
Uncommon fractures usually associated with high energy trauma. Most commonly involve scapula body or spine (50%), glenoid fossa and glenoid neck. Important to exclude associated life threatening injury.

Imaging
Plain radiographs should include true anteroposterior (AP), axillary lateral and/or scapula Y view. CT scanning is useful for defining intra-articular involvement, displacement and for three dimensional reconstruction.

Classification
Based on the location of the fracture (coracoid, acromion, glenoid neck, glenoid fossa, scapula body). Beware of ipsilateral glenoid neck and clavicle fracture -floating shoulder - where limb is effectively dissociated from axial skeleton.

Treatment
The vast majority of scapula fractures are amenable to conservative management, consisting of sling immobilisation for two weeks followed by early rehabilitation. Floating shoulder will usually require fixation, and consideration of surgery should also be given to intra-articular and displaced/angulated glenoid fractures.

Dislocations

Types
Dislocations around the shoulder joint include glenohumeral dislocation, acromioclavicular joint disruption and sternoclavicular dislocation. Only glenohumeral dislocation will be covered here.

Glenohumeral dislocation
Diagnosis, classification and management are covered here.

Background
Shoulder dislocation is commonly seen in A&E. It has a high recurrence rate that is as high as 80% in teenagers. Initial management requires emergent reduction to prevent lasting chondral damage.

Early assessment and management
Usually a traumatic cause (multi-directional instability in frequent dislocations requires discussion with orthopaedics and is not covered here). Careful history, examination and documentation of neurovascular status of the limb, in particular the axillary nerve (regimental badge sensation). This should be re-assessed post manipulation. Early radiographs to confirm direction of dislocation.

Initial management consists of emergent closed reduction under under entanox and analgesia, but often requires conscious sedation. Arm should then be immobilised in a polysling, and XR to confirm relocation.

Imaging - True anteroposterior (AP), axillary lateral and/or scapula Y view. Reduced humeral head should lie between acromion and coracoid on lateral/scapula view.

Types
Direction Features Cause Examination Reduction techniques
Anterior Most Common >90% Usually traumatic - anterior force on arm when shoulder is abducted, eternally rotated Loss of shoulder contour - sulcus sign. Humeral head can be felt anteriorly.
Hippocratic.
Milch.
Stimson.

Kocher not advised due to complication of fracture
Posterior 50% missed in A&E 50% traumatic, but classically post seizure or electrocution Shoulder locked in internal rotation. XR may show lightbulb appearance. Gentle lateral traction to adducted arm.
Inferior Rare Associated with pectorals and rotator cuff tears, and glenoid fracture As for primary injury Management of primary injury

Superior Rare Associated with acrominon/clavicle fracture As for primary injury Management of primary injury

Associated injuries
Bankart lesion - avulsion of the anterior glenoid labrum with an anterior shoulder dislocation (reverse Bankart if poster labrum in posterior dislocation).
Hill Sachs defect - chondral impaction on posteriosuperior humeral head from contact with gleonoid rim. Can be large enough to lock shoulder, requiring open reduction. (Reverse Hill Sachs in posterior dislocation).
Rotator cuff tear - increases with age.
Greater or lesser tuberosity fracture - increases with age.
Humeral neck fracture - shoulder fracture dislocation. More common in high energy trauma and elderly. Should be discussed with orthopaedics prior to any attempted reduction.

Rotator Cuff Disease

Rotator cuff disease is a spectrum of conditions that ranges from subacromial impingement to rotator cuff tears and eventually to rotator cuff arthropathy (arthritis).

Anatomy
The rotator cuff is a group of four muscles that are important in shoulder movements, and maintenance of glenohumeral stability.
Muscle Scapular attachment Humeral attachment Action Innervation
Supraspinatus Supraspinatus fossa Superior facet of greater tuberosity Initiation of abduction of humerus Suprascapular nerve
Infraspinatus Infraspinatus fossa Posterior facet of greater tuberosity External rotation of humerus Suprascapular nerve
Teres Minor Lateral border Inferior facet of greater tuberosity External rotation of humerus Axillary Nerve
Subscapularis Subscapular fossa Lesser tuberosity Internal rotation of humerus Upper and lower subscapular nerve

The inferior rotator cuff muscles (infraspinatus, teres minor, and subscapularis) balance the superior pull of the deltoid. Injury/tear results in upward migration of the humeral head on the glenoid (can be seen on AP radiograph).
Likewise, the anterior muscles (subscapularis) are balanced with the posterior muscles (infraspinatus, teres minor).

Subacromial Impingement

The most common cause of shoulder pain, which results from impingement of the superior cuff on the undersurface of the acromion, and an inflammatory bursitis.
Associated with certain types of acromial morphology (Bigliani classification).
Presents as insidious pain which is exacerbated by overhead activities.

Rotator Cuff Tear

Often presents as an acute event on the background of chronic subacromial impingement in the older patient, but can present as an avulsion injury in younger patients.
Majority of tears are to the superior cuff (supraspinatus, infraspinatus, teres minor), though a tear to subscapularis is associated with subcoracoid impingement.
Tears present as pain and weakness when using the muscles in question.

Rotator Cuff Arthropathy

Defined as shoulder arthritis in the setting of rotator cuff dysfunction. Results from superior migration due to the loss of rotator cuff function and integrity. Unopposed deltoid pulls the humeral head superiorly.
Associated with massive chronic cuff tears.

Imaging

Plain radiographs
AP of the shoulder may show superior migration of the humerus with a cuff tear, and features of arthritis with arthropathy. Other causes of pain may also be identified (e.g. calcific tendonitis/fracture)
Outlet view is useful for defining the acromial morphology

USS
Allows dynamic imaging of the cuff, and is inexpensive. However, it is very user dependent.

MRI
Best imaging modality for cuff pathology.
Also allows imaging of the rest of the shoulder. When intra-articular pathology is suspected, can be combined with an arthrogram for improved sensitivity and specificity.

Treatment

Subacromial impingement
Physiotherapy, oral anti-inflammatory medication
Subacromial steroid injection can settle inflammation
Arthroscopic subacromial decompression by shaving away the undersurface of the acromion, more space is created for the rotator cuff. Cuff integrity is assessed also at time of surgery, and can be repaired if necessary.

Rotator cuff tear
When considering repair of a cuff tear, the age and activity of the patient, the nature of the tear (degenerative vs. acute traumatic), and the size and retraction of the tear should be considered when making a surgical plan.
Mild tears or tears in the elderly can be managed conservatively, as outlined above.
Moderate tears can be repaired arthroscopically. Massive or retracted tears will often require an open repair (occasionally with a tendon transfer). Subacromial decompression is performed at the same time to reduce impingement, symptoms and recurrence.

Calcific tendonitis
Calcific tendonitis involves calcific deposits within tendons anywhere in the body, but most commonly in the rotator cuff (specifically the supraspinatus tendon). When present in the shoulder, it is associated with subacromial impingement and pain.

Pathology
More common in women aged 30-60 years.
Association with diabetes and hypothyroidism

There are three stages of calcification
Formative phase characterized by calcific deposits
Resting phase deposit is stable, but presents with impingement problems
Resorptive phase phagocytic resorption. Most painful stage.

Presentation
Similar in presentation to subacromial impingement, with pain especially with over head activities. Atraumatic in nature.

Imaging
Plain radiographs show calcification of the rotator cuff, usually within 1.5cm of its insertion on the humerus. Supraspinatus outlet views can show level of impingment. Further imaging is rarely needed.

Treatment
Non-operative NSAIDS, steroid injection (controversial, but practiced) and physiotherapy. Approximately 75% will resolve by 6 months with conservative management.
Ultrasound guided or surgical needle barbotage can break down deposits and resolve symptoms. Occasionally surgical excision is required.

Adhesive capsulitis (Frozen Shoulder)
Pain and loss of movement of shoulder joint, which involves fibroplastic proliferation of capsular tissue, causing soft tissue scarring and contracture. Patients present with a painful and decreased arc of motion.
Associated with prolonged immobilization, previous surgery, thyroid disorders (AI) and diabetes
Classically three stages which can take up to two years to resolve:
Stage one the freezing and painful stage
Stage two the frozen and stiff stage
Stage three the thawing stage, where shoulder movement slowly improves

Imaging
Plain radiographs to exclude other causes of a painful shoulder
MRI arthrogram may show capsular contracture, and again may be used to exclude cuff pathology. However, often not performed as diagnosis is largely clinical.

Treatment
Non-operative NSAIDS, steroid injection and physiotherapy. Patience is required as condition can take up to 2 years to improve.
Operative MUA or arthroscopic adhesiolysis (release of adhesions) can expedite recovery, followed by intensive physiotherapy.

Glenohumeral Arthritis
Background
May be osteoarthritis (primary or secondary to cuff tear or trauma), rheumatoid arthritis, or as part of a spondyloarthropathy. Majority of those with RA will develop symptoms.
More common in the elderly
Presents like any other arthritis - pain at night and with movement

Imaging
AP and axillary radiographs will show features of arthritis.
CT/MRI is often useful to classify the shape of the glenoid and extent of bone loss when considering arthroplasty. MRI also essential to asses integrity of rotator cuff if considering shoulder replacement.

Treatment
Like all orthopaedics, start with simple measures:
NSAIDS, management of RA, physiotherapy, steroid injection.
Hemiarthroplasty can sometimes be considered if glenoid is in excellent condition or if patient has large comorbidity.
Arthroscopic debridement is useful if patient has isolated ACJ arthritis, but is rarely used for glenohumeral arthritis.
Total shoulder replacement is shown to produce superior outcome when compared to hemiarthroplasty in terms of pain relief, function and implant survival.
Total shoulder replacement can be anatomical (ball on humerus, with cup on glenoid), or reverse geometry (ball on glenoid, with cup on humerus). Anatomical TSR requires an in tact rotator cuff, so often reverse is preferable when the cuff if questionable in integrity.

How well did you know this?
1
Not at all
2
3
4
5
Perfectly
19
Q

An 82 year old female presents to A&E after tripping on a step. She complains of shoulder pain. On examination there is pain to 90o on abduction.

A.	Glenohumeral dislocation
B.	Acromioclavicular dislocation
C.	Sternoclavicular dislocation
D.	Biceps tendon tear
E.	Supraspinatus tear
F.	Fracture of the surgical neck of the humerus
G.	Infra spinatus tear
A

Supraspinatus tear

A supraspinatus tear is the most common of rotator cuff tears. It occurs as a result of degeneration and is rare in younger adults.

Shoulder fractures and dislocations
Fractures
Proximal humerus
Background
Third most common fragility fracture in the elderly.
Results from low energy fall in predominantly elderly females, or from high energy trauma in young males.
Can be associated with nerve injury (commonly axillary), and fracture-dislocation of the humeral head. Detailed neurological assessment is essential for all upper limb injuries.

Anatomy
Osteology
Consists of articular head, greater tuberosity, lesser tuberosity, metaphysis and diaphysis. Between the articular head and the tuberosities is the anatomical neck (previous physis). Between the tuberosities and the metaphysis is the surgical neck.
The supraspinatus, infraspinatus and teres minor muscles attach to the greater tuberosity. The subscapularis muscle attaches to the lesser tuberosity.

Vascular Supply
Humeral head is supplied by the anterior and posterior humeral circumflex arteries. Anatomical neck fractures are at greatest risk of osteonecrosis.

Imaging
Imaging aims to both delineate the fracture pattern, and confirm/exlude the presence of an associated dislocation.
Radiographs - True anteroposterior (AP), axillary lateral and/or scapula Y view.
CT - indicated to better define intra-articular involvement and to aid pre-operative planning. MRI is not useful for fracture imaging.

Classification
Description of the fracture is often more useful than classification. Particular attention should be paid to humeral alignment, fracture displacement, and greater tuberosity position (rotator cuff will pull the GT supero-posterioly, which can cause impingement problems with malunion).
- Neer Classification: Most commonly used. Describes fracture as 2,3,or 4 part depending upon the number main fragments. Also comments on the degree of displacement. Fragments:
-greater tuberosity
-lesser tuberosity
- articular surface
- shaft
Displacement: >1cm or angulation >45 degrees.

Treatment
The vast majority of proximal humeral fractures are minimally displaced, and therefore can be managed conservatively. This involves immobilisation in a polysling, and progressive mobilisation. Pendular exercise can commence at 14 days, and active abduction from 4-6 weeks.

Irreducible fracture dislocation is an indication for operative management. Other indications include large displacement, younger patient, head splitting (intra-articular fractures). However, the recent PROFHER trial (1) has suggested no benefit to operative intervention on patient outcome (it must be applied cautiously as majority of patients were elderly with extraarticular fractures). Options available for surgical management include:

ORIF Most commonly used. Plate and screw fixation. Can reconstruct complex fractures.
Intramedullary nail Suitable for extra-articular configuration, predominantly surgical neck +/- GT fractures.

Hemiarthroplasty Used for un-reconstructable fractures in the older patient who has good glenoid quality.
Total shoulder arthroplasty Unconstructable fractures where high functioning shoulder is required (hemiarthroplasty will cause glenoid erosion)
Reverse shoulder arthroplasty Total shoulder arthroplasty that provides better functional outcome than conventional total shoulder replacement.

Scapula
Background
Uncommon fractures usually associated with high energy trauma. Most commonly involve scapula body or spine (50%), glenoid fossa and glenoid neck. Important to exclude associated life threatening injury.

Imaging
Plain radiographs should include true anteroposterior (AP), axillary lateral and/or scapula Y view. CT scanning is useful for defining intra-articular involvement, displacement and for three dimensional reconstruction.

Classification
Based on the location of the fracture (coracoid, acromion, glenoid neck, glenoid fossa, scapula body). Beware of ipsilateral glenoid neck and clavicle fracture -floating shoulder - where limb is effectively dissociated from axial skeleton.

Treatment
The vast majority of scapula fractures are amenable to conservative management, consisting of sling immobilisation for two weeks followed by early rehabilitation. Floating shoulder will usually require fixation, and consideration of surgery should also be given to intra-articular and displaced/angulated glenoid fractures.

Dislocations

Types
Dislocations around the shoulder joint include glenohumeral dislocation, acromioclavicular joint disruption and sternoclavicular dislocation. Only glenohumeral dislocation will be covered here.

Glenohumeral dislocation
Diagnosis, classification and management are covered here.

Background
Shoulder dislocation is commonly seen in A&E. It has a high recurrence rate that is as high as 80% in teenagers. Initial management requires emergent reduction to prevent lasting chondral damage.

Early assessment and management
Usually a traumatic cause (multi-directional instability in frequent dislocations requires discussion with orthopaedics and is not covered here). Careful history, examination and documentation of neurovascular status of the limb, in particular the axillary nerve (regimental badge sensation). This should be re-assessed post manipulation. Early radiographs to confirm direction of dislocation.

Initial management consists of emergent closed reduction under under entanox and analgesia, but often requires conscious sedation. Arm should then be immobilised in a polysling, and XR to confirm relocation.

Imaging - True anteroposterior (AP), axillary lateral and/or scapula Y view. Reduced humeral head should lie between acromion and coracoid on lateral/scapula view.

Types
Direction Features Cause Examination Reduction techniques
Anterior Most Common >90% Usually traumatic - anterior force on arm when shoulder is abducted, eternally rotated Loss of shoulder contour - sulcus sign. Humeral head can be felt anteriorly.
Hippocratic.
Milch.
Stimson.

Kocher not advised due to complication of fracture
Posterior 50% missed in A&E 50% traumatic, but classically post seizure or electrocution Shoulder locked in internal rotation. XR may show lightbulb appearance. Gentle lateral traction to adducted arm.
Inferior Rare Associated with pectorals and rotator cuff tears, and glenoid fracture As for primary injury Management of primary injury

Superior Rare Associated with acrominon/clavicle fracture As for primary injury Management of primary injury

Associated injuries
Bankart lesion - avulsion of the anterior glenoid labrum with an anterior shoulder dislocation (reverse Bankart if poster labrum in posterior dislocation).
Hill Sachs defect - chondral impaction on posteriosuperior humeral head from contact with gleonoid rim. Can be large enough to lock shoulder, requiring open reduction. (Reverse Hill Sachs in posterior dislocation).
Rotator cuff tear - increases with age.
Greater or lesser tuberosity fracture - increases with age.
Humeral neck fracture - shoulder fracture dislocation. More common in high energy trauma and elderly. Should be discussed with orthopaedics prior to any attempted reduction.

Rotator Cuff Disease

Rotator cuff disease is a spectrum of conditions that ranges from subacromial impingement to rotator cuff tears and eventually to rotator cuff arthropathy (arthritis).

Anatomy
The rotator cuff is a group of four muscles that are important in shoulder movements, and maintenance of glenohumeral stability.
Muscle Scapular attachment Humeral attachment Action Innervation
Supraspinatus Supraspinatus fossa Superior facet of greater tuberosity Initiation of abduction of humerus Suprascapular nerve
Infraspinatus Infraspinatus fossa Posterior facet of greater tuberosity External rotation of humerus Suprascapular nerve
Teres Minor Lateral border Inferior facet of greater tuberosity External rotation of humerus Axillary Nerve
Subscapularis Subscapular fossa Lesser tuberosity Internal rotation of humerus Upper and lower subscapular nerve

The inferior rotator cuff muscles (infraspinatus, teres minor, and subscapularis) balance the superior pull of the deltoid. Injury/tear results in upward migration of the humeral head on the glenoid (can be seen on AP radiograph).
Likewise, the anterior muscles (subscapularis) are balanced with the posterior muscles (infraspinatus, teres minor).

Subacromial Impingement

The most common cause of shoulder pain, which results from impingement of the superior cuff on the undersurface of the acromion, and an inflammatory bursitis.
Associated with certain types of acromial morphology (Bigliani classification).
Presents as insidious pain which is exacerbated by overhead activities.

Rotator Cuff Tear

Often presents as an acute event on the background of chronic subacromial impingement in the older patient, but can present as an avulsion injury in younger patients.
Majority of tears are to the superior cuff (supraspinatus, infraspinatus, teres minor), though a tear to subscapularis is associated with subcoracoid impingement.
Tears present as pain and weakness when using the muscles in question.

Rotator Cuff Arthropathy

Defined as shoulder arthritis in the setting of rotator cuff dysfunction. Results from superior migration due to the loss of rotator cuff function and integrity. Unopposed deltoid pulls the humeral head superiorly.
Associated with massive chronic cuff tears.

Imaging

Plain radiographs
AP of the shoulder may show superior migration of the humerus with a cuff tear, and features of arthritis with arthropathy. Other causes of pain may also be identified (e.g. calcific tendonitis/fracture)
Outlet view is useful for defining the acromial morphology

USS
Allows dynamic imaging of the cuff, and is inexpensive. However, it is very user dependent.

MRI
Best imaging modality for cuff pathology.
Also allows imaging of the rest of the shoulder. When intra-articular pathology is suspected, can be combined with an arthrogram for improved sensitivity and specificity.

Treatment

Subacromial impingement
Physiotherapy, oral anti-inflammatory medication
Subacromial steroid injection can settle inflammation
Arthroscopic subacromial decompression by shaving away the undersurface of the acromion, more space is created for the rotator cuff. Cuff integrity is assessed also at time of surgery, and can be repaired if necessary.

Rotator cuff tear
When considering repair of a cuff tear, the age and activity of the patient, the nature of the tear (degenerative vs. acute traumatic), and the size and retraction of the tear should be considered when making a surgical plan.
Mild tears or tears in the elderly can be managed conservatively, as outlined above.
Moderate tears can be repaired arthroscopically. Massive or retracted tears will often require an open repair (occasionally with a tendon transfer). Subacromial decompression is performed at the same time to reduce impingement, symptoms and recurrence.

Calcific tendonitis
Calcific tendonitis involves calcific deposits within tendons anywhere in the body, but most commonly in the rotator cuff (specifically the supraspinatus tendon). When present in the shoulder, it is associated with subacromial impingement and pain.

Pathology
More common in women aged 30-60 years.
Association with diabetes and hypothyroidism

There are three stages of calcification
Formative phase characterized by calcific deposits
Resting phase deposit is stable, but presents with impingement problems
Resorptive phase phagocytic resorption. Most painful stage.

Presentation
Similar in presentation to subacromial impingement, with pain especially with over head activities. Atraumatic in nature.

Imaging
Plain radiographs show calcification of the rotator cuff, usually within 1.5cm of its insertion on the humerus. Supraspinatus outlet views can show level of impingment. Further imaging is rarely needed.

Treatment
Non-operative NSAIDS, steroid injection (controversial, but practiced) and physiotherapy. Approximately 75% will resolve by 6 months with conservative management.
Ultrasound guided or surgical needle barbotage can break down deposits and resolve symptoms. Occasionally surgical excision is required.

Adhesive capsulitis (Frozen Shoulder)
Pain and loss of movement of shoulder joint, which involves fibroplastic proliferation of capsular tissue, causing soft tissue scarring and contracture. Patients present with a painful and decreased arc of motion.
Associated with prolonged immobilization, previous surgery, thyroid disorders (AI) and diabetes
Classically three stages which can take up to two years to resolve:
Stage one the freezing and painful stage
Stage two the frozen and stiff stage
Stage three the thawing stage, where shoulder movement slowly improves

Imaging
Plain radiographs to exclude other causes of a painful shoulder
MRI arthrogram may show capsular contracture, and again may be used to exclude cuff pathology. However, often not performed as diagnosis is largely clinical.

Treatment
Non-operative NSAIDS, steroid injection and physiotherapy. Patience is required as condition can take up to 2 years to improve.
Operative MUA or arthroscopic adhesiolysis (release of adhesions) can expedite recovery, followed by intensive physiotherapy.

Glenohumeral Arthritis
Background
May be osteoarthritis (primary or secondary to cuff tear or trauma), rheumatoid arthritis, or as part of a spondyloarthropathy. Majority of those with RA will develop symptoms.
More common in the elderly
Presents like any other arthritis - pain at night and with movement

Imaging
AP and axillary radiographs will show features of arthritis.
CT/MRI is often useful to classify the shape of the glenoid and extent of bone loss when considering arthroplasty. MRI also essential to asses integrity of rotator cuff if considering shoulder replacement.

Treatment
Like all orthopaedics, start with simple measures:
NSAIDS, management of RA, physiotherapy, steroid injection.
Hemiarthroplasty can sometimes be considered if glenoid is in excellent condition or if patient has large comorbidity.
Arthroscopic debridement is useful if patient has isolated ACJ arthritis, but is rarely used for glenohumeral arthritis.
Total shoulder replacement is shown to produce superior outcome when compared to hemiarthroplasty in terms of pain relief, function and implant survival.
Total shoulder replacement can be anatomical (ball on humerus, with cup on glenoid), or reverse geometry (ball on glenoid, with cup on humerus). Anatomical TSR requires an in tact rotator cuff, so often reverse is preferable when the cuff if questionable in integrity.

How well did you know this?
1
Not at all
2
3
4
5
Perfectly
20
Q

Which of the following statements relating to menisceal tears is false?

The medial meniscus is most often affected
True locking of the knee joint may occur
Most established tears will heal with conservative management
In the chronic setting there is typically little to find on examination if the knee is not locked
An arthroscopic approach may be used to treat most lesions
A

Most established tears will heal with conservative management - FALSE

Menisci have no nerve or blood supply and thus heal poorly. Established tears with associated symptoms are best managed by arthroscopic menisectomy.

Ruptured anterior cruciate ligament
Sport injury
Mechanism: high twisting force applied to a bent knee
Typically presents with: loud crack, pain and RAPID joint swelling (haemoarthrosis)
Poor healing
Management: intense physiotherapy or surgery
Ruptured posterior cruciate ligament
Mechanism: hyperextension injuries
Tibia lies back on the femur
Paradoxical anterior draw test
Rupture of medial collateral ligament
Mechanism: leg forced into valgus via force outside the leg
Knee unstable when put into valgus position
Menisceal tear
Rotational sporting injuries
Delayed knee swelling
Joint locking (Patient may develop skills to “unlock” the knee
Recurrent episodes of pain and effusions are common, often following minor trauma
Chondromalacia patellae
Teenage girls, following an injury to knee e.g. Dislocation patella
Typical history of pain on going downstairs or at rest
Tenderness, quadriceps wasting
Dislocation of the patella
Most commonly occurs as a traumatic primary event, either through direct trauma or through severe contraction of quadriceps with knee stretched in valgus and external rotation
Genu valgum, tibial torsion and high riding patella are risk factors
Skyline x-ray views of patella are required, although displaced patella may be clinically obvious
An osteochondral fracture is present in 5%
The condition has a 20% recurrence rate
Fractured patella
2 types:
i. Direct blow to patella causing undisplaced fragments
ii. Avulsion fracture
Tibial plateau fracture
Occur in the elderly (or following significant trauma in young)
Mechanism: knee forced into valgus or varus, but the knee fractures before the ligaments rupture
Varus injury affects medial plateau and if valgus injury, lateral plateau depressed fracture occurs
Classified using the Schatzker system (see below)

Schatzker Classification system for tibial plateau fractures
Type Anatomical description Features
1 Vertical split of lateral condyle Fracture through dense bone, usually in the young. It may be virtually undisplaced, or the condylar fragment may be pushed inferiorly and tilted
2 Vertical split of the lateral condyle combined with an adjacent load bearing part of the condyle The wedge fragment (which may be of variable size), is displaced laterally; the joint is widened. Untreated, a valgus deformity may develop
3 Depression of the articular surface with intact condylar rim The split does not extend to the edge of the plateau. Depressed fragments may be firmly embedded in subchondral bone, the joint is stable
4 Fragment of the medial tibial condyle Two injuries are seen in this category; (1) a depressed fracture of osteoporotic bone in the elderly. (2) a high energy fracture resulting in a condylar split that runs from the intercondylar eminence to the medial cortex. Associated ligamentous injury may be severe
5 Fracture of both condyles Both condyles fractured but the column of the metaphysis remains in continuity with the tibial shaft
6 Combined condylar and subcondylar fractures High energy fracture with marked comminution

How well did you know this?
1
Not at all
2
3
4
5
Perfectly
21
Q

A 15 year-old boy presents to the out-patient clinic with tiredness, recurrent throat and chest infections, and gradual loss of vision. Multiple x-rays show brittle bones with no differentiation between the cortex and the medulla.

A.	Rickets
B.	Craniocleidodysostosis
C.	Achondroplasia
D.	Scurvy
E.	Pagets disease
F.	Multiple myeloma
G.	Osteogenesis imperfecta
H.	Osteomalacia
I.	Osteopetrosis
J.	None of the above
A

Osteopetrosis

Osteopetrosis is an autosomal recessive condition. It is commonest in young adults. They may present with symptoms of anaemia or thrombocytopaenia due to decreased marrow space. Radiology reveals a lack of differentiation between the cortex and the medulla described as marble bone. These bones are very dense and brittle.

Paediatric fracture types
Type Injury pattern
Complete fracture Both sides of cortex are breached
Toddlers fracture Oblique tibial fracture in infants
Plastic deformity Stress on bone resulting in deformity without cortical disruption
Greenstick fracture Unilateral cortical breach only
Buckle fracture Incomplete cortical disruption resulting in periosteal haematoma only

Growth plate fractures
In paediatric practice fractures may also involve the growth plate and these injuries are classified according to the Salter- Harris system (given below):

Type Injury pattern
I Fracture through the physis only (x-ray often normal)
II Fracture through the physis and metaphysis
III Fracture through the physis and epiphyisis to include the joint
IV Fracture involving the physis, metaphysis and epiphysis
V Crush injury involving the physis (x-ray may resemble type I, and appear normal)

As a general rule it is safer to assume that growth plate tenderness is indicative of an underlying fracture even if the x-ray appears normal. Injuries of Types III, IV and V will usually require surgery. Type V injuries are often associated with disruption to growth.

Non accidental injury
Delayed presentation
Delay in attaining milestones
Lack of concordance between proposed and actual mechanism of injury
Multiple injuries
Injuries at sites not commonly exposed to trauma
Children on the at risk register

Pathological fractures
Genetic conditions, such as osteogenesis imperfecta, may cause pathological fractures.

Osteogenesis imperfecta
Defective osteoid formation due to congenital inability to produce adequate intercellular substances like osteoid, collagen and dentine.
Failure of maturation of collagen in all the connective tissues.
Radiology may show translucent bones, multiple fractures, particularly of the long bones, wormian bones (irregular patches of ossification) and a trefoil pelvis.

Subtypes
Type I The collagen is normal quality but insufficient quantity.
Type II- Poor collagen quantity and quality.
Type III- Collagen poorly formed. Normal quantity.
Type IV- Sufficient collagen quantity but poor quality.

Osteopetrosis
Bones become harder and more dense.
Autosomal recessive condition.
It is commonest in young adults.
Radiology reveals a lack of differentiation between the cortex and the medulla described as marble bone.
How well did you know this?
1
Not at all
2
3
4
5
Perfectly
22
Q

A 12 year-old boy who is small for his age presents to the clinic with poor muscular development and hyper-mobile fingers. His x-rays show multiple fractures of the long bones and irregular patches of ossification.

A.	Rickets
B.	Craniocleidodysostosis
C.	Achondroplasia
D.	Scurvy
E.	Pagets disease
F.	Multiple myeloma
G.	Osteogenesis imperfecta
H.	Osteomalacia
I.	Osteopetrosis
J.	None of the above
A

Osteogenesis imperfecta

Osteogenesis imperfecta is caused by defective osteoid formation due to congenital inability to produce adequate intercellular substances like osteoid, collagen and dentine. There is a failure of maturation of collagen in all the connective tissues.Radiology may show translucent bones, multiple fractures, particularly of the long bones, wormian bones (irregular patches of ossification) and a trefoil pelvis.

Paediatric fracture types
Type Injury pattern
Complete fracture Both sides of cortex are breached
Toddlers fracture Oblique tibial fracture in infants
Plastic deformity Stress on bone resulting in deformity without cortical disruption
Greenstick fracture Unilateral cortical breach only
Buckle fracture Incomplete cortical disruption resulting in periosteal haematoma only

Growth plate fractures
In paediatric practice fractures may also involve the growth plate and these injuries are classified according to the Salter- Harris system (given below):

Type Injury pattern
I Fracture through the physis only (x-ray often normal)
II Fracture through the physis and metaphysis
III Fracture through the physis and epiphyisis to include the joint
IV Fracture involving the physis, metaphysis and epiphysis
V Crush injury involving the physis (x-ray may resemble type I, and appear normal)

As a general rule it is safer to assume that growth plate tenderness is indicative of an underlying fracture even if the x-ray appears normal. Injuries of Types III, IV and V will usually require surgery. Type V injuries are often associated with disruption to growth.

Non accidental injury
Delayed presentation
Delay in attaining milestones
Lack of concordance between proposed and actual mechanism of injury
Multiple injuries
Injuries at sites not commonly exposed to trauma
Children on the at risk register

Pathological fractures
Genetic conditions, such as osteogenesis imperfecta, may cause pathological fractures.

Osteogenesis imperfecta
Defective osteoid formation due to congenital inability to produce adequate intercellular substances like osteoid, collagen and dentine.
Failure of maturation of collagen in all the connective tissues.
Radiology may show translucent bones, multiple fractures, particularly of the long bones, wormian bones (irregular patches of ossification) and a trefoil pelvis.

Subtypes
Type I The collagen is normal quality but insufficient quantity.
Type II- Poor collagen quantity and quality.
Type III- Collagen poorly formed. Normal quantity.
Type IV- Sufficient collagen quantity but poor quality.

Osteopetrosis
Bones become harder and more dense.
Autosomal recessive condition.
It is commonest in young adults.
Radiology reveals a lack of differentiation between the cortex and the medulla described as marble bone.
How well did you know this?
1
Not at all
2
3
4
5
Perfectly
23
Q

A 1 year-old is brought to the Emergency Department with a history of failure to thrive. On examination, the child is small for age and has a large head. X-ray shows a cupped appearance of the epiphysis of the wrist.

A.	Rickets
B.	Craniocleidodysostosis
C.	Achondroplasia
D.	Scurvy
E.	Pagets disease
F.	Multiple myeloma
G.	Osteogenesis imperfecta
H.	Osteomalacia
I.	Osteopetrosis
J.	None of the above
A

Rickets

Rickets is the childhood form of osteomalacia. It is due to the failure of the osteoid to ossify due to vitamin D deficiency. Symptoms start about the age of one. The child is small for age and there is a history of failure to thrive. Bony deformities include bowing of the femur and tibia, a large head, deformity of the chest wall with thickening of the costochondral junction (rickettary rosary), and a transverse sulcus in the chest caused by the pull of the diaphragm (Harrison’s sulcus). X- Rays show widening and cupping of the epiphysis of the long bones, most readily apparent in the wrist.

Paediatric fracture types
Type Injury pattern
Complete fracture Both sides of cortex are breached
Toddlers fracture Oblique tibial fracture in infants
Plastic deformity Stress on bone resulting in deformity without cortical disruption
Greenstick fracture Unilateral cortical breach only
Buckle fracture Incomplete cortical disruption resulting in periosteal haematoma only

Growth plate fractures
In paediatric practice fractures may also involve the growth plate and these injuries are classified according to the Salter- Harris system (given below):

Type Injury pattern
I Fracture through the physis only (x-ray often normal)
II Fracture through the physis and metaphysis
III Fracture through the physis and epiphyisis to include the joint
IV Fracture involving the physis, metaphysis and epiphysis
V Crush injury involving the physis (x-ray may resemble type I, and appear normal)

As a general rule it is safer to assume that growth plate tenderness is indicative of an underlying fracture even if the x-ray appears normal. Injuries of Types III, IV and V will usually require surgery. Type V injuries are often associated with disruption to growth.

Non accidental injury
Delayed presentation
Delay in attaining milestones
Lack of concordance between proposed and actual mechanism of injury
Multiple injuries
Injuries at sites not commonly exposed to trauma
Children on the at risk register

Pathological fractures
Genetic conditions, such as osteogenesis imperfecta, may cause pathological fractures.

Osteogenesis imperfecta
Defective osteoid formation due to congenital inability to produce adequate intercellular substances like osteoid, collagen and dentine.
Failure of maturation of collagen in all the connective tissues.
Radiology may show translucent bones, multiple fractures, particularly of the long bones, wormian bones (irregular patches of ossification) and a trefoil pelvis.

Subtypes
Type I The collagen is normal quality but insufficient quantity.
Type II- Poor collagen quantity and quality.
Type III- Collagen poorly formed. Normal quantity.
Type IV- Sufficient collagen quantity but poor quality.

Osteopetrosis
Bones become harder and more dense.
Autosomal recessive condition.
It is commonest in young adults.
Radiology reveals a lack of differentiation between the cortex and the medulla described as marble bone.
How well did you know this?
1
Not at all
2
3
4
5
Perfectly
24
Q

A 60 year old male is admitted to A&E with a fall. He lives with his wife and still works as a restaurant manager. He has a past history of benign prostatic hypertrophy and is currently taking tamsulosin. He is otherwise fit and healthy. On examination there is right hip tenderness on movement in all directions. A hip x-ray confirms an intertrochanteric fracture.

 A.	Conservative management
B.	Percutaneous pinning
C.	Fracture reduction and internal fixation
D.	Hemiarthroplasty
E.	Total hip replacement
F.	Dynamic hip screw
G.	Intramedullary femoral nail
A

Dynamic hip screw

The blood supply to the femoral head may be intact and the fracture should heal with compression type devices such as gamma nails or dynamic hip screws. The latter device being the most commonly performed therapeutic intervention.

Neck of femur (NOF) fracture is a common orthopaedic presentation, with over 65000 fractures in the UK per year. Like many orthopaedic injuries, there is a bimodal age distribution. It is imperative to distinguish between the high energy injury in a young patient, and the low energy osteoporotic fracture in the elderly, as their management aims are very different:

Young patient - Usually high energy trauma (e.g road traffic accident, horse riding) and needs treating in accordance with Advanced Trauma Life Support (ATLS) principles. Will often have associated injuries. Aim is to retain the patients own anatomy, and optimise their function.

Elderly patient - Predominantly female, fall from standing height (fragility fracture). Often patients have multiple comorbidities that will ultimately dictate their prognosis. Aim of orthopaedic treatment is to immediately regain patient mobility so that morbidity (infection, thromboembolic events, pressure sores etc) and mortality associated with prolonged bed rest is avoided. Left untreated, a neck of femur fracture can be considered a terminal event. Historically, mortality associated with elderly hip fracture is 10% at one month, and 30% at one year. However, this has been improved in the UK with the introduction of multidisciplinary, orthogeriatric lead care and the National Hip Fracture Database and Best Practice Tariff.

Pertinent anatomy
Osteology - normal neck-shaft angle is 130 +/- 7 degrees, and 10 +/- 7 degrees of neck anteversion.
Vascular supply - The predominant blood supply to the femoral head and neck is from the medial and lateral femoral circumflex arteries (branches of profunda femoris). These anastomose and pierce the joint capsule at the base of the neck, mainly posteriorly. There is a small vascular contribution from the artery of the ligament teres. Understanding the blood supply is fundamental to the decision making process in treating NOF fractures.

Presentation and initial management
Typically, patients present with pain in the hip/groin, a shortened, abducted, externally rotated leg (due to the unopposed pull of the muscles that act across the hip joint) and the inability to straight-leg-raise. With undisplaced fractures, signs are more subtle.
High energy injuries should be treated in line with ATLS principles. All patients should be fluid resuscitated, have adequate pain relief (often with a fascio-iliiaca nerve block), and be optimised for surgery. In addition, elderly patients should be assessed by an orthogeriatrician.

Imaging
Anteroposterior and cross-table lateral plain radiographs are sufficient to diagnose the majority of NOF fractures. If the fracture extends below the level of the lesser trochanter, or there is any possibility of pathological fracture, full length femur views are essential to plan surgery.

Where there is a high index of suspicion of fracture, but plain radiographs are inconclusive, gold standard investigation is MRI. However, if unavailable within 24 hours, or if the patient will not tolerate MRI, CT is appropriate. The majority of fractures can be seen with modern CT techniques, and so this is becoming first line in many hospitals.

Classification
There has been a move away from named classification systems towards descriptive classification systems.
Two main types of NOF exist: Intra-capsular, and extra-capsular. Extra-capsular fractures are further divided into pertrochanteric or subtrochanteric (within 5cm distal to the lesser trochanter). All fractures are then described as undisplaced, minimally displaced, or displaced.
Femoral neck and head blood supply disruption is common with intracapsular NOF fractures, and rare with extracapsular fractures. This fundamental principle underpins the practise of arthroplasty for intracapsular fractures, and fixation for extracapsular fractures.

If you wish to use a named classification system, the most commonly used are below:
Elderly intracapsular - Garden Classification
Young intrasapsular - Pauvels Classification
Intertrochanteric - Evans
Subtrochanteric - Russell Taylor

Treatment
In general, NOF fractures are treated operatively except if the patient is deemed unlikely to survive an anaesthetic. Best Practice Tarif (BPT) dictates that surgery should happen within 36 hours, as delay of greater than 48 hours is associated with increased morbidity and mortality.

  • The priority with the young patient is to retain the femoral head if possible, even with a displaced intracapsular fracture. The risk of avascular necrosis and non-union (and therefore revision surgery) associated with internal fixation needs weighing up against the sequelae of total hip replacement in the young (wear, dislocation, revision). Discussion is necessary with the patient, on a case by case basis.

** Undisplaced fractures in the elderly can be treated with internal fixation, often with cannulated screws. This is appropriate for valgus impacted subcapital fractures which are inherently stable, to prevent secondary displacement. This does still carry the risk of AVN or non-union, and therefore a future revision. For this reason, many surgeons advocate arthroplasty as a single surgery.

*** NICE guidance - patients who fulfil these criteria should be offered total hip replacement which conveys better function and prosthetic survivorship, compared with hemiarthroplasty, but at an increased risk of dislocation.

  • Intertrochanteric fractures vary greatly in their stability. If the trochanter (and therefore lateral wall), and medial calcar is in tact, then the fracture configuration bears stability. This can be treated with a DHS, as collapse of the fracture is predictable. Where either or both structures are involved in the fracture, stability becomes compromised and many surgeons will favour using an intramedullary device. This is an ongoing debate, and difficult to test in an exam setting.

Post operative management
Patients should be mobilised fully weight bearing where possible. Care is multidisciplinary in its delivery. Elderly patients should have orthogeriatrician assessment of comorbidity, and bone health with secondary prevention measures if appropriate. There should be early involvement of physiotherapy and occupational therapy services.

How well did you know this?
1
Not at all
2
3
4
5
Perfectly
25
Q

An 86 year old retired pharmacist is admitted to A&E following a fall. She complains of right hip pain. She is known to have hypertension and is currently on bendrofluazide. She lives alone and mobilises with a Zimmer frame. Her right leg is shortened and externally rotated. A hip x-ray confirms a displaced intracapsular fracture.

A.	Conservative management
B.	Percutaneous pinning
C.	Fracture reduction and internal fixation
D.	Hemiarthroplasty
E.	Total hip replacement
F.	Dynamic hip screw
G.	Intramedullary femoral nail
A

Hemiarthroplasty

Hemiarthroplasty is offered to older, less mobile individuals compared to fracture reduction and fixation in younger patients.

Neck of femur (NOF) fracture is a common orthopaedic presentation, with over 65000 fractures in the UK per year. Like many orthopaedic injuries, there is a bimodal age distribution. It is imperative to distinguish between the high energy injury in a young patient, and the low energy osteoporotic fracture in the elderly, as their management aims are very different:

Young patient - Usually high energy trauma (e.g road traffic accident, horse riding) and needs treating in accordance with Advanced Trauma Life Support (ATLS) principles. Will often have associated injuries. Aim is to retain the patients own anatomy, and optimise their function.

Elderly patient - Predominantly female, fall from standing height (fragility fracture). Often patients have multiple comorbidities that will ultimately dictate their prognosis. Aim of orthopaedic treatment is to immediately regain patient mobility so that morbidity (infection, thromboembolic events, pressure sores etc) and mortality associated with prolonged bed rest is avoided. Left untreated, a neck of femur fracture can be considered a terminal event. Historically, mortality associated with elderly hip fracture is 10% at one month, and 30% at one year. However, this has been improved in the UK with the introduction of multidisciplinary, orthogeriatric lead care and the National Hip Fracture Database and Best Practice Tariff.

Pertinent anatomy
Osteology - normal neck-shaft angle is 130 +/- 7 degrees, and 10 +/- 7 degrees of neck anteversion.
Vascular supply - The predominant blood supply to the femoral head and neck is from the medial and lateral femoral circumflex arteries (branches of profunda femoris). These anastomose and pierce the joint capsule at the base of the neck, mainly posteriorly. There is a small vascular contribution from the artery of the ligament teres. Understanding the blood supply is fundamental to the decision making process in treating NOF fractures.

Presentation and initial management
Typically, patients present with pain in the hip/groin, a shortened, abducted, externally rotated leg (due to the unopposed pull of the muscles that act across the hip joint) and the inability to straight-leg-raise. With undisplaced fractures, signs are more subtle.
High energy injuries should be treated in line with ATLS principles. All patients should be fluid resuscitated, have adequate pain relief (often with a fascio-iliiaca nerve block), and be optimised for surgery. In addition, elderly patients should be assessed by an orthogeriatrician.

Imaging
Anteroposterior and cross-table lateral plain radiographs are sufficient to diagnose the majority of NOF fractures. If the fracture extends below the level of the lesser trochanter, or there is any possibility of pathological fracture, full length femur views are essential to plan surgery.

Where there is a high index of suspicion of fracture, but plain radiographs are inconclusive, gold standard investigation is MRI. However, if unavailable within 24 hours, or if the patient will not tolerate MRI, CT is appropriate. The majority of fractures can be seen with modern CT techniques, and so this is becoming first line in many hospitals.

Classification
There has been a move away from named classification systems towards descriptive classification systems.
Two main types of NOF exist: Intra-capsular, and extra-capsular. Extra-capsular fractures are further divided into pertrochanteric or subtrochanteric (within 5cm distal to the lesser trochanter). All fractures are then described as undisplaced, minimally displaced, or displaced.
Femoral neck and head blood supply disruption is common with intracapsular NOF fractures, and rare with extracapsular fractures. This fundamental principle underpins the practise of arthroplasty for intracapsular fractures, and fixation for extracapsular fractures.

If you wish to use a named classification system, the most commonly used are below:
Elderly intracapsular - Garden Classification
Young intrasapsular - Pauvels Classification
Intertrochanteric - Evans
Subtrochanteric - Russell Taylor

Treatment
In general, NOF fractures are treated operatively except if the patient is deemed unlikely to survive an anaesthetic. Best Practice Tarif (BPT) dictates that surgery should happen within 36 hours, as delay of greater than 48 hours is associated with increased morbidity and mortality.

  • The priority with the young patient is to retain the femoral head if possible, even with a displaced intracapsular fracture. The risk of avascular necrosis and non-union (and therefore revision surgery) associated with internal fixation needs weighing up against the sequelae of total hip replacement in the young (wear, dislocation, revision). Discussion is necessary with the patient, on a case by case basis.

** Undisplaced fractures in the elderly can be treated with internal fixation, often with cannulated screws. This is appropriate for valgus impacted subcapital fractures which are inherently stable, to prevent secondary displacement. This does still carry the risk of AVN or non-union, and therefore a future revision. For this reason, many surgeons advocate arthroplasty as a single surgery.

*** NICE guidance - patients who fulfil these criteria should be offered total hip replacement which conveys better function and prosthetic survivorship, compared with hemiarthroplasty, but at an increased risk of dislocation.

  • Intertrochanteric fractures vary greatly in their stability. If the trochanter (and therefore lateral wall), and medial calcar is in tact, then the fracture configuration bears stability. This can be treated with a DHS, as collapse of the fracture is predictable. Where either or both structures are involved in the fracture, stability becomes compromised and many surgeons will favour using an intramedullary device. This is an ongoing debate, and difficult to test in an exam setting.

Post operative management
Patients should be mobilised fully weight bearing where possible. Care is multidisciplinary in its delivery. Elderly patients should have orthogeriatrician assessment of comorbidity, and bone health with secondary prevention measures if appropriate. There should be early involvement of physiotherapy and occupational therapy services.

How well did you know this?
1
Not at all
2
3
4
5
Perfectly
26
Q

A 74 year old male is admitted to A&E with a fall. He is known to have rheumatoid arthritis and is on methotrexate and paracetamol. He lives alone in a bungalow and enjoys playing golf. He is independent with his ADLs. He complains of left groin pain, therefore has a hip x-ray which confirms a displaced intracapsular fracture.

A.	Conservative management
B.	Percutaneous pinning
C.	Fracture reduction and internal fixation
D.	Hemiarthroplasty
E.	Total hip replacement
F.	Dynamic hip screw
G.	Intramedullary femoral nail
A

Total hip replacement

This patient has pre-existing joint disease, good level of activity and a relatively high life expectancy, therefore THR is preferable to hemiarthroplasty.

Neck of femur (NOF) fracture is a common orthopaedic presentation, with over 65000 fractures in the UK per year. Like many orthopaedic injuries, there is a bimodal age distribution. It is imperative to distinguish between the high energy injury in a young patient, and the low energy osteoporotic fracture in the elderly, as their management aims are very different:

Young patient - Usually high energy trauma (e.g road traffic accident, horse riding) and needs treating in accordance with Advanced Trauma Life Support (ATLS) principles. Will often have associated injuries. Aim is to retain the patients own anatomy, and optimise their function.

Elderly patient - Predominantly female, fall from standing height (fragility fracture). Often patients have multiple comorbidities that will ultimately dictate their prognosis. Aim of orthopaedic treatment is to immediately regain patient mobility so that morbidity (infection, thromboembolic events, pressure sores etc) and mortality associated with prolonged bed rest is avoided. Left untreated, a neck of femur fracture can be considered a terminal event. Historically, mortality associated with elderly hip fracture is 10% at one month, and 30% at one year. However, this has been improved in the UK with the introduction of multidisciplinary, orthogeriatric lead care and the National Hip Fracture Database and Best Practice Tariff.

Pertinent anatomy
Osteology - normal neck-shaft angle is 130 +/- 7 degrees, and 10 +/- 7 degrees of neck anteversion.
Vascular supply - The predominant blood supply to the femoral head and neck is from the medial and lateral femoral circumflex arteries (branches of profunda femoris). These anastomose and pierce the joint capsule at the base of the neck, mainly posteriorly. There is a small vascular contribution from the artery of the ligament teres. Understanding the blood supply is fundamental to the decision making process in treating NOF fractures.

Presentation and initial management
Typically, patients present with pain in the hip/groin, a shortened, abducted, externally rotated leg (due to the unopposed pull of the muscles that act across the hip joint) and the inability to straight-leg-raise. With undisplaced fractures, signs are more subtle.
High energy injuries should be treated in line with ATLS principles. All patients should be fluid resuscitated, have adequate pain relief (often with a fascio-iliiaca nerve block), and be optimised for surgery. In addition, elderly patients should be assessed by an orthogeriatrician.

Imaging
Anteroposterior and cross-table lateral plain radiographs are sufficient to diagnose the majority of NOF fractures. If the fracture extends below the level of the lesser trochanter, or there is any possibility of pathological fracture, full length femur views are essential to plan surgery.

Where there is a high index of suspicion of fracture, but plain radiographs are inconclusive, gold standard investigation is MRI. However, if unavailable within 24 hours, or if the patient will not tolerate MRI, CT is appropriate. The majority of fractures can be seen with modern CT techniques, and so this is becoming first line in many hospitals.

Classification
There has been a move away from named classification systems towards descriptive classification systems.
Two main types of NOF exist: Intra-capsular, and extra-capsular. Extra-capsular fractures are further divided into pertrochanteric or subtrochanteric (within 5cm distal to the lesser trochanter). All fractures are then described as undisplaced, minimally displaced, or displaced.
Femoral neck and head blood supply disruption is common with intracapsular NOF fractures, and rare with extracapsular fractures. This fundamental principle underpins the practise of arthroplasty for intracapsular fractures, and fixation for extracapsular fractures.

If you wish to use a named classification system, the most commonly used are below:
Elderly intracapsular - Garden Classification
Young intrasapsular - Pauvels Classification
Intertrochanteric - Evans
Subtrochanteric - Russell Taylor

Treatment
In general, NOF fractures are treated operatively except if the patient is deemed unlikely to survive an anaesthetic. Best Practice Tarif (BPT) dictates that surgery should happen within 36 hours, as delay of greater than 48 hours is associated with increased morbidity and mortality.

  • The priority with the young patient is to retain the femoral head if possible, even with a displaced intracapsular fracture. The risk of avascular necrosis and non-union (and therefore revision surgery) associated with internal fixation needs weighing up against the sequelae of total hip replacement in the young (wear, dislocation, revision). Discussion is necessary with the patient, on a case by case basis.

** Undisplaced fractures in the elderly can be treated with internal fixation, often with cannulated screws. This is appropriate for valgus impacted subcapital fractures which are inherently stable, to prevent secondary displacement. This does still carry the risk of AVN or non-union, and therefore a future revision. For this reason, many surgeons advocate arthroplasty as a single surgery.

*** NICE guidance - patients who fulfil these criteria should be offered total hip replacement which conveys better function and prosthetic survivorship, compared with hemiarthroplasty, but at an increased risk of dislocation.

  • Intertrochanteric fractures vary greatly in their stability. If the trochanter (and therefore lateral wall), and medial calcar is in tact, then the fracture configuration bears stability. This can be treated with a DHS, as collapse of the fracture is predictable. Where either or both structures are involved in the fracture, stability becomes compromised and many surgeons will favour using an intramedullary device. This is an ongoing debate, and difficult to test in an exam setting.

Post operative management
Patients should be mobilised fully weight bearing where possible. Care is multidisciplinary in its delivery. Elderly patients should have orthogeriatrician assessment of comorbidity, and bone health with secondary prevention measures if appropriate. There should be early involvement of physiotherapy and occupational therapy services.

How well did you know this?
1
Not at all
2
3
4
5
Perfectly
27
Q

Of the list below, which is not a cause of avascular necrosis?

	Steroids
	Sickle cell disease
	Radiotherapy
	Myeloma
	Caisson disease
A

Myeloma
Steroid containing therapy for myeloma may induce avascular necrosis, however the disease itself does not cause it. Caisson disease as may occur in deep sea divers is a recognised cause.

Avascular necrosis
Cellular death of bone components due to interruption of the blood supply, causing bone destruction
Main joints affected are hip, scaphoid, lunate and the talus.
It is not the same as non union. The fracture has usually united.
Radiological evidence is slow to appear.
Vascular ingrowth into the affected bone may occur. However, many joints will develop secondary osteoarthritis.

Causes
P ancreatitis 
L upus 
A lcohol 
S teroids 
T rauma 
I diopathic, infection 
C aisson disease, collagen vascular disease 
R adiation, rheumatoid arthritis 
A myloid 
G aucher disease 
S ickle cell disease

Presentation
Usually pain. Often despite apparent fracture union.

Investigation
MRI scanning will show changes earlier than plain films.

Treatment
In fractures at high risk sites anticipation is key. Early prompt and accurate reduction is essential.

Non weight bearing may help to facilitate vascular regeneration.

Joint replacement may be necessary, or even the preferred option (e.g. Hip in the elderly).

How well did you know this?
1
Not at all
2
3
4
5
Perfectly
28
Q

Which of the following is the first radiological change likely to be apparent in a plain radiograph of a 12 year old presenting with suspected Perthes disease

	Multiple bone cysts
	Sclerosis of the femoral head
	Loss of bone density
	Joint space narrowing
	Collapse of the femoral head
A

Sclerosis of the femoral head
In Catterall stage I disease there may be no radiological abnormality at all. In Stage II disease there may be sclerosis of the femoral head.

Indication for treatment (aide memoire):Half a dozen, half a head
Those aged greater than 6 years with >50% involvement of the femoral head should almost always be treated.

Perthes disease
Idiopathic avascular necrosis of the femoral epiphysis of the femoral head
Impaired blood supply to femoral head, causing bone infarction. New vessels develop and ossification occurs. The bone either heals or a subchondral fracture occurs.

Clinical features
Males 4x's greater than females
Age between 2-12 years (the younger the age of onset, the better the prognosis)
Limp
Hip pain
Bilateral in 20%

Diagnosis
Plain x-ray, Technetium bone scan or magnetic resonance imaging if normal x-ray and symptoms persist.

Catterall staging
Stage Features
Stage 1 Clinical and histological features only
Stage 2 Sclerosis with or without cystic changes and preservation of the articular surface
Stage 3 Loss of structural integrity of the femoral head
Stage 4 Loss of acetabular integrity

Management
To keep the femoral head within the acetabulum: cast, braces
If less than 6 years: observation
Older: surgical management with moderate results
Operate on severe deformities

Prognosis
Most cases will resolve with conservative management. Early diagnosis improves outcomes.

How well did you know this?
1
Not at all
2
3
4
5
Perfectly
29
Q

A footballer is injured in a match and is being assessed in the outpatient department. On examination he has a positive valgus stress test and minimal joint effusion. What is the most likely underlying injury?

Injury to the lateral collateral ligament
Injury to the medial collateral ligament
Injury to the anterior cruciate ligament
Injury to the posterior cruciate ligament
Injury to the patellar tendon
A

Injury to the medial collateral ligament

A knee injury in the footballer with a positive valgus stress test is usually associated with MCL injury.

Knee collateral ligament

Anatomy
The tibial collateral ligament is a broad, flat band. Its upper end has an extensive attachment to the medial epicondyle of the femur with some fibres projecting onto the adductor magnus tendon. The ligament passes downwards and forwards to the medial side of the tibia. The deepest fibres are fused with the medial meniscus.
The fibular collateral ligament is round and cord like and stands clear of the thin, lateral part of the fibrous capsule. It is enclosed within the fascia lata. It passes from the lateral epicondyle of the femur to the head of the fibula in front of its highest point and splits the tendon of biceps femoris. On the lateral side of the joint the fibres are short and weak and bridge the interval between the femoral and tibial condyles. The popliteus tendon intervenes between the lateral meniscus and the capsule.
The tibial and fibular collateral ligaments prevent disruption of the joint at the sides. They are most tightly stretched in extension, and then their direction- the fibular ligament downwards and backwards, the tibial downwards and forwards- prevents rotation of the tibia laterally or the femur medially. Rotation may be demonstrated in the flexed knee.

Injury
The collateral ligaments are commonly injured, the medial is most often affected. It requires a significant force such as sporting tackle or motor vehicle to strike the side of the leg. Associated injuries to both the tibial plateau or menisci are not uncommon.

Grading and treatment
Grade of injury Features Treatment
1 Minor tearing of ligament fibres
Negative instability tests Conservative (analgesia and physiotherapy)
2 Ligament laxity (seen with knee in 30o flexion)
Knee stable when joint extended Usually splinting or casting for 4-6 weeks
3 Ligament completely torn
Joint instability Surgical ligament reconstruction

How well did you know this?
1
Not at all
2
3
4
5
Perfectly
30
Q

A 30 year old man is admitted overnight, following a road traffic accident. He has an open tibial fracture with a 20 cm wound and extensive periosteal stripping. He is neurovascularly intact and IV antibiotics and wound dressing have been administered in the emergency department.

A. Immediate skeletal stabilisation and application of negative pressure dressing
B. Combined skeletal and soft tissue reconstruction on a scheduled operating list
C. Thorough wound debridement in the emergency department
D. Immediate vascuIar shunting, followed by temporary skeletal stabilisation and vascular reconstruction
E. Intravenous antibiotics, photography and application of saline soaked gauze with impermeable dressing
F. Application of external fixator and conversion to internal fixation after two weeks
G. Fasciotomy with four compartment decompression
H. Skeletal fixation followed by vascular reconstruction

A

Combined skeletal and soft tissue reconstruction on a scheduled operating list

This patient has a Gustillo-Anderson Grade 3B open fracture. He will require definitive skeletal and soft tissue reconstruction, which should be performed on a combined ortho-plastic scheduled operating list, as per the BOA/BAPRAS guidelines. The surgery does not have to be performed out of scheduled hours unless there is marine/ sewage contamination, vascular compromise or it is a polytrauma.
Whilst it is reasonable to apply an external fixator prior to definitive skeletal and soft tissue reconstruction, this should be converted to internal fixation within 72 hours.

Open fractures

The term open fracture refers to a disruption of the bony cortex associated with a breach in the overlying skin. Any wound that is present in the same limb as a fracture should be suspected as being representative of an open fracture. One of the main problems with open fractures is the associated injuries to the surrounding soft tissues. Whilst the skin is usually relatively resistant to trauma, underlying muscle can be damaged or devitalised, nerves, blood vessels and periosteum may all be disrupted the degree to which this occurs correlates with the severity of the injury and the outcome. These can be graded using the Gustilo and Anderson system (see below).

Grade Injury
1 Low energy wound <1cm
2 Greater than 1cm wound with moderate soft tissue damage
3 High energy wound > 1cm with extensive soft tissue damage
3 A (sub group of 3) Adequate soft tissue coverage
3 B (sub group of 3) Inadequate soft tissue coverage
3 C (sub group of 3) Associated arterial injury

In Type IIIc injuries, the mangled extremity scoring system (MESS) can help to predict the need for primary amputation.

Initial management should focus on careful patient examination to check for associated injuries, control of haemorrhage and the extent of injury. The area should be carefully imaged, distal neurovascular status established the wound covered with a dressing and antibiotics administered. Early debridement is the cornerstone of the management of open fractures. The aim of the debridement is to remove foreign material and devitalised tissue. In most cases the wound is left open. The wound should be irrigated, generally, 6 litres of saline is used. The fracture should be stabilised and an external fixator is often used in the first instance

How well did you know this?
1
Not at all
2
3
4
5
Perfectly
31
Q

A 50 year old man is admitted after falling from scaffolding. He has an open fracture of his tibia with a 15 cm wound. He is neurovascularly intact.

A. Immediate skeletal stabilisation and application of negative pressure dressing
B. Combined skeletal and soft tissue reconstruction on a scheduled operating list
C. Thorough wound debridement in the emergency department
D. Immediate vascuIar shunting, followed by temporary skeletal stabilisation and vascular reconstruction
E. Intravenous antibiotics, photography and application of saline soaked gauze with impermeable dressing
F. Application of external fixator and conversion to internal fixation after two weeks
G. Fasciotomy with four compartment decompression
H. Skeletal fixation followed by vascular reconstruction

A

Intravenous antibiotics, photography and application of saline soaked gauze with impermeable dressing

The initial management of open fractures should include administration of intravenous antibiotics, photography of wound and application of a sterile soaked gauze and impermeable film. The wound should only be handled to remove gross contamination. The patient is then likely to require definitive skeletal and soft tissue reconstruction.

Open fractures

The term open fracture refers to a disruption of the bony cortex associated with a breach in the overlying skin. Any wound that is present in the same limb as a fracture should be suspected as being representative of an open fracture. One of the main problems with open fractures is the associated injuries to the surrounding soft tissues. Whilst the skin is usually relatively resistant to trauma, underlying muscle can be damaged or devitalised, nerves, blood vessels and periosteum may all be disrupted the degree to which this occurs correlates with the severity of the injury and the outcome. These can be graded using the Gustilo and Anderson system (see below).

Grade Injury
1 Low energy wound <1cm
2 Greater than 1cm wound with moderate soft tissue damage
3 High energy wound > 1cm with extensive soft tissue damage
3 A (sub group of 3) Adequate soft tissue coverage
3 B (sub group of 3) Inadequate soft tissue coverage
3 C (sub group of 3) Associated arterial injury

In Type IIIc injuries, the mangled extremity scoring system (MESS) can help to predict the need for primary amputation.

Initial management should focus on careful patient examination to check for associated injuries, control of haemorrhage and the extent of injury. The area should be carefully imaged, distal neurovascular status established the wound covered with a dressing and antibiotics administered. Early debridement is the cornerstone of the management of open fractures. The aim of the debridement is to remove foreign material and devitalised tissue. In most cases the wound is left open. The wound should be irrigated, generally, 6 litres of saline is used. The fracture should be stabilised and an external fixator is often used in the first instance

How well did you know this?
1
Not at all
2
3
4
5
Perfectly
32
Q

A 40 year old woman is admitted after being knocked off her bike. She has an open fracture of her tibia, with a 10 cm wound. No peripheral pulses are palpable. Intravenous antibiotics have been administered in the emergency department and the wound has been dressed.

A. Immediate skeletal stabilisation and application of negative pressure dressing
B. Combined skeletal and soft tissue reconstruction on a scheduled operating list
C. Thorough wound debridement in the emergency department
D. Immediate vascuIar shunting, followed by temporary skeletal stabilisation and vascular reconstruction
E. Intravenous antibiotics, photography and application of saline soaked gauze with impermeable dressing
F. Application of external fixator and conversion to internal fixation after two weeks
G. Fasciotomy with four compartment decompression
H. Skeletal fixation followed by vascular reconstruction

A

Immediate vascuIar shunting, followed by temporary skeletal stabilisation and vascular reconstruction

This patient has a Gustillo-Anderson Grade 3C open fracture with vascular injury. Vascular impairment requires immediate surgery and restoration of circulation, ideally within 3-4 hours. This should follow the sequence of shunting, temporary skeletal stabilisation and then vascular reconstruction as per BOA / BAPRAS guidelines. Revascularisation using vascular shunts should be performed before skeletal fixation.

Open fractures

The term open fracture refers to a disruption of the bony cortex associated with a breach in the overlying skin. Any wound that is present in the same limb as a fracture should be suspected as being representative of an open fracture. One of the main problems with open fractures is the associated injuries to the surrounding soft tissues. Whilst the skin is usually relatively resistant to trauma, underlying muscle can be damaged or devitalised, nerves, blood vessels and periosteum may all be disrupted the degree to which this occurs correlates with the severity of the injury and the outcome. These can be graded using the Gustilo and Anderson system (see below).

Grade Injury
1 Low energy wound <1cm
2 Greater than 1cm wound with moderate soft tissue damage
3 High energy wound > 1cm with extensive soft tissue damage
3 A (sub group of 3) Adequate soft tissue coverage
3 B (sub group of 3) Inadequate soft tissue coverage
3 C (sub group of 3) Associated arterial injury

In Type IIIc injuries, the mangled extremity scoring system (MESS) can help to predict the need for primary amputation.

Initial management should focus on careful patient examination to check for associated injuries, control of haemorrhage and the extent of injury. The area should be carefully imaged, distal neurovascular status established the wound covered with a dressing and antibiotics administered. Early debridement is the cornerstone of the management of open fractures. The aim of the debridement is to remove foreign material and devitalised tissue. In most cases the wound is left open. The wound should be irrigated, generally, 6 litres of saline is used. The fracture should be stabilised and an external fixator is often used in the first instance

How well did you know this?
1
Not at all
2
3
4
5
Perfectly
33
Q

A 32 year old man presents with a painful swelling over the volar aspect of his hand after receiving a hard blow to his palm. On examination, he experiences pain on moving the wrist and on longitudinal compression of the thumb.

 A.	Pulled elbow
B.	Fracture of the coronoid process
C.	Scaphoid fracture
D.	Fracture of the distal humerus
E.	Bennets fracture
F.	Fracture of the shaft of the radius and ulnar
G.	Galeazzi fracture
H.	Fracture of the olecranon
I.	Fracture of the radial head
A

Scaphoid fracture

Scaphoid fractures usually occur as a result of direct hard blow to the palm or following a fall on the out-stretched hand. The main physical signs are swelling and tenderness in the anatomical snuff box, and pain on wrist movements and on longitudinal compression of the thumb

Colles’ fracture
Fall onto extended outstretched hands
Described as a dinner fork type deformity
Classical Colles’ fractures have the following 3 features:

Features of the injury

  1. Transverse fracture of the radius
  2. 1 inch proximal to the radio-carpal joint
  3. Dorsal displacement and angulation

Smith’s fracture (reverse Colles’ fracture)
Volar angulation of distal radius fragment (Garden spade deformity)
Caused by falling backwards onto the palm of an outstretched hand or falling with wrists flexed

Bennett’s fracture
Intra-articular fracture of the first carpometacarpal joint
Impact on flexed metacarpal, caused by fist fights
X-ray: triangular fragment at ulnar base of metacarpal

Monteggia’s fracture
Dislocation of the proximal radioulnar joint in association with an ulna fracture
Fall on outstretched hand with forced pronation
Needs prompt diagnosis to avoid disability

Galeazzi fracture
Radial shaft fracture with associated dislocation of the distal radioulnar joint
Occur after a fall on the hand with a rotational force superimposed on it.
On examination, there is bruising, swelling and tenderness over the lower end of the forearm.
X Rays reveal the displaced fracture of the radius and a prominent ulnar head due to dislocation of the inferior radio-ulnar joint.

Barton’s fracture
Distal radius fracture (Colles’/Smith’s) with associated radiocarpal dislocation
Fall onto extended and pronated wrist

Scaphoid fractures
Scaphoid fractures are the commonest carpal fractures.
Surface of scaphoid is covered by articular cartilage with small area available for blood vessels (fracture risks blood supply)
Forms floor of anatomical snuffbox
Risk of fracture associated with fall onto outstretched hand (tubercle, waist, or proximal 1/3)
The main physical signs are swelling and tenderness in the anatomical snuff box, and pain on wrist movements and on longitudinal compression of the thumb.
Ulnar deviation AP needed for visualization of scaphoid
Immobilization of scaphoid fractures difficult

Radial head fracture
Fracture of the radial head is common in young adults.
It is usually caused by a fall on the outstretched hand.
On examination, there is marked local tenderness over the head of the radius, impaired movements at the elbow, and a sharp pain at the lateral side of the elbow at the extremes of rotation (pronation and supination).

How well did you know this?
1
Not at all
2
3
4
5
Perfectly
34
Q

A 26 year old man presents to the emergency department with a swelling over his left elbow after a fall on an outstretched hand. On examination, he has tenderness over the proximal part of his forearm, and has severely restricted supination and pronation movements.

A.	Pulled elbow
B.	Fracture of the coronoid process
C.	Scaphoid fracture
D.	Fracture of the distal humerus
E.	Bennets fracture
F.	Fracture of the shaft of the radius and ulnar
G.	Galeazzi fracture
H.	Fracture of the olecranon
I.	Fracture of the radial head
A

Fracture of the radial head

Fracture of the radial head is common in young adults. It is usually caused by a fall on the outstretched hand. On examination, there is marked local tenderness over the head of the radius, impaired movements at the elbow, and a sharp pain at the lateral side of the elbow at the extremes of rotation (pronation and supination).

Colles’ fracture
Fall onto extended outstretched hands
Described as a dinner fork type deformity
Classical Colles’ fractures have the following 3 features:

Features of the injury

  1. Transverse fracture of the radius
  2. 1 inch proximal to the radio-carpal joint
  3. Dorsal displacement and angulation

Smith’s fracture (reverse Colles’ fracture)
Volar angulation of distal radius fragment (Garden spade deformity)
Caused by falling backwards onto the palm of an outstretched hand or falling with wrists flexed

Bennett’s fracture
Intra-articular fracture of the first carpometacarpal joint
Impact on flexed metacarpal, caused by fist fights
X-ray: triangular fragment at ulnar base of metacarpal

Monteggia’s fracture
Dislocation of the proximal radioulnar joint in association with an ulna fracture
Fall on outstretched hand with forced pronation
Needs prompt diagnosis to avoid disability

Galeazzi fracture
Radial shaft fracture with associated dislocation of the distal radioulnar joint
Occur after a fall on the hand with a rotational force superimposed on it.
On examination, there is bruising, swelling and tenderness over the lower end of the forearm.
X Rays reveal the displaced fracture of the radius and a prominent ulnar head due to dislocation of the inferior radio-ulnar joint.

Barton’s fracture
Distal radius fracture (Colles’/Smith’s) with associated radiocarpal dislocation
Fall onto extended and pronated wrist

Scaphoid fractures
Scaphoid fractures are the commonest carpal fractures.
Surface of scaphoid is covered by articular cartilage with small area available for blood vessels (fracture risks blood supply)
Forms floor of anatomical snuffbox
Risk of fracture associated with fall onto outstretched hand (tubercle, waist, or proximal 1/3)
The main physical signs are swelling and tenderness in the anatomical snuff box, and pain on wrist movements and on longitudinal compression of the thumb.
Ulnar deviation AP needed for visualization of scaphoid
Immobilization of scaphoid fractures difficult

Radial head fracture
Fracture of the radial head is common in young adults.
It is usually caused by a fall on the outstretched hand.
On examination, there is marked local tenderness over the head of the radius, impaired movements at the elbow, and a sharp pain at the lateral side of the elbow at the extremes of rotation (pronation and supination).

How well did you know this?
1
Not at all
2
3
4
5
Perfectly
35
Q

A 56 year old lady presents with a painful swelling over the lower end of the forearm following a fall. Imaging reveals a distal radial fracture with disruption of the distal radio-ulnar joint.

 A.	Pulled elbow
B.	Fracture of the coronoid process
C.	Scaphoid fracture
D.	Fracture of the distal humerus
E.	Bennets fracture
F.	Fracture of the shaft of the radius and ulnar
G.	Galeazzi fracture
H.	Fracture of the olecranon
I.	Fracture of the radial head
A

Galeazzi fracture

Galeazzi fractures occur after a fall on the hand with a rotational force superimposed on it. On examination, there is bruising, swelling and tenderness over the lower end of the forearm. X- Rays reveal a displaced fracture of the radius and a prominent ulnar head due to dislocation of the inferior radio-ulnar joint.

Colles’ fracture
Fall onto extended outstretched hands
Described as a dinner fork type deformity
Classical Colles’ fractures have the following 3 features:

Features of the injury

  1. Transverse fracture of the radius
  2. 1 inch proximal to the radio-carpal joint
  3. Dorsal displacement and angulation

Smith’s fracture (reverse Colles’ fracture)
Volar angulation of distal radius fragment (Garden spade deformity)
Caused by falling backwards onto the palm of an outstretched hand or falling with wrists flexed

Bennett’s fracture
Intra-articular fracture of the first carpometacarpal joint
Impact on flexed metacarpal, caused by fist fights
X-ray: triangular fragment at ulnar base of metacarpal

Monteggia’s fracture
Dislocation of the proximal radioulnar joint in association with an ulna fracture
Fall on outstretched hand with forced pronation
Needs prompt diagnosis to avoid disability

Galeazzi fracture
Radial shaft fracture with associated dislocation of the distal radioulnar joint
Occur after a fall on the hand with a rotational force superimposed on it.
On examination, there is bruising, swelling and tenderness over the lower end of the forearm.
X Rays reveal the displaced fracture of the radius and a prominent ulnar head due to dislocation of the inferior radio-ulnar joint.

Barton’s fracture
Distal radius fracture (Colles’/Smith’s) with associated radiocarpal dislocation
Fall onto extended and pronated wrist

Scaphoid fractures
Scaphoid fractures are the commonest carpal fractures.
Surface of scaphoid is covered by articular cartilage with small area available for blood vessels (fracture risks blood supply)
Forms floor of anatomical snuffbox
Risk of fracture associated with fall onto outstretched hand (tubercle, waist, or proximal 1/3)
The main physical signs are swelling and tenderness in the anatomical snuff box, and pain on wrist movements and on longitudinal compression of the thumb.
Ulnar deviation AP needed for visualization of scaphoid
Immobilization of scaphoid fractures difficult

Radial head fracture
Fracture of the radial head is common in young adults.
It is usually caused by a fall on the outstretched hand.
On examination, there is marked local tenderness over the head of the radius, impaired movements at the elbow, and a sharp pain at the lateral side of the elbow at the extremes of rotation (pronation and supination).

How well did you know this?
1
Not at all
2
3
4
5
Perfectly
36
Q

A 42 year old skier falls and impacts his hand on his ski pole. On examination he is tender in the anatomical snuffbox and on bimanual palpation. X-rays with scaphoid views show no evidence of fracture.

A. Admission and surgical debridement
B. Application of futura splint and fracture clinic review
C. Application of tubigrip bandage and fracture clinic review
D. Admission for open reduction and fixation
E. Discharge with reassurance
F. Commence oral prednisolone
G. Commence oral diclofenac

A

Application of futura splint and fracture clinic review

A fracture may still be present and should be immobilised until repeat imaging can be performed. If clinical suspicion persists then subsequent imaging should be with MRI scanning or CT if MRI is contra-indicated.

Scaphoid fractures:
80% of all carpal fractures
80% occur in men
80% occur at the waist of the scaphoid

Scaphoid fractures are the commonest carpal fractures.
Surface of scaphoid is covered by articular cartilage with small area available for blood vessels (fracture risks blood supply)
Forms floor of anatomical snuffbox
Risk of fracture associated with fall onto outstretched hand (tubercle, waist, or proximal third)
Ulnar deviation AP needed for visualization of scaphoid
Immobilization of scaphoid fractures difficult

Management
Non-displaced fractures - Casts or splints
- Percutaneous scaphoid fixation
Displaced fracture Surgical fixation, usually with a screw

Complications
Non union of scaphoid
Avascular necrosis of the scaphoid
Scapholunate disruption and wrist collapse
Degenerative changes of the adjacent joint

How well did you know this?
1
Not at all
2
3
4
5
Perfectly
37
Q

A 43 year old man falls over landing on his left hand. Although there was anatomical snuffbox tenderness no x-rays either at the time or subsequently have shown evidence of scaphoid fracture. He has been immobilised in a futura splint for two weeks and is now asymptomatic.

A. Admission and surgical debridement
B. Application of futura splint and fracture clinic review
C. Application of tubigrip bandage and fracture clinic review
D. Admission for open reduction and fixation
E. Discharge with reassurance
F. Commence oral prednisolone
G. Commence oral diclofenac

A

Discharge with reassurance

This patient is at extremely low risk of having sustained a scaphoid injury and may be discharged.

Scaphoid fractures:
80% of all carpal fractures
80% occur in men
80% occur at the waist of the scaphoid

Scaphoid fractures are the commonest carpal fractures.
Surface of scaphoid is covered by articular cartilage with small area available for blood vessels (fracture risks blood supply)
Forms floor of anatomical snuffbox
Risk of fracture associated with fall onto outstretched hand (tubercle, waist, or proximal third)
Ulnar deviation AP needed for visualization of scaphoid
Immobilization of scaphoid fractures difficult

Management
Non-displaced fractures - Casts or splints
- Percutaneous scaphoid fixation
Displaced fracture Surgical fixation, usually with a screw

Complications
Non union of scaphoid
Avascular necrosis of the scaphoid
Scapholunate disruption and wrist collapse
Degenerative changes of the adjacent joint

How well did you know this?
1
Not at all
2
3
4
5
Perfectly
38
Q

A builder falls from scaffolding and lands on his left hand he suffers a severe laceration to his palm. An x-ray shows evidence of scaphoid fracture that is minimally displaced.

A. Admission and surgical debridement
B. Application of futura splint and fracture clinic review
C. Application of tubigrip bandage and fracture clinic review
D. Admission for open reduction and fixation
E. Discharge with reassurance
F. Commence oral prednisolone
G. Commence oral diclofenac

A

Admission and surgical debridement

This is technically an open fracture and should be debrided prior to attempted fixation (which should occur soon after).

Scaphoid fractures:
80% of all carpal fractures
80% occur in men
80% occur at the waist of the scaphoid

Scaphoid fractures are the commonest carpal fractures.
Surface of scaphoid is covered by articular cartilage with small area available for blood vessels (fracture risks blood supply)
Forms floor of anatomical snuffbox
Risk of fracture associated with fall onto outstretched hand (tubercle, waist, or proximal third)
Ulnar deviation AP needed for visualization of scaphoid
Immobilization of scaphoid fractures difficult

Management
Non-displaced fractures - Casts or splints
- Percutaneous scaphoid fixation
Displaced fracture Surgical fixation, usually with a screw

Complications
Non union of scaphoid
Avascular necrosis of the scaphoid
Scapholunate disruption and wrist collapse
Degenerative changes of the adjacent joint

How well did you know this?
1
Not at all
2
3
4
5
Perfectly
39
Q

A 4 year boy presents with an abnormal gait. He has a history of recent viral illness. His WCC is 11 and ESR is 30.

 A.	Musculoskeletal pain
B.	Congenital dysplasia of the hip
C.	Slipped upper femoral epiphysis
D.	Transient synovitis
E.	Septic arthritis
F.	Perthes disease
G.	Tibial fracture
A

Transient synovitis

Viral illnesses can be associated with transient synovitis. The WCC should ideally be > 12 and the ESR > 40 to suggest septic arthritis.

Developmental dysplasia of the hip Usually diagnosed in infancy by screening tests. May be bilateral, when disease is unilateral there may be leg length inequality. As disease progresses child may limp and then early onset arthritis. More common in extended breech babies. Splints and harnesses or traction. In later years osteotomy and hip realignment procedures may be needed. In arthritis a joint replacement may be needed. However, this is best deferred if possible as it will almost certainly require revision Initially no obvious change on plain films and USS gives best resolution until 3 months of age. On plain films Shentons line should form a smooth arc

Perthes Disease Hip pain (may be referred to the knee) usually occurring between 5 and 12 years of age. Bilateral disease in 20%. Remove pressure from joint to allow normal development. Physiotherapy. Usually self-limiting if diagnosed and treated promptly. X-rays will show flattened femoral head. Eventually in untreated cases the femoral head will fragment.

Slipped upper femoral epiphysis Typically seen in obese male adolescents. Pain is often referred to the knee. Limitation to internal rotation is usually seen. Knee pain is usually present 2 months prior to hip slipping. Bilateral in 20%. Bed rest and non-weight bearing. Aim to avoid avascular necrosis. If severe slippage or risk of it occurring then percutaneous pinning of the hip may be required. X-rays will show the femoral head displaced and falling inferolaterally (like a melting ice cream cone) The Southwick angle gives indication of disease severity

How well did you know this?
1
Not at all
2
3
4
5
Perfectly
40
Q

A 6 year old boy presents with groin pain. He is known to be disruptive in class. He reports that he is bullied for being short. On examination he has an antalgic gait and pain on internal rotation of the right hip.

A.	Musculoskeletal pain
B.	Congenital dysplasia of the hip
C.	Slipped upper femoral epiphysis
D.	Transient synovitis
E.	Septic arthritis
F.	Perthes disease
G.	Tibial fracture
A

Perthes disease

This child is short, has hyperactivity (disruptive behaviour) and is within the age range for Perthes disease. Hyperactivity and short stature are associated with Perthes disease.

Developmental dysplasia of the hip Usually diagnosed in infancy by screening tests. May be bilateral, when disease is unilateral there may be leg length inequality. As disease progresses child may limp and then early onset arthritis. More common in extended breech babies. Splints and harnesses or traction. In later years osteotomy and hip realignment procedures may be needed. In arthritis a joint replacement may be needed. However, this is best deferred if possible as it will almost certainly require revision Initially no obvious change on plain films and USS gives best resolution until 3 months of age. On plain films Shentons line should form a smooth arc

Perthes Disease Hip pain (may be referred to the knee) usually occurring between 5 and 12 years of age. Bilateral disease in 20%. Remove pressure from joint to allow normal development. Physiotherapy. Usually self-limiting if diagnosed and treated promptly. X-rays will show flattened femoral head. Eventually in untreated cases the femoral head will fragment.

Slipped upper femoral epiphysis Typically seen in obese male adolescents. Pain is often referred to the knee. Limitation to internal rotation is usually seen. Knee pain is usually present 2 months prior to hip slipping. Bilateral in 20%. Bed rest and non-weight bearing. Aim to avoid avascular necrosis. If severe slippage or risk of it occurring then percutaneous pinning of the hip may be required. X-rays will show the femoral head displaced and falling inferolaterally (like a melting ice cream cone) The Southwick angle gives indication of disease severity

How well did you know this?
1
Not at all
2
3
4
5
Perfectly
41
Q

An obese 12 year old boy is referred with pain in the left knee and hip. On examination he has an antaglic gait and limitation of internal rotation. His knee has normal range of passive and active movement.

 A.	Musculoskeletal pain
B.	Congenital dysplasia of the hip
C.	Slipped upper femoral epiphysis
D.	Transient synovitis
E.	Septic arthritis
F.	Perthes disease
G.	Tibial fracture
A

Slipped upper femoral epiphysis

Slipped upper femoral epiphysis is commonest in obese adolescent males. The x-ray will show displacement of the femoral epiphysis inferolaterally. Treatment is usually with rest and non weight bearing crutches.

Developmental dysplasia of the hip Usually diagnosed in infancy by screening tests. May be bilateral, when disease is unilateral there may be leg length inequality. As disease progresses child may limp and then early onset arthritis. More common in extended breech babies. Splints and harnesses or traction. In later years osteotomy and hip realignment procedures may be needed. In arthritis a joint replacement may be needed. However, this is best deferred if possible as it will almost certainly require revision Initially no obvious change on plain films and USS gives best resolution until 3 months of age. On plain films Shentons line should form a smooth arc

Perthes Disease Hip pain (may be referred to the knee) usually occurring between 5 and 12 years of age. Bilateral disease in 20%. Remove pressure from joint to allow normal development. Physiotherapy. Usually self-limiting if diagnosed and treated promptly. X-rays will show flattened femoral head. Eventually in untreated cases the femoral head will fragment.

Slipped upper femoral epiphysis Typically seen in obese male adolescents. Pain is often referred to the knee. Limitation to internal rotation is usually seen. Knee pain is usually present 2 months prior to hip slipping. Bilateral in 20%. Bed rest and non-weight bearing. Aim to avoid avascular necrosis. If severe slippage or risk of it occurring then percutaneous pinning of the hip may be required. X-rays will show the femoral head displaced and falling inferolaterally (like a melting ice cream cone) The Southwick angle gives indication of disease severity

How well did you know this?
1
Not at all
2
3
4
5
Perfectly
42
Q

A 28 year old man falls on the back of his hand. On x-ray he has a fractured distal radius demonstrating volar displacement of the fracture.

 A.	Smith's
B.	Bennett's
C.	Monteggia's
D.	Colles'
E.	Galeazzi
F.	Pott's
G.	Barton's
A

Smith’s

This is a Smith fracture (reverse Colles’ fracture); unlike a Colles’ this is a high velocity injury and may require surgical correction. Note that Colles’ fractures are usually dorsally displaced.

Eponymous fractures

Colles’ fracture (dinner fork deformity)
Fall onto extended outstretched hand
Classical Colles’ fractures have the following 3 features:

  1. Transverse fracture of the radius
  2. 1 inch proximal to the radio-carpal joint
  3. Dorsal displacement and angulation

Smith’s fracture (reverse Colles’ fracture)
Volar angulation of distal radius fragment (Garden spade deformity)
Caused by falling backwards onto the palm of an outstretched hand or falling with wrists flexed

Bennett’s fracture
Intra-articular fracture of the first carpometacarpal joint
Impact on flexed metacarpal, caused by fist fights
X-ray: triangular fragment at ulnar base of metacarpal

Image sourced from Wikipedia
Monteggia’s fracture
Dislocation of the proximal radioulnar joint in association with an ulna fracture
Fall on outstretched hand with forced pronation
Needs prompt diagnosis to avoid disability

Image sourced from Wikipedia
Galeazzi fracture
Radial shaft fracture with associated dislocation of the distal radioulnar joint
Direct blow

Pott’s fracture
Bimalleolar ankle fracture
Forced foot eversion

Barton’s fracture
Distal radius fracture (Colles’/Smith’s) with associated radiocarpal dislocation
Fall onto extended and pronated wrist
Involvement of the joint is a defining feature

How well did you know this?
1
Not at all
2
3
4
5
Perfectly
43
Q

A 38 year old window cleaner falls from his ladder. He lands on his left arm and notices an obvious injury. An x-ray and clinical examination demonstrate that he has a fracture of the proximal ulna and associated radial dislocation.

A.	Smith's
B.	Bennett's
C.	Monteggia's
D.	Colles'
E.	Galeazzi
F.	Pott's
G.	Barton's
A

Monteggia’s

This constellation of injuries is referred to as a Monteggia’s fracture.

Eponymous fractures

Colles’ fracture (dinner fork deformity)
Fall onto extended outstretched hand
Classical Colles’ fractures have the following 3 features:

  1. Transverse fracture of the radius
  2. 1 inch proximal to the radio-carpal joint
  3. Dorsal displacement and angulation

Smith’s fracture (reverse Colles’ fracture)
Volar angulation of distal radius fragment (Garden spade deformity)
Caused by falling backwards onto the palm of an outstretched hand or falling with wrists flexed

Bennett’s fracture
Intra-articular fracture of the first carpometacarpal joint
Impact on flexed metacarpal, caused by fist fights
X-ray: triangular fragment at ulnar base of metacarpal

Image sourced from Wikipedia
Monteggia’s fracture
Dislocation of the proximal radioulnar joint in association with an ulna fracture
Fall on outstretched hand with forced pronation
Needs prompt diagnosis to avoid disability

Image sourced from Wikipedia
Galeazzi fracture
Radial shaft fracture with associated dislocation of the distal radioulnar joint
Direct blow

Pott’s fracture
Bimalleolar ankle fracture
Forced foot eversion

Barton’s fracture
Distal radius fracture (Colles’/Smith’s) with associated radiocarpal dislocation
Fall onto extended and pronated wrist
Involvement of the joint is a defining feature

How well did you know this?
1
Not at all
2
3
4
5
Perfectly
44
Q

A 32 year old man falls from scaffolding and sustains an injury to his forearm. Clinical examination and x-ray shows that he has sustained a radial fracture with dislocation of the distal radio-ulna joint.

 A.	Smith's
B.	Bennett's
C.	Monteggia's
D.	Colles'
E.	Galeazzi
F.	Pott's
G.	Barton's
A

Galeazzi

Isolated fracture of the radius alone can occur but is rare. Always check for associated injury.

Eponymous fractures

Colles’ fracture (dinner fork deformity)
Fall onto extended outstretched hand
Classical Colles’ fractures have the following 3 features:

  1. Transverse fracture of the radius
  2. 1 inch proximal to the radio-carpal joint
  3. Dorsal displacement and angulation

Smith’s fracture (reverse Colles’ fracture)
Volar angulation of distal radius fragment (Garden spade deformity)
Caused by falling backwards onto the palm of an outstretched hand or falling with wrists flexed

Bennett’s fracture
Intra-articular fracture of the first carpometacarpal joint
Impact on flexed metacarpal, caused by fist fights
X-ray: triangular fragment at ulnar base of metacarpal

Image sourced from Wikipedia
Monteggia’s fracture
Dislocation of the proximal radioulnar joint in association with an ulna fracture
Fall on outstretched hand with forced pronation
Needs prompt diagnosis to avoid disability

Image sourced from Wikipedia
Galeazzi fracture
Radial shaft fracture with associated dislocation of the distal radioulnar joint
Direct blow

Pott’s fracture
Bimalleolar ankle fracture
Forced foot eversion

Barton’s fracture
Distal radius fracture (Colles’/Smith’s) with associated radiocarpal dislocation
Fall onto extended and pronated wrist
Involvement of the joint is a defining feature

How well did you know this?
1
Not at all
2
3
4
5
Perfectly
45
Q

A 54-year-old man presents to the Emergency Department with a 2 day history of a swollen, painful left knee. You aspirate the joint to avoid admission to the orthopaedic wards. Aspirated joint fluid shows calcium pyrophosphate crystals. Which of the following blood tests is most useful in revealing an underlying cause?

	Transferrin saturation
	ACTH
	ANA
	Serum ferritin
	LDH
A

This is a typical presentation of pseudogout. An elevated transferrin saturation may indicate haemochromatosis, a recognised cause of pseudogout.

A high ferritin level is also seen in haemochromatosis but can be raised in a variety of infective and inflammatory processes, including pseudogout, as part of an acute phase response.

Pseudogout is a form of microcrystal synovitis caused by the deposition of calcium pyrophosphate dihydrate in the synovium

Risk factors
hyperparathyroidism
hypothyroidism
haemochromatosis
acromegaly
low magnesium, low phosphate
Wilson's disease

Features
knee, wrist and shoulders most commonly affected
joint aspiration: weakly-positively birefringent rhomboid shaped crystals
x-ray: chondrocalcinosis

Management
aspiration of joint fluid, to exclude septic arthritis
NSAIDs or intra-articular, intra-muscular or oral steroids as for gout

How well did you know this?
1
Not at all
2
3
4
5
Perfectly
46
Q

A 19 year old soldier has just returned from a prolonged marching exercise and presents with a sudden onset, severe pain, in the forefoot. Clinical examination reveals tenderness along the second metatarsal. Plain x-rays are taken of the area, these demonstrate callus surrounding the shaft of the second metatarsal. What is the most likely diagnosis?

	Stress fracture
	Mortons neuroma
	Osteochondroma
	Acute osteomyelitis
	Freiberg's disease
A

Stress Fracture

A short history of pain together with clinical examination and radiological signs affecting the second metatarsal favour a stress fracture. The fact that callus is present suggests that immobilisation is unlikely to be beneficial. Freibergs disease is an anterior metatarsalgia affecting the head of the second metarsal, it typically occurs in the pubertal growth spurt. The initial injury was thought to be due to stress microfractures at the growth plate. The key feature in the history which distinguishes the injury as being stress fracture is the radiology. In Freibergs disease the x-ray changes include; joint space widening, formation of bony spurs, sclerosis and flattening of the metatarsal head.

How well did you know this?
1
Not at all
2
3
4
5
Perfectly
47
Q

A 65-year-old Asian female presents with an extracapsular neck of femur fracture. Investigations show:

Calcium 2.07 mmol/l (2.20-2.60 mmol/l)
Phosphate 0.66 mmol/l (0.8-1.40 mmol/l)
ALP 256 IU/l (44-147 IU/l)

What is the most likely diagnosis?

	Bone tuberculosis
	Hypoparathyroidism
	Myeloma
	Osteomalacia
	Paget's disease
A

Osteomalacia

low: calcium, phosphate
raised: alkaline phosphatase

The low calcium and phosphate combined with the raised alkaline phosphatase point towards osteomalacia.

Osteomalacia

Basics
normal bony tissue but decreased mineral content
rickets if when growing
osteomalacia if after epiphysis fusion

Types
vitamin D deficiency e.g. malabsorption, lack of sunlight, diet
renal failure
drug induced e.g. anticonvulsants
vitamin D resistant; inherited
liver disease, e.g. cirrhosis

Features

rickets: knock-knee, bow leg, features of hypocalcaemia
osteomalacia: bone pain, fractures, muscle tenderness, proximal myopathy

Investigation
low calcium, phosphate, 25(OH) vitamin D
raised alkaline phosphatase
x-ray: children - cupped, ragged metaphyseal surfaces; adults - translucent bands (Looser’s zones or pseudofractures)

Treatment
calcium with vitamin D tablets

How well did you know this?
1
Not at all
2
3
4
5
Perfectly
48
Q

A 78-year-old woman is discharged following a fractured neck of femur. On review, she is making good progress but consideration is given to secondary prevention of further fractures. Unfortunately the orthogeriatricians are all on annual leave and the consultant has asked you to arrange suitable management. Which is the best option?

	Alendronate
	Alendronate, calcium and vitamin D supplementation
	Strontium
	Arrange a DEXA scan
	Hormone replacement therapy
A

Alendronate, calcium and vitamin D supplementation

A bisphosphonate, calcium and vitamin D supplementation should be given to all patients aged over 75 years after having a fracture. A DEXA scan is only needed of the patient is aged below 75 years. Hormone replacement therapy has been shown to reduce vertebral and non vertebral fractures, however the risks of cardiovascular disease and breast malignancy make this a less favourable option.

Osteoporosis: secondary prevention

NICE guidelines were updated in 2008 on the secondary prevention of osteoporotic fractures in postmenopausal women.

Key points include
Treatment is indicated following osteoporotic fragility fractures in postmenopausal women who are confirmed to have osteoporosis (a T-score of - 2.5 SD or below).
In women aged 75 years or older, a DEXA scan may not be required ‘if the responsible clinician considers it to be clinically inappropriate or unfeasible’
Vitamin D and calcium supplementation should be offered to all women unless the clinician is confident they have adequate calcium intake and are vitamin D replete
Alendronate is first-line
Around 25% of patients cannot tolerate alendronate, usually due to upper gastrointestinal problems. These patients should be offered risedronate or etidronate (see treatment criteria below)
Strontium ranelate and raloxifene are recommended if patients cannot tolerate bisphosphonates (see treatment criteria below)

Supplementary notes on treatment

Bisphosphonates
Alendronate, risedronate and etidronate are all licensed for the prevention and treatment of post-menopausal and glucocorticoid-induced osteoporosis
All three have been shown to reduce the risk of both vertebral and non-vertebral fractures although alendronate, risedronate may be superior to etidronate in preventing hip fractures
Ibandronate is a once-monthly oral bisphosphonate

Vitamin D and calcium
Poor evidence base to suggest reduced fracture rates in the general population at risk of osteoporotic fractures - may reduce rates in frail, housebound patients

Raloxifene - selective oestrogen receptor modulator (SERM)
Has been shown to prevent bone loss and to reduce the risk of vertebral fractures, but has not yet been shown to reduce the risk of non-vertebral fractures
Has been shown to increase bone density in the spine and proximal femur
May worsen menopausal symptoms
Increased risk of thromboembolic events
May decrease risk of breast cancer

Strontium ranelate
‘Dual action bone agent’ - increases deposition of new bone by osteoblasts and reduces the resorption of bone by osteoclasts
Strong evidence base, may be second-line treatment in near future
Increased risk of thromboembolic events

How well did you know this?
1
Not at all
2
3
4
5
Perfectly
49
Q

Which of the following statements relating to avascular necrosis is false?

When associated with fracture may occur despite the radiological evidence of fracture union.
Pain and stiffness will typically precede radiological evidence of the condition.
Drilling of affected bony fragments may be used to facilitate angiogenesis where arthroplasty is not warranted.
The earliest detectable radiological evidence is a radiolucency of the affected area coupled with subchondral collapse.
It is less likely when prompt anatomical alignment of fracture fragments is achieved.
A

The earliest detectable radiological evidence is a radiolucency of the affected area coupled with subchondral collapse. - FALSE
Avascular necrosis- radiological changes occur late.

Radiolucency and subchondral collapse are late changes. The earliest evidence on plain films is the affected area appearing as being more radio-opaque due to hyperaemia and resorption of the neighboring area. It may be diagnosed earlier using bone scans and MRI.

Avascular necrosis

Cellular death of bone components due to interruption of the blood supply, causing bone destruction
Main joints affected are hip, scaphoid, lunate and the talus.
It is not the same as non union. The fracture has usually united.
Radiological evidence is slow to appear.
Vascular ingrowth into the affected bone may occur. However, many joints will develop secondary osteoarthritis.

Causes
P ancreatitis 
L upus 
A lcohol 
S teroids 
T rauma 
I diopathic, infection 
C aisson disease, collagen vascular disease 
R adiation, rheumatoid arthritis 
A myloid 
G aucher disease 
S ickle cell disease

Presentation
Usually pain. Often despite apparent fracture union.

Investigation
MRI scanning will show changes earlier than plain films.

Treatment
In fractures at high risk sites anticipation is key. Early prompt and accurate reduction is essential.

Non weight bearing may help to facilitate vascular regeneration.

Joint replacement may be necessary, or even the preferred option (e.g. Hip in the elderly).

How well did you know this?
1
Not at all
2
3
4
5
Perfectly
50
Q

A 22 year old rugby player falls onto an outstretched hand and sustains a fracture of the distal radius. The x-ray shows a dorsally angulated comminuted fracture.

A. Discharge home with arm sling and fracture clinic appointment
B. Discharge home with futura splint and fracture clinic appointment
C. Admit for open reduction and fixation
D. Fasciotomy
E. Active observation for progression of neurovascular compromise
F. Reduction of fracture in casualty and application of plaster backslab, followed by discharge home.

A

Admit for open reduction and fixation

Unlike an osteoporotic fracture in an elderly lady this is a high velocity injury and will require surgical fixation.

Fracture management

Bony injury resulting in a fracture may arise from trauma (excessive forces applied to bone), stress related (repetitive low velocity injury) or pathological (abnormal bone which fractures during normal use of following minimal trauma)
Diagnosis involves not just evaluating the fracture ; such as site and type of injury but also other associated injuries and distal neurovascular deficits. This may entail not just clinical examination but radiographs of proximal and distal joints.
When assessing x-rays it is important to assess for changes in length of the bone, the angulation of the distal bone, rotational effects, presence of material such as glass.

Fracture types
Fracture type Description
Oblique fracture Fracture lies obliquely to long axis of bone
Comminuted fracture >2 fragments
Segmental fracture More than one fracture along a bone
Transverse fracture Perpendicular to long axis of bone
Spiral fracture Severe oblique fracture with rotation along long axis of bone

Open Vs Closed
It is also important to distinguish open from closed injuries. The most common classification system for open fractures is the Gustilo and Anderson classification system (given below):

Grade Injury
1 Low energy wound <1cm
2 Greater than 1cm wound with moderate soft tissue damage
3 High energy wound > 1cm with extensive soft tissue damage
3 A (sub group of 3) Adequate soft tissue coverage
3 B (sub group of 3) Inadequate soft tissue coverage
3 C (sub group of 3) Associated arterial injury

Key points in management of fractures
Immobilise the fracture including the proximal and distal joints
Carefully monitor and document neurovascular status, particularly following reduction and immobilisation
Manage infection including tetanus prophylaxis
IV broad spectrum antibiotics for open injuries
As a general principle all open fractures should be thoroughly debrided ( and internal fixation devices avoided or used with extreme caution)
Open fractures constitute an emergency and should be debrided and lavaged within 6 hours of injury

How well did you know this?
1
Not at all
2
3
4
5
Perfectly
51
Q

A 10 year old boy undergoes a delayed open reduction and fixation of a significantly displaced supracondylar fracture. On the ward he complains of significant forearm pain and paraesthesia of the hand. Radial pulse is normal.

A. Discharge home with arm sling and fracture clinic appointment
B. Discharge home with futura splint and fracture clinic appointment
C. Admit for open reduction and fixation
D. Fasciotomy
E. Active observation for progression of neurovascular compromise
F. Reduction of fracture in casualty and application of plaster backslab, followed by discharge home.

A

Fasciotomy

The delay is the significant factor here. These injuries often have neurovascular compromise and inactivity now places him at risk of developing complications. In compartment syndrome the loss of arterial pulsation occurs late.

Fracture management

Bony injury resulting in a fracture may arise from trauma (excessive forces applied to bone), stress related (repetitive low velocity injury) or pathological (abnormal bone which fractures during normal use of following minimal trauma)
Diagnosis involves not just evaluating the fracture ; such as site and type of injury but also other associated injuries and distal neurovascular deficits. This may entail not just clinical examination but radiographs of proximal and distal joints.
When assessing x-rays it is important to assess for changes in length of the bone, the angulation of the distal bone, rotational effects, presence of material such as glass.

Fracture types
Fracture type Description
Oblique fracture Fracture lies obliquely to long axis of bone
Comminuted fracture >2 fragments
Segmental fracture More than one fracture along a bone
Transverse fracture Perpendicular to long axis of bone
Spiral fracture Severe oblique fracture with rotation along long axis of bone

Open Vs Closed
It is also important to distinguish open from closed injuries. The most common classification system for open fractures is the Gustilo and Anderson classification system (given below):

Grade Injury
1 Low energy wound <1cm
2 Greater than 1cm wound with moderate soft tissue damage
3 High energy wound > 1cm with extensive soft tissue damage
3 A (sub group of 3) Adequate soft tissue coverage
3 B (sub group of 3) Inadequate soft tissue coverage
3 C (sub group of 3) Associated arterial injury

Key points in management of fractures
Immobilise the fracture including the proximal and distal joints
Carefully monitor and document neurovascular status, particularly following reduction and immobilisation
Manage infection including tetanus prophylaxis
IV broad spectrum antibiotics for open injuries
As a general principle all open fractures should be thoroughly debrided ( and internal fixation devices avoided or used with extreme caution)
Open fractures constitute an emergency and should be debrided and lavaged within 6 hours of injury

How well did you know this?
1
Not at all
2
3
4
5
Perfectly
52
Q

A 28 year old man falls onto an outstretched hand. On examination there is tenderness of the anatomical snuffbox. However, forearm and hand x-rays are normal.

A. Discharge home with arm sling and fracture clinic appointment
B. Discharge home with futura splint and fracture clinic appointment
C. Admit for open reduction and fixation
D. Fasciotomy
E. Active observation for progression of neurovascular compromise
F. Reduction of fracture in casualty and application of plaster backslab, followed by discharge home.

A

Discharge home with futura splint and fracture clinic appointment

This could well be a scaphoid fracture and should be temporarily immobilised pending further review. A futura splint will immobilise better than an arm sling for this problem.

Fracture management

Bony injury resulting in a fracture may arise from trauma (excessive forces applied to bone), stress related (repetitive low velocity injury) or pathological (abnormal bone which fractures during normal use of following minimal trauma)
Diagnosis involves not just evaluating the fracture ; such as site and type of injury but also other associated injuries and distal neurovascular deficits. This may entail not just clinical examination but radiographs of proximal and distal joints.
When assessing x-rays it is important to assess for changes in length of the bone, the angulation of the distal bone, rotational effects, presence of material such as glass.

Fracture types
Fracture type Description
Oblique fracture Fracture lies obliquely to long axis of bone
Comminuted fracture >2 fragments
Segmental fracture More than one fracture along a bone
Transverse fracture Perpendicular to long axis of bone
Spiral fracture Severe oblique fracture with rotation along long axis of bone

Open Vs Closed
It is also important to distinguish open from closed injuries. The most common classification system for open fractures is the Gustilo and Anderson classification system (given below):

Grade Injury
1 Low energy wound <1cm
2 Greater than 1cm wound with moderate soft tissue damage
3 High energy wound > 1cm with extensive soft tissue damage
3 A (sub group of 3) Adequate soft tissue coverage
3 B (sub group of 3) Inadequate soft tissue coverage
3 C (sub group of 3) Associated arterial injury

Key points in management of fractures
Immobilise the fracture including the proximal and distal joints
Carefully monitor and document neurovascular status, particularly following reduction and immobilisation
Manage infection including tetanus prophylaxis
IV broad spectrum antibiotics for open injuries
As a general principle all open fractures should be thoroughly debrided ( and internal fixation devices avoided or used with extreme caution)
Open fractures constitute an emergency and should be debrided and lavaged within 6 hours of injury

How well did you know this?
1
Not at all
2
3
4
5
Perfectly
53
Q

A 4 year old boy falls and sustains a fracture to the growth plate of his right wrist. Which of the following systems is used to classify the injury?

	Salter - Harris system
	Weber system
	Gustilo - Anderson system
	Garden system
	None of the above
A

The mnemonic ‘SALTR’ can be used to help remember the first five types. This mnemonic requires the reader to imagine the bones as long bones, with the epiphyses at the base.
I “S” = Slip (separated or straight across). Fracture of the cartilage of the physis (growth plate)
II “A” = Above. The fracture lies above the physis, or Away from the joint.
III “L” = Lower. The fracture is below the physis in the epiphysis.
IV “TE” = Through Everything. The fracture is through the metaphysis, physis, and epiphysis.
V “R” = Rammed (crushed). The physis has been crushed

The Salter - Harris system is most commonly used. The radiological signs in Type 1 and 5 injuries may be identical. Which is unfortunate as type 5 injuries do not do well (and may be missed!). One of our users has helpfully supplied a mnemonic for remembering the types (see above).

Paediatric fracture types
Type Injury pattern
Complete fracture Both sides of cortex are breached
Toddlers fracture Oblique tibial fracture in infants
Plastic deformity Stress on bone resulting in deformity without cortical disruption
Greenstick fracture Unilateral cortical breach only
Buckle fracture Incomplete cortical disruption resulting in periosteal haematoma only

Growth plate fractures
In paediatric practice fractures may also involve the growth plate and these injuries are classified according to the Salter- Harris system (given below):

Type Injury pattern
I Fracture through the physis only (x-ray often normal)
II Fracture through the physis and metaphysis
III Fracture through the physis and epiphyisis to include the joint
IV Fracture involving the physis, metaphysis and epiphysis
V Crush injury involving the physis (x-ray may resemble type I, and appear normal)

As a general rule it is safer to assume that growth plate tenderness is indicative of an underlying fracture even if the x-ray appears normal. Injuries of Types III, IV and V will usually require surgery. Type V injuries are often associated with disruption to growth.

Non accidental injury
Delayed presentation
Delay in attaining milestones
Lack of concordance between proposed and actual mechanism of injury
Multiple injuries
Injuries at sites not commonly exposed to trauma
Children on the at risk register

Pathological fractures
Genetic conditions, such as osteogenesis imperfecta, may cause pathological fractures.

Osteogenesis imperfecta
Defective osteoid formation due to congenital inability to produce adequate intercellular substances like osteoid, collagen and dentine.
Failure of maturation of collagen in all the connective tissues.
Radiology may show translucent bones, multiple fractures, particularly of the long bones, wormian bones (irregular patches of ossification) and a trefoil pelvis.

Subtypes
Type I The collagen is normal quality but insufficient quantity.
Type II- Poor collagen quantity and quality.
Type III- Collagen poorly formed. Normal quantity.
Type IV- Sufficient collagen quantity but poor quality.

Osteopetrosis
Bones become harder and more dense.
Autosomal recessive condition.
It is commonest in young adults.
Radiology reveals a lack of differentiation between the cortex and the medulla described as marble bone.
How well did you know this?
1
Not at all
2
3
4
5
Perfectly
54
Q

A 66 year old lady presents with pain in her right hip. It has been increasing over the previous three weeks and waking her from sleep. On examination she is tender on internal rotation. Blood tests reveal a mildly elevated serum calcium and alkaline phosphatase levels.

A.	Osteogenesis imperfecta
B.	Osteoporosis
C.	Rickets
D.	Pagets disease
E.	Chondrosarcoma
F.	Metastatic breast cancer
A

Metastatic breast cancer

Increasing pain at rest, together with increased serum calcium and alkaline phosphatase are most likely to represent metastatic tumour to bone. Chondrosarcomas do occur in the pelvis but are not associated with increased serum calcium and typically have a longer history.

Pagets
Focal bone resorption followed by excessive and chaotic bone deposition
Affects (in order): spine, skull, pelvis and femur
Serum alkaline phosphatase raised (other parameters normal)
Abnormal thickened, sclerotic bone on x-rays
Risk of cardiac failure with >15% bony involvement
Small risk of sarcomatous change
Bisphosphonates

Osteoporosis
Excessive bone resorption resulting in demineralised bone
Commoner in old age
Increased risk of pathological fracture, otherwise asymptomatic
Alkaline phosphatase normal, calcium normal
Bisphosphonates, calcium and vitamin D

Secondary bone tumours
Bone destruction and tumour infiltration
Mirel scoring used to predict risk of fracture
Appearances depend on primary (e.g.sclerotic - prostate, lytic - breast)
Elevated serum calcium and alkaline phosphatase may be seen
Radiotherapy, prophylactic fixation and analgesia

How well did you know this?
1
Not at all
2
3
4
5
Perfectly
55
Q

A 73 year old man presents with pain in the right leg. It is most uncomfortable on walking. On examination he has a deformity of his right femur, which on x-ray is thickened and sclerotic. His serum alkaline phosphatase is elevated, but calcium is within normal limits.

 A.	Osteogenesis imperfecta
B.	Osteoporosis
C.	Rickets
D.	Pagets disease
E.	Chondrosarcoma
F.	Metastatic breast cancer
A

Pagets disease

This is a typical scenario for Pagets disease.

Pagets
Focal bone resorption followed by excessive and chaotic bone deposition
Affects (in order): spine, skull, pelvis and femur
Serum alkaline phosphatase raised (other parameters normal)
Abnormal thickened, sclerotic bone on x-rays
Risk of cardiac failure with >15% bony involvement
Small risk of sarcomatous change
Bisphosphonates

Osteoporosis
Excessive bone resorption resulting in demineralised bone
Commoner in old age
Increased risk of pathological fracture, otherwise asymptomatic
Alkaline phosphatase normal, calcium normal
Bisphosphonates, calcium and vitamin D

Secondary bone tumours
Bone destruction and tumour infiltration
Mirel scoring used to predict risk of fracture
Appearances depend on primary (e.g.sclerotic - prostate, lytic - breast)
Elevated serum calcium and alkaline phosphatase may be seen
Radiotherapy, prophylactic fixation and analgesia

How well did you know this?
1
Not at all
2
3
4
5
Perfectly
56
Q

A 73 year old lady presents with pain in her left hip. She was walking around the house when she tripped over a rug and fell over. Apart from temporal arteritis which is well controlled with prednisolone she is otherwise well. On examination he leg is shorted and externally rotated.Her serum alkaline phosphatase and calcium are normal.

A.	Osteogenesis imperfecta
B.	Osteoporosis
C.	Rickets
D.	Pagets disease
E.	Chondrosarcoma
F.	Metastatic breast cancer
A

Osteoporosis

The combination of age, female gender and steroids coupled with hip pain on minor trauma are strongly suggestive of osteoporosis.

Pagets
Focal bone resorption followed by excessive and chaotic bone deposition
Affects (in order): spine, skull, pelvis and femur
Serum alkaline phosphatase raised (other parameters normal)
Abnormal thickened, sclerotic bone on x-rays
Risk of cardiac failure with >15% bony involvement
Small risk of sarcomatous change
Bisphosphonates

Osteoporosis
Excessive bone resorption resulting in demineralised bone
Commoner in old age
Increased risk of pathological fracture, otherwise asymptomatic
Alkaline phosphatase normal, calcium normal
Bisphosphonates, calcium and vitamin D

Secondary bone tumours
Bone destruction and tumour infiltration
Mirel scoring used to predict risk of fracture
Appearances depend on primary (e.g.sclerotic - prostate, lytic - breast)
Elevated serum calcium and alkaline phosphatase may be seen
Radiotherapy, prophylactic fixation and analgesia

Ankylosing spondylitis
Chronic inflammatory disorder affecting the axial skeleton
Sacro-ilitis is a usually visible in plain films
Up to 20% of those who are HLA B27 positive will develop the condition
Affected articulations develop bony or fibrous changes
Typical spinal features include loss of the lumbar lordosis and progressive kyphosis of the cervico-thoracic spine
Scheuermann’s disease
Epiphysitis of the vertebral joints is the main pathological process
Predominantly affects adolescents
Symptoms include back pain and stiffness
X-ray changes include epiphyseal plate disturbance and anterior wedging
Clinical features include progressive kyphosis (at least 3 vertebrae must be involved)
Minor cases may be managed with physiotherapy and analgesia, more severe cases may require bracing or surgical stabilisation
Scoliosis
Consists of curvature of the spine in the coronal plane
Divisible into structural and non structural, the latter being commonest in adolescent females who develop minor postural changes only. Postural scoliosis will typically disappear on manoeuvres such as bending forwards
Structural scoliosis affects > 1 vertebral body and is divisible into idiopathic, congential and neuromuscular in origin. It is not correctable by alterations in posture
Within structural scoliosis, idiopathic is the most common type
Severe, or progressive structural disease is often managed surgically with bilateral rod stabilisation of the spine
Spina bifida
Non fusion of the vertebral arches during embryonic development
Three categories; myelomeningocele, spina bifida occulta and meningocele
Myelomeningocele is the most severe type with associated neurological defects that may persist in spite of anatomical closure of the defect
Up to 10% of the population may have spina bifida occulta, in this condition the skin and tissues (but not not bones) may develop over the distal cord. The site may be identifiable by a birth mark or hair patch
The incidence of the condition is reduced by use of folic acid supplements during pregnancy
Spondylolysis
Congenital or acquired deficiency of the pars interarticularis of the neural arch of a particular vertebral body, usually affects L4/ L5
May be asymptomatic and affects up to 5% of the population
Spondylolysis is the commonest cause of spondylolisthesis in children
Asymptomatic cases do not require treatment
Spondylolisthesis
This occurs when one vertebra is displaced relative to its immediate inferior vertebral body
May occur as a result of stress fracture or spondylolysis
Traumatic cases may show the classic “Scotty Dog” appearance on plain films
Treatment depends upon the extent of deformity and associated neurological symptoms, minor cases may be actively monitored. Individuals with radicular symptoms or signs will usually require spinal decompression and stabilisation

How well did you know this?
1
Not at all
2
3
4
5
Perfectly
57
Q

A 19 year old female is involved in an athletics event. She has just completed the high jump when she suddenly develops severe back pain and weakness affecting both her legs. On examination, she has a prominent sacrum and her lower back is painful.

A.	Spondylolysis
B.	Spina bifida occulta
C.	Spondylolisthesis
D.	Meningomyelocele
E.	Meningocele
F.	Functional scoliosis
G.	Structural scoliosis
H.	Ankylosing spondylitis
I.	Scheuermanns disease
A

Spondylolisthesis

Young athletic females are the group most frequently affected by spondylolythesis who have a background of spondylolysis. Whilst the latter condition is a risk factor for spondylolythesis the former condition is most likely in a young athletic female who presents with sudden pain.

Ankylosing spondylitis
Chronic inflammatory disorder affecting the axial skeleton
Sacro-ilitis is a usually visible in plain films
Up to 20% of those who are HLA B27 positive will develop the condition
Affected articulations develop bony or fibrous changes
Typical spinal features include loss of the lumbar lordosis and progressive kyphosis of the cervico-thoracic spine
Scheuermann’s disease
Epiphysitis of the vertebral joints is the main pathological process
Predominantly affects adolescents
Symptoms include back pain and stiffness
X-ray changes include epiphyseal plate disturbance and anterior wedging
Clinical features include progressive kyphosis (at least 3 vertebrae must be involved)
Minor cases may be managed with physiotherapy and analgesia, more severe cases may require bracing or surgical stabilisation
Scoliosis
Consists of curvature of the spine in the coronal plane
Divisible into structural and non structural, the latter being commonest in adolescent females who develop minor postural changes only. Postural scoliosis will typically disappear on manoeuvres such as bending forwards
Structural scoliosis affects > 1 vertebral body and is divisible into idiopathic, congential and neuromuscular in origin. It is not correctable by alterations in posture
Within structural scoliosis, idiopathic is the most common type
Severe, or progressive structural disease is often managed surgically with bilateral rod stabilisation of the spine
Spina bifida
Non fusion of the vertebral arches during embryonic development
Three categories; myelomeningocele, spina bifida occulta and meningocele
Myelomeningocele is the most severe type with associated neurological defects that may persist in spite of anatomical closure of the defect
Up to 10% of the population may have spina bifida occulta, in this condition the skin and tissues (but not not bones) may develop over the distal cord. The site may be identifiable by a birth mark or hair patch
The incidence of the condition is reduced by use of folic acid supplements during pregnancy
Spondylolysis
Congenital or acquired deficiency of the pars interarticularis of the neural arch of a particular vertebral body, usually affects L4/ L5
May be asymptomatic and affects up to 5% of the population
Spondylolysis is the commonest cause of spondylolisthesis in children
Asymptomatic cases do not require treatment
Spondylolisthesis
This occurs when one vertebra is displaced relative to its immediate inferior vertebral body
May occur as a result of stress fracture or spondylolysis
Traumatic cases may show the classic “Scotty Dog” appearance on plain films
Treatment depends upon the extent of deformity and associated neurological symptoms, minor cases may be actively monitored. Individuals with radicular symptoms or signs will usually require spinal decompression and stabilisation

58
Q

A 15 year old boy is brought to the clinic by his mother who is concerned that he has a mark overlying his lower spine. On examination the boy has a patch of hair overlying his lower lumbar spine and a birth mark at the same location. Lower limb neurological examination is normal.

 A.	Spondylolysis
B.	Spina bifida occulta
C.	Spondylolisthesis
D.	Meningomyelocele
E.	Meningocele
F.	Functional scoliosis
G.	Structural scoliosis
H.	Ankylosing spondylitis
I.	Scheuermanns disease
A

Spina bifida occulta

Spina bifida occulta is a common condition and may affect up to 10% of the population. The more severe types of spina bifida have more characteristic skin changes. Occasionally the unwary surgeon is persuaded to operate on these cutaneous changes and we would advocate performing an MRI scan prior to any such surgical procedure in this region.

Ankylosing spondylitis
Chronic inflammatory disorder affecting the axial skeleton
Sacro-ilitis is a usually visible in plain films
Up to 20% of those who are HLA B27 positive will develop the condition
Affected articulations develop bony or fibrous changes
Typical spinal features include loss of the lumbar lordosis and progressive kyphosis of the cervico-thoracic spine
Scheuermann’s disease
Epiphysitis of the vertebral joints is the main pathological process
Predominantly affects adolescents
Symptoms include back pain and stiffness
X-ray changes include epiphyseal plate disturbance and anterior wedging
Clinical features include progressive kyphosis (at least 3 vertebrae must be involved)
Minor cases may be managed with physiotherapy and analgesia, more severe cases may require bracing or surgical stabilisation
Scoliosis
Consists of curvature of the spine in the coronal plane
Divisible into structural and non structural, the latter being commonest in adolescent females who develop minor postural changes only. Postural scoliosis will typically disappear on manoeuvres such as bending forwards
Structural scoliosis affects > 1 vertebral body and is divisible into idiopathic, congential and neuromuscular in origin. It is not correctable by alterations in posture
Within structural scoliosis, idiopathic is the most common type
Severe, or progressive structural disease is often managed surgically with bilateral rod stabilisation of the spine
Spina bifida
Non fusion of the vertebral arches during embryonic development
Three categories; myelomeningocele, spina bifida occulta and meningocele
Myelomeningocele is the most severe type with associated neurological defects that may persist in spite of anatomical closure of the defect
Up to 10% of the population may have spina bifida occulta, in this condition the skin and tissues (but not not bones) may develop over the distal cord. The site may be identifiable by a birth mark or hair patch
The incidence of the condition is reduced by use of folic acid supplements during pregnancy
Spondylolysis
Congenital or acquired deficiency of the pars interarticularis of the neural arch of a particular vertebral body, usually affects L4/ L5
May be asymptomatic and affects up to 5% of the population
Spondylolysis is the commonest cause of spondylolisthesis in children
Asymptomatic cases do not require treatment
Spondylolisthesis
This occurs when one vertebra is displaced relative to its immediate inferior vertebral body
May occur as a result of stress fracture or spondylolysis
Traumatic cases may show the classic “Scotty Dog” appearance on plain films
Treatment depends upon the extent of deformity and associated neurological symptoms, minor cases may be actively monitored. Individuals with radicular symptoms or signs will usually require spinal decompression and stabilisation

59
Q

A 19 year old female presents to the clinic with progressive pain in her neck and back. The condition has been progressively worsening over the past 6 months. She has not presented previously because she was an inpatient with a disease flare of ulcerative colitis. On examination, she has a stiff back with limited spinal extension on bending forwards.

A.	Spondylolysis
B.	Spina bifida occulta
C.	Spondylolisthesis
D.	Meningomyelocele
E.	Meningocele
F.	Functional scoliosis
G.	Structural scoliosis
H.	Ankylosing spondylitis
I.	Scheuermanns disease
A

Ankylosing spondylitis

Ankylosing spondylitis is associated with HLA B27, there is a strong association with ulcerative colitis in such individuals. The clinical findings are usually of a kyphosis affecting the cervical and thoracic spine. Considerable symptomatic benefit may be obtained using non steroidal anti inflammatory drugs. These should be used carefully in patients with inflammatory bowel disease who may be taking steroids.

Ankylosing spondylitis
Chronic inflammatory disorder affecting the axial skeleton
Sacro-ilitis is a usually visible in plain films
Up to 20% of those who are HLA B27 positive will develop the condition
Affected articulations develop bony or fibrous changes
Typical spinal features include loss of the lumbar lordosis and progressive kyphosis of the cervico-thoracic spine
Scheuermann’s disease
Epiphysitis of the vertebral joints is the main pathological process
Predominantly affects adolescents
Symptoms include back pain and stiffness
X-ray changes include epiphyseal plate disturbance and anterior wedging
Clinical features include progressive kyphosis (at least 3 vertebrae must be involved)
Minor cases may be managed with physiotherapy and analgesia, more severe cases may require bracing or surgical stabilisation
Scoliosis
Consists of curvature of the spine in the coronal plane
Divisible into structural and non structural, the latter being commonest in adolescent females who develop minor postural changes only. Postural scoliosis will typically disappear on manoeuvres such as bending forwards
Structural scoliosis affects > 1 vertebral body and is divisible into idiopathic, congential and neuromuscular in origin. It is not correctable by alterations in posture
Within structural scoliosis, idiopathic is the most common type
Severe, or progressive structural disease is often managed surgically with bilateral rod stabilisation of the spine
Spina bifida
Non fusion of the vertebral arches during embryonic development
Three categories; myelomeningocele, spina bifida occulta and meningocele
Myelomeningocele is the most severe type with associated neurological defects that may persist in spite of anatomical closure of the defect
Up to 10% of the population may have spina bifida occulta, in this condition the skin and tissues (but not not bones) may develop over the distal cord. The site may be identifiable by a birth mark or hair patch
The incidence of the condition is reduced by use of folic acid supplements during pregnancy
Spondylolysis
Congenital or acquired deficiency of the pars interarticularis of the neural arch of a particular vertebral body, usually affects L4/ L5
May be asymptomatic and affects up to 5% of the population
Spondylolysis is the commonest cause of spondylolisthesis in children
Asymptomatic cases do not require treatment
Spondylolisthesis
This occurs when one vertebra is displaced relative to its immediate inferior vertebral body
May occur as a result of stress fracture or spondylolysis
Traumatic cases may show the classic “Scotty Dog” appearance on plain films
Treatment depends upon the extent of deformity and associated neurological symptoms, minor cases may be actively monitored. Individuals with radicular symptoms or signs will usually require spinal decompression and stabilisation

60
Q

A 63 year old lady undergoes an axillary clearance for breast cancer. She makes steady progress. However, 8 weeks post operatively she still suffers from severe shoulder pain. On examination she has reduced active movements in all planes and loss of passive external rotation.

A.	Impingement syndrome
B.	Rotator cuff tear
C.	Adhesive capsulitis
D.	Calcific tendonitis
E.	Biceps tendon rupture
F.	Parsonage - Turner syndrome
G.	Labral tear
A

Adhesive capsulitis

Frozen shoulder passes through an initial painful stage followed by a period of joint stiffness. With physiotherapy the problem will usually resolve although it may take up to 2 years to do so.

Shoulder fractures and dislocations
Fractures
Proximal humerus
Background
Third most common fragility fracture in the elderly.
Results from low energy fall in predominantly elderly females, or from high energy trauma in young males.
Can be associated with nerve injury (commonly axillary), and fracture-dislocation of the humeral head. Detailed neurological assessment is essential for all upper limb injuries.

Anatomy
Osteology
Consists of articular head, greater tuberosity, lesser tuberosity, metaphysis and diaphysis. Between the articular head and the tuberosities is the anatomical neck (previous physis). Between the tuberosities and the metaphysis is the surgical neck.
The supraspinatus, infraspinatus and teres minor muscles attach to the greater tuberosity. The subscapularis muscle attaches to the lesser tuberosity.

Vascular Supply
Humeral head is supplied by the anterior and posterior humeral circumflex arteries. Anatomical neck fractures are at greatest risk of osteonecrosis.

Imaging
Imaging aims to both delineate the fracture pattern, and confirm/exlude the presence of an associated dislocation.
Radiographs - True anteroposterior (AP), axillary lateral and/or scapula Y view.
CT - indicated to better define intra-articular involvement and to aid pre-operative planning. MRI is not useful for fracture imaging.

Classification
Description of the fracture is often more useful than classification. Particular attention should be paid to humeral alignment, fracture displacement, and greater tuberosity position (rotator cuff will pull the GT supero-posterioly, which can cause impingement problems with malunion).
- Neer Classification: Most commonly used. Describes fracture as 2,3,or 4 part depending upon the number main fragments. Also comments on the degree of displacement. Fragments:
-greater tuberosity
-lesser tuberosity
- articular surface
- shaft
Displacement: >1cm or angulation >45 degrees.

Treatment
The vast majority of proximal humeral fractures are minimally displaced, and therefore can be managed conservatively. This involves immobilisation in a polysling, and progressive mobilisation. Pendular exercise can commence at 14 days, and active abduction from 4-6 weeks.

Irreducible fracture dislocation is an indication for operative management. Other indications include large displacement, younger patient, head splitting (intra-articular fractures). However, the recent PROFHER trial (1) has suggested no benefit to operative intervention on patient outcome (it must be applied cautiously as majority of patients were elderly with extraarticular fractures). Options available for surgical management include:

ORIF Most commonly used. Plate and screw fixation. Can reconstruct complex fractures.
Intramedullary nail Suitable for extra-articular configuration, predominantly surgical neck +/- GT fractures.

Hemiarthroplasty Used for un-reconstructable fractures in the older patient who has good glenoid quality.
Total shoulder arthroplasty Unconstructable fractures where high functioning shoulder is required (hemiarthroplasty will cause glenoid erosion)
Reverse shoulder arthroplasty Total shoulder arthroplasty that provides better functional outcome than conventional total shoulder replacement.

Scapula
Background
Uncommon fractures usually associated with high energy trauma. Most commonly involve scapula body or spine (50%), glenoid fossa and glenoid neck. Important to exclude associated life threatening injury.

Imaging
Plain radiographs should include true anteroposterior (AP), axillary lateral and/or scapula Y view. CT scanning is useful for defining intra-articular involvement, displacement and for three dimensional reconstruction.

Classification
Based on the location of the fracture (coracoid, acromion, glenoid neck, glenoid fossa, scapula body). Beware of ipsilateral glenoid neck and clavicle fracture -floating shoulder - where limb is effectively dissociated from axial skeleton.

Treatment
The vast majority of scapula fractures are amenable to conservative management, consisting of sling immobilisation for two weeks followed by early rehabilitation. Floating shoulder will usually require fixation, and consideration of surgery should also be given to intra-articular and displaced/angulated glenoid fractures.

Dislocations

Types
Dislocations around the shoulder joint include glenohumeral dislocation, acromioclavicular joint disruption and sternoclavicular dislocation. Only glenohumeral dislocation will be covered here.

Glenohumeral dislocation
Diagnosis, classification and management are covered here.

Background
Shoulder dislocation is commonly seen in A&E. It has a high recurrence rate that is as high as 80% in teenagers. Initial management requires emergent reduction to prevent lasting chondral damage.

Early assessment and management
Usually a traumatic cause (multi-directional instability in frequent dislocations requires discussion with orthopaedics and is not covered here). Careful history, examination and documentation of neurovascular status of the limb, in particular the axillary nerve (regimental badge sensation). This should be re-assessed post manipulation. Early radiographs to confirm direction of dislocation.

Initial management consists of emergent closed reduction under under entanox and analgesia, but often requires conscious sedation. Arm should then be immobilised in a polysling, and XR to confirm relocation.

Imaging - True anteroposterior (AP), axillary lateral and/or scapula Y view. Reduced humeral head should lie between acromion and coracoid on lateral/scapula view.

Types
Direction Features Cause Examination Reduction techniques
Anterior Most Common >90% Usually traumatic - anterior force on arm when shoulder is abducted, eternally rotated Loss of shoulder contour - sulcus sign. Humeral head can be felt anteriorly.
Hippocratic.
Milch.
Stimson.

Kocher not advised due to complication of fracture
Posterior 50% missed in A&E 50% traumatic, but classically post seizure or electrocution Shoulder locked in internal rotation. XR may show lightbulb appearance. Gentle lateral traction to adducted arm.
Inferior Rare Associated with pectorals and rotator cuff tears, and glenoid fracture As for primary injury Management of primary injury

Superior Rare Associated with acrominon/clavicle fracture As for primary injury Management of primary injury

Associated injuries
Bankart lesion - avulsion of the anterior glenoid labrum with an anterior shoulder dislocation (reverse Bankart if poster labrum in posterior dislocation).
Hill Sachs defect - chondral impaction on posteriosuperior humeral head from contact with gleonoid rim. Can be large enough to lock shoulder, requiring open reduction. (Reverse Hill Sachs in posterior dislocation).
Rotator cuff tear - increases with age.
Greater or lesser tuberosity fracture - increases with age.
Humeral neck fracture - shoulder fracture dislocation. More common in high energy trauma and elderly. Should be discussed with orthopaedics prior to any attempted reduction.

Rotator Cuff Disease

Rotator cuff disease is a spectrum of conditions that ranges from subacromial impingement to rotator cuff tears and eventually to rotator cuff arthropathy (arthritis).

Anatomy
The rotator cuff is a group of four muscles that are important in shoulder movements, and maintenance of glenohumeral stability.
Muscle Scapular attachment Humeral attachment Action Innervation
Supraspinatus Supraspinatus fossa Superior facet of greater tuberosity Initiation of abduction of humerus Suprascapular nerve
Infraspinatus Infraspinatus fossa Posterior facet of greater tuberosity External rotation of humerus Suprascapular nerve
Teres Minor Lateral border Inferior facet of greater tuberosity External rotation of humerus Axillary Nerve
Subscapularis Subscapular fossa Lesser tuberosity Internal rotation of humerus Upper and lower subscapular nerve

The inferior rotator cuff muscles (infraspinatus, teres minor, and subscapularis) balance the superior pull of the deltoid. Injury/tear results in upward migration of the humeral head on the glenoid (can be seen on AP radiograph).
Likewise, the anterior muscles (subscapularis) are balanced with the posterior muscles (infraspinatus, teres minor).

Subacromial Impingement

The most common cause of shoulder pain, which results from impingement of the superior cuff on the undersurface of the acromion, and an inflammatory bursitis.
Associated with certain types of acromial morphology (Bigliani classification).
Presents as insidious pain which is exacerbated by overhead activities.

Rotator Cuff Tear

Often presents as an acute event on the background of chronic subacromial impingement in the older patient, but can present as an avulsion injury in younger patients.
Majority of tears are to the superior cuff (supraspinatus, infraspinatus, teres minor), though a tear to subscapularis is associated with subcoracoid impingement.
Tears present as pain and weakness when using the muscles in question.

Rotator Cuff Arthropathy

Defined as shoulder arthritis in the setting of rotator cuff dysfunction. Results from superior migration due to the loss of rotator cuff function and integrity. Unopposed deltoid pulls the humeral head superiorly.
Associated with massive chronic cuff tears.

Imaging

Plain radiographs
AP of the shoulder may show superior migration of the humerus with a cuff tear, and features of arthritis with arthropathy. Other causes of pain may also be identified (e.g. calcific tendonitis/fracture)
Outlet view is useful for defining the acromial morphology

USS
Allows dynamic imaging of the cuff, and is inexpensive. However, it is very user dependent.

MRI
Best imaging modality for cuff pathology.
Also allows imaging of the rest of the shoulder. When intra-articular pathology is suspected, can be combined with an arthrogram for improved sensitivity and specificity.

Treatment

Subacromial impingement
Physiotherapy, oral anti-inflammatory medication
Subacromial steroid injection can settle inflammation
Arthroscopic subacromial decompression by shaving away the undersurface of the acromion, more space is created for the rotator cuff. Cuff integrity is assessed also at time of surgery, and can be repaired if necessary.

Rotator cuff tear
When considering repair of a cuff tear, the age and activity of the patient, the nature of the tear (degenerative vs. acute traumatic), and the size and retraction of the tear should be considered when making a surgical plan.
Mild tears or tears in the elderly can be managed conservatively, as outlined above.
Moderate tears can be repaired arthroscopically. Massive or retracted tears will often require an open repair (occasionally with a tendon transfer). Subacromial decompression is performed at the same time to reduce impingement, symptoms and recurrence.

Calcific tendonitis
Calcific tendonitis involves calcific deposits within tendons anywhere in the body, but most commonly in the rotator cuff (specifically the supraspinatus tendon). When present in the shoulder, it is associated with subacromial impingement and pain.

Pathology
More common in women aged 30-60 years.
Association with diabetes and hypothyroidism

There are three stages of calcification
Formative phase characterized by calcific deposits
Resting phase deposit is stable, but presents with impingement problems
Resorptive phase phagocytic resorption. Most painful stage.

Presentation
Similar in presentation to subacromial impingement, with pain especially with over head activities. Atraumatic in nature.

Imaging
Plain radiographs show calcification of the rotator cuff, usually within 1.5cm of its insertion on the humerus. Supraspinatus outlet views can show level of impingment. Further imaging is rarely needed.

Treatment
Non-operative NSAIDS, steroid injection (controversial, but practiced) and physiotherapy. Approximately 75% will resolve by 6 months with conservative management.
Ultrasound guided or surgical needle barbotage can break down deposits and resolve symptoms. Occasionally surgical excision is required.

Adhesive capsulitis (Frozen Shoulder)
Pain and loss of movement of shoulder joint, which involves fibroplastic proliferation of capsular tissue, causing soft tissue scarring and contracture. Patients present with a painful and decreased arc of motion.
Associated with prolonged immobilization, previous surgery, thyroid disorders (AI) and diabetes
Classically three stages which can take up to two years to resolve:
Stage one the freezing and painful stage
Stage two the frozen and stiff stage
Stage three the thawing stage, where shoulder movement slowly improves

Imaging
Plain radiographs to exclude other causes of a painful shoulder
MRI arthrogram may show capsular contracture, and again may be used to exclude cuff pathology. However, often not performed as diagnosis is largely clinical.

Treatment
Non-operative NSAIDS, steroid injection and physiotherapy. Patience is required as condition can take up to 2 years to improve.
Operative MUA or arthroscopic adhesiolysis (release of adhesions) can expedite recovery, followed by intensive physiotherapy.

Glenohumeral Arthritis
Background
May be osteoarthritis (primary or secondary to cuff tear or trauma), rheumatoid arthritis, or as part of a spondyloarthropathy. Majority of those with RA will develop symptoms.
More common in the elderly
Presents like any other arthritis - pain at night and with movement

Imaging
AP and axillary radiographs will show features of arthritis.
CT/MRI is often useful to classify the shape of the glenoid and extent of bone loss when considering arthroplasty. MRI also essential to asses integrity of rotator cuff if considering shoulder replacement.

Treatment
Like all orthopaedics, start with simple measures:
NSAIDS, management of RA, physiotherapy, steroid injection.
Hemiarthroplasty can sometimes be considered if glenoid is in excellent condition or if patient has large comorbidity.
Arthroscopic debridement is useful if patient has isolated ACJ arthritis, but is rarely used for glenohumeral arthritis.
Total shoulder replacement is shown to produce superior outcome when compared to hemiarthroplasty in terms of pain relief, function and implant survival.
Total shoulder replacement can be anatomical (ball on humerus, with cup on glenoid), or reverse geometry (ball on glenoid, with cup on humerus). Anatomical TSR requires an in tact rotator cuff, so often reverse is preferable when the cuff if questionable in integrity.

61
Q

A 78 year old man complains of a long history of shoulder pain and more recently weakness. On examination active attempts at abduction are impaired. Passive movements are normal.

A.	Impingement syndrome
B.	Rotator cuff tear
C.	Adhesive capsulitis
D.	Calcific tendonitis
E.	Biceps tendon rupture
F.	Parsonage - Turner syndrome
G.	Labral tear
A

Rotator cuff tear

Rotator cuff tears are common in elderly people and may occur following minor trauma or as a result of long standing impingement. Tears greater than 2cm should generally be repaired surgically. The length of the history in this scenario is suggestive of a tear complicating impingement.

Shoulder fractures and dislocations
Fractures
Proximal humerus
Background
Third most common fragility fracture in the elderly.
Results from low energy fall in predominantly elderly females, or from high energy trauma in young males.
Can be associated with nerve injury (commonly axillary), and fracture-dislocation of the humeral head. Detailed neurological assessment is essential for all upper limb injuries.

Anatomy
Osteology
Consists of articular head, greater tuberosity, lesser tuberosity, metaphysis and diaphysis. Between the articular head and the tuberosities is the anatomical neck (previous physis). Between the tuberosities and the metaphysis is the surgical neck.
The supraspinatus, infraspinatus and teres minor muscles attach to the greater tuberosity. The subscapularis muscle attaches to the lesser tuberosity.

Vascular Supply
Humeral head is supplied by the anterior and posterior humeral circumflex arteries. Anatomical neck fractures are at greatest risk of osteonecrosis.

Imaging
Imaging aims to both delineate the fracture pattern, and confirm/exlude the presence of an associated dislocation.
Radiographs - True anteroposterior (AP), axillary lateral and/or scapula Y view.
CT - indicated to better define intra-articular involvement and to aid pre-operative planning. MRI is not useful for fracture imaging.

Classification
Description of the fracture is often more useful than classification. Particular attention should be paid to humeral alignment, fracture displacement, and greater tuberosity position (rotator cuff will pull the GT supero-posterioly, which can cause impingement problems with malunion).
- Neer Classification: Most commonly used. Describes fracture as 2,3,or 4 part depending upon the number main fragments. Also comments on the degree of displacement. Fragments:
-greater tuberosity
-lesser tuberosity
- articular surface
- shaft
Displacement: >1cm or angulation >45 degrees.

Treatment
The vast majority of proximal humeral fractures are minimally displaced, and therefore can be managed conservatively. This involves immobilisation in a polysling, and progressive mobilisation. Pendular exercise can commence at 14 days, and active abduction from 4-6 weeks.

Irreducible fracture dislocation is an indication for operative management. Other indications include large displacement, younger patient, head splitting (intra-articular fractures). However, the recent PROFHER trial (1) has suggested no benefit to operative intervention on patient outcome (it must be applied cautiously as majority of patients were elderly with extraarticular fractures). Options available for surgical management include:

ORIF Most commonly used. Plate and screw fixation. Can reconstruct complex fractures.
Intramedullary nail Suitable for extra-articular configuration, predominantly surgical neck +/- GT fractures.

Hemiarthroplasty Used for un-reconstructable fractures in the older patient who has good glenoid quality.
Total shoulder arthroplasty Unconstructable fractures where high functioning shoulder is required (hemiarthroplasty will cause glenoid erosion)
Reverse shoulder arthroplasty Total shoulder arthroplasty that provides better functional outcome than conventional total shoulder replacement.

Scapula
Background
Uncommon fractures usually associated with high energy trauma. Most commonly involve scapula body or spine (50%), glenoid fossa and glenoid neck. Important to exclude associated life threatening injury.

Imaging
Plain radiographs should include true anteroposterior (AP), axillary lateral and/or scapula Y view. CT scanning is useful for defining intra-articular involvement, displacement and for three dimensional reconstruction.

Classification
Based on the location of the fracture (coracoid, acromion, glenoid neck, glenoid fossa, scapula body). Beware of ipsilateral glenoid neck and clavicle fracture -floating shoulder - where limb is effectively dissociated from axial skeleton.

Treatment
The vast majority of scapula fractures are amenable to conservative management, consisting of sling immobilisation for two weeks followed by early rehabilitation. Floating shoulder will usually require fixation, and consideration of surgery should also be given to intra-articular and displaced/angulated glenoid fractures.

Dislocations

Types
Dislocations around the shoulder joint include glenohumeral dislocation, acromioclavicular joint disruption and sternoclavicular dislocation. Only glenohumeral dislocation will be covered here.

Glenohumeral dislocation
Diagnosis, classification and management are covered here.

Background
Shoulder dislocation is commonly seen in A&E. It has a high recurrence rate that is as high as 80% in teenagers. Initial management requires emergent reduction to prevent lasting chondral damage.

Early assessment and management
Usually a traumatic cause (multi-directional instability in frequent dislocations requires discussion with orthopaedics and is not covered here). Careful history, examination and documentation of neurovascular status of the limb, in particular the axillary nerve (regimental badge sensation). This should be re-assessed post manipulation. Early radiographs to confirm direction of dislocation.

Initial management consists of emergent closed reduction under under entanox and analgesia, but often requires conscious sedation. Arm should then be immobilised in a polysling, and XR to confirm relocation.

Imaging - True anteroposterior (AP), axillary lateral and/or scapula Y view. Reduced humeral head should lie between acromion and coracoid on lateral/scapula view.

Types
Direction Features Cause Examination Reduction techniques
Anterior Most Common >90% Usually traumatic - anterior force on arm when shoulder is abducted, eternally rotated Loss of shoulder contour - sulcus sign. Humeral head can be felt anteriorly.
Hippocratic.
Milch.
Stimson.

Kocher not advised due to complication of fracture
Posterior 50% missed in A&E 50% traumatic, but classically post seizure or electrocution Shoulder locked in internal rotation. XR may show lightbulb appearance. Gentle lateral traction to adducted arm.
Inferior Rare Associated with pectorals and rotator cuff tears, and glenoid fracture As for primary injury Management of primary injury

Superior Rare Associated with acrominon/clavicle fracture As for primary injury Management of primary injury

Associated injuries
Bankart lesion - avulsion of the anterior glenoid labrum with an anterior shoulder dislocation (reverse Bankart if poster labrum in posterior dislocation).
Hill Sachs defect - chondral impaction on posteriosuperior humeral head from contact with gleonoid rim. Can be large enough to lock shoulder, requiring open reduction. (Reverse Hill Sachs in posterior dislocation).
Rotator cuff tear - increases with age.
Greater or lesser tuberosity fracture - increases with age.
Humeral neck fracture - shoulder fracture dislocation. More common in high energy trauma and elderly. Should be discussed with orthopaedics prior to any attempted reduction.

Rotator Cuff Disease

Rotator cuff disease is a spectrum of conditions that ranges from subacromial impingement to rotator cuff tears and eventually to rotator cuff arthropathy (arthritis).

Anatomy
The rotator cuff is a group of four muscles that are important in shoulder movements, and maintenance of glenohumeral stability.
Muscle Scapular attachment Humeral attachment Action Innervation
Supraspinatus Supraspinatus fossa Superior facet of greater tuberosity Initiation of abduction of humerus Suprascapular nerve
Infraspinatus Infraspinatus fossa Posterior facet of greater tuberosity External rotation of humerus Suprascapular nerve
Teres Minor Lateral border Inferior facet of greater tuberosity External rotation of humerus Axillary Nerve
Subscapularis Subscapular fossa Lesser tuberosity Internal rotation of humerus Upper and lower subscapular nerve

The inferior rotator cuff muscles (infraspinatus, teres minor, and subscapularis) balance the superior pull of the deltoid. Injury/tear results in upward migration of the humeral head on the glenoid (can be seen on AP radiograph).
Likewise, the anterior muscles (subscapularis) are balanced with the posterior muscles (infraspinatus, teres minor).

Subacromial Impingement

The most common cause of shoulder pain, which results from impingement of the superior cuff on the undersurface of the acromion, and an inflammatory bursitis.
Associated with certain types of acromial morphology (Bigliani classification).
Presents as insidious pain which is exacerbated by overhead activities.

Rotator Cuff Tear

Often presents as an acute event on the background of chronic subacromial impingement in the older patient, but can present as an avulsion injury in younger patients.
Majority of tears are to the superior cuff (supraspinatus, infraspinatus, teres minor), though a tear to subscapularis is associated with subcoracoid impingement.
Tears present as pain and weakness when using the muscles in question.

Rotator Cuff Arthropathy

Defined as shoulder arthritis in the setting of rotator cuff dysfunction. Results from superior migration due to the loss of rotator cuff function and integrity. Unopposed deltoid pulls the humeral head superiorly.
Associated with massive chronic cuff tears.

Imaging

Plain radiographs
AP of the shoulder may show superior migration of the humerus with a cuff tear, and features of arthritis with arthropathy. Other causes of pain may also be identified (e.g. calcific tendonitis/fracture)
Outlet view is useful for defining the acromial morphology

USS
Allows dynamic imaging of the cuff, and is inexpensive. However, it is very user dependent.

MRI
Best imaging modality for cuff pathology.
Also allows imaging of the rest of the shoulder. When intra-articular pathology is suspected, can be combined with an arthrogram for improved sensitivity and specificity.

Treatment

Subacromial impingement
Physiotherapy, oral anti-inflammatory medication
Subacromial steroid injection can settle inflammation
Arthroscopic subacromial decompression by shaving away the undersurface of the acromion, more space is created for the rotator cuff. Cuff integrity is assessed also at time of surgery, and can be repaired if necessary.

Rotator cuff tear
When considering repair of a cuff tear, the age and activity of the patient, the nature of the tear (degenerative vs. acute traumatic), and the size and retraction of the tear should be considered when making a surgical plan.
Mild tears or tears in the elderly can be managed conservatively, as outlined above.
Moderate tears can be repaired arthroscopically. Massive or retracted tears will often require an open repair (occasionally with a tendon transfer). Subacromial decompression is performed at the same time to reduce impingement, symptoms and recurrence.

Calcific tendonitis
Calcific tendonitis involves calcific deposits within tendons anywhere in the body, but most commonly in the rotator cuff (specifically the supraspinatus tendon). When present in the shoulder, it is associated with subacromial impingement and pain.

Pathology
More common in women aged 30-60 years.
Association with diabetes and hypothyroidism

There are three stages of calcification
Formative phase characterized by calcific deposits
Resting phase deposit is stable, but presents with impingement problems
Resorptive phase phagocytic resorption. Most painful stage.

Presentation
Similar in presentation to subacromial impingement, with pain especially with over head activities. Atraumatic in nature.

Imaging
Plain radiographs show calcification of the rotator cuff, usually within 1.5cm of its insertion on the humerus. Supraspinatus outlet views can show level of impingment. Further imaging is rarely needed.

Treatment
Non-operative NSAIDS, steroid injection (controversial, but practiced) and physiotherapy. Approximately 75% will resolve by 6 months with conservative management.
Ultrasound guided or surgical needle barbotage can break down deposits and resolve symptoms. Occasionally surgical excision is required.

Adhesive capsulitis (Frozen Shoulder)
Pain and loss of movement of shoulder joint, which involves fibroplastic proliferation of capsular tissue, causing soft tissue scarring and contracture. Patients present with a painful and decreased arc of motion.
Associated with prolonged immobilization, previous surgery, thyroid disorders (AI) and diabetes
Classically three stages which can take up to two years to resolve:
Stage one the freezing and painful stage
Stage two the frozen and stiff stage
Stage three the thawing stage, where shoulder movement slowly improves

Imaging
Plain radiographs to exclude other causes of a painful shoulder
MRI arthrogram may show capsular contracture, and again may be used to exclude cuff pathology. However, often not performed as diagnosis is largely clinical.

Treatment
Non-operative NSAIDS, steroid injection and physiotherapy. Patience is required as condition can take up to 2 years to improve.
Operative MUA or arthroscopic adhesiolysis (release of adhesions) can expedite recovery, followed by intensive physiotherapy.

Glenohumeral Arthritis
Background
May be osteoarthritis (primary or secondary to cuff tear or trauma), rheumatoid arthritis, or as part of a spondyloarthropathy. Majority of those with RA will develop symptoms.
More common in the elderly
Presents like any other arthritis - pain at night and with movement

Imaging
AP and axillary radiographs will show features of arthritis.
CT/MRI is often useful to classify the shape of the glenoid and extent of bone loss when considering arthroplasty. MRI also essential to asses integrity of rotator cuff if considering shoulder replacement.

Treatment
Like all orthopaedics, start with simple measures:
NSAIDS, management of RA, physiotherapy, steroid injection.
Hemiarthroplasty can sometimes be considered if glenoid is in excellent condition or if patient has large comorbidity.
Arthroscopic debridement is useful if patient has isolated ACJ arthritis, but is rarely used for glenohumeral arthritis.
Total shoulder replacement is shown to produce superior outcome when compared to hemiarthroplasty in terms of pain relief, function and implant survival.
Total shoulder replacement can be anatomical (ball on humerus, with cup on glenoid), or reverse geometry (ball on glenoid, with cup on humerus). Anatomical TSR requires an in tact rotator cuff, so often reverse is preferable when the cuff if questionable in integrity.

62
Q

A 28 year old man complains of pain and weakness in the shoulder. He has recently been unwell with glandular fever from which he is fully recovered. On examination there is some evidence of muscle wasting and a degree of winging of the scapula. Power during active movements is impaired.

A.	Impingement syndrome
B.	Rotator cuff tear
C.	Adhesive capsulitis
D.	Calcific tendonitis
E.	Biceps tendon rupture
F.	Parsonage - Turner syndrome
G.	Labral tear
A

Parsonage - Turner syndrome

This is a peripheral neuropathy that may complicate viral illnesses and usually resolves spontaneously.

Shoulder fractures and dislocations
Fractures
Proximal humerus
Background
Third most common fragility fracture in the elderly.
Results from low energy fall in predominantly elderly females, or from high energy trauma in young males.
Can be associated with nerve injury (commonly axillary), and fracture-dislocation of the humeral head. Detailed neurological assessment is essential for all upper limb injuries.

Anatomy
Osteology
Consists of articular head, greater tuberosity, lesser tuberosity, metaphysis and diaphysis. Between the articular head and the tuberosities is the anatomical neck (previous physis). Between the tuberosities and the metaphysis is the surgical neck.
The supraspinatus, infraspinatus and teres minor muscles attach to the greater tuberosity. The subscapularis muscle attaches to the lesser tuberosity.

Vascular Supply
Humeral head is supplied by the anterior and posterior humeral circumflex arteries. Anatomical neck fractures are at greatest risk of osteonecrosis.

Imaging
Imaging aims to both delineate the fracture pattern, and confirm/exlude the presence of an associated dislocation.
Radiographs - True anteroposterior (AP), axillary lateral and/or scapula Y view.
CT - indicated to better define intra-articular involvement and to aid pre-operative planning. MRI is not useful for fracture imaging.

Classification
Description of the fracture is often more useful than classification. Particular attention should be paid to humeral alignment, fracture displacement, and greater tuberosity position (rotator cuff will pull the GT supero-posterioly, which can cause impingement problems with malunion).
- Neer Classification: Most commonly used. Describes fracture as 2,3,or 4 part depending upon the number main fragments. Also comments on the degree of displacement. Fragments:
-greater tuberosity
-lesser tuberosity
- articular surface
- shaft
Displacement: >1cm or angulation >45 degrees.

Treatment
The vast majority of proximal humeral fractures are minimally displaced, and therefore can be managed conservatively. This involves immobilisation in a polysling, and progressive mobilisation. Pendular exercise can commence at 14 days, and active abduction from 4-6 weeks.

Irreducible fracture dislocation is an indication for operative management. Other indications include large displacement, younger patient, head splitting (intra-articular fractures). However, the recent PROFHER trial (1) has suggested no benefit to operative intervention on patient outcome (it must be applied cautiously as majority of patients were elderly with extraarticular fractures). Options available for surgical management include:

ORIF Most commonly used. Plate and screw fixation. Can reconstruct complex fractures.
Intramedullary nail Suitable for extra-articular configuration, predominantly surgical neck +/- GT fractures.

Hemiarthroplasty Used for un-reconstructable fractures in the older patient who has good glenoid quality.
Total shoulder arthroplasty Unconstructable fractures where high functioning shoulder is required (hemiarthroplasty will cause glenoid erosion)
Reverse shoulder arthroplasty Total shoulder arthroplasty that provides better functional outcome than conventional total shoulder replacement.

Scapula
Background
Uncommon fractures usually associated with high energy trauma. Most commonly involve scapula body or spine (50%), glenoid fossa and glenoid neck. Important to exclude associated life threatening injury.

Imaging
Plain radiographs should include true anteroposterior (AP), axillary lateral and/or scapula Y view. CT scanning is useful for defining intra-articular involvement, displacement and for three dimensional reconstruction.

Classification
Based on the location of the fracture (coracoid, acromion, glenoid neck, glenoid fossa, scapula body). Beware of ipsilateral glenoid neck and clavicle fracture -floating shoulder - where limb is effectively dissociated from axial skeleton.

Treatment
The vast majority of scapula fractures are amenable to conservative management, consisting of sling immobilisation for two weeks followed by early rehabilitation. Floating shoulder will usually require fixation, and consideration of surgery should also be given to intra-articular and displaced/angulated glenoid fractures.

Dislocations

Types
Dislocations around the shoulder joint include glenohumeral dislocation, acromioclavicular joint disruption and sternoclavicular dislocation. Only glenohumeral dislocation will be covered here.

Glenohumeral dislocation
Diagnosis, classification and management are covered here.

Background
Shoulder dislocation is commonly seen in A&E. It has a high recurrence rate that is as high as 80% in teenagers. Initial management requires emergent reduction to prevent lasting chondral damage.

Early assessment and management
Usually a traumatic cause (multi-directional instability in frequent dislocations requires discussion with orthopaedics and is not covered here). Careful history, examination and documentation of neurovascular status of the limb, in particular the axillary nerve (regimental badge sensation). This should be re-assessed post manipulation. Early radiographs to confirm direction of dislocation.

Initial management consists of emergent closed reduction under under entanox and analgesia, but often requires conscious sedation. Arm should then be immobilised in a polysling, and XR to confirm relocation.

Imaging - True anteroposterior (AP), axillary lateral and/or scapula Y view. Reduced humeral head should lie between acromion and coracoid on lateral/scapula view.

Types
Direction Features Cause Examination Reduction techniques
Anterior Most Common >90% Usually traumatic - anterior force on arm when shoulder is abducted, eternally rotated Loss of shoulder contour - sulcus sign. Humeral head can be felt anteriorly.
Hippocratic.
Milch.
Stimson.

Kocher not advised due to complication of fracture
Posterior 50% missed in A&E 50% traumatic, but classically post seizure or electrocution Shoulder locked in internal rotation. XR may show lightbulb appearance. Gentle lateral traction to adducted arm.
Inferior Rare Associated with pectorals and rotator cuff tears, and glenoid fracture As for primary injury Management of primary injury

Superior Rare Associated with acrominon/clavicle fracture As for primary injury Management of primary injury

Associated injuries
Bankart lesion - avulsion of the anterior glenoid labrum with an anterior shoulder dislocation (reverse Bankart if poster labrum in posterior dislocation).
Hill Sachs defect - chondral impaction on posteriosuperior humeral head from contact with gleonoid rim. Can be large enough to lock shoulder, requiring open reduction. (Reverse Hill Sachs in posterior dislocation).
Rotator cuff tear - increases with age.
Greater or lesser tuberosity fracture - increases with age.
Humeral neck fracture - shoulder fracture dislocation. More common in high energy trauma and elderly. Should be discussed with orthopaedics prior to any attempted reduction.

Rotator Cuff Disease

Rotator cuff disease is a spectrum of conditions that ranges from subacromial impingement to rotator cuff tears and eventually to rotator cuff arthropathy (arthritis).

Anatomy
The rotator cuff is a group of four muscles that are important in shoulder movements, and maintenance of glenohumeral stability.
Muscle Scapular attachment Humeral attachment Action Innervation
Supraspinatus Supraspinatus fossa Superior facet of greater tuberosity Initiation of abduction of humerus Suprascapular nerve
Infraspinatus Infraspinatus fossa Posterior facet of greater tuberosity External rotation of humerus Suprascapular nerve
Teres Minor Lateral border Inferior facet of greater tuberosity External rotation of humerus Axillary Nerve
Subscapularis Subscapular fossa Lesser tuberosity Internal rotation of humerus Upper and lower subscapular nerve

The inferior rotator cuff muscles (infraspinatus, teres minor, and subscapularis) balance the superior pull of the deltoid. Injury/tear results in upward migration of the humeral head on the glenoid (can be seen on AP radiograph).
Likewise, the anterior muscles (subscapularis) are balanced with the posterior muscles (infraspinatus, teres minor).

Subacromial Impingement

The most common cause of shoulder pain, which results from impingement of the superior cuff on the undersurface of the acromion, and an inflammatory bursitis.
Associated with certain types of acromial morphology (Bigliani classification).
Presents as insidious pain which is exacerbated by overhead activities.

Rotator Cuff Tear

Often presents as an acute event on the background of chronic subacromial impingement in the older patient, but can present as an avulsion injury in younger patients.
Majority of tears are to the superior cuff (supraspinatus, infraspinatus, teres minor), though a tear to subscapularis is associated with subcoracoid impingement.
Tears present as pain and weakness when using the muscles in question.

Rotator Cuff Arthropathy

Defined as shoulder arthritis in the setting of rotator cuff dysfunction. Results from superior migration due to the loss of rotator cuff function and integrity. Unopposed deltoid pulls the humeral head superiorly.
Associated with massive chronic cuff tears.

Imaging

Plain radiographs
AP of the shoulder may show superior migration of the humerus with a cuff tear, and features of arthritis with arthropathy. Other causes of pain may also be identified (e.g. calcific tendonitis/fracture)
Outlet view is useful for defining the acromial morphology

USS
Allows dynamic imaging of the cuff, and is inexpensive. However, it is very user dependent.

MRI
Best imaging modality for cuff pathology.
Also allows imaging of the rest of the shoulder. When intra-articular pathology is suspected, can be combined with an arthrogram for improved sensitivity and specificity.

Treatment

Subacromial impingement
Physiotherapy, oral anti-inflammatory medication
Subacromial steroid injection can settle inflammation
Arthroscopic subacromial decompression by shaving away the undersurface of the acromion, more space is created for the rotator cuff. Cuff integrity is assessed also at time of surgery, and can be repaired if necessary.

Rotator cuff tear
When considering repair of a cuff tear, the age and activity of the patient, the nature of the tear (degenerative vs. acute traumatic), and the size and retraction of the tear should be considered when making a surgical plan.
Mild tears or tears in the elderly can be managed conservatively, as outlined above.
Moderate tears can be repaired arthroscopically. Massive or retracted tears will often require an open repair (occasionally with a tendon transfer). Subacromial decompression is performed at the same time to reduce impingement, symptoms and recurrence.

Calcific tendonitis
Calcific tendonitis involves calcific deposits within tendons anywhere in the body, but most commonly in the rotator cuff (specifically the supraspinatus tendon). When present in the shoulder, it is associated with subacromial impingement and pain.

Pathology
More common in women aged 30-60 years.
Association with diabetes and hypothyroidism

There are three stages of calcification
Formative phase characterized by calcific deposits
Resting phase deposit is stable, but presents with impingement problems
Resorptive phase phagocytic resorption. Most painful stage.

Presentation
Similar in presentation to subacromial impingement, with pain especially with over head activities. Atraumatic in nature.

Imaging
Plain radiographs show calcification of the rotator cuff, usually within 1.5cm of its insertion on the humerus. Supraspinatus outlet views can show level of impingment. Further imaging is rarely needed.

Treatment
Non-operative NSAIDS, steroid injection (controversial, but practiced) and physiotherapy. Approximately 75% will resolve by 6 months with conservative management.
Ultrasound guided or surgical needle barbotage can break down deposits and resolve symptoms. Occasionally surgical excision is required.

Adhesive capsulitis (Frozen Shoulder)
Pain and loss of movement of shoulder joint, which involves fibroplastic proliferation of capsular tissue, causing soft tissue scarring and contracture. Patients present with a painful and decreased arc of motion.
Associated with prolonged immobilization, previous surgery, thyroid disorders (AI) and diabetes
Classically three stages which can take up to two years to resolve:
Stage one the freezing and painful stage
Stage two the frozen and stiff stage
Stage three the thawing stage, where shoulder movement slowly improves

Imaging
Plain radiographs to exclude other causes of a painful shoulder
MRI arthrogram may show capsular contracture, and again may be used to exclude cuff pathology. However, often not performed as diagnosis is largely clinical.

Treatment
Non-operative NSAIDS, steroid injection and physiotherapy. Patience is required as condition can take up to 2 years to improve.
Operative MUA or arthroscopic adhesiolysis (release of adhesions) can expedite recovery, followed by intensive physiotherapy.

Glenohumeral Arthritis
Background
May be osteoarthritis (primary or secondary to cuff tear or trauma), rheumatoid arthritis, or as part of a spondyloarthropathy. Majority of those with RA will develop symptoms.
More common in the elderly
Presents like any other arthritis - pain at night and with movement

Imaging
AP and axillary radiographs will show features of arthritis.
CT/MRI is often useful to classify the shape of the glenoid and extent of bone loss when considering arthroplasty. MRI also essential to asses integrity of rotator cuff if considering shoulder replacement.

Treatment
Like all orthopaedics, start with simple measures:
NSAIDS, management of RA, physiotherapy, steroid injection.
Hemiarthroplasty can sometimes be considered if glenoid is in excellent condition or if patient has large comorbidity.
Arthroscopic debridement is useful if patient has isolated ACJ arthritis, but is rarely used for glenohumeral arthritis.
Total shoulder replacement is shown to produce superior outcome when compared to hemiarthroplasty in terms of pain relief, function and implant survival.
Total shoulder replacement can be anatomical (ball on humerus, with cup on glenoid), or reverse geometry (ball on glenoid, with cup on humerus). Anatomical TSR requires an in tact rotator cuff, so often reverse is preferable when the cuff if questionable in integrity.

63
Q

A 38 year old man is playing football when he slips over during a tackle. His knee is painful immediately following the fall. Several hours later he notices that the knee has become swollen. Following a course of non steroidal anti inflammatory drugs and rest the situation improves. However, complains of recurrent pain. On assessment in clinic you notice that it is impossible to fully extend the knee, although the patient is able to do so when asked.

A.	Anterior cruciate ligament rupture
B.	Posterior cruciate ligament rupture
C.	Medial collateral ligament tear
D.	Lateral collateral ligament tear
E.	Torn meniscus
F.	Chondromalacia patellae
G.	Dislocated patella
H.	Fractured patella
I.	Tibial plateau fracture
A

Torn meniscus

Twisting sporting injuries followed by delayed onset of knee swelling and locking are strongly suggestive of a menisceal tear. Arthroscopic menisectomy is the usual treatment.

Ruptured anterior cruciate ligament
Sport injury
Mechanism: high twisting force applied to a bent knee
Typically presents with: loud crack, pain and RAPID joint swelling (haemoarthrosis)
Poor healing
Management: intense physiotherapy or surgery
Ruptured posterior cruciate ligament
Mechanism: hyperextension injuries
Tibia lies back on the femur
Paradoxical anterior draw test
Rupture of medial collateral ligament
Mechanism: leg forced into valgus via force outside the leg
Knee unstable when put into valgus position
Menisceal tear
Rotational sporting injuries
Delayed knee swelling
Joint locking (Patient may develop skills to “unlock” the knee
Recurrent episodes of pain and effusions are common, often following minor trauma
Chondromalacia patellae
Teenage girls, following an injury to knee e.g. Dislocation patella
Typical history of pain on going downstairs or at rest
Tenderness, quadriceps wasting
Dislocation of the patella
Most commonly occurs as a traumatic primary event, either through direct trauma or through severe contraction of quadriceps with knee stretched in valgus and external rotation
Genu valgum, tibial torsion and high riding patella are risk factors
Skyline x-ray views of patella are required, although displaced patella may be clinically obvious
An osteochondral fracture is present in 5%
The condition has a 20% recurrence rate
Fractured patella
2 types:
i. Direct blow to patella causing undisplaced fragments
ii. Avulsion fracture
Tibial plateau fracture
Occur in the elderly (or following significant trauma in young)
Mechanism: knee forced into valgus or varus, but the knee fractures before the ligaments rupture
Varus injury affects medial plateau and if valgus injury, lateral plateau depressed fracture occurs
Classified using the Schatzker system (see below)

Schatzker Classification system for tibial plateau fractures
Type Anatomical description Features
1 Vertical split of lateral condyle Fracture through dense bone, usually in the young. It may be virtually undisplaced, or the condylar fragment may be pushed inferiorly and tilted
2 Vertical split of the lateral condyle combined with an adjacent load bearing part of the condyle The wedge fragment (which may be of variable size), is displaced laterally; the joint is widened. Untreated, a valgus deformity may develop
3 Depression of the articular surface with intact condylar rim The split does not extend to the edge of the plateau. Depressed fragments may be firmly embedded in subchondral bone, the joint is stable
4 Fragment of the medial tibial condyle Two injuries are seen in this category; (1) a depressed fracture of osteoporotic bone in the elderly. (2) a high energy fracture resulting in a condylar split that runs from the intercondylar eminence to the medial cortex. Associated ligamentous injury may be severe
5 Fracture of both condyles Both condyles fractured but the column of the metaphysis remains in continuity with the tibial shaft
6 Combined condylar and subcondylar fractures High energy fracture with marked comminution

64
Q

A 34 year old woman is a passenger in a car during an accident. Her knee hits the dashboard. On examination the tibia looks posterior compared to the non injured knee.

A.	Anterior cruciate ligament rupture
B.	Posterior cruciate ligament rupture
C.	Medial collateral ligament tear
D.	Lateral collateral ligament tear
E.	Torn meniscus
F.	Chondromalacia patellae
G.	Dislocated patella
H.	Fractured patella
I.	Tibial plateau fracture
A

Posterior cruciate ligament rupture

In ruptured posterior cruciate ligament the tibia lies back on the femur and can be drawn forward during a paradoxical draw test.

Ruptured anterior cruciate ligament
Sport injury
Mechanism: high twisting force applied to a bent knee
Typically presents with: loud crack, pain and RAPID joint swelling (haemoarthrosis)
Poor healing
Management: intense physiotherapy or surgery
Ruptured posterior cruciate ligament
Mechanism: hyperextension injuries
Tibia lies back on the femur
Paradoxical anterior draw test
Rupture of medial collateral ligament
Mechanism: leg forced into valgus via force outside the leg
Knee unstable when put into valgus position
Menisceal tear
Rotational sporting injuries
Delayed knee swelling
Joint locking (Patient may develop skills to “unlock” the knee
Recurrent episodes of pain and effusions are common, often following minor trauma
Chondromalacia patellae
Teenage girls, following an injury to knee e.g. Dislocation patella
Typical history of pain on going downstairs or at rest
Tenderness, quadriceps wasting
Dislocation of the patella
Most commonly occurs as a traumatic primary event, either through direct trauma or through severe contraction of quadriceps with knee stretched in valgus and external rotation
Genu valgum, tibial torsion and high riding patella are risk factors
Skyline x-ray views of patella are required, although displaced patella may be clinically obvious
An osteochondral fracture is present in 5%
The condition has a 20% recurrence rate
Fractured patella
2 types:
i. Direct blow to patella causing undisplaced fragments
ii. Avulsion fracture
Tibial plateau fracture
Occur in the elderly (or following significant trauma in young)
Mechanism: knee forced into valgus or varus, but the knee fractures before the ligaments rupture
Varus injury affects medial plateau and if valgus injury, lateral plateau depressed fracture occurs
Classified using the Schatzker system (see below)

Schatzker Classification system for tibial plateau fractures
Type Anatomical description Features
1 Vertical split of lateral condyle Fracture through dense bone, usually in the young. It may be virtually undisplaced, or the condylar fragment may be pushed inferiorly and tilted
2 Vertical split of the lateral condyle combined with an adjacent load bearing part of the condyle The wedge fragment (which may be of variable size), is displaced laterally; the joint is widened. Untreated, a valgus deformity may develop
3 Depression of the articular surface with intact condylar rim The split does not extend to the edge of the plateau. Depressed fragments may be firmly embedded in subchondral bone, the joint is stable
4 Fragment of the medial tibial condyle Two injuries are seen in this category; (1) a depressed fracture of osteoporotic bone in the elderly. (2) a high energy fracture resulting in a condylar split that runs from the intercondylar eminence to the medial cortex. Associated ligamentous injury may be severe
5 Fracture of both condyles Both condyles fractured but the column of the metaphysis remains in continuity with the tibial shaft
6 Combined condylar and subcondylar fractures High energy fracture with marked comminution

65
Q

A 28 year old professional footballer is admitted to the emergency department. During a tackle he is twisted with his knee flexed. He hears a loud crack and his knee rapidly becomes swollen.

A.	Anterior cruciate ligament rupture
B.	Posterior cruciate ligament rupture
C.	Medial collateral ligament tear
D.	Lateral collateral ligament tear
E.	Torn meniscus
F.	Chondromalacia patellae
G.	Dislocated patella
H.	Fractured patella
I.	Tibial plateau fracture
A

Anterior cruciate ligament rupture

This is common in footballers as the football boot studs stick to the ground and high twisting force is applied to a flexed knee. Rapid joint swelling also supports the diagnosis.

Ruptured anterior cruciate ligament
Sport injury
Mechanism: high twisting force applied to a bent knee
Typically presents with: loud crack, pain and RAPID joint swelling (haemoarthrosis)
Poor healing
Management: intense physiotherapy or surgery
Ruptured posterior cruciate ligament
Mechanism: hyperextension injuries
Tibia lies back on the femur
Paradoxical anterior draw test
Rupture of medial collateral ligament
Mechanism: leg forced into valgus via force outside the leg
Knee unstable when put into valgus position
Menisceal tear
Rotational sporting injuries
Delayed knee swelling
Joint locking (Patient may develop skills to “unlock” the knee
Recurrent episodes of pain and effusions are common, often following minor trauma
Chondromalacia patellae
Teenage girls, following an injury to knee e.g. Dislocation patella
Typical history of pain on going downstairs or at rest
Tenderness, quadriceps wasting
Dislocation of the patella
Most commonly occurs as a traumatic primary event, either through direct trauma or through severe contraction of quadriceps with knee stretched in valgus and external rotation
Genu valgum, tibial torsion and high riding patella are risk factors
Skyline x-ray views of patella are required, although displaced patella may be clinically obvious
An osteochondral fracture is present in 5%
The condition has a 20% recurrence rate
Fractured patella
2 types:
i. Direct blow to patella causing undisplaced fragments
ii. Avulsion fracture
Tibial plateau fracture
Occur in the elderly (or following significant trauma in young)
Mechanism: knee forced into valgus or varus, but the knee fractures before the ligaments rupture
Varus injury affects medial plateau and if valgus injury, lateral plateau depressed fracture occurs
Classified using the Schatzker system (see below)

Schatzker Classification system for tibial plateau fractures
Type Anatomical description Features
1 Vertical split of lateral condyle Fracture through dense bone, usually in the young. It may be virtually undisplaced, or the condylar fragment may be pushed inferiorly and tilted
2 Vertical split of the lateral condyle combined with an adjacent load bearing part of the condyle The wedge fragment (which may be of variable size), is displaced laterally; the joint is widened. Untreated, a valgus deformity may develop
3 Depression of the articular surface with intact condylar rim The split does not extend to the edge of the plateau. Depressed fragments may be firmly embedded in subchondral bone, the joint is stable
4 Fragment of the medial tibial condyle Two injuries are seen in this category; (1) a depressed fracture of osteoporotic bone in the elderly. (2) a high energy fracture resulting in a condylar split that runs from the intercondylar eminence to the medial cortex. Associated ligamentous injury may be severe
5 Fracture of both condyles Both condyles fractured but the column of the metaphysis remains in continuity with the tibial shaft
6 Combined condylar and subcondylar fractures High energy fracture with marked comminution

66
Q

A 10 year old boy is referred to the orthopaedic clinic with symptoms of right knee pain. He has suffered pain for the past 3 months and the pain typically lasts for several hours. On examination he walks with an antalgic gait and has apparent right leg shortening. The right knee is normal but the right hip reveals pain on internal and external rotation. Imaging shows flattening of the femoral head. Which of the following is the most likely underlying diagnosis?

	Osteogenesis imperfecta
	Child abuse
	Osteosarcoma
	Osteopetrosis
	Perthes disease
A

This is a typical description of Perthes disease. Management involves keeping the femoral head in the acetabulum by braces, casts or surgery.

Perthes disease
Idiopathic avascular necrosis of the femoral epiphysis of the femoral head
Impaired blood supply to femoral head, causing bone infarction. New vessels develop and ossification occurs. The bone either heals or a subchondral fracture occurs.

Clinical features
Males 4x's greater than females
Age between 2-12 years (the younger the age of onset, the better the prognosis)
Limp
Hip pain
Bilateral in 20%

Diagnosis
Plain x-ray, Technetium bone scan or magnetic resonance imaging if normal x-ray and symptoms persist.

Catterall staging
Stage Features
Stage 1 Clinical and histological features only
Stage 2 Sclerosis with or without cystic changes and preservation of the articular surface
Stage 3 Loss of structural integrity of the femoral head
Stage 4 Loss of acetabular integrity

Management
To keep the femoral head within the acetabulum: cast, braces
If less than 6 years: observation
Older: surgical management with moderate results
Operate on severe deformities

Prognosis
Most cases will resolve with conservative management. Early diagnosis improves outcomes.

67
Q

Which statement relating to talipes equinovarus is untrue?

It has an annual incidence of around 1 in 1000 in the UK
The muscles involved in the disorder are intrinsically abnormal
The cuboid is classically displaced laterally
All cases should be treated with an Ilizarov frame initially unless there is minor deformity
The talocalcaneal angle is typically less than 20 degrees in club foot
A

All cases should be treated with an Ilizarov frame initially unless there is minor deformity - FALSE

In most cases of Club Foot conservative measures should be tried first. The Ponsetti method is a popular approach. Severe cases may benefit from Ilizarov frame re-aligment.
Congenital talipes equinovarus.
Features:
Equinus of the hindfoot.
Adduction and varus of the midfoot.
High arch.

Most cases in developing countries. Incidence in UK is 1 per 1000 live births. It is more common in males and is bilateral in 50% cases. There is a strong familial link(1). It may also be associated with other developmental disorders such as Down’s syndrome.

Key anatomical deformities (2):
Adducted and inverted calcaneus
Wedge shaped distal calcaneal articular surface
Severe Tibio-talar plantar flexion.
Medial Talar neck inclination
Displacement of the navicular bone (medially)
Wedge shaped head of talus
Displacement of the cuboid (medially)

Management
Conservative first, the Ponseti method is best described and gives comparable results to surgery. It consists of serial casting to mold the foot into correct shape. Following casting around 90% will require a Achilles tenotomy. This is then followed by a phase of walking braces to maintain the correction.

Surgical correction is reserved for those cases that fail to respond to conservative measures. The procedures involve multiple tenotomies and lengthening procedures. In patients who fail to respond surgically an Ilizarov frame reconstruction may be attempted and gives good results.

68
Q

Which of the following is least likely to impair bone fracture healing?

	Radiotherapy
	Osteoporosis
	Administration of non steroidal anti inflammatory drugs
	Preservation of periosteum
	Presence of osteomyelitic sequestra
A

Periosteal preservation helps fractures to heal.

Bone fracture

  • Bleeding vessels in the bone and periosteum
  • Clot and haematoma formation
  • The clot organises over a week (improved structure and collagen)
  • The periosteum contains osteoblasts which produce new bone
  • Mesenchymal cells produce cartilage (fibrocartilage and hyaline cartilage) in the soft tissue around the fracture
  • Connective tissue + hyaline cartilage = callus
  • As the new bone approaches the new cartilage, endochondral ossification occurs to bridge the gap
  • Trabecular bone forms
  • Trabecular bone is resorbed by osteoclasts and replaced with compact bone

Factors affecting fracture healing
Age
Malnutrition
Bone disorders: osteoporosis
Systemic disorders: diabetes, Marfan’s syndrome and Ehlers-Danlos syndrome cause abnormal musculoskeletal healing.
Drugs: steroids, non steroidal anti inflammatory agents.
Type of bone: Cancellous (spongy) bone fractures are usually more stable, involve greater surface areas, and have a better blood supply than cortical (compact) bone fractures.
Degree of Trauma: The more extensive the injury to bone and surrounding soft tissue, the poorer the outcome.
Vascular Injury: Especially the femoral head, talus, and scaphoid bones.
Degree of Immobilization
Intra-articular Fractures: These fractures communicate with synovial fluid, which contains collagenases that retard bone healing.
Separation of Bone Ends: Normal apposition of fracture fragments is needed for union to occur. Inadequate reduction, excessive traction, or interposition of soft tissue will prevent healing.
Infection

69
Q

An obese 14 year old boy presents with difficulty running and mild knee and hip pain. There is no antecedent history of trauma. On examination internal rotation is restricted but the knee is normal with full range of passive movement possible and no evidence of effusions. Both the C-reactive protein and white cell count are normal.

 A.	Perthes disease
B.	Developmental dysplasia of the hip
C.	Osteoarthritis
D.	Slipped upper femoral epiphysis
E.	Septic arthritis
F.	Rheumatoid arthritis
G.	Intra capsular fracture of the femoral neck
H.	Extra capsular fracture of the femoral neck
A

Slipped upper femoral epiphysis

Slipped upper femoral epiphysis is the commonest adolescent hip disorder. It occurs most commonly in obese males. It may often present as knee pain which is usually referred from the ipsilateral hip. The knee itself is normal. The hip often limits internal rotation. The diagnosis is easily missed. X-rays will show displacement of the femoral epiphysis and the degree of its displacement may be calculated using the Southwick angle. Treatment is directed at preventing further slippage which may result in avascular necrosis of the femoral head.

Developmental dysplasia of the hip Usually diagnosed in infancy by screening tests. May be bilateral, when disease is unilateral there may be leg length inequality. As disease progresses child may limp and then early onset arthritis. More common in extended breech babies. Splints and harnesses or traction. In later years osteotomy and hip realignment procedures may be needed. In arthritis a joint replacement may be needed. However, this is best deferred if possible as it will almost certainly require revision Initially no obvious change on plain films and USS gives best resolution until 3 months of age. On plain films Shentons line should form a smooth arc
Perthes Disease Hip pain (may be referred to the knee) usually occurring between 5 and 12 years of age. Bilateral disease in 20%. Remove pressure from joint to allow normal development. Physiotherapy. Usually self-limiting if diagnosed and treated promptly. X-rays will show flattened femoral head. Eventually in untreated cases the femoral head will fragment.
Slipped upper femoral epiphysis Typically seen in obese male adolescents. Pain is often referred to the knee. Limitation to internal rotation is usually seen. Knee pain is usually present 2 months prior to hip slipping. Bilateral in 20%. Bed rest and non-weight bearing. Aim to avoid avascular necrosis. If severe slippage or risk of it occurring then percutaneous pinning of the hip may be required. X-rays will show the femoral head displaced and falling inferolaterally (like a melting ice cream cone) The Southwick angle gives indication of disease severity

70
Q

A 6 year old boy presents with pain in the hip it is present on activity and has been worsening over the past few weeks. There is no history of trauma. He was born by normal vaginal delivery at 38 weeks gestation On examination he has an antalgic gait and limitation of active and passive movement of the hip joint in all directions. C-reactive protein is mildly elevated at 10 but the white cell count is normal.

A.	Perthes disease
B.	Developmental dysplasia of the hip
C.	Osteoarthritis
D.	Slipped upper femoral epiphysis
E.	Septic arthritis
F.	Rheumatoid arthritis
G.	Intra capsular fracture of the femoral neck
H.	Extra capsular fracture of the femoral neck
A

Perthes disease

This is a typical presentation for Perthes disease. X-ray may show flattening of the femoral head or fragmentation in more advanced cases.
Early plain x-ray changes in Perthes Disease:
Widening of the joint space.
Sub chondral linear lucency

Developmental dysplasia of the hip Usually diagnosed in infancy by screening tests. May be bilateral, when disease is unilateral there may be leg length inequality. As disease progresses child may limp and then early onset arthritis. More common in extended breech babies. Splints and harnesses or traction. In later years osteotomy and hip realignment procedures may be needed. In arthritis a joint replacement may be needed. However, this is best deferred if possible as it will almost certainly require revision Initially no obvious change on plain films and USS gives best resolution until 3 months of age. On plain films Shentons line should form a smooth arc
Perthes Disease Hip pain (may be referred to the knee) usually occurring between 5 and 12 years of age. Bilateral disease in 20%. Remove pressure from joint to allow normal development. Physiotherapy. Usually self-limiting if diagnosed and treated promptly. X-rays will show flattened femoral head. Eventually in untreated cases the femoral head will fragment.
Slipped upper femoral epiphysis Typically seen in obese male adolescents. Pain is often referred to the knee. Limitation to internal rotation is usually seen. Knee pain is usually present 2 months prior to hip slipping. Bilateral in 20%. Bed rest and non-weight bearing. Aim to avoid avascular necrosis. If severe slippage or risk of it occurring then percutaneous pinning of the hip may be required. X-rays will show the femoral head displaced and falling inferolaterally (like a melting ice cream cone) The Southwick angle gives indication of disease severity

71
Q

A 30 year old man presents with severe pain in the left hip it has been present on and off for many years. He was born at 39 weeks gestation by emergency caesarean section after a long obstructed breech delivery. He was slow to walk and as a child was noted to have an antalgic gait. He was a frequent attender at the primary care centre and the pains dismissed as growing pains. X-rays show almost complete destruction of the femoral head and a narrow acetabulum.

 A.	Perthes disease
B.	Developmental dysplasia of the hip
C.	Osteoarthritis
D.	Slipped upper femoral epiphysis
E.	Septic arthritis
F.	Rheumatoid arthritis
G.	Intra capsular fracture of the femoral neck
H.	Extra capsular fracture of the femoral neck
A

Developmental dysplasia of the hip

Developmental dysplasia of the hip. Usually diagnosed by Barlow and Ortolani tests in early childhood. Most Breech deliveries are also routinely subjected to USS of the hip joint. At this young age an arthrodesis may be preferable to hip replacement.

Developmental dysplasia of the hip Usually diagnosed in infancy by screening tests. May be bilateral, when disease is unilateral there may be leg length inequality. As disease progresses child may limp and then early onset arthritis. More common in extended breech babies. Splints and harnesses or traction. In later years osteotomy and hip realignment procedures may be needed. In arthritis a joint replacement may be needed. However, this is best deferred if possible as it will almost certainly require revision Initially no obvious change on plain films and USS gives best resolution until 3 months of age. On plain films Shentons line should form a smooth arc
Perthes Disease Hip pain (may be referred to the knee) usually occurring between 5 and 12 years of age. Bilateral disease in 20%. Remove pressure from joint to allow normal development. Physiotherapy. Usually self-limiting if diagnosed and treated promptly. X-rays will show flattened femoral head. Eventually in untreated cases the femoral head will fragment.
Slipped upper femoral epiphysis Typically seen in obese male adolescents. Pain is often referred to the knee. Limitation to internal rotation is usually seen. Knee pain is usually present 2 months prior to hip slipping. Bilateral in 20%. Bed rest and non-weight bearing. Aim to avoid avascular necrosis. If severe slippage or risk of it occurring then percutaneous pinning of the hip may be required. X-rays will show the femoral head displaced and falling inferolaterally (like a melting ice cream cone) The Southwick angle gives indication of disease severity

72
Q

A toddler aged 3 years presents to the Emergency Department with swelling of his leg and is found to have a spiral fracture of the tibia. His mother reports that he had tripped and fallen the previous day but she had not noticed any sign of injury at the time. She is a single parent with little family support. The child is not on the child protection register.

 A.	Non accidental injury
B.	Accidental fracture
C.	Rickets
D.	Metabolic bone disease of prematurity
E.	Hypophosphataemic rickets
F.	Osteopetrosis
G.	Osteogenesis imperfecta
H.	Hypoparathyroidism
I.	Osteoporosis
A

Non accidental injury

Delayed presentation is unusual and should raise concern. In addition spiral fractures are usually the result of rotational injury which is not compatible with the mechanism proposed by the parent.

Paediatric fracture types
Type Injury pattern
Complete fracture Both sides of cortex are breached
Toddlers fracture Oblique tibial fracture in infants
Plastic deformity Stress on bone resulting in deformity without cortical disruption
Greenstick fracture Unilateral cortical breach only
Buckle fracture Incomplete cortical disruption resulting in periosteal haematoma only

Growth plate fractures
In paediatric practice fractures may also involve the growth plate and these injuries are classified according to the Salter- Harris system (given below):

Type Injury pattern
I Fracture through the physis only (x-ray often normal)
II Fracture through the physis and metaphysis
III Fracture through the physis and epiphyisis to include the joint
IV Fracture involving the physis, metaphysis and epiphysis
V Crush injury involving the physis (x-ray may resemble type I, and appear normal)

As a general rule it is safer to assume that growth plate tenderness is indicative of an underlying fracture even if the x-ray appears normal. Injuries of Types III, IV and V will usually require surgery. Type V injuries are often associated with disruption to growth.

Non accidental injury
Delayed presentation
Delay in attaining milestones
Lack of concordance between proposed and actual mechanism of injury
Multiple injuries
Injuries at sites not commonly exposed to trauma
Children on the at risk register

Pathological fractures
Genetic conditions, such as osteogenesis imperfecta, may cause pathological fractures.

Osteogenesis imperfecta
Defective osteoid formation due to congenital inability to produce adequate intercellular substances like osteoid, collagen and dentine.
Failure of maturation of collagen in all the connective tissues.
Radiology may show translucent bones, multiple fractures, particularly of the long bones, wormian bones (irregular patches of ossification) and a trefoil pelvis.

Subtypes
Type I The collagen is normal quality but insufficient quantity.
Type II- Poor collagen quantity and quality.
Type III- Collagen poorly formed. Normal quantity.
Type IV- Sufficient collagen quantity but poor quality.

Osteopetrosis
Bones become harder and more dense.
Autosomal recessive condition.
It is commonest in young adults.
Radiology reveals a lack of differentiation between the cortex and the medulla described as marble bone.
73
Q

A 5 month baby boy presents with swelling of his right arm and is found to have a spiral fracture of the humerus. He had been in the care of her mother’s boyfriend who reported that he had nearly dropped him that day when reaching for his bottle and had inadvertently pulled on his arm to save him. He was immediately taken to the Emergency Department.

A.	Non accidental injury
B.	Accidental fracture
C.	Rickets
D.	Metabolic bone disease of prematurity
E.	Hypophosphataemic rickets
F.	Osteopetrosis
G.	Osteogenesis imperfecta
H.	Hypoparathyroidism
I.	Osteoporosis
A

Accidental fracture

The mechanism fits with the fracture pattern and the presentation is not delayed.

Paediatric fracture types
Type Injury pattern
Complete fracture Both sides of cortex are breached
Toddlers fracture Oblique tibial fracture in infants
Plastic deformity Stress on bone resulting in deformity without cortical disruption
Greenstick fracture Unilateral cortical breach only
Buckle fracture Incomplete cortical disruption resulting in periosteal haematoma only

Growth plate fractures
In paediatric practice fractures may also involve the growth plate and these injuries are classified according to the Salter- Harris system (given below):

Type Injury pattern
I Fracture through the physis only (x-ray often normal)
II Fracture through the physis and metaphysis
III Fracture through the physis and epiphyisis to include the joint
IV Fracture involving the physis, metaphysis and epiphysis
V Crush injury involving the physis (x-ray may resemble type I, and appear normal)

As a general rule it is safer to assume that growth plate tenderness is indicative of an underlying fracture even if the x-ray appears normal. Injuries of Types III, IV and V will usually require surgery. Type V injuries are often associated with disruption to growth.

Non accidental injury
Delayed presentation
Delay in attaining milestones
Lack of concordance between proposed and actual mechanism of injury
Multiple injuries
Injuries at sites not commonly exposed to trauma
Children on the at risk register

Pathological fractures
Genetic conditions, such as osteogenesis imperfecta, may cause pathological fractures.

Osteogenesis imperfecta
Defective osteoid formation due to congenital inability to produce adequate intercellular substances like osteoid, collagen and dentine.
Failure of maturation of collagen in all the connective tissues.
Radiology may show translucent bones, multiple fractures, particularly of the long bones, wormian bones (irregular patches of ossification) and a trefoil pelvis.

Subtypes
Type I The collagen is normal quality but insufficient quantity.
Type II- Poor collagen quantity and quality.
Type III- Collagen poorly formed. Normal quantity.
Type IV- Sufficient collagen quantity but poor quality.

Osteopetrosis
Bones become harder and more dense.
Autosomal recessive condition.
It is commonest in young adults.
Radiology reveals a lack of differentiation between the cortex and the medulla described as marble bone.
74
Q

An infant is admitted with symptoms and signs of respiratory infection and is found to have several posterior rib fractures on chest radiograph. He was born prematurely at 37 weeks’ gestation and was observed overnight on the special care baby unit for tachypnoea which settled by the following day. On assessment it is also apparent that his head circumference has increased at an excessive rate and has crossed 3 centiles since birth.

A.	Non accidental injury
B.	Accidental fracture
C.	Rickets
D.	Metabolic bone disease of prematurity
E.	Hypophosphataemic rickets
F.	Osteopetrosis
G.	Osteogenesis imperfecta
H.	Hypoparathyroidism
I.	Osteoporosis
A

Non accidental injury

Posterior rib fractures are extremely unusual in neonates. The change in head size may be accounted for by hydrocephalus which may occur as a sequelae from head injury.

Paediatric fracture types
Type Injury pattern
Complete fracture Both sides of cortex are breached
Toddlers fracture Oblique tibial fracture in infants
Plastic deformity Stress on bone resulting in deformity without cortical disruption
Greenstick fracture Unilateral cortical breach only
Buckle fracture Incomplete cortical disruption resulting in periosteal haematoma only

Growth plate fractures
In paediatric practice fractures may also involve the growth plate and these injuries are classified according to the Salter- Harris system (given below):

Type Injury pattern
I Fracture through the physis only (x-ray often normal)
II Fracture through the physis and metaphysis
III Fracture through the physis and epiphyisis to include the joint
IV Fracture involving the physis, metaphysis and epiphysis
V Crush injury involving the physis (x-ray may resemble type I, and appear normal)

As a general rule it is safer to assume that growth plate tenderness is indicative of an underlying fracture even if the x-ray appears normal. Injuries of Types III, IV and V will usually require surgery. Type V injuries are often associated with disruption to growth.

Non accidental injury
Delayed presentation
Delay in attaining milestones
Lack of concordance between proposed and actual mechanism of injury
Multiple injuries
Injuries at sites not commonly exposed to trauma
Children on the at risk register

Pathological fractures
Genetic conditions, such as osteogenesis imperfecta, may cause pathological fractures.

Osteogenesis imperfecta
Defective osteoid formation due to congenital inability to produce adequate intercellular substances like osteoid, collagen and dentine.
Failure of maturation of collagen in all the connective tissues.
Radiology may show translucent bones, multiple fractures, particularly of the long bones, wormian bones (irregular patches of ossification) and a trefoil pelvis.

Subtypes
Type I The collagen is normal quality but insufficient quantity.
Type II- Poor collagen quantity and quality.
Type III- Collagen poorly formed. Normal quantity.
Type IV- Sufficient collagen quantity but poor quality.

Osteopetrosis
Bones become harder and more dense.
Autosomal recessive condition.
It is commonest in young adults.
Radiology reveals a lack of differentiation between the cortex and the medulla described as marble bone.
75
Q

In paediatric orthopaedic surgery, which of the following does not fulfill the Kocher criteria for septic arthritis?

	ESR > 40mm/h
	Positive blood culture
	Fever
	White cell count > 12, 000
	Non weight bearing on the affected side
A

Kocher criteria

  1. Non weight bearing on affected side
  2. ESR > 40 mm/hr
  3. Fever
  4. WBC count of >12,000 mm3
    - When 4/4 criteria are met, there is a 99% chance that the child has septic arthritis

The Kocher criteria do not consider blood culture results.
Septic arthritis
Staph aureus commonest organism
Urgent washout and antibiotics otherwise high risk of joint destruction

Diagnosis
Plain x-rays
Consider aspiration
Utilise the Kocher criteria (see below)

Kocher criteria:

  1. Non weight bearing on affected side
  2. ESR > 40 mm/hr
  3. Fever
  4. WBC count of >12,000 mm3
    - when 4/4 criteria are met, there is a 99% chance that the child has septic arthritis

Treatment
Surgical drainage of the affected joint is required, this should be done as soon as possible since permanent damage to the joint may occur. In some cases repeated procedures are necessary. Appropriate intravenous antibiotics should be administered.

76
Q

An otherwise fit 74 year old man presents with pain in the right hip following minimal trauma. On examination his leg is shortened and externally rotated. Plain films demonstrate a displaced intracapsular fracture of the femoral neck.

 A.	MRI scan
B.	Hemiarthroplasty
C.	Bone scintigraphy
D.	Conservative management
E.	Total hip replacement
F.	Insertion of intra medullary nail
G.	Hip arthrodesis
H.	Internal fixation
A

Total hip replacement

In otherwise fit patients aged over 70, the best long term functional outcomes are obtained with total hip arthroplasty.

Neck of femur (NOF) fracture is a common orthopaedic presentation, with over 65000 fractures in the UK per year. Like many orthopaedic injuries, there is a bimodal age distribution. It is imperative to distinguish between the high energy injury in a young patient, and the low energy osteoporotic fracture in the elderly, as their management aims are very different:

Young patient - Usually high energy trauma (e.g road traffic accident, horse riding) and needs treating in accordance with Advanced Trauma Life Support (ATLS) principles. Will often have associated injuries. Aim is to retain the patients own anatomy, and optimise their function.

Elderly patient - Predominantly female, fall from standing height (fragility fracture). Often patients have multiple comorbidities that will ultimately dictate their prognosis. Aim of orthopaedic treatment is to immediately regain patient mobility so that morbidity (infection, thromboembolic events, pressure sores etc) and mortality associated with prolonged bed rest is avoided. Left untreated, a neck of femur fracture can be considered a terminal event. Historically, mortality associated with elderly hip fracture is 10% at one month, and 30% at one year. However, this has been improved in the UK with the introduction of multidisciplinary, orthogeriatric lead care and the National Hip Fracture Database and Best Practice Tariff.

Pertinent anatomy
Osteology - normal neck-shaft angle is 130 +/- 7 degrees, and 10 +/- 7 degrees of neck anteversion.
Vascular supply - The predominant blood supply to the femoral head and neck is from the medial and lateral femoral circumflex arteries (branches of profunda femoris). These anastomose and pierce the joint capsule at the base of the neck, mainly posteriorly. There is a small vascular contribution from the artery of the ligament teres. Understanding the blood supply is fundamental to the decision making process in treating NOF fractures.

Presentation and initial management
Typically, patients present with pain in the hip/groin, a shortened, abducted, externally rotated leg (due to the unopposed pull of the muscles that act across the hip joint) and the inability to straight-leg-raise. With undisplaced fractures, signs are more subtle.
High energy injuries should be treated in line with ATLS principles. All patients should be fluid resuscitated, have adequate pain relief (often with a fascio-iliiaca nerve block), and be optimised for surgery. In addition, elderly patients should be assessed by an orthogeriatrician.

Imaging
Anteroposterior and cross-table lateral plain radiographs are sufficient to diagnose the majority of NOF fractures. If the fracture extends below the level of the lesser trochanter, or there is any possibility of pathological fracture, full length femur views are essential to plan surgery.

Where there is a high index of suspicion of fracture, but plain radiographs are inconclusive, gold standard investigation is MRI. However, if unavailable within 24 hours, or if the patient will not tolerate MRI, CT is appropriate. The majority of fractures can be seen with modern CT techniques, and so this is becoming first line in many hospitals.

Classification
There has been a move away from named classification systems towards descriptive classification systems.
Two main types of NOF exist: Intra-capsular, and extra-capsular. Extra-capsular fractures are further divided into pertrochanteric or subtrochanteric (within 5cm distal to the lesser trochanter). All fractures are then described as undisplaced, minimally displaced, or displaced.
Femoral neck and head blood supply disruption is common with intracapsular NOF fractures, and rare with extracapsular fractures. This fundamental principle underpins the practise of arthroplasty for intracapsular fractures, and fixation for extracapsular fractures.

If you wish to use a named classification system, the most commonly used are below:
Elderly intracapsular - Garden Classification
Young intrasapsular - Pauvels Classification
Intertrochanteric - Evans
Subtrochanteric - Russell Taylor

Treatment
In general, NOF fractures are treated operatively except if the patient is deemed unlikely to survive an anaesthetic. Best Practice Tarif (BPT) dictates that surgery should happen within 36 hours, as delay of greater than 48 hours is associated with increased morbidity and mortality. Below are suggested algorithms for the treatment of NOF. There are some areas of debate/controversy which are detailed below.

Image sourced from Wikipedia
* The priority with the young patient is to retain the femoral head if possible, even with a displaced intracapsular fracture. The risk of avascular necrosis and non-union (and therefore revision surgery) associated with internal fixation needs weighing up against the sequelae of total hip replacement in the young (wear, dislocation, revision). Discussion is necessary with the patient, on a case by case basis.

** Undisplaced fractures in the elderly can be treated with internal fixation, often with cannulated screws. This is appropriate for valgus impacted subcapital fractures which are inherently stable, to prevent secondary displacement. This does still carry the risk of AVN or non-union, and therefore a future revision. For this reason, many surgeons advocate arthroplasty as a single surgery.

*** NICE guidance - patients who fulfil these criteria should be offered total hip replacement which conveys better function and prosthetic survivorship, compared with hemiarthroplasty, but at an increased risk of dislocation.

Image sourced from Wikipedia
* Intertrochanteric fractures vary greatly in their stability. If the trochanter (and therefore lateral wall), and medial calcar is in tact, then the fracture configuration bears stability. This can be treated with a DHS, as collapse of the fracture is predictable. Where either or both structures are involved in the fracture, stability becomes compromised and many surgeons will favour using an intramedullary device. This is an ongoing debate, and difficult to test in an exam setting.

Post operative management
Patients should be mobilised fully weight bearing where possible. Care is multidisciplinary in its delivery. Elderly patients should have orthogeriatrician assessment of comorbidity, and bone health with secondary prevention measures if appropriate. There should be early involvement of physiotherapy and occupational therapy services.

77
Q

A 72 year old retired teacher is admitted to A&E with a fall and hip pain. He is normally fit and well. He lives with his son in a detached, 2 storey house. A hip x-ray and femur views confirm a sub trochanteric fracture.

A.	MRI scan
B.	Hemiarthroplasty
C.	Bone scintigraphy
D.	Conservative management
E.	Total hip replacement
F.	Insertion of intra medullary nail
G.	Hip arthrodesis
H.	Internal fixation
A

Insertion of intra medullary nail

Intramedullary devices are normally recommended for reverse oblique, transverse subtrochanteric fractures.

Neck of femur (NOF) fracture is a common orthopaedic presentation, with over 65000 fractures in the UK per year. Like many orthopaedic injuries, there is a bimodal age distribution. It is imperative to distinguish between the high energy injury in a young patient, and the low energy osteoporotic fracture in the elderly, as their management aims are very different:

Young patient - Usually high energy trauma (e.g road traffic accident, horse riding) and needs treating in accordance with Advanced Trauma Life Support (ATLS) principles. Will often have associated injuries. Aim is to retain the patients own anatomy, and optimise their function.

Elderly patient - Predominantly female, fall from standing height (fragility fracture). Often patients have multiple comorbidities that will ultimately dictate their prognosis. Aim of orthopaedic treatment is to immediately regain patient mobility so that morbidity (infection, thromboembolic events, pressure sores etc) and mortality associated with prolonged bed rest is avoided. Left untreated, a neck of femur fracture can be considered a terminal event. Historically, mortality associated with elderly hip fracture is 10% at one month, and 30% at one year. However, this has been improved in the UK with the introduction of multidisciplinary, orthogeriatric lead care and the National Hip Fracture Database and Best Practice Tariff.

Pertinent anatomy
Osteology - normal neck-shaft angle is 130 +/- 7 degrees, and 10 +/- 7 degrees of neck anteversion.
Vascular supply - The predominant blood supply to the femoral head and neck is from the medial and lateral femoral circumflex arteries (branches of profunda femoris). These anastomose and pierce the joint capsule at the base of the neck, mainly posteriorly. There is a small vascular contribution from the artery of the ligament teres. Understanding the blood supply is fundamental to the decision making process in treating NOF fractures.

Presentation and initial management
Typically, patients present with pain in the hip/groin, a shortened, abducted, externally rotated leg (due to the unopposed pull of the muscles that act across the hip joint) and the inability to straight-leg-raise. With undisplaced fractures, signs are more subtle.
High energy injuries should be treated in line with ATLS principles. All patients should be fluid resuscitated, have adequate pain relief (often with a fascio-iliiaca nerve block), and be optimised for surgery. In addition, elderly patients should be assessed by an orthogeriatrician.

Imaging
Anteroposterior and cross-table lateral plain radiographs are sufficient to diagnose the majority of NOF fractures. If the fracture extends below the level of the lesser trochanter, or there is any possibility of pathological fracture, full length femur views are essential to plan surgery.

Where there is a high index of suspicion of fracture, but plain radiographs are inconclusive, gold standard investigation is MRI. However, if unavailable within 24 hours, or if the patient will not tolerate MRI, CT is appropriate. The majority of fractures can be seen with modern CT techniques, and so this is becoming first line in many hospitals.

Classification
There has been a move away from named classification systems towards descriptive classification systems.
Two main types of NOF exist: Intra-capsular, and extra-capsular. Extra-capsular fractures are further divided into pertrochanteric or subtrochanteric (within 5cm distal to the lesser trochanter). All fractures are then described as undisplaced, minimally displaced, or displaced.
Femoral neck and head blood supply disruption is common with intracapsular NOF fractures, and rare with extracapsular fractures. This fundamental principle underpins the practise of arthroplasty for intracapsular fractures, and fixation for extracapsular fractures.

If you wish to use a named classification system, the most commonly used are below:
Elderly intracapsular - Garden Classification
Young intrasapsular - Pauvels Classification
Intertrochanteric - Evans
Subtrochanteric - Russell Taylor

Treatment
In general, NOF fractures are treated operatively except if the patient is deemed unlikely to survive an anaesthetic. Best Practice Tarif (BPT) dictates that surgery should happen within 36 hours, as delay of greater than 48 hours is associated with increased morbidity and mortality. Below are suggested algorithms for the treatment of NOF. There are some areas of debate/controversy which are detailed below.

Image sourced from Wikipedia
* The priority with the young patient is to retain the femoral head if possible, even with a displaced intracapsular fracture. The risk of avascular necrosis and non-union (and therefore revision surgery) associated with internal fixation needs weighing up against the sequelae of total hip replacement in the young (wear, dislocation, revision). Discussion is necessary with the patient, on a case by case basis.

** Undisplaced fractures in the elderly can be treated with internal fixation, often with cannulated screws. This is appropriate for valgus impacted subcapital fractures which are inherently stable, to prevent secondary displacement. This does still carry the risk of AVN or non-union, and therefore a future revision. For this reason, many surgeons advocate arthroplasty as a single surgery.

*** NICE guidance - patients who fulfil these criteria should be offered total hip replacement which conveys better function and prosthetic survivorship, compared with hemiarthroplasty, but at an increased risk of dislocation.

Image sourced from Wikipedia
* Intertrochanteric fractures vary greatly in their stability. If the trochanter (and therefore lateral wall), and medial calcar is in tact, then the fracture configuration bears stability. This can be treated with a DHS, as collapse of the fracture is predictable. Where either or both structures are involved in the fracture, stability becomes compromised and many surgeons will favour using an intramedullary device. This is an ongoing debate, and difficult to test in an exam setting.

Post operative management
Patients should be mobilised fully weight bearing where possible. Care is multidisciplinary in its delivery. Elderly patients should have orthogeriatrician assessment of comorbidity, and bone health with secondary prevention measures if appropriate. There should be early involvement of physiotherapy and occupational therapy services.

78
Q

A 72 year old lady stumbles and falls. On examination she is tender in the left groin and unable to weight bear. Attempts at internal rotation produce severe pain. Plain films of the hip show no obvious fracture.

A.	MRI scan
B.	Hemiarthroplasty
C.	Bone scintigraphy
D.	Conservative management
E.	Total hip replacement
F.	Insertion of intra medullary nail
G.	Hip arthrodesis
H.	Internal fixation
A

MRI scan

In those patients who present with a suspected hip fracture, but normal plain films, the most accurate investigation is an MRI or CT scan.

Neck of femur (NOF) fracture is a common orthopaedic presentation, with over 65000 fractures in the UK per year. Like many orthopaedic injuries, there is a bimodal age distribution. It is imperative to distinguish between the high energy injury in a young patient, and the low energy osteoporotic fracture in the elderly, as their management aims are very different:

Young patient - Usually high energy trauma (e.g road traffic accident, horse riding) and needs treating in accordance with Advanced Trauma Life Support (ATLS) principles. Will often have associated injuries. Aim is to retain the patients own anatomy, and optimise their function.

Elderly patient - Predominantly female, fall from standing height (fragility fracture). Often patients have multiple comorbidities that will ultimately dictate their prognosis. Aim of orthopaedic treatment is to immediately regain patient mobility so that morbidity (infection, thromboembolic events, pressure sores etc) and mortality associated with prolonged bed rest is avoided. Left untreated, a neck of femur fracture can be considered a terminal event. Historically, mortality associated with elderly hip fracture is 10% at one month, and 30% at one year. However, this has been improved in the UK with the introduction of multidisciplinary, orthogeriatric lead care and the National Hip Fracture Database and Best Practice Tariff.

Pertinent anatomy
Osteology - normal neck-shaft angle is 130 +/- 7 degrees, and 10 +/- 7 degrees of neck anteversion.
Vascular supply - The predominant blood supply to the femoral head and neck is from the medial and lateral femoral circumflex arteries (branches of profunda femoris). These anastomose and pierce the joint capsule at the base of the neck, mainly posteriorly. There is a small vascular contribution from the artery of the ligament teres. Understanding the blood supply is fundamental to the decision making process in treating NOF fractures.

Presentation and initial management
Typically, patients present with pain in the hip/groin, a shortened, abducted, externally rotated leg (due to the unopposed pull of the muscles that act across the hip joint) and the inability to straight-leg-raise. With undisplaced fractures, signs are more subtle.
High energy injuries should be treated in line with ATLS principles. All patients should be fluid resuscitated, have adequate pain relief (often with a fascio-iliiaca nerve block), and be optimised for surgery. In addition, elderly patients should be assessed by an orthogeriatrician.

Imaging
Anteroposterior and cross-table lateral plain radiographs are sufficient to diagnose the majority of NOF fractures. If the fracture extends below the level of the lesser trochanter, or there is any possibility of pathological fracture, full length femur views are essential to plan surgery.

Where there is a high index of suspicion of fracture, but plain radiographs are inconclusive, gold standard investigation is MRI. However, if unavailable within 24 hours, or if the patient will not tolerate MRI, CT is appropriate. The majority of fractures can be seen with modern CT techniques, and so this is becoming first line in many hospitals.

Classification
There has been a move away from named classification systems towards descriptive classification systems.
Two main types of NOF exist: Intra-capsular, and extra-capsular. Extra-capsular fractures are further divided into pertrochanteric or subtrochanteric (within 5cm distal to the lesser trochanter). All fractures are then described as undisplaced, minimally displaced, or displaced.
Femoral neck and head blood supply disruption is common with intracapsular NOF fractures, and rare with extracapsular fractures. This fundamental principle underpins the practise of arthroplasty for intracapsular fractures, and fixation for extracapsular fractures.

If you wish to use a named classification system, the most commonly used are below:
Elderly intracapsular - Garden Classification
Young intrasapsular - Pauvels Classification
Intertrochanteric - Evans
Subtrochanteric - Russell Taylor

Treatment
In general, NOF fractures are treated operatively except if the patient is deemed unlikely to survive an anaesthetic. Best Practice Tarif (BPT) dictates that surgery should happen within 36 hours, as delay of greater than 48 hours is associated with increased morbidity and mortality. Below are suggested algorithms for the treatment of NOF. There are some areas of debate/controversy which are detailed below.

Image sourced from Wikipedia
* The priority with the young patient is to retain the femoral head if possible, even with a displaced intracapsular fracture. The risk of avascular necrosis and non-union (and therefore revision surgery) associated with internal fixation needs weighing up against the sequelae of total hip replacement in the young (wear, dislocation, revision). Discussion is necessary with the patient, on a case by case basis.

** Undisplaced fractures in the elderly can be treated with internal fixation, often with cannulated screws. This is appropriate for valgus impacted subcapital fractures which are inherently stable, to prevent secondary displacement. This does still carry the risk of AVN or non-union, and therefore a future revision. For this reason, many surgeons advocate arthroplasty as a single surgery.

*** NICE guidance - patients who fulfil these criteria should be offered total hip replacement which conveys better function and prosthetic survivorship, compared with hemiarthroplasty, but at an increased risk of dislocation.

Image sourced from Wikipedia
* Intertrochanteric fractures vary greatly in their stability. If the trochanter (and therefore lateral wall), and medial calcar is in tact, then the fracture configuration bears stability. This can be treated with a DHS, as collapse of the fracture is predictable. Where either or both structures are involved in the fracture, stability becomes compromised and many surgeons will favour using an intramedullary device. This is an ongoing debate, and difficult to test in an exam setting.

Post operative management
Patients should be mobilised fully weight bearing where possible. Care is multidisciplinary in its delivery. Elderly patients should have orthogeriatrician assessment of comorbidity, and bone health with secondary prevention measures if appropriate. There should be early involvement of physiotherapy and occupational therapy services.

79
Q

A 45 year old man has been admitted after being knocked off his bicycle. His ankle is grossly deformed with bilateral malleolar tenderness with severe ankle swelling and tenting of the medial soft tissues.

A. Surgical fixation
B. Below knee amputation
C. Application of below knee plaster
D. Application of ankle boot
E. Application of external fixation device
F. Application of compression dressing and physiotherapy
G. Immediate reduction and application of backslab

A

Immediate reduction and application of backslab

This is an unstable ankle injury that is likely to require surgical fixation. The immediate management of a displaced ankle fracture is to reduce the fracture to prevent soft tissues compromise and help reduce swelling. This can be performed before an x-ray is obtained if performing the x-ray will significantly delay reduction.

An ankle fracture relates to a fracture around the tibio-talar joint. It generally refers to a fracture involving the lateral, and/or medial and/or posterior malleolus. Pilon and Tillaux fractures are also considered to be ankle fractures, but are not covered here.
Ankle fractures are common. They effect men and women in equal numbers, but men have a higher rate as young adults (sports and contact injuries), and women a higher rate post-menopausal (fragility type fracture).

Osseous anatomy
The ankle (or mortise) joint consists of the distal tibia (tibial plafond and posterior malleolus), the distal fibula (lateral malleolus), and the talus. The main movement at the ankle joint is plantar and dorsiflexion.

Ligamentous anatomy
Medial side: Deltoid ligament. This is divided into superficial and deep portions. It is the primary restraint to valgus tilting of the talus.

Lateral side: Lateral ligament complex consisting from anterior to posterior of the anterior talofibular ligament (ATFL), calcaneofibular ligament (CFL), and the posterior talofibular ligament (PTFL). Together they resist valgus stress to the ankle, and are a restraint to anterior translation of the talus within the mortise joint.

Syndesmosis: The syndesmosis is a ligament complex between the distal tibia and fibula, holding the two bones together. It is fundamental to the integrity of the ankle joint, and its disruption leads to instability. It consists of (from anterior to posterior) the anterior-inferior tibiofibular ligament (AITFL), the transverse tibiofibular ligament (TTFL), the interosseous membrane, and the posterior-inferior tibiofibular ligament (PITFL).

Presentation and initial management
Patients will present following a traumatic event with a painful, swollen ankle, and reluctance/inability to weight bear. The Ottawa rules can be applied to differentiate between an ankle fracture and sprain, but can be unreliable.
In high energy injuries, management should follow ATLS principles to identify more significant injuries first. Neurovascular status of the foot should be documented, and open injuries should be excluded. If an open injury is identified, it should be managed in line with BOAST 4 principles1. If an obvious deformity exists, it should be reduced as soon as possible with appropriate analgesia or conscious sedation. Radiographs of clearly deformed or dislocated joints are not necessary, and removing the pressure on the surrounding soft tissues from the underlying bony deformity is the priority. If the fracture pattern is not clinically obvious then plain radiographs are appropriate and will guide the subsequent manipulation during plaster-of-paris below knee backslab application.

Imaging
AP, lateral and mortise views (20o internal rotation) are essential to evaluate fracture displacement and syndesmotic injury. Decreased tibiofibular overlap, medial joint clear space and lateral talar shift all indicate a syndesmotic injury. (In subtle cases of shift, imaging the uninjured ankle can be helpful as a proportion of the population have little or no tibiotalar overlap 2.)

Where there is suspicion of syndesmosis involvement in the absence of radiographic evidence, stress radiographs can be diagnostic.
Complex fracture patterns (and increasingly posterior malleolar fractures) are best defined using CT.

Classification
The most commonly used classifications are Lauge-Hansen and Danis-Weber.

Lauge-Hansen
Comprises two parts: first part is the foot position, and the second part is the force applied. Useful for understanding the forces involved and therefore predict the ligamentous or bony injury. Results in four injury patterns:
Supination - Adduction (SA) - 10-20%
Supination - External rotation (SER) - 40-75%
Pronation - Abduction (PA) - 5-20%
Pronation - External rotation (PER) - 5-20%

Not often used in clinical practice but good for understanding the principles of ankle fracture.

Danis-Weber
Commonly used. Based on the level of the fibula fracture in relation to the syndesmosis. The more proximal, the greater the risk of syndesmotic injury and therefore fracture instability.
A - fracture below the level of the syndesmosis
B - fracture at the level of the syndesmosis / level of the tibial plafond
C - fracture above the level of the syndesmosis. This includes Maisonneuve fractures (proximal fibula fracture), which can be associated with ankle instability. Beware the high fibula fracture - it may be an ankle fracture!

The Weber classification is based purely on the the lateral side. All injuries can include a medial or posterior bony or ligamentous injury which also dictates fracture stability (bimalleolar and trimalleolar fractures are more unstable).

Treatment
When deciding upon treatment for an ankle fracture, one must consider both the fracture and the patient. Diabetic patients and smokers are at greater risk of post-operative complication, especially wound problems and infection. Likewise, the long term outcome of post-traumatic arthritis from a malunited ankle fracture is extremely important for a young patient, but not as relevant in the elderly. Therefore, normal surgical decision processes apply as with all fractures.

Defining stability of an ankle fracture underpins the treatment decision.
Weber A - Unimalleolar Weber A Weber fractures by definition are stable and therefore can be mobilised fully weight bearing in an ankle boot.

Weber C - Fractures tend to include syndesmotic disruption and are usually bimalleolar (either bony or ligamentous). They are therefore unstable and usually require operative fixation. In addition to the fracture fixation, the syndesmosis usually requires reconstruction/augmentation with screws to restore the joint integrity and function.

Weber B - B fractures vary greatly. They can be part of a trimalleolar injury and therefore extremely unstable, requiring fixation. Alternatively, a uni-malleolar Weber B fracture can be a stable injury, and therefore mobilised immediately in an ankle boot. Defining the stability can be challenging, and often involves stress radiographs, or a trial of mobilisation and repeat radiographs. Defining stability is the subject of much ongoing research. However, treating undisplaced ankle fractures in a below knee plaster, non-weight bearing for six weeks is still widely practised, and a safe approach.

When operative fixation is appropriate, it is usually via open reduction and internal fixation using plates and screws. It must be carried out when soft tissue swelling has settled in order to minimise the risk of wound problems. This can often take a week to settle.

The use of fibula nails is expanding, but is not yet mainstream. Ankle fractures can also be treated with external fixation, or with a hind foot nail in patients who need fixation but where soft tissue or bone quality is poor.

Post operative management
Ankle fractures generally take 6 weeks to unite enough to prevent secondary displacement. This is therefore an appropriate time period to keep a cast on in a conservatively managed patient. Weight bearing post-operatively depends on the quality of the fixation and bone quality, and preference varies between surgeons, ranging from aggressive early mobilisation to a period of non-weight bearing. Return to activities takes approximately three months, and often requires assistance of a physiotherapist to improve range-of-movement and muscle strengthening.

80
Q

A 40 year old marine injures his ankle on an assault course. On examination he has a severely swollen ankle, as well as tenderness over the medial malleolus and proximal fibula. X-rays demonstrate a medial malleolar fracture, spiral fracture of the proximal fibula and widening of the syndesmosis.

A. Surgical fixation
B. Below knee amputation
C. Application of below knee plaster
D. Application of ankle boot
E. Application of external fixation device
F. Application of compression dressing and physiotherapy
G. Immediate reduction and application of backslab

A

Surgical fixation

This is a Maisonneuve fracture of the proximal fibula. It indicates an unstable ankle injury with likely injury to the interosseous membrane. In the setting of radiographic evidence of syndesmotic widening, this requires surgical fixation to reduce and stabilise the syndesmosis.

An ankle fracture relates to a fracture around the tibio-talar joint. It generally refers to a fracture involving the lateral, and/or medial and/or posterior malleolus. Pilon and Tillaux fractures are also considered to be ankle fractures, but are not covered here.
Ankle fractures are common. They effect men and women in equal numbers, but men have a higher rate as young adults (sports and contact injuries), and women a higher rate post-menopausal (fragility type fracture).

Osseous anatomy
The ankle (or mortise) joint consists of the distal tibia (tibial plafond and posterior malleolus), the distal fibula (lateral malleolus), and the talus. The main movement at the ankle joint is plantar and dorsiflexion.

Ligamentous anatomy
Medial side: Deltoid ligament. This is divided into superficial and deep portions. It is the primary restraint to valgus tilting of the talus.

Lateral side: Lateral ligament complex consisting from anterior to posterior of the anterior talofibular ligament (ATFL), calcaneofibular ligament (CFL), and the posterior talofibular ligament (PTFL). Together they resist valgus stress to the ankle, and are a restraint to anterior translation of the talus within the mortise joint.

Syndesmosis: The syndesmosis is a ligament complex between the distal tibia and fibula, holding the two bones together. It is fundamental to the integrity of the ankle joint, and its disruption leads to instability. It consists of (from anterior to posterior) the anterior-inferior tibiofibular ligament (AITFL), the transverse tibiofibular ligament (TTFL), the interosseous membrane, and the posterior-inferior tibiofibular ligament (PITFL).

Presentation and initial management
Patients will present following a traumatic event with a painful, swollen ankle, and reluctance/inability to weight bear. The Ottawa rules can be applied to differentiate between an ankle fracture and sprain, but can be unreliable.
In high energy injuries, management should follow ATLS principles to identify more significant injuries first. Neurovascular status of the foot should be documented, and open injuries should be excluded. If an open injury is identified, it should be managed in line with BOAST 4 principles1. If an obvious deformity exists, it should be reduced as soon as possible with appropriate analgesia or conscious sedation. Radiographs of clearly deformed or dislocated joints are not necessary, and removing the pressure on the surrounding soft tissues from the underlying bony deformity is the priority. If the fracture pattern is not clinically obvious then plain radiographs are appropriate and will guide the subsequent manipulation during plaster-of-paris below knee backslab application.

Imaging
AP, lateral and mortise views (20o internal rotation) are essential to evaluate fracture displacement and syndesmotic injury. Decreased tibiofibular overlap, medial joint clear space and lateral talar shift all indicate a syndesmotic injury. (In subtle cases of shift, imaging the uninjured ankle can be helpful as a proportion of the population have little or no tibiotalar overlap 2.)

Where there is suspicion of syndesmosis involvement in the absence of radiographic evidence, stress radiographs can be diagnostic.
Complex fracture patterns (and increasingly posterior malleolar fractures) are best defined using CT.

Classification
The most commonly used classifications are Lauge-Hansen and Danis-Weber.

Lauge-Hansen
Comprises two parts: first part is the foot position, and the second part is the force applied. Useful for understanding the forces involved and therefore predict the ligamentous or bony injury. Results in four injury patterns:
Supination - Adduction (SA) - 10-20%
Supination - External rotation (SER) - 40-75%
Pronation - Abduction (PA) - 5-20%
Pronation - External rotation (PER) - 5-20%

Not often used in clinical practice but good for understanding the principles of ankle fracture.

Danis-Weber
Commonly used. Based on the level of the fibula fracture in relation to the syndesmosis. The more proximal, the greater the risk of syndesmotic injury and therefore fracture instability.
A - fracture below the level of the syndesmosis
B - fracture at the level of the syndesmosis / level of the tibial plafond
C - fracture above the level of the syndesmosis. This includes Maisonneuve fractures (proximal fibula fracture), which can be associated with ankle instability. Beware the high fibula fracture - it may be an ankle fracture!

The Weber classification is based purely on the the lateral side. All injuries can include a medial or posterior bony or ligamentous injury which also dictates fracture stability (bimalleolar and trimalleolar fractures are more unstable).

Treatment
When deciding upon treatment for an ankle fracture, one must consider both the fracture and the patient. Diabetic patients and smokers are at greater risk of post-operative complication, especially wound problems and infection. Likewise, the long term outcome of post-traumatic arthritis from a malunited ankle fracture is extremely important for a young patient, but not as relevant in the elderly. Therefore, normal surgical decision processes apply as with all fractures.

Defining stability of an ankle fracture underpins the treatment decision.
Weber A - Unimalleolar Weber A Weber fractures by definition are stable and therefore can be mobilised fully weight bearing in an ankle boot.

Weber C - Fractures tend to include syndesmotic disruption and are usually bimalleolar (either bony or ligamentous). They are therefore unstable and usually require operative fixation. In addition to the fracture fixation, the syndesmosis usually requires reconstruction/augmentation with screws to restore the joint integrity and function.

Weber B - B fractures vary greatly. They can be part of a trimalleolar injury and therefore extremely unstable, requiring fixation. Alternatively, a uni-malleolar Weber B fracture can be a stable injury, and therefore mobilised immediately in an ankle boot. Defining the stability can be challenging, and often involves stress radiographs, or a trial of mobilisation and repeat radiographs. Defining stability is the subject of much ongoing research. However, treating undisplaced ankle fractures in a below knee plaster, non-weight bearing for six weeks is still widely practised, and a safe approach.

When operative fixation is appropriate, it is usually via open reduction and internal fixation using plates and screws. It must be carried out when soft tissue swelling has settled in order to minimise the risk of wound problems. This can often take a week to settle.

The use of fibula nails is expanding, but is not yet mainstream. Ankle fractures can also be treated with external fixation, or with a hind foot nail in patients who need fixation but where soft tissue or bone quality is poor.

Post operative management
Ankle fractures generally take 6 weeks to unite enough to prevent secondary displacement. This is therefore an appropriate time period to keep a cast on in a conservatively managed patient. Weight bearing post-operatively depends on the quality of the fixation and bone quality, and preference varies between surgeons, ranging from aggressive early mobilisation to a period of non-weight bearing. Return to activities takes approximately three months, and often requires assistance of a physiotherapist to improve range-of-movement and muscle strengthening.

81
Q

A 60 year old female injures her ankle after a falling from a horse. On examination she has severe swelling and bruising of the ankle with x-rays demonstrating a comminuted intra-articular distal tibia fracture. The ankle has been temporarily reduced and splinted in the emergency department.

A. Surgical fixation
B. Below knee amputation
C. Application of below knee plaster
D. Application of ankle boot
E. Application of external fixation device
F. Application of compression dressing and physiotherapy
G. Immediate reduction and application of backslab

A

Application of external fixation device

This is a pilon fracture, which a high energy injury of the distal tibia. The patient will ultimately require surgical fixation but early management involves applying a spanning external fixator to temporarily reduce the fracture and allow soft tissue swelling to settle. A CT scan should then be performed to aid surgical planning.

An ankle fracture relates to a fracture around the tibio-talar joint. It generally refers to a fracture involving the lateral, and/or medial and/or posterior malleolus. Pilon and Tillaux fractures are also considered to be ankle fractures, but are not covered here.
Ankle fractures are common. They effect men and women in equal numbers, but men have a higher rate as young adults (sports and contact injuries), and women a higher rate post-menopausal (fragility type fracture).

Osseous anatomy
The ankle (or mortise) joint consists of the distal tibia (tibial plafond and posterior malleolus), the distal fibula (lateral malleolus), and the talus. The main movement at the ankle joint is plantar and dorsiflexion.

Ligamentous anatomy
Medial side: Deltoid ligament. This is divided into superficial and deep portions. It is the primary restraint to valgus tilting of the talus.

Lateral side: Lateral ligament complex consisting from anterior to posterior of the anterior talofibular ligament (ATFL), calcaneofibular ligament (CFL), and the posterior talofibular ligament (PTFL). Together they resist valgus stress to the ankle, and are a restraint to anterior translation of the talus within the mortise joint.

Syndesmosis: The syndesmosis is a ligament complex between the distal tibia and fibula, holding the two bones together. It is fundamental to the integrity of the ankle joint, and its disruption leads to instability. It consists of (from anterior to posterior) the anterior-inferior tibiofibular ligament (AITFL), the transverse tibiofibular ligament (TTFL), the interosseous membrane, and the posterior-inferior tibiofibular ligament (PITFL).

Presentation and initial management
Patients will present following a traumatic event with a painful, swollen ankle, and reluctance/inability to weight bear. The Ottawa rules can be applied to differentiate between an ankle fracture and sprain, but can be unreliable.
In high energy injuries, management should follow ATLS principles to identify more significant injuries first. Neurovascular status of the foot should be documented, and open injuries should be excluded. If an open injury is identified, it should be managed in line with BOAST 4 principles1. If an obvious deformity exists, it should be reduced as soon as possible with appropriate analgesia or conscious sedation. Radiographs of clearly deformed or dislocated joints are not necessary, and removing the pressure on the surrounding soft tissues from the underlying bony deformity is the priority. If the fracture pattern is not clinically obvious then plain radiographs are appropriate and will guide the subsequent manipulation during plaster-of-paris below knee backslab application.

Imaging
AP, lateral and mortise views (20o internal rotation) are essential to evaluate fracture displacement and syndesmotic injury. Decreased tibiofibular overlap, medial joint clear space and lateral talar shift all indicate a syndesmotic injury. (In subtle cases of shift, imaging the uninjured ankle can be helpful as a proportion of the population have little or no tibiotalar overlap 2.)

Where there is suspicion of syndesmosis involvement in the absence of radiographic evidence, stress radiographs can be diagnostic.
Complex fracture patterns (and increasingly posterior malleolar fractures) are best defined using CT.

Classification
The most commonly used classifications are Lauge-Hansen and Danis-Weber.

Lauge-Hansen
Comprises two parts: first part is the foot position, and the second part is the force applied. Useful for understanding the forces involved and therefore predict the ligamentous or bony injury. Results in four injury patterns:
Supination - Adduction (SA) - 10-20%
Supination - External rotation (SER) - 40-75%
Pronation - Abduction (PA) - 5-20%
Pronation - External rotation (PER) - 5-20%

Not often used in clinical practice but good for understanding the principles of ankle fracture.

Danis-Weber
Commonly used. Based on the level of the fibula fracture in relation to the syndesmosis. The more proximal, the greater the risk of syndesmotic injury and therefore fracture instability.
A - fracture below the level of the syndesmosis
B - fracture at the level of the syndesmosis / level of the tibial plafond
C - fracture above the level of the syndesmosis. This includes Maisonneuve fractures (proximal fibula fracture), which can be associated with ankle instability. Beware the high fibula fracture - it may be an ankle fracture!

The Weber classification is based purely on the the lateral side. All injuries can include a medial or posterior bony or ligamentous injury which also dictates fracture stability (bimalleolar and trimalleolar fractures are more unstable).

Treatment
When deciding upon treatment for an ankle fracture, one must consider both the fracture and the patient. Diabetic patients and smokers are at greater risk of post-operative complication, especially wound problems and infection. Likewise, the long term outcome of post-traumatic arthritis from a malunited ankle fracture is extremely important for a young patient, but not as relevant in the elderly. Therefore, normal surgical decision processes apply as with all fractures.

Defining stability of an ankle fracture underpins the treatment decision.
Weber A - Unimalleolar Weber A Weber fractures by definition are stable and therefore can be mobilised fully weight bearing in an ankle boot.

Weber C - Fractures tend to include syndesmotic disruption and are usually bimalleolar (either bony or ligamentous). They are therefore unstable and usually require operative fixation. In addition to the fracture fixation, the syndesmosis usually requires reconstruction/augmentation with screws to restore the joint integrity and function.

Weber B - B fractures vary greatly. They can be part of a trimalleolar injury and therefore extremely unstable, requiring fixation. Alternatively, a uni-malleolar Weber B fracture can be a stable injury, and therefore mobilised immediately in an ankle boot. Defining the stability can be challenging, and often involves stress radiographs, or a trial of mobilisation and repeat radiographs. Defining stability is the subject of much ongoing research. However, treating undisplaced ankle fractures in a below knee plaster, non-weight bearing for six weeks is still widely practised, and a safe approach.

When operative fixation is appropriate, it is usually via open reduction and internal fixation using plates and screws. It must be carried out when soft tissue swelling has settled in order to minimise the risk of wound problems. This can often take a week to settle.

The use of fibula nails is expanding, but is not yet mainstream. Ankle fractures can also be treated with external fixation, or with a hind foot nail in patients who need fixation but where soft tissue or bone quality is poor.

Post operative management
Ankle fractures generally take 6 weeks to unite enough to prevent secondary displacement. This is therefore an appropriate time period to keep a cast on in a conservatively managed patient. Weight bearing post-operatively depends on the quality of the fixation and bone quality, and preference varies between surgeons, ranging from aggressive early mobilisation to a period of non-weight bearing. Return to activities takes approximately three months, and often requires assistance of a physiotherapist to improve range-of-movement and muscle strengthening.

82
Q

An obese 12 year old boy presents with knee pain. On examination he has pain on internal rotation of the hip. His knee is clinically normal.

A.	USS hip
B.	Hip x-ray
C.	Anteroposterior pelvic x-ray
D.	CT scan
E.	Discharge and reassure
F.	MRI
G.	USS knee
H.	X-ray knee
A

Hip x-ray

The main differential diagnosis in a boy over 10 years old is of slipped upper femoral epiphysis. Knee pain is a common presenting feature. An anteroposterior pelvic x-ray may miss a minor slip, therefore request a hip film.

Developmental dysplasia of the hip Usually diagnosed in infancy by screening tests. May be bilateral, when disease is unilateral there may be leg length inequality. As disease progresses child may limp and then early onset arthritis. More common in extended breech babies. Splints and harnesses or traction. In later years osteotomy and hip realignment procedures may be needed. In arthritis a joint replacement may be needed. However, this is best deferred if possible as it will almost certainly require revision Initially no obvious change on plain films and USS gives best resolution until 3 months of age. On plain films Shentons line should form a smooth arc
Perthes Disease Hip pain (may be referred to the knee) usually occurring between 5 and 12 years of age. Bilateral disease in 20%. Remove pressure from joint to allow normal development. Physiotherapy. Usually self-limiting if diagnosed and treated promptly. X-rays will show flattened femoral head. Eventually in untreated cases the femoral head will fragment.
Slipped upper femoral epiphysis Typically seen in obese male adolescents. Pain is often referred to the knee. Limitation to internal rotation is usually seen. Knee pain is usually present 2 months prior to hip slipping. Bilateral in 20%. Bed rest and non-weight bearing. Aim to avoid avascular necrosis. If severe slippage or risk of it occurring then percutaneous pinning of the hip may be required. X-rays will show the femoral head displaced and falling inferolaterally (like a melting ice cream cone) The Southwick angle gives indication of disease severity

83
Q

A baby is delivered in the breech position. Barlows and Ortolani tests are normal

A.	USS hip
B.	Hip x-ray
C.	Anteroposterior pelvic x-ray
D.	CT scan
E.	Discharge and reassure
F.	MRI
G.	USS knee
H.	X-ray knee
A

USS hip

This child is at risk of developmental dysplasia of the hip (up to 20% will have DDH), so should have the hip joints scanned to exclude this.

Developmental dysplasia of the hip Usually diagnosed in infancy by screening tests. May be bilateral, when disease is unilateral there may be leg length inequality. As disease progresses child may limp and then early onset arthritis. More common in extended breech babies. Splints and harnesses or traction. In later years osteotomy and hip realignment procedures may be needed. In arthritis a joint replacement may be needed. However, this is best deferred if possible as it will almost certainly require revision Initially no obvious change on plain films and USS gives best resolution until 3 months of age. On plain films Shentons line should form a smooth arc
Perthes Disease Hip pain (may be referred to the knee) usually occurring between 5 and 12 years of age. Bilateral disease in 20%. Remove pressure from joint to allow normal development. Physiotherapy. Usually self-limiting if diagnosed and treated promptly. X-rays will show flattened femoral head. Eventually in untreated cases the femoral head will fragment.
Slipped upper femoral epiphysis Typically seen in obese male adolescents. Pain is often referred to the knee. Limitation to internal rotation is usually seen. Knee pain is usually present 2 months prior to hip slipping. Bilateral in 20%. Bed rest and non-weight bearing. Aim to avoid avascular necrosis. If severe slippage or risk of it occurring then percutaneous pinning of the hip may be required. X-rays will show the femoral head displaced and falling inferolaterally (like a melting ice cream cone) The Southwick angle gives indication of disease severity

84
Q

A 5 year old boy presents with a painful limp. The symptoms have been present for 8 weeks. Two hip x-rays have been performed and appear normal.

A.	USS hip
B.	Hip x-ray
C.	Anteroposterior pelvic x-ray
D.	CT scan
E.	Discharge and reassure
F.	MRI
G.	USS knee
H.	X-ray knee
A

MRI

Perthes disease should be suspected in boys over 4 years old presenting with a limp. Early disease can be missed on x-ray. An MRI will often demonstrate areas of hypoperfusion and subtle changes that allow for earlier diagnosis. A bone scan is an alternative option.

Developmental dysplasia of the hip Usually diagnosed in infancy by screening tests. May be bilateral, when disease is unilateral there may be leg length inequality. As disease progresses child may limp and then early onset arthritis. More common in extended breech babies. Splints and harnesses or traction. In later years osteotomy and hip realignment procedures may be needed. In arthritis a joint replacement may be needed. However, this is best deferred if possible as it will almost certainly require revision Initially no obvious change on plain films and USS gives best resolution until 3 months of age. On plain films Shentons line should form a smooth arc
Perthes Disease Hip pain (may be referred to the knee) usually occurring between 5 and 12 years of age. Bilateral disease in 20%. Remove pressure from joint to allow normal development. Physiotherapy. Usually self-limiting if diagnosed and treated promptly. X-rays will show flattened femoral head. Eventually in untreated cases the femoral head will fragment.
Slipped upper femoral epiphysis Typically seen in obese male adolescents. Pain is often referred to the knee. Limitation to internal rotation is usually seen. Knee pain is usually present 2 months prior to hip slipping. Bilateral in 20%. Bed rest and non-weight bearing. Aim to avoid avascular necrosis. If severe slippage or risk of it occurring then percutaneous pinning of the hip may be required. X-rays will show the femoral head displaced and falling inferolaterally (like a melting ice cream cone) The Southwick angle gives indication of disease severity

85
Q

A 5 year old boy is playing in a tree when he falls and lands on his right forearm. He is brought to the emergency department by his parents. On examination he has bony tenderness and bruising. An X-ray is taken and shows unilateral cortical disruption and development of periosteal haematoma. What is the most likely diagnosis?

	Buckle fracture
	Greenstick fracture
	Toddlers fracture
	Complete fracture
	None of the above
A

Greenstick fractures are common childhood injuries. Unilateral cortical disruption is the main radiological feature, since involvement of both cortices makes the injury a complete fracture. Buckle fractures will show periosteal haematoma formation only.

Paediatric fracture types
Type Injury pattern
Complete fracture Both sides of cortex are breached
Toddlers fracture Oblique tibial fracture in infants
Plastic deformity Stress on bone resulting in deformity without cortical disruption
Greenstick fracture Unilateral cortical breach only
Buckle fracture Incomplete cortical disruption resulting in periosteal haematoma only

Growth plate fractures
In paediatric practice fractures may also involve the growth plate and these injuries are classified according to the Salter- Harris system (given below):

Type Injury pattern
I Fracture through the physis only (x-ray often normal)
II Fracture through the physis and metaphysis
III Fracture through the physis and epiphyisis to include the joint
IV Fracture involving the physis, metaphysis and epiphysis
V Crush injury involving the physis (x-ray may resemble type I, and appear normal)

As a general rule it is safer to assume that growth plate tenderness is indicative of an underlying fracture even if the x-ray appears normal. Injuries of Types III, IV and V will usually require surgery. Type V injuries are often associated with disruption to growth.

Non accidental injury
Delayed presentation
Delay in attaining milestones
Lack of concordance between proposed and actual mechanism of injury
Multiple injuries
Injuries at sites not commonly exposed to trauma
Children on the at risk register

Pathological fractures
Genetic conditions, such as osteogenesis imperfecta, may cause pathological fractures.

Osteogenesis imperfecta
Defective osteoid formation due to congenital inability to produce adequate intercellular substances like osteoid, collagen and dentine.
Failure of maturation of collagen in all the connective tissues.
Radiology may show translucent bones, multiple fractures, particularly of the long bones, wormian bones (irregular patches of ossification) and a trefoil pelvis.

Subtypes
Type I The collagen is normal quality but insufficient quantity.
Type II- Poor collagen quantity and quality.
Type III- Collagen poorly formed. Normal quantity.
Type IV- Sufficient collagen quantity but poor quality.

Osteopetrosis
Bones become harder and more dense.
Autosomal recessive condition.
It is commonest in young adults.
Radiology reveals a lack of differentiation between the cortex and the medulla described as marble bone.
86
Q

Which of the following is not typically seen in patients with a femoral neck fracture?

	Malunion
	Non union
	Avascular necrosis
	Shortening
	External rotation
A

Malunion would be unusual with a femoral neck fracture. Because it is a weight bearing joint, if the fracture is not united then it does not heal at all. It is for this reason that most femoral neck fractures are fixed. Avascular necrosis is a well recognised complication and a total hip replacement or hemiarthroplasty is usually considered in the elderly.

Neck of femur (NOF) fracture is a common orthopaedic presentation, with over 65000 fractures in the UK per year. Like many orthopaedic injuries, there is a bimodal age distribution. It is imperative to distinguish between the high energy injury in a young patient, and the low energy osteoporotic fracture in the elderly, as their management aims are very different:

Young patient - Usually high energy trauma (e.g road traffic accident, horse riding) and needs treating in accordance with Advanced Trauma Life Support (ATLS) principles. Will often have associated injuries. Aim is to retain the patients own anatomy, and optimise their function.

Elderly patient - Predominantly female, fall from standing height (fragility fracture). Often patients have multiple comorbidities that will ultimately dictate their prognosis. Aim of orthopaedic treatment is to immediately regain patient mobility so that morbidity (infection, thromboembolic events, pressure sores etc) and mortality associated with prolonged bed rest is avoided. Left untreated, a neck of femur fracture can be considered a terminal event. Historically, mortality associated with elderly hip fracture is 10% at one month, and 30% at one year. However, this has been improved in the UK with the introduction of multidisciplinary, orthogeriatric lead care and the National Hip Fracture Database and Best Practice Tariff.

Pertinent anatomy
Osteology - normal neck-shaft angle is 130 +/- 7 degrees, and 10 +/- 7 degrees of neck anteversion.
Vascular supply - The predominant blood supply to the femoral head and neck is from the medial and lateral femoral circumflex arteries (branches of profunda femoris). These anastomose and pierce the joint capsule at the base of the neck, mainly posteriorly. There is a small vascular contribution from the artery of the ligament teres. Understanding the blood supply is fundamental to the decision making process in treating NOF fractures.

Presentation and initial management
Typically, patients present with pain in the hip/groin, a shortened, abducted, externally rotated leg (due to the unopposed pull of the muscles that act across the hip joint) and the inability to straight-leg-raise. With undisplaced fractures, signs are more subtle.
High energy injuries should be treated in line with ATLS principles. All patients should be fluid resuscitated, have adequate pain relief (often with a fascio-iliiaca nerve block), and be optimised for surgery. In addition, elderly patients should be assessed by an orthogeriatrician.

Imaging
Anteroposterior and cross-table lateral plain radiographs are sufficient to diagnose the majority of NOF fractures. If the fracture extends below the level of the lesser trochanter, or there is any possibility of pathological fracture, full length femur views are essential to plan surgery.

Where there is a high index of suspicion of fracture, but plain radiographs are inconclusive, gold standard investigation is MRI. However, if unavailable within 24 hours, or if the patient will not tolerate MRI, CT is appropriate. The majority of fractures can be seen with modern CT techniques, and so this is becoming first line in many hospitals.

Classification
There has been a move away from named classification systems towards descriptive classification systems.
Two main types of NOF exist: Intra-capsular, and extra-capsular. Extra-capsular fractures are further divided into pertrochanteric or subtrochanteric (within 5cm distal to the lesser trochanter). All fractures are then described as undisplaced, minimally displaced, or displaced.
Femoral neck and head blood supply disruption is common with intracapsular NOF fractures, and rare with extracapsular fractures. This fundamental principle underpins the practise of arthroplasty for intracapsular fractures, and fixation for extracapsular fractures.

If you wish to use a named classification system, the most commonly used are below:
Elderly intracapsular - Garden Classification
Young intrasapsular - Pauvels Classification
Intertrochanteric - Evans
Subtrochanteric - Russell Taylor

Treatment
In general, NOF fractures are treated operatively except if the patient is deemed unlikely to survive an anaesthetic. Best Practice Tarif (BPT) dictates that surgery should happen within 36 hours, as delay of greater than 48 hours is associated with increased morbidity and mortality. Below are suggested algorithms for the treatment of NOF. There are some areas of debate/controversy which are detailed below.

Image sourced from Wikipedia
* The priority with the young patient is to retain the femoral head if possible, even with a displaced intracapsular fracture. The risk of avascular necrosis and non-union (and therefore revision surgery) associated with internal fixation needs weighing up against the sequelae of total hip replacement in the young (wear, dislocation, revision). Discussion is necessary with the patient, on a case by case basis.

** Undisplaced fractures in the elderly can be treated with internal fixation, often with cannulated screws. This is appropriate for valgus impacted subcapital fractures which are inherently stable, to prevent secondary displacement. This does still carry the risk of AVN or non-union, and therefore a future revision. For this reason, many surgeons advocate arthroplasty as a single surgery.

*** NICE guidance - patients who fulfil these criteria should be offered total hip replacement which conveys better function and prosthetic survivorship, compared with hemiarthroplasty, but at an increased risk of dislocation.

Image sourced from Wikipedia
* Intertrochanteric fractures vary greatly in their stability. If the trochanter (and therefore lateral wall), and medial calcar is in tact, then the fracture configuration bears stability. This can be treated with a DHS, as collapse of the fracture is predictable. Where either or both structures are involved in the fracture, stability becomes compromised and many surgeons will favour using an intramedullary device. This is an ongoing debate, and difficult to test in an exam setting.

Post operative management
Patients should be mobilised fully weight bearing where possible. Care is multidisciplinary in its delivery. Elderly patients should have orthogeriatrician assessment of comorbidity, and bone health with secondary prevention measures if appropriate. There should be early involvement of physiotherapy and occupational therapy services.

87
Q

A 14 year old boy jumps off a 10 foot wall and lands on both feet. An x-ray shows a bimalleolar fracture of the right ankle.

 A.	Smith's
B.	Bennett's
C.	Monteggia's
D.	Colle's
E.	Galeazzi
F.	Pott's
G.	Barton's
A

Pott’s

Colles’ fracture (dinner fork deformity)
Fall onto extended outstretched hand
Classical Colles’ fractures have the following 3 features:

  1. Transverse fracture of the radius
  2. 1 inch proximal to the radio-carpal joint
  3. Dorsal displacement and angulation

Smith’s fracture (reverse Colles’ fracture)
Volar angulation of distal radius fragment (Garden spade deformity)
Caused by falling backwards onto the palm of an outstretched hand or falling with wrists flexed

Bennett’s fracture
Intra-articular fracture of the first carpometacarpal joint
Impact on flexed metacarpal, caused by fist fights
X-ray: triangular fragment at ulnar base of metacarpal

Image sourced from Wikipedia
Monteggia’s fracture
Dislocation of the proximal radioulnar joint in association with an ulna fracture
Fall on outstretched hand with forced pronation
Needs prompt diagnosis to avoid disability

Image sourced from Wikipedia
Galeazzi fracture
Radial shaft fracture with associated dislocation of the distal radioulnar joint
Direct blow

Pott’s fracture
Bimalleolar ankle fracture
Forced foot eversion

Barton’s fracture
Distal radius fracture (Colles’/Smith’s) with associated radiocarpal dislocation
Fall onto extended and pronated wrist
Involvement of the joint is a defining feature

88
Q

A 22 year old drunk man is involved in a fight. He hurts his thumb when he punches his opponent.

A.	Smith's
B.	Bennett's
C.	Monteggia's
D.	Colle's
E.	Galeazzi
F.	Pott's
G.	Barton's
A

Bennett’s

Colles’ fracture (dinner fork deformity)
Fall onto extended outstretched hand
Classical Colles’ fractures have the following 3 features:

  1. Transverse fracture of the radius
  2. 1 inch proximal to the radio-carpal joint
  3. Dorsal displacement and angulation

Smith’s fracture (reverse Colles’ fracture)
Volar angulation of distal radius fragment (Garden spade deformity)
Caused by falling backwards onto the palm of an outstretched hand or falling with wrists flexed

Bennett’s fracture
Intra-articular fracture of the first carpometacarpal joint
Impact on flexed metacarpal, caused by fist fights
X-ray: triangular fragment at ulnar base of metacarpal

Image sourced from Wikipedia
Monteggia’s fracture
Dislocation of the proximal radioulnar joint in association with an ulna fracture
Fall on outstretched hand with forced pronation
Needs prompt diagnosis to avoid disability

Image sourced from Wikipedia
Galeazzi fracture
Radial shaft fracture with associated dislocation of the distal radioulnar joint
Direct blow

Pott’s fracture
Bimalleolar ankle fracture
Forced foot eversion

Barton’s fracture
Distal radius fracture (Colles’/Smith’s) with associated radiocarpal dislocation
Fall onto extended and pronated wrist
Involvement of the joint is a defining feature

89
Q

A 63 year nurse falls on an extended and pronated wrist. An x-ray shows a distal radial fracture with radiocarpal dislocation.

A.	Smith's
B.	Bennett's
C.	Monteggia's
D.	Colle's
E.	Galeazzi
F.	Pott's
G.	Barton's
A

Barton’s

Bartons fractures tend to have intra-articular involvement and dislocation may sometimes be present.

Colles’ fracture (dinner fork deformity)
Fall onto extended outstretched hand
Classical Colles’ fractures have the following 3 features:

  1. Transverse fracture of the radius
  2. 1 inch proximal to the radio-carpal joint
  3. Dorsal displacement and angulation

Smith’s fracture (reverse Colles’ fracture)
Volar angulation of distal radius fragment (Garden spade deformity)
Caused by falling backwards onto the palm of an outstretched hand or falling with wrists flexed

Bennett’s fracture
Intra-articular fracture of the first carpometacarpal joint
Impact on flexed metacarpal, caused by fist fights
X-ray: triangular fragment at ulnar base of metacarpal

Image sourced from Wikipedia
Monteggia’s fracture
Dislocation of the proximal radioulnar joint in association with an ulna fracture
Fall on outstretched hand with forced pronation
Needs prompt diagnosis to avoid disability

Image sourced from Wikipedia
Galeazzi fracture
Radial shaft fracture with associated dislocation of the distal radioulnar joint
Direct blow

Pott’s fracture
Bimalleolar ankle fracture
Forced foot eversion

Barton’s fracture
Distal radius fracture (Colles’/Smith’s) with associated radiocarpal dislocation
Fall onto extended and pronated wrist
Involvement of the joint is a defining feature

90
Q

A 32 year old man falls from a ladder and sustains a fracture of his proximal radius. On examination, he has severe pain in his forearm and diminished distal sensation. There is a single puncture wound present at the fracture site.

A. Application of external fixator
B. Open reduction and internal fixation
C. Fasciotomy
D. Skeletal traction

A

Fasciotomy

Pain and neurological symptoms in a tight fascial compartment coupled with a high velocity injury carry a high risk of compartment syndrome and prompt fasciotomy should be performed.

Fracture management

Bony injury resulting in a fracture may arise from trauma (excessive forces applied to bone), stress related (repetitive low velocity injury) or pathological (abnormal bone which fractures during normal use of following minimal trauma)
Diagnosis involves not just evaluating the fracture ; such as site and type of injury but also other associated injuries and distal neurovascular deficits. This may entail not just clinical examination but radiographs of proximal and distal joints.
When assessing x-rays it is important to assess for changes in length of the bone, the angulation of the distal bone, rotational effects, presence of material such as glass.

Fracture types
Fracture type Description
Oblique fracture Fracture lies obliquely to long axis of bone
Comminuted fracture >2 fragments
Segmental fracture More than one fracture along a bone
Transverse fracture Perpendicular to long axis of bone
Spiral fracture Severe oblique fracture with rotation along long axis of bone

Open Vs Closed
It is also important to distinguish open from closed injuries. The most common classification system for open fractures is the Gustilo and Anderson classification system (given below):

Grade Injury
1 Low energy wound <1cm
2 Greater than 1cm wound with moderate soft tissue damage
3 High energy wound > 1cm with extensive soft tissue damage
3 A (sub group of 3) Adequate soft tissue coverage
3 B (sub group of 3) Inadequate soft tissue coverage
3 C (sub group of 3) Associated arterial injury

Key points in management of fractures
Immobilise the fracture including the proximal and distal joints
Carefully monitor and document neurovascular status, particularly following reduction and immobilisation
Manage infection including tetanus prophylaxis
IV broad spectrum antibiotics for open injuries
As a general principle all open fractures should be thoroughly debrided ( and internal fixation devices avoided or used with extreme caution)
Open fractures constitute an emergency and should be debrided and lavaged within 6 hours of injury

91
Q

A 32 year old man falls a sustains a fracture of his distal humerus. The fracture segment is markedly angulated and unstable. There is a puncture site overlying the fracture site.

A. Application of external fixator
B. Open reduction and internal fixation
C. Fasciotomy
D. Skeletal traction

A

Application of external fixator

Wide exposure to plate the humerus is generally inadvisable owing to its many important anatomical relations. Both intramedullary nailing and external fixation are reasonable treatments. However, in the presence of an open fracture application of an external fixator and appropriate tissue debridement would be most appropriate.

Fracture management

Bony injury resulting in a fracture may arise from trauma (excessive forces applied to bone), stress related (repetitive low velocity injury) or pathological (abnormal bone which fractures during normal use of following minimal trauma)
Diagnosis involves not just evaluating the fracture ; such as site and type of injury but also other associated injuries and distal neurovascular deficits. This may entail not just clinical examination but radiographs of proximal and distal joints.
When assessing x-rays it is important to assess for changes in length of the bone, the angulation of the distal bone, rotational effects, presence of material such as glass.

Fracture types
Fracture type Description
Oblique fracture Fracture lies obliquely to long axis of bone
Comminuted fracture >2 fragments
Segmental fracture More than one fracture along a bone
Transverse fracture Perpendicular to long axis of bone
Spiral fracture Severe oblique fracture with rotation along long axis of bone

Open Vs Closed
It is also important to distinguish open from closed injuries. The most common classification system for open fractures is the Gustilo and Anderson classification system (given below):

Grade Injury
1 Low energy wound <1cm
2 Greater than 1cm wound with moderate soft tissue damage
3 High energy wound > 1cm with extensive soft tissue damage
3 A (sub group of 3) Adequate soft tissue coverage
3 B (sub group of 3) Inadequate soft tissue coverage
3 C (sub group of 3) Associated arterial injury

Key points in management of fractures
Immobilise the fracture including the proximal and distal joints
Carefully monitor and document neurovascular status, particularly following reduction and immobilisation
Manage infection including tetanus prophylaxis
IV broad spectrum antibiotics for open injuries
As a general principle all open fractures should be thoroughly debrided ( and internal fixation devices avoided or used with extreme caution)
Open fractures constitute an emergency and should be debrided and lavaged within 6 hours of injury

92
Q

A 24 year old man sustains a distal radius fracture during a game of rugby. Imaging shows a comminuted fracture with involvement of the articular surface.

A. Application of external fixator
B. Open reduction and internal fixation
C. Fasciotomy
D. Skeletal traction

A

Open reduction and internal fixation

Meticulous anatomical alignment of the fracture segments is crucial to avoid the development of osteoarthritis and risk of malunion.

Fracture management

Bony injury resulting in a fracture may arise from trauma (excessive forces applied to bone), stress related (repetitive low velocity injury) or pathological (abnormal bone which fractures during normal use of following minimal trauma)
Diagnosis involves not just evaluating the fracture ; such as site and type of injury but also other associated injuries and distal neurovascular deficits. This may entail not just clinical examination but radiographs of proximal and distal joints.
When assessing x-rays it is important to assess for changes in length of the bone, the angulation of the distal bone, rotational effects, presence of material such as glass.

Fracture types
Fracture type Description
Oblique fracture Fracture lies obliquely to long axis of bone
Comminuted fracture >2 fragments
Segmental fracture More than one fracture along a bone
Transverse fracture Perpendicular to long axis of bone
Spiral fracture Severe oblique fracture with rotation along long axis of bone

Open Vs Closed
It is also important to distinguish open from closed injuries. The most common classification system for open fractures is the Gustilo and Anderson classification system (given below):

Grade Injury
1 Low energy wound <1cm
2 Greater than 1cm wound with moderate soft tissue damage
3 High energy wound > 1cm with extensive soft tissue damage
3 A (sub group of 3) Adequate soft tissue coverage
3 B (sub group of 3) Inadequate soft tissue coverage
3 C (sub group of 3) Associated arterial injury

Key points in management of fractures
Immobilise the fracture including the proximal and distal joints
Carefully monitor and document neurovascular status, particularly following reduction and immobilisation
Manage infection including tetanus prophylaxis
IV broad spectrum antibiotics for open injuries
As a general principle all open fractures should be thoroughly debrided ( and internal fixation devices avoided or used with extreme caution)
Open fractures constitute an emergency and should be debrided and lavaged within 6 hours of injury

93
Q

A 55 year old motorcyclist is involved in a road traffic accident and sustained a Gustilo and Anderson IIIc type fracture to the distal tibia. He was trapped in the wreckage for 7 hours during which time he bled profusely from the fracture site. He has an established distal neurovascular deficit.

A. Copious lavage and generous surgical debridement, followed by external fixation
B. Intramedullary nail
C. Open reduction and internal fixation
D. Immobilisation in plaster cast
E. External fixation using a frame device
F. Amputation
G. Application of external fixation device
H. Primary closure of wound and application of plaster cast

A

Amputation

This man is unstable, and at 7 hours after extraction, the limb is not viable. The safest option is primary amputation.

Fracture management

Bony injury resulting in a fracture may arise from trauma (excessive forces applied to bone), stress related (repetitive low velocity injury) or pathological (abnormal bone which fractures during normal use of following minimal trauma)
Diagnosis involves not just evaluating the fracture ; such as site and type of injury but also other associated injuries and distal neurovascular deficits. This may entail not just clinical examination but radiographs of proximal and distal joints.
When assessing x-rays it is important to assess for changes in length of the bone, the angulation of the distal bone, rotational effects, presence of material such as glass.

Fracture types
Fracture type Description
Oblique fracture Fracture lies obliquely to long axis of bone
Comminuted fracture >2 fragments
Segmental fracture More than one fracture along a bone
Transverse fracture Perpendicular to long axis of bone
Spiral fracture Severe oblique fracture with rotation along long axis of bone

Open Vs Closed
It is also important to distinguish open from closed injuries. The most common classification system for open fractures is the Gustilo and Anderson classification system (given below):

Grade Injury
1 Low energy wound <1cm
2 Greater than 1cm wound with moderate soft tissue damage
3 High energy wound > 1cm with extensive soft tissue damage
3 A (sub group of 3) Adequate soft tissue coverage
3 B (sub group of 3) Inadequate soft tissue coverage
3 C (sub group of 3) Associated arterial injury

Key points in management of fractures
Immobilise the fracture including the proximal and distal joints
Carefully monitor and document neurovascular status, particularly following reduction and immobilisation
Manage infection including tetanus prophylaxis
IV broad spectrum antibiotics for open injuries
As a general principle all open fractures should be thoroughly debrided ( and internal fixation devices avoided or used with extreme caution)
Open fractures constitute an emergency and should be debrided and lavaged within 6 hours of injury

94
Q

A 25 year old ski instructor who falls off a ski lift and sustains a spiral fracture of the mid shaft of the tibia. Attempts to achieve satisfactory position in plaster have failed. Overlying tissues are healthy.

A. Copious lavage and generous surgical debridement, followed by external fixation
B. Intramedullary nail
C. Open reduction and internal fixation
D. Immobilisation in plaster cast
E. External fixation using a frame device
F. Amputation
G. Application of external fixation device
H. Primary closure of wound and application of plaster cast

A

Intramedullary nail

This would be a good case for intramedullary nailing. Open reduction and external fixation would strip off otherwise healthy tissues and hence is unsuitable. In some units the injury may be managed with an Ilizarov frame device but the majority would treat with IM nailing.

Fracture management

Bony injury resulting in a fracture may arise from trauma (excessive forces applied to bone), stress related (repetitive low velocity injury) or pathological (abnormal bone which fractures during normal use of following minimal trauma)
Diagnosis involves not just evaluating the fracture ; such as site and type of injury but also other associated injuries and distal neurovascular deficits. This may entail not just clinical examination but radiographs of proximal and distal joints.
When assessing x-rays it is important to assess for changes in length of the bone, the angulation of the distal bone, rotational effects, presence of material such as glass.

Fracture types
Fracture type Description
Oblique fracture Fracture lies obliquely to long axis of bone
Comminuted fracture >2 fragments
Segmental fracture More than one fracture along a bone
Transverse fracture Perpendicular to long axis of bone
Spiral fracture Severe oblique fracture with rotation along long axis of bone

Open Vs Closed
It is also important to distinguish open from closed injuries. The most common classification system for open fractures is the Gustilo and Anderson classification system (given below):

Grade Injury
1 Low energy wound <1cm
2 Greater than 1cm wound with moderate soft tissue damage
3 High energy wound > 1cm with extensive soft tissue damage
3 A (sub group of 3) Adequate soft tissue coverage
3 B (sub group of 3) Inadequate soft tissue coverage
3 C (sub group of 3) Associated arterial injury

Key points in management of fractures
Immobilise the fracture including the proximal and distal joints
Carefully monitor and document neurovascular status, particularly following reduction and immobilisation
Manage infection including tetanus prophylaxis
IV broad spectrum antibiotics for open injuries
As a general principle all open fractures should be thoroughly debrided ( and internal fixation devices avoided or used with extreme caution)
Open fractures constitute an emergency and should be debrided and lavaged within 6 hours of injury

95
Q

A 35 year old mechanic is hit by a fork lift truck. He sustains a Gustilo and Anderson type IIIA fracture of the shaft of the left femur.

A. Copious lavage and generous surgical debridement, followed by external fixation
B. Intramedullary nail
C. Open reduction and internal fixation
D. Immobilisation in plaster cast
E. External fixation using a frame device
F. Amputation
G. Application of external fixation device
H. Primary closure of wound and application of plaster cast

A

Copious lavage and generous surgical debridement, followed by external fixation

At the tissues are in better shape than in the first case and as there is no associated vascular injury the patient may be suitable for debridement of the area and external fixation. If debridement leaves a tissue defect then plastic surgical repair will be needed at a later stage.

Fracture management

Bony injury resulting in a fracture may arise from trauma (excessive forces applied to bone), stress related (repetitive low velocity injury) or pathological (abnormal bone which fractures during normal use of following minimal trauma)
Diagnosis involves not just evaluating the fracture ; such as site and type of injury but also other associated injuries and distal neurovascular deficits. This may entail not just clinical examination but radiographs of proximal and distal joints.
When assessing x-rays it is important to assess for changes in length of the bone, the angulation of the distal bone, rotational effects, presence of material such as glass.

Fracture types
Fracture type Description
Oblique fracture Fracture lies obliquely to long axis of bone
Comminuted fracture >2 fragments
Segmental fracture More than one fracture along a bone
Transverse fracture Perpendicular to long axis of bone
Spiral fracture Severe oblique fracture with rotation along long axis of bone

Open Vs Closed
It is also important to distinguish open from closed injuries. The most common classification system for open fractures is the Gustilo and Anderson classification system (given below):

Grade Injury
1 Low energy wound <1cm
2 Greater than 1cm wound with moderate soft tissue damage
3 High energy wound > 1cm with extensive soft tissue damage
3 A (sub group of 3) Adequate soft tissue coverage
3 B (sub group of 3) Inadequate soft tissue coverage
3 C (sub group of 3) Associated arterial injury

Key points in management of fractures
Immobilise the fracture including the proximal and distal joints
Carefully monitor and document neurovascular status, particularly following reduction and immobilisation
Manage infection including tetanus prophylaxis
IV broad spectrum antibiotics for open injuries
As a general principle all open fractures should be thoroughly debrided ( and internal fixation devices avoided or used with extreme caution)
Open fractures constitute an emergency and should be debrided and lavaged within 6 hours of injury

96
Q

A 30 year old man presents with severe pain in the left hip it has been present on and off for many years. He was born at 39 weeks gestation by emergency caesarean section after a long obstructed breech delivery. He was slow to walk and as a child was noted to have an antalgic gait. He was a frequent attender at the primary care centre and the pains dismissed as growing pains. X-rays show almost complete destruction of the femoral head and a narrow acetabulum. What is the most likely underlying disease process?

	Developmental dysplasia of the hip
	Slipped upper femoral epiphysis
	Extra capsular fracture of the femoral neck
	Rheumatoid arthritis
	Perthes disease
A

Developmental dysplasia of the hip. Usually diagnosed by Barlow and Ortolani tests in early childhood. Most Breech deliveries are also routinely subjected to USS of the hip joint. At this young age an arthrodesis may be preferable to hip replacement.
Developmental dysplasia of the hip Usually diagnosed in infancy by screening tests. May be bilateral, when disease is unilateral there may be leg length inequality. As disease progresses child may limp and then early onset arthritis. More common in extended breech babies. Splints and harnesses or traction. In later years osteotomy and hip realignment procedures may be needed. In arthritis a joint replacement may be needed. However, this is best deferred if possible as it will almost certainly require revision Initially no obvious change on plain films and USS gives best resolution until 3 months of age. On plain films Shentons line should form a smooth arc
Perthes Disease Hip pain (may be referred to the knee) usually occurring between 5 and 12 years of age. Bilateral disease in 20%. Remove pressure from joint to allow normal development. Physiotherapy. Usually self-limiting if diagnosed and treated promptly. X-rays will show flattened femoral head. Eventually in untreated cases the femoral head will fragment.
Slipped upper femoral epiphysis Typically seen in obese male adolescents. Pain is often referred to the knee. Limitation to internal rotation is usually seen. Knee pain is usually present 2 months prior to hip slipping. Bilateral in 20%. Bed rest and non-weight bearing. Aim to avoid avascular necrosis. If severe slippage or risk of it occurring then percutaneous pinning of the hip may be required. X-rays will show the femoral head displaced and falling inferolaterally (like a melting ice cream cone) The Southwick angle gives indication of disease severity

97
Q

An 80 year old woman has a hip fracture. Her calcium is normal. She has never been given a diagnosis of osteoporosis. Apart from treating the hip fracture what additional intervention should be considered?

	Vitamin D and calcium supplements alone
	Vitamin D, calcium supplements and bisphosphonates
	Vitamin D alone
	Calcium supplements alone
	DEXA scan
A

Vitamin D, calcium supplements and bisphosphonates

The osteoporosis guidelines state if a postmenopausal woman has a fracture she should be put on bisphosphonates (there is no need for a DEXA scan).

Osteoporosis is a condition of bone atrophy. It is a lesion in which the volume of bone tissue per unit volume of anatomical bone is reduced. It may be generalized (most common) or localized (following pressure or disuse). The hallmark is a reduction in the amount of osteoid matrix, which, however, remains normally mineralized. It is therefore distinct from osteomalacia in which there is abundant osteoid which is poorly calcified. In osteoporosis the bony trabeculae are greatly thinned which significantly reduces the tensile strength of the bone and renders it at increased risk of pathological fracture.
It is only visible on plain films when the calcium content is approximately halved. More subtle changes can be appreciated by use of DEXA scanning.

Treatment
Fractures are treated according to their site and mechanism of injury. Those deemed to occur as a result of osteoporosis should also be addressed medically to treat the underlying osteoporosis.
Drugs that can be prescribed to prevent fragility fractures include bisphosphonates (alendronate, ibandronate, risedronate and zoledronic acid) and non-bisphosphonates (raloxifene, denosumab, teriparatide, calcitriol and hormone replacement therapy).
Calcium and vitamin D supplements are also administered.

Individuals at risk of fragility fractures should also be considered for prophylactic medical treatment as outlined above.

98
Q

A 50 year old female slips on wet floor injuring her ankle. On examination, she has tenderness over the lateral and medial malleolus. X-rays demonstrate an undisplaced fracture of the distal fiibula at the level of the syndesmosis and a congruent ankle mortice. What is the most appropriate management?

	Application of full leg cast
	Surgical fixation
	Application of below knee plaster cast
	Application of external fixator
	Bed rest, splinting and traction
A

Application of below knee plaster cast

This is a Weber B fracture and therefore potentially unstable. Medial malleolar tenderness indicates deltoid ligament injury. As the fracture is currently undisplaced and the ankle mortice is congruent, the injury can be initially managed conservatively in a below knee plaster but the patient should be monitored in the outpatient clinic for fracture displacement in the first few weeks.

An ankle fracture relates to a fracture around the tibio-talar joint. It generally refers to a fracture involving the lateral, and/or medial and/or posterior malleolus. Pilon and Tillaux fractures are also considered to be ankle fractures, but are not covered here.
Ankle fractures are common. They effect men and women in equal numbers, but men have a higher rate as young adults (sports and contact injuries), and women a higher rate post-menopausal (fragility type fracture).

Osseous anatomy
The ankle (or mortise) joint consists of the distal tibia (tibial plafond and posterior malleolus), the distal fibula (lateral malleolus), and the talus. The main movement at the ankle joint is plantar and dorsiflexion.

Ligamentous anatomy
Medial side: Deltoid ligament. This is divided into superficial and deep portions. It is the primary restraint to valgus tilting of the talus.

Lateral side: Lateral ligament complex consisting from anterior to posterior of the anterior talofibular ligament (ATFL), calcaneofibular ligament (CFL), and the posterior talofibular ligament (PTFL). Together they resist valgus stress to the ankle, and are a restraint to anterior translation of the talus within the mortise joint.

Syndesmosis: The syndesmosis is a ligament complex between the distal tibia and fibula, holding the two bones together. It is fundamental to the integrity of the ankle joint, and its disruption leads to instability. It consists of (from anterior to posterior) the anterior-inferior tibiofibular ligament (AITFL), the transverse tibiofibular ligament (TTFL), the interosseous membrane, and the posterior-inferior tibiofibular ligament (PITFL).

Presentation and initial management
Patients will present following a traumatic event with a painful, swollen ankle, and reluctance/inability to weight bear. The Ottawa rules can be applied to differentiate between an ankle fracture and sprain, but can be unreliable.
In high energy injuries, management should follow ATLS principles to identify more significant injuries first. Neurovascular status of the foot should be documented, and open injuries should be excluded. If an open injury is identified, it should be managed in line with BOAST 4 principles1. If an obvious deformity exists, it should be reduced as soon as possible with appropriate analgesia or conscious sedation. Radiographs of clearly deformed or dislocated joints are not necessary, and removing the pressure on the surrounding soft tissues from the underlying bony deformity is the priority. If the fracture pattern is not clinically obvious then plain radiographs are appropriate and will guide the subsequent manipulation during plaster-of-paris below knee backslab application.

Imaging
AP, lateral and mortise views (20o internal rotation) are essential to evaluate fracture displacement and syndesmotic injury. Decreased tibiofibular overlap, medial joint clear space and lateral talar shift all indicate a syndesmotic injury. (In subtle cases of shift, imaging the uninjured ankle can be helpful as a proportion of the population have little or no tibiotalar overlap 2.)

Where there is suspicion of syndesmosis involvement in the absence of radiographic evidence, stress radiographs can be diagnostic.
Complex fracture patterns (and increasingly posterior malleolar fractures) are best defined using CT.

Classification
The most commonly used classifications are Lauge-Hansen and Danis-Weber.

Lauge-Hansen
Comprises two parts: first part is the foot position, and the second part is the force applied. Useful for understanding the forces involved and therefore predict the ligamentous or bony injury. Results in four injury patterns:
Supination - Adduction (SA) - 10-20%
Supination - External rotation (SER) - 40-75%
Pronation - Abduction (PA) - 5-20%
Pronation - External rotation (PER) - 5-20%

Not often used in clinical practice but good for understanding the principles of ankle fracture.

Danis-Weber
Commonly used. Based on the level of the fibula fracture in relation to the syndesmosis. The more proximal, the greater the risk of syndesmotic injury and therefore fracture instability.
A - fracture below the level of the syndesmosis
B - fracture at the level of the syndesmosis / level of the tibial plafond
C - fracture above the level of the syndesmosis. This includes Maisonneuve fractures (proximal fibula fracture), which can be associated with ankle instability. Beware the high fibula fracture - it may be an ankle fracture!

The Weber classification is based purely on the the lateral side. All injuries can include a medial or posterior bony or ligamentous injury which also dictates fracture stability (bimalleolar and trimalleolar fractures are more unstable).

Treatment
When deciding upon treatment for an ankle fracture, one must consider both the fracture and the patient. Diabetic patients and smokers are at greater risk of post-operative complication, especially wound problems and infection. Likewise, the long term outcome of post-traumatic arthritis from a malunited ankle fracture is extremely important for a young patient, but not as relevant in the elderly. Therefore, normal surgical decision processes apply as with all fractures.

Defining stability of an ankle fracture underpins the treatment decision.
Weber A - Unimalleolar Weber A Weber fractures by definition are stable and therefore can be mobilised fully weight bearing in an ankle boot.

Weber C - Fractures tend to include syndesmotic disruption and are usually bimalleolar (either bony or ligamentous). They are therefore unstable and usually require operative fixation. In addition to the fracture fixation, the syndesmosis usually requires reconstruction/augmentation with screws to restore the joint integrity and function.

Weber B - B fractures vary greatly. They can be part of a trimalleolar injury and therefore extremely unstable, requiring fixation. Alternatively, a uni-malleolar Weber B fracture can be a stable injury, and therefore mobilised immediately in an ankle boot. Defining the stability can be challenging, and often involves stress radiographs, or a trial of mobilisation and repeat radiographs. Defining stability is the subject of much ongoing research. However, treating undisplaced ankle fractures in a below knee plaster, non-weight bearing for six weeks is still widely practised, and a safe approach.

When operative fixation is appropriate, it is usually via open reduction and internal fixation using plates and screws. It must be carried out when soft tissue swelling has settled in order to minimise the risk of wound problems. This can often take a week to settle.

The use of fibula nails is expanding, but is not yet mainstream. Ankle fractures can also be treated with external fixation, or with a hind foot nail in patients who need fixation but where soft tissue or bone quality is poor.

Post operative management
Ankle fractures generally take 6 weeks to unite enough to prevent secondary displacement. This is therefore an appropriate time period to keep a cast on in a conservatively managed patient. Weight bearing post-operatively depends on the quality of the fixation and bone quality, and preference varies between surgeons, ranging from aggressive early mobilisation to a period of non-weight bearing. Return to activities takes approximately three months, and often requires assistance of a physiotherapist to improve range-of-movement and muscle strengthening.

99
Q

A 78 year old man complains of a long history of shoulder pain and more recently weakness. On examination, active attempts at abduction are impaired. Passive movements are normal. What is the most likely diagnosis?

	Rotator cuff tear
	Osteoarthritis
	Metastatic malignancy
	Adhesive capsulitis
	Calcific tendonitis
A

Rotator cuff tears are common in elderly people and may occur following minor trauma or as a result of long standing impingement. Tears greater than 2cm should generally be repaired surgically. The length of the history in this scenario is suggestive of a tear complicating impingement.

Shoulder fractures and dislocations
Fractures
Proximal humerus
Background
Third most common fragility fracture in the elderly.
Results from low energy fall in predominantly elderly females, or from high energy trauma in young males.
Can be associated with nerve injury (commonly axillary), and fracture-dislocation of the humeral head. Detailed neurological assessment is essential for all upper limb injuries.

Anatomy
Osteology
Consists of articular head, greater tuberosity, lesser tuberosity, metaphysis and diaphysis. Between the articular head and the tuberosities is the anatomical neck (previous physis). Between the tuberosities and the metaphysis is the surgical neck.
The supraspinatus, infraspinatus and teres minor muscles attach to the greater tuberosity. The subscapularis muscle attaches to the lesser tuberosity.

Vascular Supply
Humeral head is supplied by the anterior and posterior humeral circumflex arteries. Anatomical neck fractures are at greatest risk of osteonecrosis.

Imaging
Imaging aims to both delineate the fracture pattern, and confirm/exlude the presence of an associated dislocation.
Radiographs - True anteroposterior (AP), axillary lateral and/or scapula Y view.
CT - indicated to better define intra-articular involvement and to aid pre-operative planning. MRI is not useful for fracture imaging.

Classification
Description of the fracture is often more useful than classification. Particular attention should be paid to humeral alignment, fracture displacement, and greater tuberosity position (rotator cuff will pull the GT supero-posterioly, which can cause impingement problems with malunion).
- Neer Classification: Most commonly used. Describes fracture as 2,3,or 4 part depending upon the number main fragments. Also comments on the degree of displacement. Fragments:
-greater tuberosity
-lesser tuberosity
- articular surface
- shaft
Displacement: >1cm or angulation >45 degrees.

Treatment
The vast majority of proximal humeral fractures are minimally displaced, and therefore can be managed conservatively. This involves immobilisation in a polysling, and progressive mobilisation. Pendular exercise can commence at 14 days, and active abduction from 4-6 weeks.

Irreducible fracture dislocation is an indication for operative management. Other indications include large displacement, younger patient, head splitting (intra-articular fractures). However, the recent PROFHER trial (1) has suggested no benefit to operative intervention on patient outcome (it must be applied cautiously as majority of patients were elderly with extraarticular fractures). Options available for surgical management include:

ORIF Most commonly used. Plate and screw fixation. Can reconstruct complex fractures.
Intramedullary nail Suitable for extra-articular configuration, predominantly surgical neck +/- GT fractures.

Hemiarthroplasty Used for un-reconstructable fractures in the older patient who has good glenoid quality.
Total shoulder arthroplasty Unconstructable fractures where high functioning shoulder is required (hemiarthroplasty will cause glenoid erosion)
Reverse shoulder arthroplasty Total shoulder arthroplasty that provides better functional outcome than conventional total shoulder replacement.

Scapula
Background
Uncommon fractures usually associated with high energy trauma. Most commonly involve scapula body or spine (50%), glenoid fossa and glenoid neck. Important to exclude associated life threatening injury.

Imaging
Plain radiographs should include true anteroposterior (AP), axillary lateral and/or scapula Y view. CT scanning is useful for defining intra-articular involvement, displacement and for three dimensional reconstruction.

Classification
Based on the location of the fracture (coracoid, acromion, glenoid neck, glenoid fossa, scapula body). Beware of ipsilateral glenoid neck and clavicle fracture -floating shoulder - where limb is effectively dissociated from axial skeleton.

Treatment
The vast majority of scapula fractures are amenable to conservative management, consisting of sling immobilisation for two weeks followed by early rehabilitation. Floating shoulder will usually require fixation, and consideration of surgery should also be given to intra-articular and displaced/angulated glenoid fractures.

Dislocations

Types
Dislocations around the shoulder joint include glenohumeral dislocation, acromioclavicular joint disruption and sternoclavicular dislocation. Only glenohumeral dislocation will be covered here.

Glenohumeral dislocation
Diagnosis, classification and management are covered here.

Background
Shoulder dislocation is commonly seen in A&E. It has a high recurrence rate that is as high as 80% in teenagers. Initial management requires emergent reduction to prevent lasting chondral damage.

Early assessment and management
Usually a traumatic cause (multi-directional instability in frequent dislocations requires discussion with orthopaedics and is not covered here). Careful history, examination and documentation of neurovascular status of the limb, in particular the axillary nerve (regimental badge sensation). This should be re-assessed post manipulation. Early radiographs to confirm direction of dislocation.

Initial management consists of emergent closed reduction under under entanox and analgesia, but often requires conscious sedation. Arm should then be immobilised in a polysling, and XR to confirm relocation.

Imaging - True anteroposterior (AP), axillary lateral and/or scapula Y view. Reduced humeral head should lie between acromion and coracoid on lateral/scapula view.

Types
Direction Features Cause Examination Reduction techniques
Anterior Most Common >90% Usually traumatic - anterior force on arm when shoulder is abducted, eternally rotated Loss of shoulder contour - sulcus sign. Humeral head can be felt anteriorly.
Hippocratic.
Milch.
Stimson.

Kocher not advised due to complication of fracture
Posterior 50% missed in A&E 50% traumatic, but classically post seizure or electrocution Shoulder locked in internal rotation. XR may show lightbulb appearance. Gentle lateral traction to adducted arm.
Inferior Rare Associated with pectorals and rotator cuff tears, and glenoid fracture As for primary injury Management of primary injury

Superior Rare Associated with acrominon/clavicle fracture As for primary injury Management of primary injury

Associated injuries
Bankart lesion - avulsion of the anterior glenoid labrum with an anterior shoulder dislocation (reverse Bankart if poster labrum in posterior dislocation).
Hill Sachs defect - chondral impaction on posteriosuperior humeral head from contact with gleonoid rim. Can be large enough to lock shoulder, requiring open reduction. (Reverse Hill Sachs in posterior dislocation).
Rotator cuff tear - increases with age.
Greater or lesser tuberosity fracture - increases with age.
Humeral neck fracture - shoulder fracture dislocation. More common in high energy trauma and elderly. Should be discussed with orthopaedics prior to any attempted reduction.

Rotator Cuff Disease

Rotator cuff disease is a spectrum of conditions that ranges from subacromial impingement to rotator cuff tears and eventually to rotator cuff arthropathy (arthritis).

Anatomy
The rotator cuff is a group of four muscles that are important in shoulder movements, and maintenance of glenohumeral stability.
Muscle Scapular attachment Humeral attachment Action Innervation
Supraspinatus Supraspinatus fossa Superior facet of greater tuberosity Initiation of abduction of humerus Suprascapular nerve
Infraspinatus Infraspinatus fossa Posterior facet of greater tuberosity External rotation of humerus Suprascapular nerve
Teres Minor Lateral border Inferior facet of greater tuberosity External rotation of humerus Axillary Nerve
Subscapularis Subscapular fossa Lesser tuberosity Internal rotation of humerus Upper and lower subscapular nerve

The inferior rotator cuff muscles (infraspinatus, teres minor, and subscapularis) balance the superior pull of the deltoid. Injury/tear results in upward migration of the humeral head on the glenoid (can be seen on AP radiograph).
Likewise, the anterior muscles (subscapularis) are balanced with the posterior muscles (infraspinatus, teres minor).

Subacromial Impingement

The most common cause of shoulder pain, which results from impingement of the superior cuff on the undersurface of the acromion, and an inflammatory bursitis.
Associated with certain types of acromial morphology (Bigliani classification).
Presents as insidious pain which is exacerbated by overhead activities.

Rotator Cuff Tear

Often presents as an acute event on the background of chronic subacromial impingement in the older patient, but can present as an avulsion injury in younger patients.
Majority of tears are to the superior cuff (supraspinatus, infraspinatus, teres minor), though a tear to subscapularis is associated with subcoracoid impingement.
Tears present as pain and weakness when using the muscles in question.

Rotator Cuff Arthropathy

Defined as shoulder arthritis in the setting of rotator cuff dysfunction. Results from superior migration due to the loss of rotator cuff function and integrity. Unopposed deltoid pulls the humeral head superiorly.
Associated with massive chronic cuff tears.

Imaging

Plain radiographs
AP of the shoulder may show superior migration of the humerus with a cuff tear, and features of arthritis with arthropathy. Other causes of pain may also be identified (e.g. calcific tendonitis/fracture)
Outlet view is useful for defining the acromial morphology

USS
Allows dynamic imaging of the cuff, and is inexpensive. However, it is very user dependent.

MRI
Best imaging modality for cuff pathology.
Also allows imaging of the rest of the shoulder. When intra-articular pathology is suspected, can be combined with an arthrogram for improved sensitivity and specificity.

Treatment

Subacromial impingement
Physiotherapy, oral anti-inflammatory medication
Subacromial steroid injection can settle inflammation
Arthroscopic subacromial decompression by shaving away the undersurface of the acromion, more space is created for the rotator cuff. Cuff integrity is assessed also at time of surgery, and can be repaired if necessary.

Rotator cuff tear
When considering repair of a cuff tear, the age and activity of the patient, the nature of the tear (degenerative vs. acute traumatic), and the size and retraction of the tear should be considered when making a surgical plan.
Mild tears or tears in the elderly can be managed conservatively, as outlined above.
Moderate tears can be repaired arthroscopically. Massive or retracted tears will often require an open repair (occasionally with a tendon transfer). Subacromial decompression is performed at the same time to reduce impingement, symptoms and recurrence.

Calcific tendonitis
Calcific tendonitis involves calcific deposits within tendons anywhere in the body, but most commonly in the rotator cuff (specifically the supraspinatus tendon). When present in the shoulder, it is associated with subacromial impingement and pain.

Pathology
More common in women aged 30-60 years.
Association with diabetes and hypothyroidism

There are three stages of calcification
Formative phase characterized by calcific deposits
Resting phase deposit is stable, but presents with impingement problems
Resorptive phase phagocytic resorption. Most painful stage.

Presentation
Similar in presentation to subacromial impingement, with pain especially with over head activities. Atraumatic in nature.

Imaging
Plain radiographs show calcification of the rotator cuff, usually within 1.5cm of its insertion on the humerus. Supraspinatus outlet views can show level of impingment. Further imaging is rarely needed.

Treatment
Non-operative NSAIDS, steroid injection (controversial, but practiced) and physiotherapy. Approximately 75% will resolve by 6 months with conservative management.
Ultrasound guided or surgical needle barbotage can break down deposits and resolve symptoms. Occasionally surgical excision is required.

Adhesive capsulitis (Frozen Shoulder)
Pain and loss of movement of shoulder joint, which involves fibroplastic proliferation of capsular tissue, causing soft tissue scarring and contracture. Patients present with a painful and decreased arc of motion.
Associated with prolonged immobilization, previous surgery, thyroid disorders (AI) and diabetes
Classically three stages which can take up to two years to resolve:
Stage one the freezing and painful stage
Stage two the frozen and stiff stage
Stage three the thawing stage, where shoulder movement slowly improves

Imaging
Plain radiographs to exclude other causes of a painful shoulder
MRI arthrogram may show capsular contracture, and again may be used to exclude cuff pathology. However, often not performed as diagnosis is largely clinical.

Treatment
Non-operative NSAIDS, steroid injection and physiotherapy. Patience is required as condition can take up to 2 years to improve.
Operative MUA or arthroscopic adhesiolysis (release of adhesions) can expedite recovery, followed by intensive physiotherapy.

Glenohumeral Arthritis
Background
May be osteoarthritis (primary or secondary to cuff tear or trauma), rheumatoid arthritis, or as part of a spondyloarthropathy. Majority of those with RA will develop symptoms.
More common in the elderly
Presents like any other arthritis - pain at night and with movement

Imaging
AP and axillary radiographs will show features of arthritis.
CT/MRI is often useful to classify the shape of the glenoid and extent of bone loss when considering arthroplasty. MRI also essential to asses integrity of rotator cuff if considering shoulder replacement.

Treatment
Like all orthopaedics, start with simple measures:
NSAIDS, management of RA, physiotherapy, steroid injection.
Hemiarthroplasty can sometimes be considered if glenoid is in excellent condition or if patient has large comorbidity.
Arthroscopic debridement is useful if patient has isolated ACJ arthritis, but is rarely used for glenohumeral arthritis.
Total shoulder replacement is shown to produce superior outcome when compared to hemiarthroplasty in terms of pain relief, function and implant survival.
Total shoulder replacement can be anatomical (ball on humerus, with cup on glenoid), or reverse geometry (ball on glenoid, with cup on humerus). Anatomical TSR requires an in tact rotator cuff, so often reverse is preferable when the cuff if questionable in integrity.

100
Q

A 24 year old man sustains a distal radius fracture during a game of rugby. Imaging shows a comminuted fracture with involvement of the articular surface. What is the most appropriate management?

Open reduction and internal fixation
Reduction under anaesthesia and place in plaster cast
Reduction under haematoma block and place into plaster cast
Place onto skeletal traction system
Apply a futura splint and review in fracture clinic
A

Open reduction and internal fixation
Meticulous anatomical alignment of the fracture segments is crucial to avoid the development of osteoarthritis and risk of malunion.
Distal radius fracture

Common, usual mechanism is fall onto an outstretched hand in elderly females with underlying osteoporosis
Typically it is a distal radius fracture occurring about 1 inch proximal to the wrist joint with dorsal angulation of the fracture fragment. The distal end of the ulna is sometimes involved
Management is usually conservative with reduction of the fracture under either a haematoma or Biers block and immobilisation in a cast
Potentially unstable injuries should be managed with surgical fixation. Factors favoring instability include; dorsal tilt of more than 20 degrees, comminuted fracture, injury to the ulnar styloid, intra articular disruption
Young patients, who typically have a higher energy mechanism of injury are usually managed surgically
In the elderly it is important to initiate treatment for osteoporosis

101
Q

A 15 year old boy is brought to the clinic by his mother who is concerned that he has a mark overlying his lower spine. On examination, the boy has a patch of hair overlying his lower lumbar spine and a birth mark at the same location. Lower limb neurological examination is normal. What is the most likely cause?

	Spina bifida occulta
	Meningomyelocele
	Spondylolisthesis
	Scheuermanns disease
	Myelocele
A

Spina bifida occulta is a common condition and may affect up to 10% of the population. The more severe types of spina bifida have more characteristic skin changes. Occasionally the unwary surgeon is persuaded to operate on these cutaneous changes and we would advocate performing an MRI scan prior to any such surgical procedure in this region.

Ankylosing spondylitis
Chronic inflammatory disorder affecting the axial skeleton
Sacro-ilitis is a usually visible in plain films
Up to 20% of those who are HLA B27 positive will develop the condition
Affected articulations develop bony or fibrous changes
Typical spinal features include loss of the lumbar lordosis and progressive kyphosis of the cervico-thoracic spine
Scheuermann’s disease
Epiphysitis of the vertebral joints is the main pathological process
Predominantly affects adolescents
Symptoms include back pain and stiffness
X-ray changes include epiphyseal plate disturbance and anterior wedging
Clinical features include progressive kyphosis (at least 3 vertebrae must be involved)
Minor cases may be managed with physiotherapy and analgesia, more severe cases may require bracing or surgical stabilisation
Scoliosis
Consists of curvature of the spine in the coronal plane
Divisible into structural and non structural, the latter being commonest in adolescent females who develop minor postural changes only. Postural scoliosis will typically disappear on manoeuvres such as bending forwards
Structural scoliosis affects > 1 vertebral body and is divisible into idiopathic, congential and neuromuscular in origin. It is not correctable by alterations in posture
Within structural scoliosis, idiopathic is the most common type
Severe, or progressive structural disease is often managed surgically with bilateral rod stabilisation of the spine
Spina bifida
Non fusion of the vertebral arches during embryonic development
Three categories; myelomeningocele, spina bifida occulta and meningocele
Myelomeningocele is the most severe type with associated neurological defects that may persist in spite of anatomical closure of the defect
Up to 10% of the population may have spina bifida occulta, in this condition the skin and tissues (but not not bones) may develop over the distal cord. The site may be identifiable by a birth mark or hair patch
The incidence of the condition is reduced by use of folic acid supplements during pregnancy
Spondylolysis
Congenital or acquired deficiency of the pars interarticularis of the neural arch of a particular vertebral body, usually affects L4/ L5
May be asymptomatic and affects up to 5% of the population
Spondylolysis is the commonest cause of spondylolisthesis in children
Asymptomatic cases do not require treatment
Spondylolisthesis
This occurs when one vertebra is displaced relative to its immediate inferior vertebral body
May occur as a result of stress fracture or spondylolysis
Traumatic cases may show the classic “Scotty Dog” appearance on plain films
Treatment depends upon the extent of deformity and associated neurological symptoms, minor cases may be actively monitored. Individuals with radicular symptoms or signs will usually require spinal decompression and stabilisation

102
Q

An 82 year old female presents to the emergency room after tripping on a step. She complains of shoulder pain. On examination there is pain to 90o on abduction. What is the most likely diagnosis?

	Glenohumeral dislocation
	Fracture of the anatomical neck of the humerus
	Sternoclavicular dislocation
	Supraspinatus tear
	Infraspinatus tear
A

A supraspinatus tear is the most common of rotator cuff tears. It occurs as a result of degeneration and is rare in younger adults

Shoulder fractures and dislocations
Fractures
Proximal humerus
Background
Third most common fragility fracture in the elderly.
Results from low energy fall in predominantly elderly females, or from high energy trauma in young males.
Can be associated with nerve injury (commonly axillary), and fracture-dislocation of the humeral head. Detailed neurological assessment is essential for all upper limb injuries.

Anatomy
Osteology
Consists of articular head, greater tuberosity, lesser tuberosity, metaphysis and diaphysis. Between the articular head and the tuberosities is the anatomical neck (previous physis). Between the tuberosities and the metaphysis is the surgical neck.
The supraspinatus, infraspinatus and teres minor muscles attach to the greater tuberosity. The subscapularis muscle attaches to the lesser tuberosity.

Vascular Supply
Humeral head is supplied by the anterior and posterior humeral circumflex arteries. Anatomical neck fractures are at greatest risk of osteonecrosis.

Imaging
Imaging aims to both delineate the fracture pattern, and confirm/exlude the presence of an associated dislocation.
Radiographs - True anteroposterior (AP), axillary lateral and/or scapula Y view.
CT - indicated to better define intra-articular involvement and to aid pre-operative planning. MRI is not useful for fracture imaging.

Classification
Description of the fracture is often more useful than classification. Particular attention should be paid to humeral alignment, fracture displacement, and greater tuberosity position (rotator cuff will pull the GT supero-posterioly, which can cause impingement problems with malunion).
- Neer Classification: Most commonly used. Describes fracture as 2,3,or 4 part depending upon the number main fragments. Also comments on the degree of displacement. Fragments:
-greater tuberosity
-lesser tuberosity
- articular surface
- shaft
Displacement: >1cm or angulation >45 degrees.

Treatment
The vast majority of proximal humeral fractures are minimally displaced, and therefore can be managed conservatively. This involves immobilisation in a polysling, and progressive mobilisation. Pendular exercise can commence at 14 days, and active abduction from 4-6 weeks.

Irreducible fracture dislocation is an indication for operative management. Other indications include large displacement, younger patient, head splitting (intra-articular fractures). However, the recent PROFHER trial (1) has suggested no benefit to operative intervention on patient outcome (it must be applied cautiously as majority of patients were elderly with extraarticular fractures). Options available for surgical management include:

ORIF Most commonly used. Plate and screw fixation. Can reconstruct complex fractures.
Intramedullary nail Suitable for extra-articular configuration, predominantly surgical neck +/- GT fractures.

Hemiarthroplasty Used for un-reconstructable fractures in the older patient who has good glenoid quality.
Total shoulder arthroplasty Unconstructable fractures where high functioning shoulder is required (hemiarthroplasty will cause glenoid erosion)
Reverse shoulder arthroplasty Total shoulder arthroplasty that provides better functional outcome than conventional total shoulder replacement.

Scapula
Background
Uncommon fractures usually associated with high energy trauma. Most commonly involve scapula body or spine (50%), glenoid fossa and glenoid neck. Important to exclude associated life threatening injury.

Imaging
Plain radiographs should include true anteroposterior (AP), axillary lateral and/or scapula Y view. CT scanning is useful for defining intra-articular involvement, displacement and for three dimensional reconstruction.

Classification
Based on the location of the fracture (coracoid, acromion, glenoid neck, glenoid fossa, scapula body). Beware of ipsilateral glenoid neck and clavicle fracture -floating shoulder - where limb is effectively dissociated from axial skeleton.

Treatment
The vast majority of scapula fractures are amenable to conservative management, consisting of sling immobilisation for two weeks followed by early rehabilitation. Floating shoulder will usually require fixation, and consideration of surgery should also be given to intra-articular and displaced/angulated glenoid fractures.

Dislocations

Types
Dislocations around the shoulder joint include glenohumeral dislocation, acromioclavicular joint disruption and sternoclavicular dislocation. Only glenohumeral dislocation will be covered here.

Glenohumeral dislocation
Diagnosis, classification and management are covered here.

Background
Shoulder dislocation is commonly seen in A&E. It has a high recurrence rate that is as high as 80% in teenagers. Initial management requires emergent reduction to prevent lasting chondral damage.

Early assessment and management
Usually a traumatic cause (multi-directional instability in frequent dislocations requires discussion with orthopaedics and is not covered here). Careful history, examination and documentation of neurovascular status of the limb, in particular the axillary nerve (regimental badge sensation). This should be re-assessed post manipulation. Early radiographs to confirm direction of dislocation.

Initial management consists of emergent closed reduction under under entanox and analgesia, but often requires conscious sedation. Arm should then be immobilised in a polysling, and XR to confirm relocation.

Imaging - True anteroposterior (AP), axillary lateral and/or scapula Y view. Reduced humeral head should lie between acromion and coracoid on lateral/scapula view.

Types
Direction Features Cause Examination Reduction techniques
Anterior Most Common >90% Usually traumatic - anterior force on arm when shoulder is abducted, eternally rotated Loss of shoulder contour - sulcus sign. Humeral head can be felt anteriorly.
Hippocratic.
Milch.
Stimson.

Kocher not advised due to complication of fracture
Posterior 50% missed in A&E 50% traumatic, but classically post seizure or electrocution Shoulder locked in internal rotation. XR may show lightbulb appearance. Gentle lateral traction to adducted arm.
Inferior Rare Associated with pectorals and rotator cuff tears, and glenoid fracture As for primary injury Management of primary injury

Superior Rare Associated with acrominon/clavicle fracture As for primary injury Management of primary injury

Associated injuries
Bankart lesion - avulsion of the anterior glenoid labrum with an anterior shoulder dislocation (reverse Bankart if poster labrum in posterior dislocation).
Hill Sachs defect - chondral impaction on posteriosuperior humeral head from contact with gleonoid rim. Can be large enough to lock shoulder, requiring open reduction. (Reverse Hill Sachs in posterior dislocation).
Rotator cuff tear - increases with age.
Greater or lesser tuberosity fracture - increases with age.
Humeral neck fracture - shoulder fracture dislocation. More common in high energy trauma and elderly. Should be discussed with orthopaedics prior to any attempted reduction.

Rotator Cuff Disease

Rotator cuff disease is a spectrum of conditions that ranges from subacromial impingement to rotator cuff tears and eventually to rotator cuff arthropathy (arthritis).

Anatomy
The rotator cuff is a group of four muscles that are important in shoulder movements, and maintenance of glenohumeral stability.
Muscle Scapular attachment Humeral attachment Action Innervation
Supraspinatus Supraspinatus fossa Superior facet of greater tuberosity Initiation of abduction of humerus Suprascapular nerve
Infraspinatus Infraspinatus fossa Posterior facet of greater tuberosity External rotation of humerus Suprascapular nerve
Teres Minor Lateral border Inferior facet of greater tuberosity External rotation of humerus Axillary Nerve
Subscapularis Subscapular fossa Lesser tuberosity Internal rotation of humerus Upper and lower subscapular nerve

The inferior rotator cuff muscles (infraspinatus, teres minor, and subscapularis) balance the superior pull of the deltoid. Injury/tear results in upward migration of the humeral head on the glenoid (can be seen on AP radiograph).
Likewise, the anterior muscles (subscapularis) are balanced with the posterior muscles (infraspinatus, teres minor).

Subacromial Impingement

The most common cause of shoulder pain, which results from impingement of the superior cuff on the undersurface of the acromion, and an inflammatory bursitis.
Associated with certain types of acromial morphology (Bigliani classification).
Presents as insidious pain which is exacerbated by overhead activities.

Rotator Cuff Tear

Often presents as an acute event on the background of chronic subacromial impingement in the older patient, but can present as an avulsion injury in younger patients.
Majority of tears are to the superior cuff (supraspinatus, infraspinatus, teres minor), though a tear to subscapularis is associated with subcoracoid impingement.
Tears present as pain and weakness when using the muscles in question.

Rotator Cuff Arthropathy

Defined as shoulder arthritis in the setting of rotator cuff dysfunction. Results from superior migration due to the loss of rotator cuff function and integrity. Unopposed deltoid pulls the humeral head superiorly.
Associated with massive chronic cuff tears.

Imaging

Plain radiographs
AP of the shoulder may show superior migration of the humerus with a cuff tear, and features of arthritis with arthropathy. Other causes of pain may also be identified (e.g. calcific tendonitis/fracture)
Outlet view is useful for defining the acromial morphology

USS
Allows dynamic imaging of the cuff, and is inexpensive. However, it is very user dependent.

MRI
Best imaging modality for cuff pathology.
Also allows imaging of the rest of the shoulder. When intra-articular pathology is suspected, can be combined with an arthrogram for improved sensitivity and specificity.

Treatment

Subacromial impingement
Physiotherapy, oral anti-inflammatory medication
Subacromial steroid injection can settle inflammation
Arthroscopic subacromial decompression by shaving away the undersurface of the acromion, more space is created for the rotator cuff. Cuff integrity is assessed also at time of surgery, and can be repaired if necessary.

Rotator cuff tear
When considering repair of a cuff tear, the age and activity of the patient, the nature of the tear (degenerative vs. acute traumatic), and the size and retraction of the tear should be considered when making a surgical plan.
Mild tears or tears in the elderly can be managed conservatively, as outlined above.
Moderate tears can be repaired arthroscopically. Massive or retracted tears will often require an open repair (occasionally with a tendon transfer). Subacromial decompression is performed at the same time to reduce impingement, symptoms and recurrence.

Calcific tendonitis
Calcific tendonitis involves calcific deposits within tendons anywhere in the body, but most commonly in the rotator cuff (specifically the supraspinatus tendon). When present in the shoulder, it is associated with subacromial impingement and pain.

Pathology
More common in women aged 30-60 years.
Association with diabetes and hypothyroidism

There are three stages of calcification
Formative phase characterized by calcific deposits
Resting phase deposit is stable, but presents with impingement problems
Resorptive phase phagocytic resorption. Most painful stage.

Presentation
Similar in presentation to subacromial impingement, with pain especially with over head activities. Atraumatic in nature.

Imaging
Plain radiographs show calcification of the rotator cuff, usually within 1.5cm of its insertion on the humerus. Supraspinatus outlet views can show level of impingment. Further imaging is rarely needed.

Treatment
Non-operative NSAIDS, steroid injection (controversial, but practiced) and physiotherapy. Approximately 75% will resolve by 6 months with conservative management.
Ultrasound guided or surgical needle barbotage can break down deposits and resolve symptoms. Occasionally surgical excision is required.

Adhesive capsulitis (Frozen Shoulder)
Pain and loss of movement of shoulder joint, which involves fibroplastic proliferation of capsular tissue, causing soft tissue scarring and contracture. Patients present with a painful and decreased arc of motion.
Associated with prolonged immobilization, previous surgery, thyroid disorders (AI) and diabetes
Classically three stages which can take up to two years to resolve:
Stage one the freezing and painful stage
Stage two the frozen and stiff stage
Stage three the thawing stage, where shoulder movement slowly improves

Imaging
Plain radiographs to exclude other causes of a painful shoulder
MRI arthrogram may show capsular contracture, and again may be used to exclude cuff pathology. However, often not performed as diagnosis is largely clinical.

Treatment
Non-operative NSAIDS, steroid injection and physiotherapy. Patience is required as condition can take up to 2 years to improve.
Operative MUA or arthroscopic adhesiolysis (release of adhesions) can expedite recovery, followed by intensive physiotherapy.

Glenohumeral Arthritis
Background
May be osteoarthritis (primary or secondary to cuff tear or trauma), rheumatoid arthritis, or as part of a spondyloarthropathy. Majority of those with RA will develop symptoms.
More common in the elderly
Presents like any other arthritis - pain at night and with movement

Imaging
AP and axillary radiographs will show features of arthritis.
CT/MRI is often useful to classify the shape of the glenoid and extent of bone loss when considering arthroplasty. MRI also essential to asses integrity of rotator cuff if considering shoulder replacement.

Treatment
Like all orthopaedics, start with simple measures:
NSAIDS, management of RA, physiotherapy, steroid injection.
Hemiarthroplasty can sometimes be considered if glenoid is in excellent condition or if patient has large comorbidity.
Arthroscopic debridement is useful if patient has isolated ACJ arthritis, but is rarely used for glenohumeral arthritis.
Total shoulder replacement is shown to produce superior outcome when compared to hemiarthroplasty in terms of pain relief, function and implant survival.
Total shoulder replacement can be anatomical (ball on humerus, with cup on glenoid), or reverse geometry (ball on glenoid, with cup on humerus). Anatomical TSR requires an in tact rotator cuff, so often reverse is preferable when the cuff if questionable in integrity.

103
Q

A 35 year old mechanic is hit by a fork lift truck. He sustains a Gustilo and Anderson type IIIA fracture of the shaft of the left femur. What is the most appropriate course of action?

Amputation
Debridement and external fixation
Open reduction and fixation
Debridement and placement of intramedullary nail
Debridement and placement of long leg plaster cast
A

As there is no associated vascular injury the patient may be suitable for debridement of the area and external fixation. If debridement leaves a tissue defect then plastic surgical repair will be needed at a later stage. With open fractures, its best not to place intramedullary metalwork as this may become infected.

Fracture management

Bony injury resulting in a fracture may arise from trauma (excessive forces applied to bone), stress related (repetitive low velocity injury) or pathological (abnormal bone which fractures during normal use of following minimal trauma)
Diagnosis involves not just evaluating the fracture ; such as site and type of injury but also other associated injuries and distal neurovascular deficits. This may entail not just clinical examination but radiographs of proximal and distal joints.
When assessing x-rays it is important to assess for changes in length of the bone, the angulation of the distal bone, rotational effects, presence of material such as glass.

Fracture types
Fracture type Description
Oblique fracture Fracture lies obliquely to long axis of bone
Comminuted fracture >2 fragments
Segmental fracture More than one fracture along a bone
Transverse fracture Perpendicular to long axis of bone
Spiral fracture Severe oblique fracture with rotation along long axis of bone

Open Vs Closed
It is also important to distinguish open from closed injuries. The most common classification system for open fractures is the Gustilo and Anderson classification system (given below):

Grade Injury
1 Low energy wound <1cm
2 Greater than 1cm wound with moderate soft tissue damage
3 High energy wound > 1cm with extensive soft tissue damage
3 A (sub group of 3) Adequate soft tissue coverage
3 B (sub group of 3) Inadequate soft tissue coverage
3 C (sub group of 3) Associated arterial injury

Key points in management of fractures
Immobilise the fracture including the proximal and distal joints
Carefully monitor and document neurovascular status, particularly following reduction and immobilisation
Manage infection including tetanus prophylaxis
IV broad spectrum antibiotics for open injuries
As a general principle all open fractures should be thoroughly debrided ( and internal fixation devices avoided or used with extreme caution)
Open fractures constitute an emergency and should be debrided and lavaged within 6 hours of injury

104
Q

A 30 year old man injures his ankle playing football. On examination, he has tenderness over both medial and lateral malleoli. X-ray demonstrates a bimalleolar fracture with a displaced distal fibula fracture, at the level of the syndesmosis and fracture of the medial malleolus with talar shift. The ankle has been provisionally reduced and splinted in the emergency department. What is the most appropriate management?

	Application of external fixation device
	Application of compression dressing and physiotherapy
	Application of ankle boot
	Surgical fixation
	Below knee amputation
A

This is an unstable fracture pattern with a Weber B fracture of the distal fibula and a fracture of the medial malleolus. Talar shift indicates loss of ankle mortice congruity. This injury should therefore be treated with surgical fixation.

An ankle fracture relates to a fracture around the tibio-talar joint. It generally refers to a fracture involving the lateral, and/or medial and/or posterior malleolus. Pilon and Tillaux fractures are also considered to be ankle fractures, but are not covered here.
Ankle fractures are common. They effect men and women in equal numbers, but men have a higher rate as young adults (sports and contact injuries), and women a higher rate post-menopausal (fragility type fracture).

Osseous anatomy
The ankle (or mortise) joint consists of the distal tibia (tibial plafond and posterior malleolus), the distal fibula (lateral malleolus), and the talus. The main movement at the ankle joint is plantar and dorsiflexion.

Ligamentous anatomy
Medial side: Deltoid ligament. This is divided into superficial and deep portions. It is the primary restraint to valgus tilting of the talus.

Lateral side: Lateral ligament complex consisting from anterior to posterior of the anterior talofibular ligament (ATFL), calcaneofibular ligament (CFL), and the posterior talofibular ligament (PTFL). Together they resist valgus stress to the ankle, and are a restraint to anterior translation of the talus within the mortise joint.

Syndesmosis: The syndesmosis is a ligament complex between the distal tibia and fibula, holding the two bones together. It is fundamental to the integrity of the ankle joint, and its disruption leads to instability. It consists of (from anterior to posterior) the anterior-inferior tibiofibular ligament (AITFL), the transverse tibiofibular ligament (TTFL), the interosseous membrane, and the posterior-inferior tibiofibular ligament (PITFL).

Presentation and initial management
Patients will present following a traumatic event with a painful, swollen ankle, and reluctance/inability to weight bear. The Ottawa rules can be applied to differentiate between an ankle fracture and sprain, but can be unreliable.
In high energy injuries, management should follow ATLS principles to identify more significant injuries first. Neurovascular status of the foot should be documented, and open injuries should be excluded. If an open injury is identified, it should be managed in line with BOAST 4 principles1. If an obvious deformity exists, it should be reduced as soon as possible with appropriate analgesia or conscious sedation. Radiographs of clearly deformed or dislocated joints are not necessary, and removing the pressure on the surrounding soft tissues from the underlying bony deformity is the priority. If the fracture pattern is not clinically obvious then plain radiographs are appropriate and will guide the subsequent manipulation during plaster-of-paris below knee backslab application.

Imaging
AP, lateral and mortise views (20o internal rotation) are essential to evaluate fracture displacement and syndesmotic injury. Decreased tibiofibular overlap, medial joint clear space and lateral talar shift all indicate a syndesmotic injury. (In subtle cases of shift, imaging the uninjured ankle can be helpful as a proportion of the population have little or no tibiotalar overlap 2.)

Where there is suspicion of syndesmosis involvement in the absence of radiographic evidence, stress radiographs can be diagnostic.
Complex fracture patterns (and increasingly posterior malleolar fractures) are best defined using CT.

Classification
The most commonly used classifications are Lauge-Hansen and Danis-Weber.

Lauge-Hansen
Comprises two parts: first part is the foot position, and the second part is the force applied. Useful for understanding the forces involved and therefore predict the ligamentous or bony injury. Results in four injury patterns:
Supination - Adduction (SA) - 10-20%
Supination - External rotation (SER) - 40-75%
Pronation - Abduction (PA) - 5-20%
Pronation - External rotation (PER) - 5-20%

Not often used in clinical practice but good for understanding the principles of ankle fracture.

Danis-Weber
Commonly used. Based on the level of the fibula fracture in relation to the syndesmosis. The more proximal, the greater the risk of syndesmotic injury and therefore fracture instability.
A - fracture below the level of the syndesmosis
B - fracture at the level of the syndesmosis / level of the tibial plafond
C - fracture above the level of the syndesmosis. This includes Maisonneuve fractures (proximal fibula fracture), which can be associated with ankle instability. Beware the high fibula fracture - it may be an ankle fracture!

The Weber classification is based purely on the the lateral side. All injuries can include a medial or posterior bony or ligamentous injury which also dictates fracture stability (bimalleolar and trimalleolar fractures are more unstable).

Treatment
When deciding upon treatment for an ankle fracture, one must consider both the fracture and the patient. Diabetic patients and smokers are at greater risk of post-operative complication, especially wound problems and infection. Likewise, the long term outcome of post-traumatic arthritis from a malunited ankle fracture is extremely important for a young patient, but not as relevant in the elderly. Therefore, normal surgical decision processes apply as with all fractures.

Defining stability of an ankle fracture underpins the treatment decision.
Weber A - Unimalleolar Weber A Weber fractures by definition are stable and therefore can be mobilised fully weight bearing in an ankle boot.

Weber C - Fractures tend to include syndesmotic disruption and are usually bimalleolar (either bony or ligamentous). They are therefore unstable and usually require operative fixation. In addition to the fracture fixation, the syndesmosis usually requires reconstruction/augmentation with screws to restore the joint integrity and function.

Weber B - B fractures vary greatly. They can be part of a trimalleolar injury and therefore extremely unstable, requiring fixation. Alternatively, a uni-malleolar Weber B fracture can be a stable injury, and therefore mobilised immediately in an ankle boot. Defining the stability can be challenging, and often involves stress radiographs, or a trial of mobilisation and repeat radiographs. Defining stability is the subject of much ongoing research. However, treating undisplaced ankle fractures in a below knee plaster, non-weight bearing for six weeks is still widely practised, and a safe approach.

When operative fixation is appropriate, it is usually via open reduction and internal fixation using plates and screws. It must be carried out when soft tissue swelling has settled in order to minimise the risk of wound problems. This can often take a week to settle.

The use of fibula nails is expanding, but is not yet mainstream. Ankle fractures can also be treated with external fixation, or with a hind foot nail in patients who need fixation but where soft tissue or bone quality is poor.

Post operative management
Ankle fractures generally take 6 weeks to unite enough to prevent secondary displacement. This is therefore an appropriate time period to keep a cast on in a conservatively managed patient. Weight bearing post-operatively depends on the quality of the fixation and bone quality, and preference varies between surgeons, ranging from aggressive early mobilisation to a period of non-weight bearing. Return to activities takes approximately three months, and often requires assistance of a physiotherapist to improve range-of-movement and muscle strengthening.

105
Q

An obese 12 year old boy is referred with pain in the left knee and hip. On examination, he has an antaglic gait and limitation of internal rotation. His knee has normal range of passive and active movement. What is the most likely diagnosis?

	Septic arthritis
	Developmental dysplasia of the hip
	Perthes disease
	Osteoarthritis of the hip
	Slipped upper femoral epiphysis
A

Slipped upper femoral epiphysis is commonest in obese adolescent males. The x-ray will show displacement of the femoral epiphysis inferolaterally. Treatment is usually with rest and non weight bearing crutches.

Developmental dysplasia of the hip Usually diagnosed in infancy by screening tests. May be bilateral, when disease is unilateral there may be leg length inequality. As disease progresses child may limp and then early onset arthritis. More common in extended breech babies. Splints and harnesses or traction. In later years osteotomy and hip realignment procedures may be needed. In arthritis a joint replacement may be needed. However, this is best deferred if possible as it will almost certainly require revision Initially no obvious change on plain films and USS gives best resolution until 3 months of age. On plain films Shentons line should form a smooth arc
Perthes Disease Hip pain (may be referred to the knee) usually occurring between 5 and 12 years of age. Bilateral disease in 20%. Remove pressure from joint to allow normal development. Physiotherapy. Usually self-limiting if diagnosed and treated promptly. X-rays will show flattened femoral head. Eventually in untreated cases the femoral head will fragment.
Slipped upper femoral epiphysis Typically seen in obese male adolescents. Pain is often referred to the knee. Limitation to internal rotation is usually seen. Knee pain is usually present 2 months prior to hip slipping. Bilateral in 20%. Bed rest and non-weight bearing. Aim to avoid avascular necrosis. If severe slippage or risk of it occurring then percutaneous pinning of the hip may be required. X-rays will show the femoral head displaced and falling inferolaterally (like a melting ice cream cone) The Southwick angle gives indication of disease severity

106
Q

A 22 year old rugby player falls onto an outstretched hand and sustains a fracture of the distal radius. The x-ray shows a dorsally angulated comminuted fracture. What is the most appropriate management?

Reduce under haematoma block and place in plaster
Admit for open reduction and internal fixation
Reduce using Biers block and place into plaster cast
Discharge home with arm sling and review in fracture clinic
Discharge home with futura splint and fracture clinic appointment
A

Unlike an osteoporotic fracture in an elderly lady this is a high velocity injury and will require surgical fixation.

Fracture management

Bony injury resulting in a fracture may arise from trauma (excessive forces applied to bone), stress related (repetitive low velocity injury) or pathological (abnormal bone which fractures during normal use of following minimal trauma)
Diagnosis involves not just evaluating the fracture ; such as site and type of injury but also other associated injuries and distal neurovascular deficits. This may entail not just clinical examination but radiographs of proximal and distal joints.
When assessing x-rays it is important to assess for changes in length of the bone, the angulation of the distal bone, rotational effects, presence of material such as glass.

Fracture types
Fracture type Description
Oblique fracture Fracture lies obliquely to long axis of bone
Comminuted fracture >2 fragments
Segmental fracture More than one fracture along a bone
Transverse fracture Perpendicular to long axis of bone
Spiral fracture Severe oblique fracture with rotation along long axis of bone

Open Vs Closed
It is also important to distinguish open from closed injuries. The most common classification system for open fractures is the Gustilo and Anderson classification system (given below):

Grade Injury
1 Low energy wound <1cm
2 Greater than 1cm wound with moderate soft tissue damage
3 High energy wound > 1cm with extensive soft tissue damage
3 A (sub group of 3) Adequate soft tissue coverage
3 B (sub group of 3) Inadequate soft tissue coverage
3 C (sub group of 3) Associated arterial injury

Key points in management of fractures
Immobilise the fracture including the proximal and distal joints
Carefully monitor and document neurovascular status, particularly following reduction and immobilisation
Manage infection including tetanus prophylaxis
IV broad spectrum antibiotics for open injuries
As a general principle all open fractures should be thoroughly debrided ( and internal fixation devices avoided or used with extreme caution)
Open fractures constitute an emergency and should be debrided and lavaged within 6 hours of injury

107
Q

A 23 year old rugby player falls directly onto his shoulder. There is pain and swelling of the shoulder joint. The clavicle is prominent and there appears to be a step deformity. What is the most likely diagnosis?

	Acromioclavicular joint dislocation
	Glenohumeral dislocation
	Sternoclavicular dislocation
	Supraspinatus tear
	Infra spinatus tear
A

Acromioclavicular joint (ACJ) dislocation normally occurs secondary to direct injury to the superior aspect of the acromion. Loss of shoulder contour and prominent clavicle are key features. Note; rotator cuff tears rarely occur in the second decade.

Shoulder fractures and dislocations
Fractures
Proximal humerus
Background
Third most common fragility fracture in the elderly.
Results from low energy fall in predominantly elderly females, or from high energy trauma in young males.
Can be associated with nerve injury (commonly axillary), and fracture-dislocation of the humeral head. Detailed neurological assessment is essential for all upper limb injuries.

Anatomy
Osteology
Consists of articular head, greater tuberosity, lesser tuberosity, metaphysis and diaphysis. Between the articular head and the tuberosities is the anatomical neck (previous physis). Between the tuberosities and the metaphysis is the surgical neck.
The supraspinatus, infraspinatus and teres minor muscles attach to the greater tuberosity. The subscapularis muscle attaches to the lesser tuberosity.

Vascular Supply
Humeral head is supplied by the anterior and posterior humeral circumflex arteries. Anatomical neck fractures are at greatest risk of osteonecrosis.

Imaging
Imaging aims to both delineate the fracture pattern, and confirm/exlude the presence of an associated dislocation.
Radiographs - True anteroposterior (AP), axillary lateral and/or scapula Y view.
CT - indicated to better define intra-articular involvement and to aid pre-operative planning. MRI is not useful for fracture imaging.

Classification
Description of the fracture is often more useful than classification. Particular attention should be paid to humeral alignment, fracture displacement, and greater tuberosity position (rotator cuff will pull the GT supero-posterioly, which can cause impingement problems with malunion).
- Neer Classification: Most commonly used. Describes fracture as 2,3,or 4 part depending upon the number main fragments. Also comments on the degree of displacement. Fragments:
-greater tuberosity
-lesser tuberosity
- articular surface
- shaft
Displacement: >1cm or angulation >45 degrees.

Treatment
The vast majority of proximal humeral fractures are minimally displaced, and therefore can be managed conservatively. This involves immobilisation in a polysling, and progressive mobilisation. Pendular exercise can commence at 14 days, and active abduction from 4-6 weeks.

Irreducible fracture dislocation is an indication for operative management. Other indications include large displacement, younger patient, head splitting (intra-articular fractures). However, the recent PROFHER trial (1) has suggested no benefit to operative intervention on patient outcome (it must be applied cautiously as majority of patients were elderly with extraarticular fractures). Options available for surgical management include:

ORIF Most commonly used. Plate and screw fixation. Can reconstruct complex fractures.
Intramedullary nail Suitable for extra-articular configuration, predominantly surgical neck +/- GT fractures.

Hemiarthroplasty Used for un-reconstructable fractures in the older patient who has good glenoid quality.
Total shoulder arthroplasty Unconstructable fractures where high functioning shoulder is required (hemiarthroplasty will cause glenoid erosion)
Reverse shoulder arthroplasty Total shoulder arthroplasty that provides better functional outcome than conventional total shoulder replacement.

Scapula
Background
Uncommon fractures usually associated with high energy trauma. Most commonly involve scapula body or spine (50%), glenoid fossa and glenoid neck. Important to exclude associated life threatening injury.

Imaging
Plain radiographs should include true anteroposterior (AP), axillary lateral and/or scapula Y view. CT scanning is useful for defining intra-articular involvement, displacement and for three dimensional reconstruction.

Classification
Based on the location of the fracture (coracoid, acromion, glenoid neck, glenoid fossa, scapula body). Beware of ipsilateral glenoid neck and clavicle fracture -floating shoulder - where limb is effectively dissociated from axial skeleton.

Treatment
The vast majority of scapula fractures are amenable to conservative management, consisting of sling immobilisation for two weeks followed by early rehabilitation. Floating shoulder will usually require fixation, and consideration of surgery should also be given to intra-articular and displaced/angulated glenoid fractures.

Dislocations

Types
Dislocations around the shoulder joint include glenohumeral dislocation, acromioclavicular joint disruption and sternoclavicular dislocation. Only glenohumeral dislocation will be covered here.

Glenohumeral dislocation
Diagnosis, classification and management are covered here.

Background
Shoulder dislocation is commonly seen in A&E. It has a high recurrence rate that is as high as 80% in teenagers. Initial management requires emergent reduction to prevent lasting chondral damage.

Early assessment and management
Usually a traumatic cause (multi-directional instability in frequent dislocations requires discussion with orthopaedics and is not covered here). Careful history, examination and documentation of neurovascular status of the limb, in particular the axillary nerve (regimental badge sensation). This should be re-assessed post manipulation. Early radiographs to confirm direction of dislocation.

Initial management consists of emergent closed reduction under under entanox and analgesia, but often requires conscious sedation. Arm should then be immobilised in a polysling, and XR to confirm relocation.

Imaging - True anteroposterior (AP), axillary lateral and/or scapula Y view. Reduced humeral head should lie between acromion and coracoid on lateral/scapula view.

Types
Direction Features Cause Examination Reduction techniques
Anterior Most Common >90% Usually traumatic - anterior force on arm when shoulder is abducted, eternally rotated Loss of shoulder contour - sulcus sign. Humeral head can be felt anteriorly.
Hippocratic.
Milch.
Stimson.

Kocher not advised due to complication of fracture
Posterior 50% missed in A&E 50% traumatic, but classically post seizure or electrocution Shoulder locked in internal rotation. XR may show lightbulb appearance. Gentle lateral traction to adducted arm.
Inferior Rare Associated with pectorals and rotator cuff tears, and glenoid fracture As for primary injury Management of primary injury

Superior Rare Associated with acrominon/clavicle fracture As for primary injury Management of primary injury

Associated injuries
Bankart lesion - avulsion of the anterior glenoid labrum with an anterior shoulder dislocation (reverse Bankart if poster labrum in posterior dislocation).
Hill Sachs defect - chondral impaction on posteriosuperior humeral head from contact with gleonoid rim. Can be large enough to lock shoulder, requiring open reduction. (Reverse Hill Sachs in posterior dislocation).
Rotator cuff tear - increases with age.
Greater or lesser tuberosity fracture - increases with age.
Humeral neck fracture - shoulder fracture dislocation. More common in high energy trauma and elderly. Should be discussed with orthopaedics prior to any attempted reduction.

Rotator Cuff Disease

Rotator cuff disease is a spectrum of conditions that ranges from subacromial impingement to rotator cuff tears and eventually to rotator cuff arthropathy (arthritis).

Anatomy
The rotator cuff is a group of four muscles that are important in shoulder movements, and maintenance of glenohumeral stability.
Muscle Scapular attachment Humeral attachment Action Innervation
Supraspinatus Supraspinatus fossa Superior facet of greater tuberosity Initiation of abduction of humerus Suprascapular nerve
Infraspinatus Infraspinatus fossa Posterior facet of greater tuberosity External rotation of humerus Suprascapular nerve
Teres Minor Lateral border Inferior facet of greater tuberosity External rotation of humerus Axillary Nerve
Subscapularis Subscapular fossa Lesser tuberosity Internal rotation of humerus Upper and lower subscapular nerve

The inferior rotator cuff muscles (infraspinatus, teres minor, and subscapularis) balance the superior pull of the deltoid. Injury/tear results in upward migration of the humeral head on the glenoid (can be seen on AP radiograph).
Likewise, the anterior muscles (subscapularis) are balanced with the posterior muscles (infraspinatus, teres minor).

Subacromial Impingement

The most common cause of shoulder pain, which results from impingement of the superior cuff on the undersurface of the acromion, and an inflammatory bursitis.
Associated with certain types of acromial morphology (Bigliani classification).
Presents as insidious pain which is exacerbated by overhead activities.

Rotator Cuff Tear

Often presents as an acute event on the background of chronic subacromial impingement in the older patient, but can present as an avulsion injury in younger patients.
Majority of tears are to the superior cuff (supraspinatus, infraspinatus, teres minor), though a tear to subscapularis is associated with subcoracoid impingement.
Tears present as pain and weakness when using the muscles in question.

Rotator Cuff Arthropathy

Defined as shoulder arthritis in the setting of rotator cuff dysfunction. Results from superior migration due to the loss of rotator cuff function and integrity. Unopposed deltoid pulls the humeral head superiorly.
Associated with massive chronic cuff tears.

Imaging

Plain radiographs
AP of the shoulder may show superior migration of the humerus with a cuff tear, and features of arthritis with arthropathy. Other causes of pain may also be identified (e.g. calcific tendonitis/fracture)
Outlet view is useful for defining the acromial morphology

USS
Allows dynamic imaging of the cuff, and is inexpensive. However, it is very user dependent.

MRI
Best imaging modality for cuff pathology.
Also allows imaging of the rest of the shoulder. When intra-articular pathology is suspected, can be combined with an arthrogram for improved sensitivity and specificity.

Treatment

Subacromial impingement
Physiotherapy, oral anti-inflammatory medication
Subacromial steroid injection can settle inflammation
Arthroscopic subacromial decompression by shaving away the undersurface of the acromion, more space is created for the rotator cuff. Cuff integrity is assessed also at time of surgery, and can be repaired if necessary.

Rotator cuff tear
When considering repair of a cuff tear, the age and activity of the patient, the nature of the tear (degenerative vs. acute traumatic), and the size and retraction of the tear should be considered when making a surgical plan.
Mild tears or tears in the elderly can be managed conservatively, as outlined above.
Moderate tears can be repaired arthroscopically. Massive or retracted tears will often require an open repair (occasionally with a tendon transfer). Subacromial decompression is performed at the same time to reduce impingement, symptoms and recurrence.

Calcific tendonitis
Calcific tendonitis involves calcific deposits within tendons anywhere in the body, but most commonly in the rotator cuff (specifically the supraspinatus tendon). When present in the shoulder, it is associated with subacromial impingement and pain.

Pathology
More common in women aged 30-60 years.
Association with diabetes and hypothyroidism

There are three stages of calcification
Formative phase characterized by calcific deposits
Resting phase deposit is stable, but presents with impingement problems
Resorptive phase phagocytic resorption. Most painful stage.

Presentation
Similar in presentation to subacromial impingement, with pain especially with over head activities. Atraumatic in nature.

Imaging
Plain radiographs show calcification of the rotator cuff, usually within 1.5cm of its insertion on the humerus. Supraspinatus outlet views can show level of impingment. Further imaging is rarely needed.

Treatment
Non-operative NSAIDS, steroid injection (controversial, but practiced) and physiotherapy. Approximately 75% will resolve by 6 months with conservative management.
Ultrasound guided or surgical needle barbotage can break down deposits and resolve symptoms. Occasionally surgical excision is required.

Adhesive capsulitis (Frozen Shoulder)
Pain and loss of movement of shoulder joint, which involves fibroplastic proliferation of capsular tissue, causing soft tissue scarring and contracture. Patients present with a painful and decreased arc of motion.
Associated with prolonged immobilization, previous surgery, thyroid disorders (AI) and diabetes
Classically three stages which can take up to two years to resolve:
Stage one the freezing and painful stage
Stage two the frozen and stiff stage
Stage three the thawing stage, where shoulder movement slowly improves

Imaging
Plain radiographs to exclude other causes of a painful shoulder
MRI arthrogram may show capsular contracture, and again may be used to exclude cuff pathology. However, often not performed as diagnosis is largely clinical.

Treatment
Non-operative NSAIDS, steroid injection and physiotherapy. Patience is required as condition can take up to 2 years to improve.
Operative MUA or arthroscopic adhesiolysis (release of adhesions) can expedite recovery, followed by intensive physiotherapy.

Glenohumeral Arthritis
Background
May be osteoarthritis (primary or secondary to cuff tear or trauma), rheumatoid arthritis, or as part of a spondyloarthropathy. Majority of those with RA will develop symptoms.
More common in the elderly
Presents like any other arthritis - pain at night and with movement

Imaging
AP and axillary radiographs will show features of arthritis.
CT/MRI is often useful to classify the shape of the glenoid and extent of bone loss when considering arthroplasty. MRI also essential to asses integrity of rotator cuff if considering shoulder replacement.

Treatment
Like all orthopaedics, start with simple measures:
NSAIDS, management of RA, physiotherapy, steroid injection.
Hemiarthroplasty can sometimes be considered if glenoid is in excellent condition or if patient has large comorbidity.
Arthroscopic debridement is useful if patient has isolated ACJ arthritis, but is rarely used for glenohumeral arthritis.
Total shoulder replacement is shown to produce superior outcome when compared to hemiarthroplasty in terms of pain relief, function and implant survival.
Total shoulder replacement can be anatomical (ball on humerus, with cup on glenoid), or reverse geometry (ball on glenoid, with cup on humerus). Anatomical TSR requires an in tact rotator cuff, so often reverse is preferable when the cuff if questionable in integrity.

108
Q

A 4 year boy presents with an abnormal gait. He has a history of recent viral illness. His WCC is 11 and ESR is 30. What is the most likely cause?

	Perthes disease
	Transient synovitis
	Septic arthritis
	Slipped upper femoral epiphysis
	Osteomyelitis
A

Viral illnesses can be associated with transient synovitis. The WCC should ideally be > 12 and the ESR > 40 to suggest septic arthritis.

Developmental dysplasia of the hip Usually diagnosed in infancy by screening tests. May be bilateral, when disease is unilateral there may be leg length inequality. As disease progresses child may limp and then early onset arthritis. More common in extended breech babies. Splints and harnesses or traction. In later years osteotomy and hip realignment procedures may be needed. In arthritis a joint replacement may be needed. However, this is best deferred if possible as it will almost certainly require revision Initially no obvious change on plain films and USS gives best resolution until 3 months of age. On plain films Shentons line should form a smooth arc
Perthes Disease Hip pain (may be referred to the knee) usually occurring between 5 and 12 years of age. Bilateral disease in 20%. Remove pressure from joint to allow normal development. Physiotherapy. Usually self-limiting if diagnosed and treated promptly. X-rays will show flattened femoral head. Eventually in untreated cases the femoral head will fragment.
Slipped upper femoral epiphysis Typically seen in obese male adolescents. Pain is often referred to the knee. Limitation to internal rotation is usually seen. Knee pain is usually present 2 months prior to hip slipping. Bilateral in 20%. Bed rest and non-weight bearing. Aim to avoid avascular necrosis. If severe slippage or risk of it occurring then percutaneous pinning of the hip may be required. X-rays will show the femoral head displaced and falling inferolaterally (like a melting ice cream cone) The Southwick angle gives indication of disease severity

109
Q

A 45 year old man has been admitted after being knocked off his bicycle. His ankle is grossly deformed with bilateral malleolar tenderness with severe ankle swelling and tenting of the medial soft tissues. What is the most appropriate initial management?

Application of compression dressing and physiotherapy
Application of external fixation device
Immediate reduction and application of backslab
Surgical fixation
Application of full leg plaster cast
A

This is an unstable ankle injury that is likely to require surgical fixation. The immediate management of a displaced ankle fracture is to reduce the fracture to prevent soft tissues compromise and help reduce swelling. This can be performed before an x-ray is obtained if performing the x-ray will significantly delay reduction.

An ankle fracture relates to a fracture around the tibio-talar joint. It generally refers to a fracture involving the lateral, and/or medial and/or posterior malleolus. Pilon and Tillaux fractures are also considered to be ankle fractures, but are not covered here.
Ankle fractures are common. They effect men and women in equal numbers, but men have a higher rate as young adults (sports and contact injuries), and women a higher rate post-menopausal (fragility type fracture).

Osseous anatomy
The ankle (or mortise) joint consists of the distal tibia (tibial plafond and posterior malleolus), the distal fibula (lateral malleolus), and the talus. The main movement at the ankle joint is plantar and dorsiflexion.

Ligamentous anatomy
Medial side: Deltoid ligament. This is divided into superficial and deep portions. It is the primary restraint to valgus tilting of the talus.

Lateral side: Lateral ligament complex consisting from anterior to posterior of the anterior talofibular ligament (ATFL), calcaneofibular ligament (CFL), and the posterior talofibular ligament (PTFL). Together they resist valgus stress to the ankle, and are a restraint to anterior translation of the talus within the mortise joint.

Syndesmosis: The syndesmosis is a ligament complex between the distal tibia and fibula, holding the two bones together. It is fundamental to the integrity of the ankle joint, and its disruption leads to instability. It consists of (from anterior to posterior) the anterior-inferior tibiofibular ligament (AITFL), the transverse tibiofibular ligament (TTFL), the interosseous membrane, and the posterior-inferior tibiofibular ligament (PITFL).

Presentation and initial management
Patients will present following a traumatic event with a painful, swollen ankle, and reluctance/inability to weight bear. The Ottawa rules can be applied to differentiate between an ankle fracture and sprain, but can be unreliable.
In high energy injuries, management should follow ATLS principles to identify more significant injuries first. Neurovascular status of the foot should be documented, and open injuries should be excluded. If an open injury is identified, it should be managed in line with BOAST 4 principles1. If an obvious deformity exists, it should be reduced as soon as possible with appropriate analgesia or conscious sedation. Radiographs of clearly deformed or dislocated joints are not necessary, and removing the pressure on the surrounding soft tissues from the underlying bony deformity is the priority. If the fracture pattern is not clinically obvious then plain radiographs are appropriate and will guide the subsequent manipulation during plaster-of-paris below knee backslab application.

Imaging
AP, lateral and mortise views (20o internal rotation) are essential to evaluate fracture displacement and syndesmotic injury. Decreased tibiofibular overlap, medial joint clear space and lateral talar shift all indicate a syndesmotic injury. (In subtle cases of shift, imaging the uninjured ankle can be helpful as a proportion of the population have little or no tibiotalar overlap 2.)

Where there is suspicion of syndesmosis involvement in the absence of radiographic evidence, stress radiographs can be diagnostic.
Complex fracture patterns (and increasingly posterior malleolar fractures) are best defined using CT.

Classification
The most commonly used classifications are Lauge-Hansen and Danis-Weber.

Lauge-Hansen
Comprises two parts: first part is the foot position, and the second part is the force applied. Useful for understanding the forces involved and therefore predict the ligamentous or bony injury. Results in four injury patterns:
Supination - Adduction (SA) - 10-20%
Supination - External rotation (SER) - 40-75%
Pronation - Abduction (PA) - 5-20%
Pronation - External rotation (PER) - 5-20%

Not often used in clinical practice but good for understanding the principles of ankle fracture.

Danis-Weber
Commonly used. Based on the level of the fibula fracture in relation to the syndesmosis. The more proximal, the greater the risk of syndesmotic injury and therefore fracture instability.
A - fracture below the level of the syndesmosis
B - fracture at the level of the syndesmosis / level of the tibial plafond
C - fracture above the level of the syndesmosis. This includes Maisonneuve fractures (proximal fibula fracture), which can be associated with ankle instability. Beware the high fibula fracture - it may be an ankle fracture!

The Weber classification is based purely on the the lateral side. All injuries can include a medial or posterior bony or ligamentous injury which also dictates fracture stability (bimalleolar and trimalleolar fractures are more unstable).

Treatment
When deciding upon treatment for an ankle fracture, one must consider both the fracture and the patient. Diabetic patients and smokers are at greater risk of post-operative complication, especially wound problems and infection. Likewise, the long term outcome of post-traumatic arthritis from a malunited ankle fracture is extremely important for a young patient, but not as relevant in the elderly. Therefore, normal surgical decision processes apply as with all fractures.

Defining stability of an ankle fracture underpins the treatment decision.
Weber A - Unimalleolar Weber A Weber fractures by definition are stable and therefore can be mobilised fully weight bearing in an ankle boot.

Weber C - Fractures tend to include syndesmotic disruption and are usually bimalleolar (either bony or ligamentous). They are therefore unstable and usually require operative fixation. In addition to the fracture fixation, the syndesmosis usually requires reconstruction/augmentation with screws to restore the joint integrity and function.

Weber B - B fractures vary greatly. They can be part of a trimalleolar injury and therefore extremely unstable, requiring fixation. Alternatively, a uni-malleolar Weber B fracture can be a stable injury, and therefore mobilised immediately in an ankle boot. Defining the stability can be challenging, and often involves stress radiographs, or a trial of mobilisation and repeat radiographs. Defining stability is the subject of much ongoing research. However, treating undisplaced ankle fractures in a below knee plaster, non-weight bearing for six weeks is still widely practised, and a safe approach.

When operative fixation is appropriate, it is usually via open reduction and internal fixation using plates and screws. It must be carried out when soft tissue swelling has settled in order to minimise the risk of wound problems. This can often take a week to settle.

The use of fibula nails is expanding, but is not yet mainstream. Ankle fractures can also be treated with external fixation, or with a hind foot nail in patients who need fixation but where soft tissue or bone quality is poor.

Post operative management
Ankle fractures generally take 6 weeks to unite enough to prevent secondary displacement. This is therefore an appropriate time period to keep a cast on in a conservatively managed patient. Weight bearing post-operatively depends on the quality of the fixation and bone quality, and preference varies between surgeons, ranging from aggressive early mobilisation to a period of non-weight bearing. Return to activities takes approximately three months, and often requires assistance of a physiotherapist to improve range-of-movement and muscle strengthening.

110
Q

A 40 year old marine injures his ankle on an assault course. On examination he has a severely swollen ankle, as well as tenderness over the medial malleolus and proximal fibula. X-rays demonstrate a medial malleolar fracture, spiral fracture of the proximal fibula and widening of the syndesmosis. What is the most appropriate definitive management?

	Surgical fixation
	Application of ankle boot
	Application of lower leg plaster cast
	Application of external fixation device
	Below knee amputation
A

This is a Maisonneuve fracture of the proximal fibula. It indicates an unstable ankle injury with likely injury to the interosseous membrane. In the setting of radiographic evidence of syndesmotic widening, this requires surgical fixation to reduce and stabilise the syndesmosis.

An ankle fracture relates to a fracture around the tibio-talar joint. It generally refers to a fracture involving the lateral, and/or medial and/or posterior malleolus. Pilon and Tillaux fractures are also considered to be ankle fractures, but are not covered here.
Ankle fractures are common. They effect men and women in equal numbers, but men have a higher rate as young adults (sports and contact injuries), and women a higher rate post-menopausal (fragility type fracture).

Osseous anatomy
The ankle (or mortise) joint consists of the distal tibia (tibial plafond and posterior malleolus), the distal fibula (lateral malleolus), and the talus. The main movement at the ankle joint is plantar and dorsiflexion.

Ligamentous anatomy
Medial side: Deltoid ligament. This is divided into superficial and deep portions. It is the primary restraint to valgus tilting of the talus.

Lateral side: Lateral ligament complex consisting from anterior to posterior of the anterior talofibular ligament (ATFL), calcaneofibular ligament (CFL), and the posterior talofibular ligament (PTFL). Together they resist valgus stress to the ankle, and are a restraint to anterior translation of the talus within the mortise joint.

Syndesmosis: The syndesmosis is a ligament complex between the distal tibia and fibula, holding the two bones together. It is fundamental to the integrity of the ankle joint, and its disruption leads to instability. It consists of (from anterior to posterior) the anterior-inferior tibiofibular ligament (AITFL), the transverse tibiofibular ligament (TTFL), the interosseous membrane, and the posterior-inferior tibiofibular ligament (PITFL).

Presentation and initial management
Patients will present following a traumatic event with a painful, swollen ankle, and reluctance/inability to weight bear. The Ottawa rules can be applied to differentiate between an ankle fracture and sprain, but can be unreliable.
In high energy injuries, management should follow ATLS principles to identify more significant injuries first. Neurovascular status of the foot should be documented, and open injuries should be excluded. If an open injury is identified, it should be managed in line with BOAST 4 principles1. If an obvious deformity exists, it should be reduced as soon as possible with appropriate analgesia or conscious sedation. Radiographs of clearly deformed or dislocated joints are not necessary, and removing the pressure on the surrounding soft tissues from the underlying bony deformity is the priority. If the fracture pattern is not clinically obvious then plain radiographs are appropriate and will guide the subsequent manipulation during plaster-of-paris below knee backslab application.

Imaging
AP, lateral and mortise views (20o internal rotation) are essential to evaluate fracture displacement and syndesmotic injury. Decreased tibiofibular overlap, medial joint clear space and lateral talar shift all indicate a syndesmotic injury. (In subtle cases of shift, imaging the uninjured ankle can be helpful as a proportion of the population have little or no tibiotalar overlap 2.)

Where there is suspicion of syndesmosis involvement in the absence of radiographic evidence, stress radiographs can be diagnostic.
Complex fracture patterns (and increasingly posterior malleolar fractures) are best defined using CT.

Classification
The most commonly used classifications are Lauge-Hansen and Danis-Weber.

Lauge-Hansen
Comprises two parts: first part is the foot position, and the second part is the force applied. Useful for understanding the forces involved and therefore predict the ligamentous or bony injury. Results in four injury patterns:
Supination - Adduction (SA) - 10-20%
Supination - External rotation (SER) - 40-75%
Pronation - Abduction (PA) - 5-20%
Pronation - External rotation (PER) - 5-20%

Not often used in clinical practice but good for understanding the principles of ankle fracture.

Danis-Weber
Commonly used. Based on the level of the fibula fracture in relation to the syndesmosis. The more proximal, the greater the risk of syndesmotic injury and therefore fracture instability.
A - fracture below the level of the syndesmosis
B - fracture at the level of the syndesmosis / level of the tibial plafond
C - fracture above the level of the syndesmosis. This includes Maisonneuve fractures (proximal fibula fracture), which can be associated with ankle instability. Beware the high fibula fracture - it may be an ankle fracture!

The Weber classification is based purely on the the lateral side. All injuries can include a medial or posterior bony or ligamentous injury which also dictates fracture stability (bimalleolar and trimalleolar fractures are more unstable).

Treatment
When deciding upon treatment for an ankle fracture, one must consider both the fracture and the patient. Diabetic patients and smokers are at greater risk of post-operative complication, especially wound problems and infection. Likewise, the long term outcome of post-traumatic arthritis from a malunited ankle fracture is extremely important for a young patient, but not as relevant in the elderly. Therefore, normal surgical decision processes apply as with all fractures.

Defining stability of an ankle fracture underpins the treatment decision.
Weber A - Unimalleolar Weber A Weber fractures by definition are stable and therefore can be mobilised fully weight bearing in an ankle boot.

Weber C - Fractures tend to include syndesmotic disruption and are usually bimalleolar (either bony or ligamentous). They are therefore unstable and usually require operative fixation. In addition to the fracture fixation, the syndesmosis usually requires reconstruction/augmentation with screws to restore the joint integrity and function.

Weber B - B fractures vary greatly. They can be part of a trimalleolar injury and therefore extremely unstable, requiring fixation. Alternatively, a uni-malleolar Weber B fracture can be a stable injury, and therefore mobilised immediately in an ankle boot. Defining the stability can be challenging, and often involves stress radiographs, or a trial of mobilisation and repeat radiographs. Defining stability is the subject of much ongoing research. However, treating undisplaced ankle fractures in a below knee plaster, non-weight bearing for six weeks is still widely practised, and a safe approach.

When operative fixation is appropriate, it is usually via open reduction and internal fixation using plates and screws. It must be carried out when soft tissue swelling has settled in order to minimise the risk of wound problems. This can often take a week to settle.

The use of fibula nails is expanding, but is not yet mainstream. Ankle fractures can also be treated with external fixation, or with a hind foot nail in patients who need fixation but where soft tissue or bone quality is poor.

Post operative management
Ankle fractures generally take 6 weeks to unite enough to prevent secondary displacement. This is therefore an appropriate time period to keep a cast on in a conservatively managed patient. Weight bearing post-operatively depends on the quality of the fixation and bone quality, and preference varies between surgeons, ranging from aggressive early mobilisation to a period of non-weight bearing. Return to activities takes approximately three months, and often requires assistance of a physiotherapist to improve range-of-movement and muscle strengthening.

111
Q

Which of the following is not a typical feature of talipes equinovarus?

	Adducted and inverted calcaneus
	Medial displacement of the navicular bone
	It is nearly always unilateral
	Wedge shaped head of talus
	Severe Tibio-talar plantar flexion
A

It is bilateral in 50% of cases.

Congenital talipes equinovarus.
Features:
Equinus of the hindfoot
Adduction and varus of the midfoot
High arch

Most cases in developing countries. Incidence in UK is 1 per 1000 live births. It is more common in males and is bilateral in 50% cases. There is a strong familial link(1). It may also be associated with other developmental disorders such as Down’s syndrome.

Key anatomical deformities (2):
Adducted and inverted calcaneus
Wedge shaped distal calcaneal articular surface
Severe Tibio-talar plantar flexion
Medial Talar neck inclination
Displacement of the navicular bone (medially)
Wedge shaped head of talus
Displacement of the cuboid (medially)

Management
Conservative first, the Ponseti method is best described and gives comparable results to surgery. It consists of serial casting to mold the foot into correct shape. Following casting around 90% will require a Achilles tenotomy. This is then followed by a phase of walking braces to maintain the correction.

Surgical correction is reserved for those cases that fail to respond to conservative measures. The procedures involve multiple tenotomies and lengthening procedures. In patients who fail to respond surgically an Ilizarov frame reconstruction may be attempted and gives good results.

112
Q

A 25 year old ski instructor who falls off a ski lift and sustains a spiral fracture of the mid shaft of the tibia. Attempts to achieve satisfactory position in plaster have failed. Overlying tissues are healthy. What is the most appropriate course of action?

	Amputation
	Open reduction and fixation using a long plate
	Intramedullary nail
	Skeletal traction
	Long limb casting
A

This would be a good case for intramedullary nailing. Open reduction and external fixation would strip off otherwise healthy tissues and hence is unsuitable. In some units the injury may be managed with an Ilizarov frame device but the majority would treat with IM nailing.

Fracture management

Bony injury resulting in a fracture may arise from trauma (excessive forces applied to bone), stress related (repetitive low velocity injury) or pathological (abnormal bone which fractures during normal use of following minimal trauma)
Diagnosis involves not just evaluating the fracture ; such as site and type of injury but also other associated injuries and distal neurovascular deficits. This may entail not just clinical examination but radiographs of proximal and distal joints.
When assessing x-rays it is important to assess for changes in length of the bone, the angulation of the distal bone, rotational effects, presence of material such as glass.

Fracture types
Fracture type Description
Oblique fracture Fracture lies obliquely to long axis of bone
Comminuted fracture >2 fragments
Segmental fracture More than one fracture along a bone
Transverse fracture Perpendicular to long axis of bone
Spiral fracture Severe oblique fracture with rotation along long axis of bone

Open Vs Closed
It is also important to distinguish open from closed injuries. The most common classification system for open fractures is the Gustilo and Anderson classification system (given below):

Grade Injury
1 Low energy wound <1cm
2 Greater than 1cm wound with moderate soft tissue damage
3 High energy wound > 1cm with extensive soft tissue damage
3 A (sub group of 3) Adequate soft tissue coverage
3 B (sub group of 3) Inadequate soft tissue coverage
3 C (sub group of 3) Associated arterial injury

Key points in management of fractures
Immobilise the fracture including the proximal and distal joints
Carefully monitor and document neurovascular status, particularly following reduction and immobilisation
Manage infection including tetanus prophylaxis
IV broad spectrum antibiotics for open injuries
As a general principle all open fractures should be thoroughly debrided ( and internal fixation devices avoided or used with extreme caution)
Open fractures constitute an emergency and should be debrided and lavaged within 6 hours of injury

113
Q

A 74 year old lady falls and injures her left arm. Following assessment she is found to have an impacted fracture affecting the surgical neck of the humerus. What is the most appropriate course of action?

Reduce the fracture and apply a plate to stabilize the fragments
Perform a hemiarthroplasty
Apply a collar and cuff system for three weeks and then commence physiotherapy
Apply an upper limb cast for 8 weeks
Apply an external fixator system
A

Apply a collar and cuff system for three weeks and then commence physiotherapy

Impacted fractures of the surgical neck are stable injuries and usually heal without complication. It is rare to need to resort to surgery. Its important to start physiotherapy early.

Proximal humerus fractures

Very common injury. Usually through the surgical neck. Number of classification systems though for practical purposes describing the number of fracture fragments is probably easier. Some key points:
It is rare to have fractures through the anatomical neck.
Anatomical neck fractures which are displaced by >1cm carry a risk of avascular necrosis to the humeral head.
In children the commonest injury pattern is a greenstick fracture through the surgical neck.
Impacted fractures of the surgical neck are usually managed with a collar and cuff for 3 weeks followed by physiotherapy.
More significant displaced fractures may require open reduction and fixation or use of an intramedullary device.

114
Q

A 63 year old lady undergoes an axillary clearance for breast cancer. She makes steady progress. However, 8 weeks post operatively she still suffers from severe shoulder pain. On examination, she has reduced active movements in all planes and loss of passive external rotation. What is the most likely cause?

	Metastatic cancer
	Adhesive capsulitis
	Rotator cuff tear
	Osteoarthritis
	Rheumatoid disease
A

Frozen shoulder is more likely. Metastatic cancer can cause pain. However, passive movements are normally unaffected.
Frozen shoulder passes through an initial painful stage followed by a period of joint stiffness. With physiotherapy the problem will usually resolve although it may take up to 2 years to do so.

Shoulder fractures and dislocations
Fractures
Proximal humerus
Background
Third most common fragility fracture in the elderly.
Results from low energy fall in predominantly elderly females, or from high energy trauma in young males.
Can be associated with nerve injury (commonly axillary), and fracture-dislocation of the humeral head. Detailed neurological assessment is essential for all upper limb injuries.

Anatomy
Osteology
Consists of articular head, greater tuberosity, lesser tuberosity, metaphysis and diaphysis. Between the articular head and the tuberosities is the anatomical neck (previous physis). Between the tuberosities and the metaphysis is the surgical neck.
The supraspinatus, infraspinatus and teres minor muscles attach to the greater tuberosity. The subscapularis muscle attaches to the lesser tuberosity.

Vascular Supply
Humeral head is supplied by the anterior and posterior humeral circumflex arteries. Anatomical neck fractures are at greatest risk of osteonecrosis.

Imaging
Imaging aims to both delineate the fracture pattern, and confirm/exlude the presence of an associated dislocation.
Radiographs - True anteroposterior (AP), axillary lateral and/or scapula Y view.
CT - indicated to better define intra-articular involvement and to aid pre-operative planning. MRI is not useful for fracture imaging.

Classification
Description of the fracture is often more useful than classification. Particular attention should be paid to humeral alignment, fracture displacement, and greater tuberosity position (rotator cuff will pull the GT supero-posterioly, which can cause impingement problems with malunion).
- Neer Classification: Most commonly used. Describes fracture as 2,3,or 4 part depending upon the number main fragments. Also comments on the degree of displacement. Fragments:
-greater tuberosity
-lesser tuberosity
- articular surface
- shaft
Displacement: >1cm or angulation >45 degrees.

Treatment
The vast majority of proximal humeral fractures are minimally displaced, and therefore can be managed conservatively. This involves immobilisation in a polysling, and progressive mobilisation. Pendular exercise can commence at 14 days, and active abduction from 4-6 weeks.

Irreducible fracture dislocation is an indication for operative management. Other indications include large displacement, younger patient, head splitting (intra-articular fractures). However, the recent PROFHER trial (1) has suggested no benefit to operative intervention on patient outcome (it must be applied cautiously as majority of patients were elderly with extraarticular fractures). Options available for surgical management include:

ORIF Most commonly used. Plate and screw fixation. Can reconstruct complex fractures.
Intramedullary nail Suitable for extra-articular configuration, predominantly surgical neck +/- GT fractures.

Hemiarthroplasty Used for un-reconstructable fractures in the older patient who has good glenoid quality.
Total shoulder arthroplasty Unconstructable fractures where high functioning shoulder is required (hemiarthroplasty will cause glenoid erosion)
Reverse shoulder arthroplasty Total shoulder arthroplasty that provides better functional outcome than conventional total shoulder replacement.

Scapula
Background
Uncommon fractures usually associated with high energy trauma. Most commonly involve scapula body or spine (50%), glenoid fossa and glenoid neck. Important to exclude associated life threatening injury.

Imaging
Plain radiographs should include true anteroposterior (AP), axillary lateral and/or scapula Y view. CT scanning is useful for defining intra-articular involvement, displacement and for three dimensional reconstruction.

Classification
Based on the location of the fracture (coracoid, acromion, glenoid neck, glenoid fossa, scapula body). Beware of ipsilateral glenoid neck and clavicle fracture -floating shoulder - where limb is effectively dissociated from axial skeleton.

Treatment
The vast majority of scapula fractures are amenable to conservative management, consisting of sling immobilisation for two weeks followed by early rehabilitation. Floating shoulder will usually require fixation, and consideration of surgery should also be given to intra-articular and displaced/angulated glenoid fractures.

Dislocations

Types
Dislocations around the shoulder joint include glenohumeral dislocation, acromioclavicular joint disruption and sternoclavicular dislocation. Only glenohumeral dislocation will be covered here.

Glenohumeral dislocation
Diagnosis, classification and management are covered here.

Background
Shoulder dislocation is commonly seen in A&E. It has a high recurrence rate that is as high as 80% in teenagers. Initial management requires emergent reduction to prevent lasting chondral damage.

Early assessment and management
Usually a traumatic cause (multi-directional instability in frequent dislocations requires discussion with orthopaedics and is not covered here). Careful history, examination and documentation of neurovascular status of the limb, in particular the axillary nerve (regimental badge sensation). This should be re-assessed post manipulation. Early radiographs to confirm direction of dislocation.

Initial management consists of emergent closed reduction under under entanox and analgesia, but often requires conscious sedation. Arm should then be immobilised in a polysling, and XR to confirm relocation.

Imaging - True anteroposterior (AP), axillary lateral and/or scapula Y view. Reduced humeral head should lie between acromion and coracoid on lateral/scapula view.

Types
Direction Features Cause Examination Reduction techniques
Anterior Most Common >90% Usually traumatic - anterior force on arm when shoulder is abducted, eternally rotated Loss of shoulder contour - sulcus sign. Humeral head can be felt anteriorly.
Hippocratic.
Milch.
Stimson.

Kocher not advised due to complication of fracture
Posterior 50% missed in A&E 50% traumatic, but classically post seizure or electrocution Shoulder locked in internal rotation. XR may show lightbulb appearance. Gentle lateral traction to adducted arm.
Inferior Rare Associated with pectorals and rotator cuff tears, and glenoid fracture As for primary injury Management of primary injury

Superior Rare Associated with acrominon/clavicle fracture As for primary injury Management of primary injury

Associated injuries
Bankart lesion - avulsion of the anterior glenoid labrum with an anterior shoulder dislocation (reverse Bankart if poster labrum in posterior dislocation).
Hill Sachs defect - chondral impaction on posteriosuperior humeral head from contact with gleonoid rim. Can be large enough to lock shoulder, requiring open reduction. (Reverse Hill Sachs in posterior dislocation).
Rotator cuff tear - increases with age.
Greater or lesser tuberosity fracture - increases with age.
Humeral neck fracture - shoulder fracture dislocation. More common in high energy trauma and elderly. Should be discussed with orthopaedics prior to any attempted reduction.

Rotator Cuff Disease

Rotator cuff disease is a spectrum of conditions that ranges from subacromial impingement to rotator cuff tears and eventually to rotator cuff arthropathy (arthritis).

Anatomy
The rotator cuff is a group of four muscles that are important in shoulder movements, and maintenance of glenohumeral stability.
Muscle Scapular attachment Humeral attachment Action Innervation
Supraspinatus Supraspinatus fossa Superior facet of greater tuberosity Initiation of abduction of humerus Suprascapular nerve
Infraspinatus Infraspinatus fossa Posterior facet of greater tuberosity External rotation of humerus Suprascapular nerve
Teres Minor Lateral border Inferior facet of greater tuberosity External rotation of humerus Axillary Nerve
Subscapularis Subscapular fossa Lesser tuberosity Internal rotation of humerus Upper and lower subscapular nerve

The inferior rotator cuff muscles (infraspinatus, teres minor, and subscapularis) balance the superior pull of the deltoid. Injury/tear results in upward migration of the humeral head on the glenoid (can be seen on AP radiograph).
Likewise, the anterior muscles (subscapularis) are balanced with the posterior muscles (infraspinatus, teres minor).

Subacromial Impingement

The most common cause of shoulder pain, which results from impingement of the superior cuff on the undersurface of the acromion, and an inflammatory bursitis.
Associated with certain types of acromial morphology (Bigliani classification).
Presents as insidious pain which is exacerbated by overhead activities.

Rotator Cuff Tear

Often presents as an acute event on the background of chronic subacromial impingement in the older patient, but can present as an avulsion injury in younger patients.
Majority of tears are to the superior cuff (supraspinatus, infraspinatus, teres minor), though a tear to subscapularis is associated with subcoracoid impingement.
Tears present as pain and weakness when using the muscles in question.

Rotator Cuff Arthropathy

Defined as shoulder arthritis in the setting of rotator cuff dysfunction. Results from superior migration due to the loss of rotator cuff function and integrity. Unopposed deltoid pulls the humeral head superiorly.
Associated with massive chronic cuff tears.

Imaging

Plain radiographs
AP of the shoulder may show superior migration of the humerus with a cuff tear, and features of arthritis with arthropathy. Other causes of pain may also be identified (e.g. calcific tendonitis/fracture)
Outlet view is useful for defining the acromial morphology

USS
Allows dynamic imaging of the cuff, and is inexpensive. However, it is very user dependent.

MRI
Best imaging modality for cuff pathology.
Also allows imaging of the rest of the shoulder. When intra-articular pathology is suspected, can be combined with an arthrogram for improved sensitivity and specificity.

Treatment

Subacromial impingement
Physiotherapy, oral anti-inflammatory medication
Subacromial steroid injection can settle inflammation
Arthroscopic subacromial decompression by shaving away the undersurface of the acromion, more space is created for the rotator cuff. Cuff integrity is assessed also at time of surgery, and can be repaired if necessary.

Rotator cuff tear
When considering repair of a cuff tear, the age and activity of the patient, the nature of the tear (degenerative vs. acute traumatic), and the size and retraction of the tear should be considered when making a surgical plan.
Mild tears or tears in the elderly can be managed conservatively, as outlined above.
Moderate tears can be repaired arthroscopically. Massive or retracted tears will often require an open repair (occasionally with a tendon transfer). Subacromial decompression is performed at the same time to reduce impingement, symptoms and recurrence.

Calcific tendonitis
Calcific tendonitis involves calcific deposits within tendons anywhere in the body, but most commonly in the rotator cuff (specifically the supraspinatus tendon). When present in the shoulder, it is associated with subacromial impingement and pain.

Pathology
More common in women aged 30-60 years.
Association with diabetes and hypothyroidism

There are three stages of calcification
Formative phase characterized by calcific deposits
Resting phase deposit is stable, but presents with impingement problems
Resorptive phase phagocytic resorption. Most painful stage.

Presentation
Similar in presentation to subacromial impingement, with pain especially with over head activities. Atraumatic in nature.

Imaging
Plain radiographs show calcification of the rotator cuff, usually within 1.5cm of its insertion on the humerus. Supraspinatus outlet views can show level of impingment. Further imaging is rarely needed.

Treatment
Non-operative NSAIDS, steroid injection (controversial, but practiced) and physiotherapy. Approximately 75% will resolve by 6 months with conservative management.
Ultrasound guided or surgical needle barbotage can break down deposits and resolve symptoms. Occasionally surgical excision is required.

Adhesive capsulitis (Frozen Shoulder)
Pain and loss of movement of shoulder joint, which involves fibroplastic proliferation of capsular tissue, causing soft tissue scarring and contracture. Patients present with a painful and decreased arc of motion.
Associated with prolonged immobilization, previous surgery, thyroid disorders (AI) and diabetes
Classically three stages which can take up to two years to resolve:
Stage one the freezing and painful stage
Stage two the frozen and stiff stage
Stage three the thawing stage, where shoulder movement slowly improves

Imaging
Plain radiographs to exclude other causes of a painful shoulder
MRI arthrogram may show capsular contracture, and again may be used to exclude cuff pathology. However, often not performed as diagnosis is largely clinical.

Treatment
Non-operative NSAIDS, steroid injection and physiotherapy. Patience is required as condition can take up to 2 years to improve.
Operative MUA or arthroscopic adhesiolysis (release of adhesions) can expedite recovery, followed by intensive physiotherapy.

Glenohumeral Arthritis
Background
May be osteoarthritis (primary or secondary to cuff tear or trauma), rheumatoid arthritis, or as part of a spondyloarthropathy. Majority of those with RA will develop symptoms.
More common in the elderly
Presents like any other arthritis - pain at night and with movement

Imaging
AP and axillary radiographs will show features of arthritis.
CT/MRI is often useful to classify the shape of the glenoid and extent of bone loss when considering arthroplasty. MRI also essential to asses integrity of rotator cuff if considering shoulder replacement.

Treatment
Like all orthopaedics, start with simple measures:
NSAIDS, management of RA, physiotherapy, steroid injection.
Hemiarthroplasty can sometimes be considered if glenoid is in excellent condition or if patient has large comorbidity.
Arthroscopic debridement is useful if patient has isolated ACJ arthritis, but is rarely used for glenohumeral arthritis.
Total shoulder replacement is shown to produce superior outcome when compared to hemiarthroplasty in terms of pain relief, function and implant survival.
Total shoulder replacement can be anatomical (ball on humerus, with cup on glenoid), or reverse geometry (ball on glenoid, with cup on humerus). Anatomical TSR requires an in tact rotator cuff, so often reverse is preferable when the cuff if questionable in integrity.

115
Q

A 19 year old female is involved in an athletics event. She has just completed the high jump when she suddenly develops severe back pain and weakness affecting both her legs. On examination, she has a prominent sacrum and her lower back is painful. What is the most likely underlying cause?

	Structural scoliosis
	Ankylosing spondylitis
	Scheuermanns disease
	Spondylolisthesis
	Spondylolysis
A

Young athletic females are the group most frequently affected by spondylolythesis who have a background of spondylolysis. Whilst the latter condition is a risk factor for spondylolythesis the former condition is most likely in a young athletic female who presents with sudden pain.

Ankylosing spondylitis
Chronic inflammatory disorder affecting the axial skeleton
Sacro-ilitis is a usually visible in plain films
Up to 20% of those who are HLA B27 positive will develop the condition
Affected articulations develop bony or fibrous changes
Typical spinal features include loss of the lumbar lordosis and progressive kyphosis of the cervico-thoracic spine
Scheuermann’s disease
Epiphysitis of the vertebral joints is the main pathological process
Predominantly affects adolescents
Symptoms include back pain and stiffness
X-ray changes include epiphyseal plate disturbance and anterior wedging
Clinical features include progressive kyphosis (at least 3 vertebrae must be involved)
Minor cases may be managed with physiotherapy and analgesia, more severe cases may require bracing or surgical stabilisation
Scoliosis
Consists of curvature of the spine in the coronal plane
Divisible into structural and non structural, the latter being commonest in adolescent females who develop minor postural changes only. Postural scoliosis will typically disappear on manoeuvres such as bending forwards
Structural scoliosis affects > 1 vertebral body and is divisible into idiopathic, congential and neuromuscular in origin. It is not correctable by alterations in posture
Within structural scoliosis, idiopathic is the most common type
Severe, or progressive structural disease is often managed surgically with bilateral rod stabilisation of the spine
Spina bifida
Non fusion of the vertebral arches during embryonic development
Three categories; myelomeningocele, spina bifida occulta and meningocele
Myelomeningocele is the most severe type with associated neurological defects that may persist in spite of anatomical closure of the defect
Up to 10% of the population may have spina bifida occulta, in this condition the skin and tissues (but not not bones) may develop over the distal cord. The site may be identifiable by a birth mark or hair patch
The incidence of the condition is reduced by use of folic acid supplements during pregnancy
Spondylolysis
Congenital or acquired deficiency of the pars interarticularis of the neural arch of a particular vertebral body, usually affects L4/ L5
May be asymptomatic and affects up to 5% of the population
Spondylolysis is the commonest cause of spondylolisthesis in children
Asymptomatic cases do not require treatment
Spondylolisthesis
This occurs when one vertebra is displaced relative to its immediate inferior vertebral body
May occur as a result of stress fracture or spondylolysis
Traumatic cases may show the classic “Scotty Dog” appearance on plain films
Treatment depends upon the extent of deformity and associated neurological symptoms, minor cases may be actively monitored. Individuals with radicular symptoms or signs will usually require spinal decompression and stabilisation

116
Q

A 28 year old man falls onto an outstretched hand. On examination, there is tenderness of the anatomical snuffbox. However, forearm and hand x-rays are normal. What is the most appropriate course of action?

	Discharge with reassurance
	Place in arm sling and discharge
	Place in futura splint and review in fracture clinic
	Admit for surgical exploration
	Apply an external fixation device
A

This could well be a scaphoid fracture and should be temporarily immobilised pending further review. A futura splint will immobilise better than an arm sling for this problem.

Fracture management

Bony injury resulting in a fracture may arise from trauma (excessive forces applied to bone), stress related (repetitive low velocity injury) or pathological (abnormal bone which fractures during normal use of following minimal trauma)
Diagnosis involves not just evaluating the fracture ; such as site and type of injury but also other associated injuries and distal neurovascular deficits. This may entail not just clinical examination but radiographs of proximal and distal joints.
When assessing x-rays it is important to assess for changes in length of the bone, the angulation of the distal bone, rotational effects, presence of material such as glass.

Fracture types
Fracture type Description
Oblique fracture Fracture lies obliquely to long axis of bone
Comminuted fracture >2 fragments
Segmental fracture More than one fracture along a bone
Transverse fracture Perpendicular to long axis of bone
Spiral fracture Severe oblique fracture with rotation along long axis of bone

Open Vs Closed
It is also important to distinguish open from closed injuries. The most common classification system for open fractures is the Gustilo and Anderson classification system (given below):

Grade Injury
1 Low energy wound <1cm
2 Greater than 1cm wound with moderate soft tissue damage
3 High energy wound > 1cm with extensive soft tissue damage
3 A (sub group of 3) Adequate soft tissue coverage
3 B (sub group of 3) Inadequate soft tissue coverage
3 C (sub group of 3) Associated arterial injury

Key points in management of fractures
Immobilise the fracture including the proximal and distal joints
Carefully monitor and document neurovascular status, particularly following reduction and immobilisation
Manage infection including tetanus prophylaxis
IV broad spectrum antibiotics for open injuries
As a general principle all open fractures should be thoroughly debrided ( and internal fixation devices avoided or used with extreme caution)
Open fractures constitute an emergency and should be debrided and lavaged within 6 hours of injury

117
Q

A 6 year old boy presents with groin pain. He is known to be disruptive in class. He reports that he is bullied for being short. On examination, he has an antalgic gait and pain on internal rotation of the right hip. What is the most likely diagnosis?

	Perthes disease
	Transient synovitis
	Slipped upper femoral epiphysis
	Developmental dysplasia of the hip
	Septic arthritis
A

This child is short, has hyperactivity (disruptive behaviour) and is within the age range for Perthes disease. Hyperactivity and short stature are associated with Perthes disease.

Developmental dysplasia of the hip Usually diagnosed in infancy by screening tests. May be bilateral, when disease is unilateral there may be leg length inequality. As disease progresses child may limp and then early onset arthritis. More common in extended breech babies. Splints and harnesses or traction. In later years osteotomy and hip realignment procedures may be needed. In arthritis a joint replacement may be needed. However, this is best deferred if possible as it will almost certainly require revision Initially no obvious change on plain films and USS gives best resolution until 3 months of age. On plain films Shentons line should form a smooth arc
Perthes Disease Hip pain (may be referred to the knee) usually occurring between 5 and 12 years of age. Bilateral disease in 20%. Remove pressure from joint to allow normal development. Physiotherapy. Usually self-limiting if diagnosed and treated promptly. X-rays will show flattened femoral head. Eventually in untreated cases the femoral head will fragment.
Slipped upper femoral epiphysis Typically seen in obese male adolescents. Pain is often referred to the knee. Limitation to internal rotation is usually seen. Knee pain is usually present 2 months prior to hip slipping. Bilateral in 20%. Bed rest and non-weight bearing. Aim to avoid avascular necrosis. If severe slippage or risk of it occurring then percutaneous pinning of the hip may be required. X-rays will show the femoral head displaced and falling inferolaterally (like a melting ice cream cone) The Southwick angle gives indication of disease severity

118
Q

An obese 14 year old boy presents with difficulty running and mild knee and hip pain. There is no antecedent history of trauma. On examination, internal rotation is restricted but the knee is normal with full range of passive movement possible and no evidence of effusions. Both the C-reactive protein and white cell count are normal. What is the most likely cause?

	Perthes disease
	Slipped upper femoral epiphysis
	Non accidental injury
	Septic arthritis
	Osteoarthritis
A

Slipped upper femoral epiphysis is the commonest adolescent hip disorder. It occurs most commonly in obese males. It may often present as knee pain which is usually referred from the ipsilateral hip. The knee itself is normal. The hip often limits internal rotation. The diagnosis is easily missed. X-rays will show displacement of the femoral epiphysis and the degree of its displacement may be calculated using the Southwick angle. Treatment is directed at preventing further slippage which may result in avascular necrosis of the femoral head.

Developmental dysplasia of the hip Usually diagnosed in infancy by screening tests. May be bilateral, when disease is unilateral there may be leg length inequality. As disease progresses child may limp and then early onset arthritis. More common in extended breech babies. Splints and harnesses or traction. In later years osteotomy and hip realignment procedures may be needed. In arthritis a joint replacement may be needed. However, this is best deferred if possible as it will almost certainly require revision Initially no obvious change on plain films and USS gives best resolution until 3 months of age. On plain films Shentons line should form a smooth arc
Perthes Disease Hip pain (may be referred to the knee) usually occurring between 5 and 12 years of age. Bilateral disease in 20%. Remove pressure from joint to allow normal development. Physiotherapy. Usually self-limiting if diagnosed and treated promptly. X-rays will show flattened femoral head. Eventually in untreated cases the femoral head will fragment.
Slipped upper femoral epiphysis Typically seen in obese male adolescents. Pain is often referred to the knee. Limitation to internal rotation is usually seen. Knee pain is usually present 2 months prior to hip slipping. Bilateral in 20%. Bed rest and non-weight bearing. Aim to avoid avascular necrosis. If severe slippage or risk of it occurring then percutaneous pinning of the hip may be required. X-rays will show the femoral head displaced and falling inferolaterally (like a melting ice cream cone) The Southwick angle gives indication of disease severity

119
Q

A 64 year old man is involved in a motor vehicle accident and is found to have a fracture affecting the anatomical neck of his humerus which is displaced. What is the most appropriate management?

Place a collar and cuff for 3 weeks and then commence physiotherapy
Hemiarthroplasty
Place in a collar and cuff for 6 weeks and then commence physiotherapy
Reduce the fracture and place in an arm sling with repeat imaging at 14 days
Reduction under anaesthesia and place in collar and cuff system for 6 weeks
A

Hemiarthroplasty - These injuries are at significant risk of avscular necrosis and consideration of surgery is important. A hemi arthroplasty may be needed.

Proximal humerus fractures

Very common injury. Usually through the surgical neck. Number of classification systems though for practical purposes describing the number of fracture fragments is probably easier. Some key points:
It is rare to have fractures through the anatomical neck.
Anatomical neck fractures which are displaced by >1cm carry a risk of avascular necrosis to the humeral head.
In children the commonest injury pattern is a greenstick fracture through the surgical neck.
Impacted fractures of the surgical neck are usually managed with a collar and cuff for 3 weeks followed by physiotherapy.
More significant displaced fractures may require open reduction and fixation or use of an intramedullary device.

120
Q

A 70 year old man undergoes a revision total hip replacement. 10 days post operatively the hip dislocates and pus is discharging from the wound. He is systemically unwell with a temperature of 38.5 and WCC 19. What is the most appropriate course of action?

	Lay open wound and apply a VAC dressing
	Hindquater amputation
	Revision arthroplasty
	Removal of metalwork and bone grafting
	Removal of metalwork and implantation of local antibiotics
A

Removal of metal work implantation of gentamicin beads and delayed revision is the mainstay of managing this complication.

Osteomyelitis

Infection of the bone

Causes
S aureus and occasionally Enterobacter or Streptococcus species
In sickle cell: Salmonella species

Clinical features
Erythema
Pain
Fever

Investigation
X-ray: lytic centre with a ring of sclerosis
Bone biopsy and culture

Treatment
Prolonged antibiotics
Sequestra may need surgical removal

121
Q

A 73 year old lady presents with pain in her left hip. She was walking around the house when she tripped over a rug and fell over. Apart from temporal arteritis which is well controlled with prednisolone she is otherwise well. On examination, her leg is shorted and externally rotated. Her serum alkaline phosphatase and calcium are normal. What is the likely underlying disease process?

	Pagets disease
	Metastatic renal cancer
	Osteoporosis
	Osteopetrosis
	Osteoclastoma
A

The combination of age, female gender and steroids coupled with hip pain on minor trauma are strongly suggestive of osteoporosis complicated by pathological fracture.

Pagets
Focal bone resorption followed by excessive and chaotic bone deposition
Affects (in order): spine, skull, pelvis and femur
Serum alkaline phosphatase raised (other parameters normal)
Abnormal thickened, sclerotic bone on x-rays
Risk of cardiac failure with >15% bony involvement
Small risk of sarcomatous change
Bisphosphonates
Osteoporosis
Excessive bone resorption resulting in demineralised bone
Commoner in old age
Increased risk of pathological fracture, otherwise asymptomatic
Alkaline phosphatase normal, calcium normal
Bisphosphonates, calcium and vitamin D
Secondary bone tumours
Bone destruction and tumour infiltration
Mirel scoring used to predict risk of fracture
Appearances depend on primary (e.g.sclerotic - prostate, lytic - breast)
Elevated serum calcium and alkaline phosphatase may be seen
Radiotherapy, prophylactic fixation and analgesia

122
Q

A 60 year old female injures her ankle after a falling from a horse. On examination she has severe swelling and bruising of the ankle with x-rays demonstrating a comminuted intra-articular distal tibia fracture. The ankle has been temporarily reduced and splinted in the emergency department. What should be done next?

	Application of external fixation device
	Application of below knee plaster cast
	Application of ankle boot
	Surgical fixation
	Below knee amputation
A

This is a pilon fracture, which a high energy injury of the distal tibia. The patient will ultimately require surgical fixation but early management involves applying a spanning external fixator to temporarily reduce the fracture and allow soft tissue swelling to settle. A CT scan should then be performed to aid surgical planning.

An ankle fracture relates to a fracture around the tibio-talar joint. It generally refers to a fracture involving the lateral, and/or medial and/or posterior malleolus. Pilon and Tillaux fractures are also considered to be ankle fractures, but are not covered here.
Ankle fractures are common. They effect men and women in equal numbers, but men have a higher rate as young adults (sports and contact injuries), and women a higher rate post-menopausal (fragility type fracture).

Osseous anatomy
The ankle (or mortise) joint consists of the distal tibia (tibial plafond and posterior malleolus), the distal fibula (lateral malleolus), and the talus. The main movement at the ankle joint is plantar and dorsiflexion.

Ligamentous anatomy
Medial side: Deltoid ligament. This is divided into superficial and deep portions. It is the primary restraint to valgus tilting of the talus.

Lateral side: Lateral ligament complex consisting from anterior to posterior of the anterior talofibular ligament (ATFL), calcaneofibular ligament (CFL), and the posterior talofibular ligament (PTFL). Together they resist valgus stress to the ankle, and are a restraint to anterior translation of the talus within the mortise joint.

Syndesmosis: The syndesmosis is a ligament complex between the distal tibia and fibula, holding the two bones together. It is fundamental to the integrity of the ankle joint, and its disruption leads to instability. It consists of (from anterior to posterior) the anterior-inferior tibiofibular ligament (AITFL), the transverse tibiofibular ligament (TTFL), the interosseous membrane, and the posterior-inferior tibiofibular ligament (PITFL).

Presentation and initial management
Patients will present following a traumatic event with a painful, swollen ankle, and reluctance/inability to weight bear. The Ottawa rules can be applied to differentiate between an ankle fracture and sprain, but can be unreliable.
In high energy injuries, management should follow ATLS principles to identify more significant injuries first. Neurovascular status of the foot should be documented, and open injuries should be excluded. If an open injury is identified, it should be managed in line with BOAST 4 principles1. If an obvious deformity exists, it should be reduced as soon as possible with appropriate analgesia or conscious sedation. Radiographs of clearly deformed or dislocated joints are not necessary, and removing the pressure on the surrounding soft tissues from the underlying bony deformity is the priority. If the fracture pattern is not clinically obvious then plain radiographs are appropriate and will guide the subsequent manipulation during plaster-of-paris below knee backslab application.

Imaging
AP, lateral and mortise views (20o internal rotation) are essential to evaluate fracture displacement and syndesmotic injury. Decreased tibiofibular overlap, medial joint clear space and lateral talar shift all indicate a syndesmotic injury. (In subtle cases of shift, imaging the uninjured ankle can be helpful as a proportion of the population have little or no tibiotalar overlap 2.)

Where there is suspicion of syndesmosis involvement in the absence of radiographic evidence, stress radiographs can be diagnostic.
Complex fracture patterns (and increasingly posterior malleolar fractures) are best defined using CT.

Classification
The most commonly used classifications are Lauge-Hansen and Danis-Weber.

Lauge-Hansen
Comprises two parts: first part is the foot position, and the second part is the force applied. Useful for understanding the forces involved and therefore predict the ligamentous or bony injury. Results in four injury patterns:
Supination - Adduction (SA) - 10-20%
Supination - External rotation (SER) - 40-75%
Pronation - Abduction (PA) - 5-20%
Pronation - External rotation (PER) - 5-20%

Not often used in clinical practice but good for understanding the principles of ankle fracture.

Danis-Weber
Commonly used. Based on the level of the fibula fracture in relation to the syndesmosis. The more proximal, the greater the risk of syndesmotic injury and therefore fracture instability.
A - fracture below the level of the syndesmosis
B - fracture at the level of the syndesmosis / level of the tibial plafond
C - fracture above the level of the syndesmosis. This includes Maisonneuve fractures (proximal fibula fracture), which can be associated with ankle instability. Beware the high fibula fracture - it may be an ankle fracture!

The Weber classification is based purely on the the lateral side. All injuries can include a medial or posterior bony or ligamentous injury which also dictates fracture stability (bimalleolar and trimalleolar fractures are more unstable).

Treatment
When deciding upon treatment for an ankle fracture, one must consider both the fracture and the patient. Diabetic patients and smokers are at greater risk of post-operative complication, especially wound problems and infection. Likewise, the long term outcome of post-traumatic arthritis from a malunited ankle fracture is extremely important for a young patient, but not as relevant in the elderly. Therefore, normal surgical decision processes apply as with all fractures.

Defining stability of an ankle fracture underpins the treatment decision.
Weber A - Unimalleolar Weber A Weber fractures by definition are stable and therefore can be mobilised fully weight bearing in an ankle boot.

Weber C - Fractures tend to include syndesmotic disruption and are usually bimalleolar (either bony or ligamentous). They are therefore unstable and usually require operative fixation. In addition to the fracture fixation, the syndesmosis usually requires reconstruction/augmentation with screws to restore the joint integrity and function.

Weber B - B fractures vary greatly. They can be part of a trimalleolar injury and therefore extremely unstable, requiring fixation. Alternatively, a uni-malleolar Weber B fracture can be a stable injury, and therefore mobilised immediately in an ankle boot. Defining the stability can be challenging, and often involves stress radiographs, or a trial of mobilisation and repeat radiographs. Defining stability is the subject of much ongoing research. However, treating undisplaced ankle fractures in a below knee plaster, non-weight bearing for six weeks is still widely practised, and a safe approach.

When operative fixation is appropriate, it is usually via open reduction and internal fixation using plates and screws. It must be carried out when soft tissue swelling has settled in order to minimise the risk of wound problems. This can often take a week to settle.

The use of fibula nails is expanding, but is not yet mainstream. Ankle fractures can also be treated with external fixation, or with a hind foot nail in patients who need fixation but where soft tissue or bone quality is poor.

Post operative management
Ankle fractures generally take 6 weeks to unite enough to prevent secondary displacement. This is therefore an appropriate time period to keep a cast on in a conservatively managed patient. Weight bearing post-operatively depends on the quality of the fixation and bone quality, and preference varies between surgeons, ranging from aggressive early mobilisation to a period of non-weight bearing. Return to activities takes approximately three months, and often requires assistance of a physiotherapist to improve range-of-movement and muscle strengthening.

123
Q

A 28 year old man complains of pain and weakness in the shoulder. He has recently been unwell with glandular fever from which he is fully recovered. On examination there is some evidence of muscle wasting and a degree of winging of the scapula. Power during active movements is impaired. What is the most likely cause?

	Parsonage-Turner syndrome
	Adhesive capsulitis
	Rotator cuff tear
	Osteoarthritis
	Calcific tendonitis
A

Parsonage-Turner syndrome - This is a peripheral neuropathy that may complicate viral illnesses and usually resolves spontaneously.

Shoulder fractures and dislocations
Fractures
Proximal humerus
Background
Third most common fragility fracture in the elderly.
Results from low energy fall in predominantly elderly females, or from high energy trauma in young males.
Can be associated with nerve injury (commonly axillary), and fracture-dislocation of the humeral head. Detailed neurological assessment is essential for all upper limb injuries.

Anatomy
Osteology
Consists of articular head, greater tuberosity, lesser tuberosity, metaphysis and diaphysis. Between the articular head and the tuberosities is the anatomical neck (previous physis). Between the tuberosities and the metaphysis is the surgical neck.
The supraspinatus, infraspinatus and teres minor muscles attach to the greater tuberosity. The subscapularis muscle attaches to the lesser tuberosity.

Vascular Supply
Humeral head is supplied by the anterior and posterior humeral circumflex arteries. Anatomical neck fractures are at greatest risk of osteonecrosis.

Imaging
Imaging aims to both delineate the fracture pattern, and confirm/exlude the presence of an associated dislocation.
Radiographs - True anteroposterior (AP), axillary lateral and/or scapula Y view.
CT - indicated to better define intra-articular involvement and to aid pre-operative planning. MRI is not useful for fracture imaging.

Classification
Description of the fracture is often more useful than classification. Particular attention should be paid to humeral alignment, fracture displacement, and greater tuberosity position (rotator cuff will pull the GT supero-posterioly, which can cause impingement problems with malunion).
- Neer Classification: Most commonly used. Describes fracture as 2,3,or 4 part depending upon the number main fragments. Also comments on the degree of displacement. Fragments:
-greater tuberosity
-lesser tuberosity
- articular surface
- shaft
Displacement: >1cm or angulation >45 degrees.

Treatment
The vast majority of proximal humeral fractures are minimally displaced, and therefore can be managed conservatively. This involves immobilisation in a polysling, and progressive mobilisation. Pendular exercise can commence at 14 days, and active abduction from 4-6 weeks.

Irreducible fracture dislocation is an indication for operative management. Other indications include large displacement, younger patient, head splitting (intra-articular fractures). However, the recent PROFHER trial (1) has suggested no benefit to operative intervention on patient outcome (it must be applied cautiously as majority of patients were elderly with extraarticular fractures). Options available for surgical management include:

ORIF Most commonly used. Plate and screw fixation. Can reconstruct complex fractures.
Intramedullary nail Suitable for extra-articular configuration, predominantly surgical neck +/- GT fractures.

Hemiarthroplasty Used for un-reconstructable fractures in the older patient who has good glenoid quality.
Total shoulder arthroplasty Unconstructable fractures where high functioning shoulder is required (hemiarthroplasty will cause glenoid erosion)
Reverse shoulder arthroplasty Total shoulder arthroplasty that provides better functional outcome than conventional total shoulder replacement.

Scapula
Background
Uncommon fractures usually associated with high energy trauma. Most commonly involve scapula body or spine (50%), glenoid fossa and glenoid neck. Important to exclude associated life threatening injury.

Imaging
Plain radiographs should include true anteroposterior (AP), axillary lateral and/or scapula Y view. CT scanning is useful for defining intra-articular involvement, displacement and for three dimensional reconstruction.

Classification
Based on the location of the fracture (coracoid, acromion, glenoid neck, glenoid fossa, scapula body). Beware of ipsilateral glenoid neck and clavicle fracture -floating shoulder - where limb is effectively dissociated from axial skeleton.

Treatment
The vast majority of scapula fractures are amenable to conservative management, consisting of sling immobilisation for two weeks followed by early rehabilitation. Floating shoulder will usually require fixation, and consideration of surgery should also be given to intra-articular and displaced/angulated glenoid fractures.

Dislocations

Types
Dislocations around the shoulder joint include glenohumeral dislocation, acromioclavicular joint disruption and sternoclavicular dislocation. Only glenohumeral dislocation will be covered here.

Glenohumeral dislocation
Diagnosis, classification and management are covered here.

Background
Shoulder dislocation is commonly seen in A&E. It has a high recurrence rate that is as high as 80% in teenagers. Initial management requires emergent reduction to prevent lasting chondral damage.

Early assessment and management
Usually a traumatic cause (multi-directional instability in frequent dislocations requires discussion with orthopaedics and is not covered here). Careful history, examination and documentation of neurovascular status of the limb, in particular the axillary nerve (regimental badge sensation). This should be re-assessed post manipulation. Early radiographs to confirm direction of dislocation.

Initial management consists of emergent closed reduction under under entanox and analgesia, but often requires conscious sedation. Arm should then be immobilised in a polysling, and XR to confirm relocation.

Imaging - True anteroposterior (AP), axillary lateral and/or scapula Y view. Reduced humeral head should lie between acromion and coracoid on lateral/scapula view.

Types
Direction Features Cause Examination Reduction techniques
Anterior Most Common >90% Usually traumatic - anterior force on arm when shoulder is abducted, eternally rotated Loss of shoulder contour - sulcus sign. Humeral head can be felt anteriorly.
Hippocratic.
Milch.
Stimson.

Kocher not advised due to complication of fracture
Posterior 50% missed in A&E 50% traumatic, but classically post seizure or electrocution Shoulder locked in internal rotation. XR may show lightbulb appearance. Gentle lateral traction to adducted arm.
Inferior Rare Associated with pectorals and rotator cuff tears, and glenoid fracture As for primary injury Management of primary injury

Superior Rare Associated with acrominon/clavicle fracture As for primary injury Management of primary injury

Associated injuries
Bankart lesion - avulsion of the anterior glenoid labrum with an anterior shoulder dislocation (reverse Bankart if poster labrum in posterior dislocation).
Hill Sachs defect - chondral impaction on posteriosuperior humeral head from contact with gleonoid rim. Can be large enough to lock shoulder, requiring open reduction. (Reverse Hill Sachs in posterior dislocation).
Rotator cuff tear - increases with age.
Greater or lesser tuberosity fracture - increases with age.
Humeral neck fracture - shoulder fracture dislocation. More common in high energy trauma and elderly. Should be discussed with orthopaedics prior to any attempted reduction.

Rotator Cuff Disease

Rotator cuff disease is a spectrum of conditions that ranges from subacromial impingement to rotator cuff tears and eventually to rotator cuff arthropathy (arthritis).

Anatomy
The rotator cuff is a group of four muscles that are important in shoulder movements, and maintenance of glenohumeral stability.
Muscle Scapular attachment Humeral attachment Action Innervation
Supraspinatus Supraspinatus fossa Superior facet of greater tuberosity Initiation of abduction of humerus Suprascapular nerve
Infraspinatus Infraspinatus fossa Posterior facet of greater tuberosity External rotation of humerus Suprascapular nerve
Teres Minor Lateral border Inferior facet of greater tuberosity External rotation of humerus Axillary Nerve
Subscapularis Subscapular fossa Lesser tuberosity Internal rotation of humerus Upper and lower subscapular nerve

The inferior rotator cuff muscles (infraspinatus, teres minor, and subscapularis) balance the superior pull of the deltoid. Injury/tear results in upward migration of the humeral head on the glenoid (can be seen on AP radiograph).
Likewise, the anterior muscles (subscapularis) are balanced with the posterior muscles (infraspinatus, teres minor).

Subacromial Impingement

The most common cause of shoulder pain, which results from impingement of the superior cuff on the undersurface of the acromion, and an inflammatory bursitis.
Associated with certain types of acromial morphology (Bigliani classification).
Presents as insidious pain which is exacerbated by overhead activities.

Rotator Cuff Tear

Often presents as an acute event on the background of chronic subacromial impingement in the older patient, but can present as an avulsion injury in younger patients.
Majority of tears are to the superior cuff (supraspinatus, infraspinatus, teres minor), though a tear to subscapularis is associated with subcoracoid impingement.
Tears present as pain and weakness when using the muscles in question.

Rotator Cuff Arthropathy

Defined as shoulder arthritis in the setting of rotator cuff dysfunction. Results from superior migration due to the loss of rotator cuff function and integrity. Unopposed deltoid pulls the humeral head superiorly.
Associated with massive chronic cuff tears.

Imaging

Plain radiographs
AP of the shoulder may show superior migration of the humerus with a cuff tear, and features of arthritis with arthropathy. Other causes of pain may also be identified (e.g. calcific tendonitis/fracture)
Outlet view is useful for defining the acromial morphology

USS
Allows dynamic imaging of the cuff, and is inexpensive. However, it is very user dependent.

MRI
Best imaging modality for cuff pathology.
Also allows imaging of the rest of the shoulder. When intra-articular pathology is suspected, can be combined with an arthrogram for improved sensitivity and specificity.

Treatment

Subacromial impingement
Physiotherapy, oral anti-inflammatory medication
Subacromial steroid injection can settle inflammation
Arthroscopic subacromial decompression by shaving away the undersurface of the acromion, more space is created for the rotator cuff. Cuff integrity is assessed also at time of surgery, and can be repaired if necessary.

Rotator cuff tear
When considering repair of a cuff tear, the age and activity of the patient, the nature of the tear (degenerative vs. acute traumatic), and the size and retraction of the tear should be considered when making a surgical plan.
Mild tears or tears in the elderly can be managed conservatively, as outlined above.
Moderate tears can be repaired arthroscopically. Massive or retracted tears will often require an open repair (occasionally with a tendon transfer). Subacromial decompression is performed at the same time to reduce impingement, symptoms and recurrence.

Calcific tendonitis
Calcific tendonitis involves calcific deposits within tendons anywhere in the body, but most commonly in the rotator cuff (specifically the supraspinatus tendon). When present in the shoulder, it is associated with subacromial impingement and pain.

Pathology
More common in women aged 30-60 years.
Association with diabetes and hypothyroidism

There are three stages of calcification
Formative phase characterized by calcific deposits
Resting phase deposit is stable, but presents with impingement problems
Resorptive phase phagocytic resorption. Most painful stage.

Presentation
Similar in presentation to subacromial impingement, with pain especially with over head activities. Atraumatic in nature.

Imaging
Plain radiographs show calcification of the rotator cuff, usually within 1.5cm of its insertion on the humerus. Supraspinatus outlet views can show level of impingment. Further imaging is rarely needed.

Treatment
Non-operative NSAIDS, steroid injection (controversial, but practiced) and physiotherapy. Approximately 75% will resolve by 6 months with conservative management.
Ultrasound guided or surgical needle barbotage can break down deposits and resolve symptoms. Occasionally surgical excision is required.

Adhesive capsulitis (Frozen Shoulder)
Pain and loss of movement of shoulder joint, which involves fibroplastic proliferation of capsular tissue, causing soft tissue scarring and contracture. Patients present with a painful and decreased arc of motion.
Associated with prolonged immobilization, previous surgery, thyroid disorders (AI) and diabetes
Classically three stages which can take up to two years to resolve:
Stage one the freezing and painful stage
Stage two the frozen and stiff stage
Stage three the thawing stage, where shoulder movement slowly improves

Imaging
Plain radiographs to exclude other causes of a painful shoulder
MRI arthrogram may show capsular contracture, and again may be used to exclude cuff pathology. However, often not performed as diagnosis is largely clinical.

Treatment
Non-operative NSAIDS, steroid injection and physiotherapy. Patience is required as condition can take up to 2 years to improve.
Operative MUA or arthroscopic adhesiolysis (release of adhesions) can expedite recovery, followed by intensive physiotherapy.

Glenohumeral Arthritis
Background
May be osteoarthritis (primary or secondary to cuff tear or trauma), rheumatoid arthritis, or as part of a spondyloarthropathy. Majority of those with RA will develop symptoms.
More common in the elderly
Presents like any other arthritis - pain at night and with movement

Imaging
AP and axillary radiographs will show features of arthritis.
CT/MRI is often useful to classify the shape of the glenoid and extent of bone loss when considering arthroplasty. MRI also essential to asses integrity of rotator cuff if considering shoulder replacement.

Treatment
Like all orthopaedics, start with simple measures:
NSAIDS, management of RA, physiotherapy, steroid injection.
Hemiarthroplasty can sometimes be considered if glenoid is in excellent condition or if patient has large comorbidity.
Arthroscopic debridement is useful if patient has isolated ACJ arthritis, but is rarely used for glenohumeral arthritis.
Total shoulder replacement is shown to produce superior outcome when compared to hemiarthroplasty in terms of pain relief, function and implant survival.
Total shoulder replacement can be anatomical (ball on humerus, with cup on glenoid), or reverse geometry (ball on glenoid, with cup on humerus). Anatomical TSR requires an in tact rotator cuff, so often reverse is preferable when the cuff if questionable in integrity.

124
Q

A 19 year old female presents to the clinic with progressive pain in her neck and back. The condition has been progressively worsening over the past 6 months. She has not presented previously because she was an inpatient with a disease flare of ulcerative colitis. On examination, she has a stiff back with limited spinal extension on bending forwards. What is the most likely explanation for this process?

	Spondylolysis
	Spondylolisthesis
	Functional scoliosis
	Scheuermanns disease
	Ankylosing spondylitis
A

Ankylosing spondylitis is associated with HLA B27, there is a strong association with ulcerative colitis in such individuals. The clinical findings are usually of a kyphosis affecting the cervical and thoracic spine. Considerable symptomatic benefit may be obtained using non steroidal anti inflammatory drugs. These should be used carefully in patients with inflammatory bowel disease who may be taking steroids.

Ankylosing spondylitis
Chronic inflammatory disorder affecting the axial skeleton
Sacro-ilitis is a usually visible in plain films
Up to 20% of those who are HLA B27 positive will develop the condition
Affected articulations develop bony or fibrous changes
Typical spinal features include loss of the lumbar lordosis and progressive kyphosis of the cervico-thoracic spine
Scheuermann’s disease
Epiphysitis of the vertebral joints is the main pathological process
Predominantly affects adolescents
Symptoms include back pain and stiffness
X-ray changes include epiphyseal plate disturbance and anterior wedging
Clinical features include progressive kyphosis (at least 3 vertebrae must be involved)
Minor cases may be managed with physiotherapy and analgesia, more severe cases may require bracing or surgical stabilisation
Scoliosis
Consists of curvature of the spine in the coronal plane
Divisible into structural and non structural, the latter being commonest in adolescent females who develop minor postural changes only. Postural scoliosis will typically disappear on manoeuvres such as bending forwards
Structural scoliosis affects > 1 vertebral body and is divisible into idiopathic, congential and neuromuscular in origin. It is not correctable by alterations in posture
Within structural scoliosis, idiopathic is the most common type
Severe, or progressive structural disease is often managed surgically with bilateral rod stabilisation of the spine
Spina bifida
Non fusion of the vertebral arches during embryonic development
Three categories; myelomeningocele, spina bifida occulta and meningocele
Myelomeningocele is the most severe type with associated neurological defects that may persist in spite of anatomical closure of the defect
Up to 10% of the population may have spina bifida occulta, in this condition the skin and tissues (but not not bones) may develop over the distal cord. The site may be identifiable by a birth mark or hair patch
The incidence of the condition is reduced by use of folic acid supplements during pregnancy
Spondylolysis
Congenital or acquired deficiency of the pars interarticularis of the neural arch of a particular vertebral body, usually affects L4/ L5
May be asymptomatic and affects up to 5% of the population
Spondylolysis is the commonest cause of spondylolisthesis in children
Asymptomatic cases do not require treatment
Spondylolisthesis
This occurs when one vertebra is displaced relative to its immediate inferior vertebral body
May occur as a result of stress fracture or spondylolysis
Traumatic cases may show the classic “Scotty Dog” appearance on plain films
Treatment depends upon the extent of deformity and associated neurological symptoms, minor cases may be actively monitored. Individuals with radicular symptoms or signs will usually require spinal decompression and stabilisation

125
Q

A 65 year old type 2 diabetic with poor glycaemic control is admitted with forefoot cellulitis. X-ray of the foot shows some evidence of osteomyelitis of the 2nd ray but overlying skin is healthy. What is the best treatment initially?

	Intravenous antibiotics
	Below knee amputation
	Transfemoral amputation
	Midfoot amputation
	Ray amputation
A

It is worth attempting to try and resolve this situation with antibiotics at first presentation. A primary amputation will not heal well and may result in progressive surgery.

Osteomyelitis

Infection of the bone

Causes
S aureus and occasionally Enterobacter or Streptococcus species
In sickle cell: Salmonella species

Clinical features
Erythema
Pain
Fever

Investigation
X-ray: lytic centre with a ring of sclerosis
Bone biopsy and culture

Treatment
Prolonged antibiotics
Sequestra may need surgical removal

126
Q

With which of the conditions listed below is a Hill- Sachs lesions classically associated?

	Fracture of the surgical neck of the humerus
	Glenohumeral dislocation
	Supraspinatus tear
	Acromioclavicular dislocation
	Sternoclavicular dislocation
A

A Hill-Sachs lesion occurs when the cartilage surface of the humerus is in contact with the rim of the glenoid. About 50% of anterior glenohumeral dislocations are associated with this lesion.

Shoulder fractures and dislocations
Fractures
Proximal humerus
Background
Third most common fragility fracture in the elderly.
Results from low energy fall in predominantly elderly females, or from high energy trauma in young males.
Can be associated with nerve injury (commonly axillary), and fracture-dislocation of the humeral head. Detailed neurological assessment is essential for all upper limb injuries.

Anatomy
Osteology
Consists of articular head, greater tuberosity, lesser tuberosity, metaphysis and diaphysis. Between the articular head and the tuberosities is the anatomical neck (previous physis). Between the tuberosities and the metaphysis is the surgical neck.
The supraspinatus, infraspinatus and teres minor muscles attach to the greater tuberosity. The subscapularis muscle attaches to the lesser tuberosity.

Vascular Supply
Humeral head is supplied by the anterior and posterior humeral circumflex arteries. Anatomical neck fractures are at greatest risk of osteonecrosis.

Imaging
Imaging aims to both delineate the fracture pattern, and confirm/exlude the presence of an associated dislocation.
Radiographs - True anteroposterior (AP), axillary lateral and/or scapula Y view.
CT - indicated to better define intra-articular involvement and to aid pre-operative planning. MRI is not useful for fracture imaging.

Classification
Description of the fracture is often more useful than classification. Particular attention should be paid to humeral alignment, fracture displacement, and greater tuberosity position (rotator cuff will pull the GT supero-posterioly, which can cause impingement problems with malunion).
- Neer Classification: Most commonly used. Describes fracture as 2,3,or 4 part depending upon the number main fragments. Also comments on the degree of displacement. Fragments:
-greater tuberosity
-lesser tuberosity
- articular surface
- shaft
Displacement: >1cm or angulation >45 degrees.

Treatment
The vast majority of proximal humeral fractures are minimally displaced, and therefore can be managed conservatively. This involves immobilisation in a polysling, and progressive mobilisation. Pendular exercise can commence at 14 days, and active abduction from 4-6 weeks.

Irreducible fracture dislocation is an indication for operative management. Other indications include large displacement, younger patient, head splitting (intra-articular fractures). However, the recent PROFHER trial (1) has suggested no benefit to operative intervention on patient outcome (it must be applied cautiously as majority of patients were elderly with extraarticular fractures). Options available for surgical management include:

ORIF Most commonly used. Plate and screw fixation. Can reconstruct complex fractures.
Intramedullary nail Suitable for extra-articular configuration, predominantly surgical neck +/- GT fractures.

Hemiarthroplasty Used for un-reconstructable fractures in the older patient who has good glenoid quality.
Total shoulder arthroplasty Unconstructable fractures where high functioning shoulder is required (hemiarthroplasty will cause glenoid erosion)
Reverse shoulder arthroplasty Total shoulder arthroplasty that provides better functional outcome than conventional total shoulder replacement.

Scapula
Background
Uncommon fractures usually associated with high energy trauma. Most commonly involve scapula body or spine (50%), glenoid fossa and glenoid neck. Important to exclude associated life threatening injury.

Imaging
Plain radiographs should include true anteroposterior (AP), axillary lateral and/or scapula Y view. CT scanning is useful for defining intra-articular involvement, displacement and for three dimensional reconstruction.

Classification
Based on the location of the fracture (coracoid, acromion, glenoid neck, glenoid fossa, scapula body). Beware of ipsilateral glenoid neck and clavicle fracture -floating shoulder - where limb is effectively dissociated from axial skeleton.

Treatment
The vast majority of scapula fractures are amenable to conservative management, consisting of sling immobilisation for two weeks followed by early rehabilitation. Floating shoulder will usually require fixation, and consideration of surgery should also be given to intra-articular and displaced/angulated glenoid fractures.

Dislocations

Types
Dislocations around the shoulder joint include glenohumeral dislocation, acromioclavicular joint disruption and sternoclavicular dislocation. Only glenohumeral dislocation will be covered here.

Glenohumeral dislocation
Diagnosis, classification and management are covered here.

Background
Shoulder dislocation is commonly seen in A&E. It has a high recurrence rate that is as high as 80% in teenagers. Initial management requires emergent reduction to prevent lasting chondral damage.

Early assessment and management
Usually a traumatic cause (multi-directional instability in frequent dislocations requires discussion with orthopaedics and is not covered here). Careful history, examination and documentation of neurovascular status of the limb, in particular the axillary nerve (regimental badge sensation). This should be re-assessed post manipulation. Early radiographs to confirm direction of dislocation.

Initial management consists of emergent closed reduction under under entanox and analgesia, but often requires conscious sedation. Arm should then be immobilised in a polysling, and XR to confirm relocation.

Imaging - True anteroposterior (AP), axillary lateral and/or scapula Y view. Reduced humeral head should lie between acromion and coracoid on lateral/scapula view.

Types
Direction Features Cause Examination Reduction techniques
Anterior Most Common >90% Usually traumatic - anterior force on arm when shoulder is abducted, eternally rotated Loss of shoulder contour - sulcus sign. Humeral head can be felt anteriorly.
Hippocratic.
Milch.
Stimson.

Kocher not advised due to complication of fracture
Posterior 50% missed in A&E 50% traumatic, but classically post seizure or electrocution Shoulder locked in internal rotation. XR may show lightbulb appearance. Gentle lateral traction to adducted arm.
Inferior Rare Associated with pectorals and rotator cuff tears, and glenoid fracture As for primary injury Management of primary injury

Superior Rare Associated with acrominon/clavicle fracture As for primary injury Management of primary injury

Associated injuries
Bankart lesion - avulsion of the anterior glenoid labrum with an anterior shoulder dislocation (reverse Bankart if poster labrum in posterior dislocation).
Hill Sachs defect - chondral impaction on posteriosuperior humeral head from contact with gleonoid rim. Can be large enough to lock shoulder, requiring open reduction. (Reverse Hill Sachs in posterior dislocation).
Rotator cuff tear - increases with age.
Greater or lesser tuberosity fracture - increases with age.
Humeral neck fracture - shoulder fracture dislocation. More common in high energy trauma and elderly. Should be discussed with orthopaedics prior to any attempted reduction.

Rotator Cuff Disease

Rotator cuff disease is a spectrum of conditions that ranges from subacromial impingement to rotator cuff tears and eventually to rotator cuff arthropathy (arthritis).

Anatomy
The rotator cuff is a group of four muscles that are important in shoulder movements, and maintenance of glenohumeral stability.
Muscle Scapular attachment Humeral attachment Action Innervation
Supraspinatus Supraspinatus fossa Superior facet of greater tuberosity Initiation of abduction of humerus Suprascapular nerve
Infraspinatus Infraspinatus fossa Posterior facet of greater tuberosity External rotation of humerus Suprascapular nerve
Teres Minor Lateral border Inferior facet of greater tuberosity External rotation of humerus Axillary Nerve
Subscapularis Subscapular fossa Lesser tuberosity Internal rotation of humerus Upper and lower subscapular nerve

The inferior rotator cuff muscles (infraspinatus, teres minor, and subscapularis) balance the superior pull of the deltoid. Injury/tear results in upward migration of the humeral head on the glenoid (can be seen on AP radiograph).
Likewise, the anterior muscles (subscapularis) are balanced with the posterior muscles (infraspinatus, teres minor).

Subacromial Impingement

The most common cause of shoulder pain, which results from impingement of the superior cuff on the undersurface of the acromion, and an inflammatory bursitis.
Associated with certain types of acromial morphology (Bigliani classification).
Presents as insidious pain which is exacerbated by overhead activities.

Rotator Cuff Tear

Often presents as an acute event on the background of chronic subacromial impingement in the older patient, but can present as an avulsion injury in younger patients.
Majority of tears are to the superior cuff (supraspinatus, infraspinatus, teres minor), though a tear to subscapularis is associated with subcoracoid impingement.
Tears present as pain and weakness when using the muscles in question.

Rotator Cuff Arthropathy

Defined as shoulder arthritis in the setting of rotator cuff dysfunction. Results from superior migration due to the loss of rotator cuff function and integrity. Unopposed deltoid pulls the humeral head superiorly.
Associated with massive chronic cuff tears.

Imaging

Plain radiographs
AP of the shoulder may show superior migration of the humerus with a cuff tear, and features of arthritis with arthropathy. Other causes of pain may also be identified (e.g. calcific tendonitis/fracture)
Outlet view is useful for defining the acromial morphology

USS
Allows dynamic imaging of the cuff, and is inexpensive. However, it is very user dependent.

MRI
Best imaging modality for cuff pathology.
Also allows imaging of the rest of the shoulder. When intra-articular pathology is suspected, can be combined with an arthrogram for improved sensitivity and specificity.

Treatment

Subacromial impingement
Physiotherapy, oral anti-inflammatory medication
Subacromial steroid injection can settle inflammation
Arthroscopic subacromial decompression by shaving away the undersurface of the acromion, more space is created for the rotator cuff. Cuff integrity is assessed also at time of surgery, and can be repaired if necessary.

Rotator cuff tear
When considering repair of a cuff tear, the age and activity of the patient, the nature of the tear (degenerative vs. acute traumatic), and the size and retraction of the tear should be considered when making a surgical plan.
Mild tears or tears in the elderly can be managed conservatively, as outlined above.
Moderate tears can be repaired arthroscopically. Massive or retracted tears will often require an open repair (occasionally with a tendon transfer). Subacromial decompression is performed at the same time to reduce impingement, symptoms and recurrence.

Calcific tendonitis
Calcific tendonitis involves calcific deposits within tendons anywhere in the body, but most commonly in the rotator cuff (specifically the supraspinatus tendon). When present in the shoulder, it is associated with subacromial impingement and pain.

Pathology
More common in women aged 30-60 years.
Association with diabetes and hypothyroidism

There are three stages of calcification
Formative phase characterized by calcific deposits
Resting phase deposit is stable, but presents with impingement problems
Resorptive phase phagocytic resorption. Most painful stage.

Presentation
Similar in presentation to subacromial impingement, with pain especially with over head activities. Atraumatic in nature.

Imaging
Plain radiographs show calcification of the rotator cuff, usually within 1.5cm of its insertion on the humerus. Supraspinatus outlet views can show level of impingment. Further imaging is rarely needed.

Treatment
Non-operative NSAIDS, steroid injection (controversial, but practiced) and physiotherapy. Approximately 75% will resolve by 6 months with conservative management.
Ultrasound guided or surgical needle barbotage can break down deposits and resolve symptoms. Occasionally surgical excision is required.

Adhesive capsulitis (Frozen Shoulder)
Pain and loss of movement of shoulder joint, which involves fibroplastic proliferation of capsular tissue, causing soft tissue scarring and contracture. Patients present with a painful and decreased arc of motion.
Associated with prolonged immobilization, previous surgery, thyroid disorders (AI) and diabetes
Classically three stages which can take up to two years to resolve:
Stage one the freezing and painful stage
Stage two the frozen and stiff stage
Stage three the thawing stage, where shoulder movement slowly improves

Imaging
Plain radiographs to exclude other causes of a painful shoulder
MRI arthrogram may show capsular contracture, and again may be used to exclude cuff pathology. However, often not performed as diagnosis is largely clinical.

Treatment
Non-operative NSAIDS, steroid injection and physiotherapy. Patience is required as condition can take up to 2 years to improve.
Operative MUA or arthroscopic adhesiolysis (release of adhesions) can expedite recovery, followed by intensive physiotherapy.

Glenohumeral Arthritis
Background
May be osteoarthritis (primary or secondary to cuff tear or trauma), rheumatoid arthritis, or as part of a spondyloarthropathy. Majority of those with RA will develop symptoms.
More common in the elderly
Presents like any other arthritis - pain at night and with movement

Imaging
AP and axillary radiographs will show features of arthritis.
CT/MRI is often useful to classify the shape of the glenoid and extent of bone loss when considering arthroplasty. MRI also essential to asses integrity of rotator cuff if considering shoulder replacement.

Treatment
Like all orthopaedics, start with simple measures:
NSAIDS, management of RA, physiotherapy, steroid injection.
Hemiarthroplasty can sometimes be considered if glenoid is in excellent condition or if patient has large comorbidity.
Arthroscopic debridement is useful if patient has isolated ACJ arthritis, but is rarely used for glenohumeral arthritis.
Total shoulder replacement is shown to produce superior outcome when compared to hemiarthroplasty in terms of pain relief, function and implant survival.
Total shoulder replacement can be anatomical (ball on humerus, with cup on glenoid), or reverse geometry (ball on glenoid, with cup on humerus). Anatomical TSR requires an in tact rotator cuff, so often reverse is preferable when the cuff if questionable in integrity.

127
Q

A 20 year old woman trips over a step, injuring her ankle. Examination reveals tenderness over the lateral malleolus and an x-ray demonstrates an undisplaced fracture distal to the syndesmosis. What is the best course of action?

	Application of ankle boot
	Surgical fixation
	Application of full leg plaster cast
	Application of external fixator
	Application of Ilizarov frame
A

This is a Weber A fracture. It is a stable ankle injury and can therefore be managed conservatively. Whilst this patient could also be treated in a below knee plaster, most clinicians would nowadays treat this injury in an ankle boot. Patients should be advised to mobilise in the ankle boot, as pain allows, and can wean themselves out of the boot as the symptoms improve.

An ankle fracture relates to a fracture around the tibio-talar joint. It generally refers to a fracture involving the lateral, and/or medial and/or posterior malleolus. Pilon and Tillaux fractures are also considered to be ankle fractures, but are not covered here.
Ankle fractures are common. They effect men and women in equal numbers, but men have a higher rate as young adults (sports and contact injuries), and women a higher rate post-menopausal (fragility type fracture).

Osseous anatomy
The ankle (or mortise) joint consists of the distal tibia (tibial plafond and posterior malleolus), the distal fibula (lateral malleolus), and the talus. The main movement at the ankle joint is plantar and dorsiflexion.

Ligamentous anatomy
Medial side: Deltoid ligament. This is divided into superficial and deep portions. It is the primary restraint to valgus tilting of the talus.

Lateral side: Lateral ligament complex consisting from anterior to posterior of the anterior talofibular ligament (ATFL), calcaneofibular ligament (CFL), and the posterior talofibular ligament (PTFL). Together they resist valgus stress to the ankle, and are a restraint to anterior translation of the talus within the mortise joint.

Syndesmosis: The syndesmosis is a ligament complex between the distal tibia and fibula, holding the two bones together. It is fundamental to the integrity of the ankle joint, and its disruption leads to instability. It consists of (from anterior to posterior) the anterior-inferior tibiofibular ligament (AITFL), the transverse tibiofibular ligament (TTFL), the interosseous membrane, and the posterior-inferior tibiofibular ligament (PITFL).

Presentation and initial management
Patients will present following a traumatic event with a painful, swollen ankle, and reluctance/inability to weight bear. The Ottawa rules can be applied to differentiate between an ankle fracture and sprain, but can be unreliable.
In high energy injuries, management should follow ATLS principles to identify more significant injuries first. Neurovascular status of the foot should be documented, and open injuries should be excluded. If an open injury is identified, it should be managed in line with BOAST 4 principles1. If an obvious deformity exists, it should be reduced as soon as possible with appropriate analgesia or conscious sedation. Radiographs of clearly deformed or dislocated joints are not necessary, and removing the pressure on the surrounding soft tissues from the underlying bony deformity is the priority. If the fracture pattern is not clinically obvious then plain radiographs are appropriate and will guide the subsequent manipulation during plaster-of-paris below knee backslab application.

Imaging
AP, lateral and mortise views (20o internal rotation) are essential to evaluate fracture displacement and syndesmotic injury. Decreased tibiofibular overlap, medial joint clear space and lateral talar shift all indicate a syndesmotic injury. (In subtle cases of shift, imaging the uninjured ankle can be helpful as a proportion of the population have little or no tibiotalar overlap 2.)

Where there is suspicion of syndesmosis involvement in the absence of radiographic evidence, stress radiographs can be diagnostic.
Complex fracture patterns (and increasingly posterior malleolar fractures) are best defined using CT.

Classification
The most commonly used classifications are Lauge-Hansen and Danis-Weber.

Lauge-Hansen
Comprises two parts: first part is the foot position, and the second part is the force applied. Useful for understanding the forces involved and therefore predict the ligamentous or bony injury. Results in four injury patterns:
Supination - Adduction (SA) - 10-20%
Supination - External rotation (SER) - 40-75%
Pronation - Abduction (PA) - 5-20%
Pronation - External rotation (PER) - 5-20%

Not often used in clinical practice but good for understanding the principles of ankle fracture.

Danis-Weber
Commonly used. Based on the level of the fibula fracture in relation to the syndesmosis. The more proximal, the greater the risk of syndesmotic injury and therefore fracture instability.
A - fracture below the level of the syndesmosis
B - fracture at the level of the syndesmosis / level of the tibial plafond
C - fracture above the level of the syndesmosis. This includes Maisonneuve fractures (proximal fibula fracture), which can be associated with ankle instability. Beware the high fibula fracture - it may be an ankle fracture!

The Weber classification is based purely on the the lateral side. All injuries can include a medial or posterior bony or ligamentous injury which also dictates fracture stability (bimalleolar and trimalleolar fractures are more unstable).

Treatment
When deciding upon treatment for an ankle fracture, one must consider both the fracture and the patient. Diabetic patients and smokers are at greater risk of post-operative complication, especially wound problems and infection. Likewise, the long term outcome of post-traumatic arthritis from a malunited ankle fracture is extremely important for a young patient, but not as relevant in the elderly. Therefore, normal surgical decision processes apply as with all fractures.

Defining stability of an ankle fracture underpins the treatment decision.
Weber A - Unimalleolar Weber A Weber fractures by definition are stable and therefore can be mobilised fully weight bearing in an ankle boot.

Weber C - Fractures tend to include syndesmotic disruption and are usually bimalleolar (either bony or ligamentous). They are therefore unstable and usually require operative fixation. In addition to the fracture fixation, the syndesmosis usually requires reconstruction/augmentation with screws to restore the joint integrity and function.

Weber B - B fractures vary greatly. They can be part of a trimalleolar injury and therefore extremely unstable, requiring fixation. Alternatively, a uni-malleolar Weber B fracture can be a stable injury, and therefore mobilised immediately in an ankle boot. Defining the stability can be challenging, and often involves stress radiographs, or a trial of mobilisation and repeat radiographs. Defining stability is the subject of much ongoing research. However, treating undisplaced ankle fractures in a below knee plaster, non-weight bearing for six weeks is still widely practised, and a safe approach.

When operative fixation is appropriate, it is usually via open reduction and internal fixation using plates and screws. It must be carried out when soft tissue swelling has settled in order to minimise the risk of wound problems. This can often take a week to settle.

The use of fibula nails is expanding, but is not yet mainstream. Ankle fractures can also be treated with external fixation, or with a hind foot nail in patients who need fixation but where soft tissue or bone quality is poor.

Post operative management
Ankle fractures generally take 6 weeks to unite enough to prevent secondary displacement. This is therefore an appropriate time period to keep a cast on in a conservatively managed patient. Weight bearing post-operatively depends on the quality of the fixation and bone quality, and preference varies between surgeons, ranging from aggressive early mobilisation to a period of non-weight bearing. Return to activities takes approximately three months, and often requires assistance of a physiotherapist to improve range-of-movement and muscle strengthening.

128
Q

A 10 year old boy undergoes a delayed open reduction and fixation of a significantly displaced supracondylar fracture. On the ward he complains of significant forearm pain and paraesthesia of the hand. Radial pulse is normal. What is the most appropriate course of action?

	Fasciotomy
	Arrange a CT angiogram
	Provide stronger analgesia
	Arrange repeat limb x-rays
	Arrange a forearm duplex scan
A

The delay is the significant factor here. These injuries often have neurovascular compromise and inactivity now places him at risk of developing complications. In compartment syndrome the loss of arterial pulsation occurs late.

Fracture management

Bony injury resulting in a fracture may arise from trauma (excessive forces applied to bone), stress related (repetitive low velocity injury) or pathological (abnormal bone which fractures during normal use of following minimal trauma)
Diagnosis involves not just evaluating the fracture ; such as site and type of injury but also other associated injuries and distal neurovascular deficits. This may entail not just clinical examination but radiographs of proximal and distal joints.
When assessing x-rays it is important to assess for changes in length of the bone, the angulation of the distal bone, rotational effects, presence of material such as glass.

Fracture types
Fracture type Description
Oblique fracture Fracture lies obliquely to long axis of bone
Comminuted fracture >2 fragments
Segmental fracture More than one fracture along a bone
Transverse fracture Perpendicular to long axis of bone
Spiral fracture Severe oblique fracture with rotation along long axis of bone

Open Vs Closed
It is also important to distinguish open from closed injuries. The most common classification system for open fractures is the Gustilo and Anderson classification system (given below):

Grade Injury
1 Low energy wound <1cm
2 Greater than 1cm wound with moderate soft tissue damage
3 High energy wound > 1cm with extensive soft tissue damage
3 A (sub group of 3) Adequate soft tissue coverage
3 B (sub group of 3) Inadequate soft tissue coverage
3 C (sub group of 3) Associated arterial injury

Key points in management of fractures
Immobilise the fracture including the proximal and distal joints
Carefully monitor and document neurovascular status, particularly following reduction and immobilisation
Manage infection including tetanus prophylaxis
IV broad spectrum antibiotics for open injuries
As a general principle all open fractures should be thoroughly debrided ( and internal fixation devices avoided or used with extreme caution)
Open fractures constitute an emergency and should be debrided and lavaged within 6 hours of injury

129
Q

A 6 year old boy presents with pain in the hip it is present on activity and has been worsening over the past few weeks. There is no history of trauma. He was born by normal vaginal delivery at 38 weeks gestation On examination he has an antalgic gait and limitation of active and passive movement of the hip joint in all directions. C-reactive protein is mildly elevated at 10 but the white cell count is normal. What is the most likely diagnosis?

	Perthes disease
	Septic arthritis
	Slipped upper femoral epiphysis
	Developmental dysplasia of the hip
	Osteoarthritis
A

Early plain x-ray changes in Perthes Disease:
Widening of the joint space.
Sub chondral linear lucency.

This is a typical presentation for Perthes disease. X-ray may show flattening of the femoral head or fragmentation in more advanced cases.

Developmental dysplasia of the hip Usually diagnosed in infancy by screening tests. May be bilateral, when disease is unilateral there may be leg length inequality. As disease progresses child may limp and then early onset arthritis. More common in extended breech babies. Splints and harnesses or traction. In later years osteotomy and hip realignment procedures may be needed. In arthritis a joint replacement may be needed. However, this is best deferred if possible as it will almost certainly require revision Initially no obvious change on plain films and USS gives best resolution until 3 months of age. On plain films Shentons line should form a smooth arc
Perthes Disease Hip pain (may be referred to the knee) usually occurring between 5 and 12 years of age. Bilateral disease in 20%. Remove pressure from joint to allow normal development. Physiotherapy. Usually self-limiting if diagnosed and treated promptly. X-rays will show flattened femoral head. Eventually in untreated cases the femoral head will fragment.
Slipped upper femoral epiphysis Typically seen in obese male adolescents. Pain is often referred to the knee. Limitation to internal rotation is usually seen. Knee pain is usually present 2 months prior to hip slipping. Bilateral in 20%. Bed rest and non-weight bearing. Aim to avoid avascular necrosis. If severe slippage or risk of it occurring then percutaneous pinning of the hip may be required. X-rays will show the femoral head displaced and falling inferolaterally (like a melting ice cream cone) The Southwick angle gives indication of disease severity

130
Q

A 23 year old man is involved in a fight outside a nightclub and sustains a laceration to his right arm. On examination, he has lost extension of the fingers in his right hand. Which of the nerves listed below is most likely to have been divided?

	Median
	Musculocutaneous
	Radial
	Ulnar
	Axillary
A

The radial nerve supplies the extensor muscle group.

Radial nerve
Continuation of posterior cord of the brachial plexus (root values C5 to T1)

Path
In the axilla: lies posterior to the axillary artery on subscapularis, latissimus dorsi and teres major.
Enters the arm between the brachial artery and the long head of triceps (medial to humerus).
Spirals around the posterior surface of the humerus in the groove for the radial nerve.
At the distal third of the lateral border of the humerus it then pierces the intermuscular septum and descends in front of the lateral epicondyle.
At the lateral epicondyle it lies deeply between brachialis and brachioradialis where it then divides into a superficial and deep terminal branch.
Deep branch crosses the supinator to become the posterior interosseous nerve.

Regions innervated
Motor (main nerve)	
Triceps
Anconeus
Brachioradialis
Extensor carpi radialis
Motor (posterior interosseous branch)	
Supinator
Extensor carpi ulnaris
Extensor digitorum
Extensor indicis
Extensor digiti minimi
Extensor pollicis longus and brevis
Abductor pollicis longus
Sensory	The area of skin supplying the proximal phalanges on the dorsal aspect of the hand is supplied by the radial nerve (this does not apply to the little finger and part of the ring finger)

Muscular innervation and effect of denervation
Anatomical location Muscle affected Effect of paralysis
Shoulder Long head of triceps Minor effects on shoulder stability in abduction
Arm Triceps Loss of elbow extension
Forearm Supinator
Brachioradialis
Extensor carpi radialis longus and brevis Weakening of supination of prone hand and elbow flexion in mid prone position

131
Q

An injured axillary artery is ligated between the thyrocervical trunk of the subclavian and subscapular artery. Subsequent collateral circulation is likely to result in reversal of blood flow in which of the vessels listed below?

	Circumflex scapular artery
	Transverse cervical artery
	Posterior intercostal arteries
	Suprascapular artery
	Profunda brachii artery
A

It’s an easy question really, we just made the wording difficult (on purpose). It is asking about the branches of the axillary artery and knowledge of the fact that there is an extensive collateral network around the shoulder joint. As a result, the occlusion of the proximal aspect of the circumflex humeral inflow (from the axillary artery) ceases and there is then retrograde flow through it from collaterals.

The circumflex scapular artery is a branch of the subscapular artery and normally supplies the muscle on the dorsal aspect of the scapula. In this instance, flow is reversed in the circumflex scapular and subscapular arteries forming a collateral circulation around the scapula.

The axillary artery extends from the outer border of the first rib to the lower border of teres major, where it becomes the brachial artery. The vessel is subdivided into three zones; the first part lies above pectoralis minor, the second part is behind the muscle and the third part lies inferior to it.

First part
Together with the axillary vein, the artery is enclosed within the cords of the brachial plexus. Both vessels are contained within the axillary sheath, a prolongation of the prevertebral fascia. Posteriomedial to the sheath lies the first intercostal space, the superior aspect of the serratus anterior and the long thoracic nerve. Within the sheath, the medial cord of the brachial plexus lies behind the artery. Anteriorly lies the clavipectoral fascia. Superolaterally, lie the lateral and posterior cords of the brachial plexus. Inferomedially lies the axillary vein.

Second part
Posterior to the second part lies the posterior cord of the brachial plexus and the subscapularis muscle. Anteriorly, lie pectoralis minor and major. The lateral cord of the brachial plexus lies laterally. Medially, lies the medial cord of the brachial plexus, here it separates the artery from the vein.

Third part
Posterior to the artery lie suscapularis, latissimus dorsi and teres major. Interspersed between the vessel and subscapularis are the axillary and radial nerves. Anterior to the vessel is the medial root of the median nerve. Laterally, the lies the median and musculocutaneous nerves and coracobrachialis. The axillary vein is related medially.

Branches of the axillary artery
Highest thoracic artery
Thoraco-acromial artery
Lateral thoracic artery
Subscapular artery
Posterior circumflex humeral artery
Anterior circumflex humeral artery
132
Q

A 20 year old lady presents with pain on the medial aspect of her thigh. Investigations show a large ovarian cyst. Compression of which of the nerves listed below is the most likely underlying cause?

	Sciatic
	Genitofemoral
	Obturator
	Ilioinguinal
	Femoral cutaneous
A

The cutaneous branch of the obturator nerve is frequently absent. However, the obturator nerve is a recognised contributor to innervation of the medial thigh and large pelvic tumours may compress this nerve with resultant pain radiating distally.

The obturator nerve arises from L2, L3 and L4 by branches from the ventral divisions of each of these nerve roots. L3 forms the main contribution and the second lumbar branch is occasionally absent. These branches unite in the substance of psoas major, descending vertically in its posterior part to emerge from its medial border at the lateral margin of the sacrum. It then crosses the sacroiliac joint to enter the lesser pelvis, it descends on obturator internus to enter the obturator groove. In the lesser pelvis the nerve lies lateral to the internal iliac vessels and ureter, and is joined by the obturator vessels lateral to the ovary or ductus deferens.

Supplies
Medial compartment of thigh
Muscles supplied: external obturator, adductor longus, adductor brevis, adductor magnus (not the lower part-sciatic nerve), gracilis
The cutaneous branch is often absent. When present, it passes between gracilis and adductor longus near the middle part of the thigh, and supplies the skin and fascia of the distal two thirds of the medial aspect.

Obturator canal
Connects the pelvis and thigh: contains the obturator artery, vein, nerve which divides into anterior and posterior branches.

133
Q

A 23 year old man falls and slips at a nightclub. A shard of glass penetrates the skin at the level of the medial epicondyle, which of the following sequelae is least likely to occur?

Atrophy of the first dorsal interosseous muscle
Difficulty in abduction of the the 2nd, 3rd, 4th and 5th fingers
Claw like appearance of the hand
Loss of sensation on the anterior aspect of the 5th finger
A

Partial denervation of flexor digitorum profundus
Injury to the ulnar nerve in the mid to distal forearm will typically produce a claw hand. This consists of flexion of the 4th and 5th interphalangeal joints and extension of the metacarpophalangeal joints. The effects are potentiated when flexor digitorum profundus is not affected, and the clawing is more pronounced.More proximally sited ulnar nerve lesions produce a milder clinical picture owing to the simultaneous paralysis of flexor digitorum profundus (ulnar half).

This is the ‘ulnar paradox’, due to the more proximal level of transection the hand will typically not have a claw like appearance that may be seen following a more distal injury. The first dorsal interosseous muscle will be affected as it is supplied by the ulnar nerve.

Ulnar nerve
Origin
C8, T1

Supplies (no muscles in the upper arm)
Flexor carpi ulnaris
Flexor digitorum profundus
Flexor digiti minimi
Abductor digiti minimi
Opponens digiti minimi
Adductor pollicis
Interossei muscle
Third and fourth lumbricals
Palmaris brevis

Path
Posteromedial aspect of upper arm to flexor compartment of forearm, then along the ulnar. Passes beneath the flexor carpi ulnaris muscle, then superficially through the flexor retinaculum into the palm of the hand

Branch Supplies
Muscular branch Flexor carpi ulnaris
Medial half of the flexor digitorum profundus
Palmar cutaneous branch (Arises near the middle of the forearm) Skin on the medial part of the palm
Dorsal cutaneous branch Dorsal surface of the medial part of the hand
Superficial branch Cutaneous fibres to the anterior surfaces of the medial one and one-half digits
Deep branch Hypothenar muscles
All the interosseous muscles
Third and fourth lumbricals
Adductor pollicis
Medial head of the flexor pollicis brevis

Effects of injury
Damage at the wrist
Wasting and paralysis of intrinsic hand muscles (claw hand)
Wasting and paralysis of hypothenar muscles
Loss of sensation medial 1 and half fingers
Damage at the elbow
Radial deviation of the wrist
Clawing less in 4th and 5th digits.

134
Q

A 43 year old man is stabbed outside a nightclub. He suffers a transection of his median nerve just as it leaves the brachial plexus. Which of the following features is least likely to ensue?

	Ulnar deviation of the wrist
	Complete loss of wrist flexion
	Loss of pronation
	Loss of flexion at the thumb joint
	Inability to oppose the thumb
A

Complete loss of wrist flexion

Loss of the median nerve will result in loss of function of the flexor muscles. However, flexor carpi ulnaris will still function and produce ulnar deviation and some residual wrist flexion. High median nerve lesions result in complete loss of flexion at the thumb joint.

Median nerve

The median nerve is formed by the union of a lateral and medial root respectively from the lateral (C5,6,7) and medial (C8 and T1) cords of the brachial plexus; the medial root passes anterior to the third part of the axillary artery. The nerve descends lateral to the brachial artery, crosses to its medial side (usually passing anterior to the artery). It passes deep to the bicipital aponeurosis and the median cubital vein at the elbow.
It passes between the two heads of the pronator teres muscle, and runs on the deep surface of flexor digitorum superficialis (within its fascial sheath).
Near the wrist it becomes superficial between the tendons of flexor digitorum superficialis and flexor carpi radialis, deep to palmaris longus tendon. It passes deep to the flexor retinaculum to enter the palm, but lies anterior to the long flexor tendons within the carpal tunnel.

Branches
Region	Branch
Upper arm	No branches, although the nerve commonly communicates with the musculocutaneous nerve
Forearm	Pronator teres
Flexor carpi radialis
Palmaris longus
Flexor digitorum superficialis
Flexor pollicis longus
Flexor digitorum profundus (only the radial half)
Distal forearm	Palmar cutaneous branch
Hand (Motor)	Motor supply (LOAF)
Lateral 2 lumbricals
Opponens pollicis
Abductor pollicis brevis
Flexor pollicis brevis
Hand (Sensory)	
Over thumb and lateral 2 ½ fingers
On the palmar aspect this projects proximally, on the dorsal aspect only the distal regions are innervated with the radial nerve providing the more proximal cutaneous innervation.

Patterns of damage
Damage at wrist
e.g. carpal tunnel syndrome
paralysis and wasting of thenar eminence muscles and opponens pollicis (ape hand deformity)
sensory loss to palmar aspect of lateral (radial) 2 ½ fingers

Damage at elbow, as above plus:
unable to pronate forearm
weak wrist flexion
ulnar deviation of wrist

Anterior interosseous nerve (branch of median nerve)
leaves just below the elbow
results in loss of pronation of forearm and weakness of long flexors of thumb and index finger

135
Q

A 78 year old man is due to undergo an endarterectomy of the internal carotid artery. Which of the following nervous structures are most at risk during the dissection?

	Recurrent laryngeal nerve
	Sympathetic chain
	Hypoglossal nerve
	Phrenic nerve
	Lingual nerve
A

Hypoglossal nerve

Nerves at risk during a carotid endarterectomy:
Hypoglossal nerve
Greater auricular nerve
Superior laryngeal nerve

During a carotid endarterectomy the sternocleidomastoid muscle is dissected, with ligation of the common facial vein and then the internal jugular is dissected exposing the common and the internal carotid arteries. The nerves at risk during the operation include:
Hypoglossal nerve
Greater auricular nerve
Superior laryngeal nerve
The sympathetic chain lies posteriorly and is less prone to injury in this procedure.

Internal carotid artery

The internal carotid artery is formed from the common carotid opposite the upper border of the thyroid cartilage. It extends superiorly to enter the skull via the carotid canal. From the carotid canal it then passes through the cavernous sinus, above which it divides into the anterior and middle cerebral arteries.

Relations in the neck
Posterior	
Longus capitis
Pre-vertebral fascia
Sympathetic chain
Superior laryngeal nerve
Medially	
External carotid (near origin)
Wall of pharynx
Ascending pharyngeal artery
Laterally	
Internal jugular vein (moves posteriorly at entrance to skull)
Vagus nerve (most posterolaterally)
Anteriorly	
Sternocleidomastoid
Lingual and facial veins
Hypoglossal nerve
Relations in the carotid canal
Internal carotid plexus
Cochlea and middle ear cavity
Trigeminal ganglion (superiorly)
Leaves canal lies above the foramen lacerum

Path and relations in the cranial cavity
The artery bends sharply forwards in the cavernous sinus, the aducens nerve lies close to its inferolateral aspect. The oculomotor, trochlear, opthalmic and, usually, the maxillary nerves lie in the lateral wall of the sinus. Near the superior orbital fissure it turns posteriorly and passes postero-medially to pierce the roof of the cavernous sinus inferior to the optic nerve. It then passes between the optic and oculomotor nerves to terminate below the anterior perforated substance by dividing into the anterior and middle cerebral arteries.

Branches
Anterior and middle cerebral artery
Ophthalmic artery
Posterior communicating artery
Anterior choroid artery
Meningeal arteries
Hypophyseal arteries
136
Q

Which of the structures listed below articulates with the head of the radius superiorly?

	Capitulum
	Trochlea
	Lateral epicondyle
	Ulna
	Medial epicondyle
A

The head of the radius articulates with the capitulum of the humerus.

The radius is one of the two long forearm bones that extends from the lateral side of the elbow to the thumb side of the wrist. It has two expanded ends, of which the distal end is the larger. Key points relating to its topography and relations are outlined below;

Upper end
Articular cartilage- covers medial > lateral side
Articulates with radial notch of the ulna by the annular ligament
Muscle attachment- biceps brachii at the tuberosity

Shaft
Muscle attachment
Upper third of the body	Supinator
Flexor digitorum superficialis
Flexor pollicis longus
Middle third of the body	Pronator teres
Lower quarter of the body	Pronator quadratus
Tendon of supinator longus
Lower end
Quadrilateral
Anterior surface- capsule of wrist joint
Medial surface- head of ulna
Lateral surface- ends in the styloid process
Posterior surface: 3 grooves containing:
1. Tendons of extensor carpi radialis longus and brevis 
2. Tendon of extensor pollicis longus 
3. Tendon of extensor indicis
137
Q

Which of the following fascial structures encases the apex of the lungs?

	Waldeyers fascia
	Sibsons fascia
	Pretracheal fascia
	Clavipectoral fascia
	None of the above
A

Sibson’s fascia overlies the apices of both lungs

The suprapleural fascia (Sibson’s fascia) runs from C7 to the first rib and overlies the apex of both lungs.It lies between the parietal pleura and the thoracic cage.
The suprapleural fascia (Sibson’s fascia) runs from C7 to the first rib and overlies the apex of both lungs.It lies between the parietal pleura and the thoracic cage.

The right lung is composed of 3 lobes divided by the oblique and transverse fissures. The left lung has two lobes divided by the oblique fissure.The apex of both lungs is approximately 4cm superior to the sterno-costal joint of the first rib. Immediately below this is a sulcus created by the subclavian artery.

Peripheral contact points of the lung
Base: diaphragm
Costal surface: corresponds to the cavity of the chest
Mediastinal surface: Contacts the mediastinal pleura. Has the cardiac impression. Above and behind this concavity is a triangular depression named the hilum, where the structures which form the root of the lung enter and leave the viscus. These structures are invested by pleura, which, below the hilum and behind the pericardial impression, forms the pulmonary ligament

Right lung
Above the hilum is the azygos vein; Superior to this is the groove for the superior vena cava and right innominate vein; behind this, and nearer the apex, is a furrow for the innominate artery. Behind the hilum and the attachment of the pulmonary ligament is a vertical groove for the oesophagus; In front and to the right of the lower part of the oesophageal groove is a deep concavity for the extrapericardiac portion of the inferior vena cava.

The root of the right lung lies behind the superior vena cava and the right atrium, and below the azygos vein.

The right main bronchus is shorter, wider and more vertical than the left main bronchus and therefore the route taken by most foreign bodies.

Left lung
Above the hilum is the furrow produced by the aortic arch, and then superiorly the groove accommodating the left subclavian artery; Behind the hilum and pulmonary ligament is a vertical groove produced by the descending aorta, and in front of this, near the base of the lung, is the lower part of the oesophagus.

The root of the left lung passes under the aortic arch and in front of the descending aorta.

Inferior borders of both lungs
6th rib in mid clavicular line
8th rib in mid axillary line
10th rib posteriorly
The pleura runs two ribs lower than the corresponding lung level.
138
Q

Which of the structures listed below inserts into the lesser trochanter

	Psoas minor
	Psoas major
	Sartorius
	Obturator externus
	Gemellus
A

Psoas major inserts onto the lesser trochanter.

Femur

Extends from a rounded head, which articulates with the acetabulum down to the knee joint where the two large condyles at it’s inferior aspect articulate with the tibia.
The superior aspect comprises a head and neck which pass inferolaterally to the body and the two trochanters. These lie at the junction between the neck and the body.
The neck meets the body of the femur at an angle of 125o.
Developmentally, the neck is part of the body but is demarcated from it by a wide rough intertrochanteric crest, this continues inferomedially as a spiral line that runs below the lesser trochanter. Medially, the intertrochanteric line gives attachment to the inferior end of the iliofemoral ligament. The neck is covered by synovial membrane up to the intertrochanteric line. The posterior aspect of the neck is demarcated from the shaft by the intertrochanteric crest and only it’s medial aspect is covered by synovium and the joint capsule.
The greater trochanter has discernible surfaces that form the site of attachment of the gluteal muscles.Laterally, the greater trochanter overhangs the body and this forms part of the origin of vastus lateralis
Viewed anteriorly, the body of the femur appears rounded. Viewed laterally, it has an anterior concavity which gives fullness to the anterior thigh. Posteriorly, there is a ridge of bone, the linea aspera. The surface of the anterior aspect of the body forms the origin of the vastus intermedius. More medially, it forms the origin of vastus medialis.
The upper and middle aspects of the linea aspera form part of the origin of the attachments of the thigh adductors. Inferiorly, it spans out to form the bony floor of the popliteal fossa. At the inferior aspect of the popliteal surface the surface curves posteriorly to form the femoral condyles.
The structures that are attached to the inferior aspect of the linea aspera split with it as it approaches the popliteal fossa. Thus the vastus medialis and adductor magnus continue with the medial split and the biceps femoris and vastus intermedius along the lateral split.

Blood supply
The femur has a rich blood supply and numerous vascular foramina exist throughout it’s length. The blood supply to the femoral head is clinically important and is provided by the medial circumflex femoral and lateral circumflex femoral arteries (Branches of profunda femoris). Also from the inferior gluteal artery. These form an anastomosis and travel to up the femoral neck to supply the head.

139
Q

As regards the internal jugular vein, which of the following statements is untrue?

It lies within the carotid sheath
It is the continuation of the sigmoid sinus
The terminal part of the thoracic duct crosses anterior to it to insert into the right subclavian vein
The hypoglossal nerve is closely related to it as it passes near the atlas
The vagus nerve is closely related to it within the carotid sheath
A

The terminal part of the thoracic duct crosses anterior to it to insert into the right subclavian vein - FALSE

Internal jugular vein

Each jugular vein begins in the jugular foramen, where they are the continuation of the sigmoid sinus. They terminate at the medial end of the clavicle where they unite with the subclavian vein.

The vein lies within the carotid sheath throughout its course. Below the skull the internal carotid artery and last four cranial nerves are anteromedial to the vein. Thereafter it is in contact medially with the internal (then common) carotid artery. The vagus lies posteromedially.

At its superior aspect, the vein is overlapped by sternocleidomastoid and covered by it at the inferior aspect of the vein.

Below the transverse process of the atlas it is crossed on its lateral side by the accessory nerve. At its mid point it is crossed by the inferior root of the ansa cervicalis.
Posterior to the vein are the transverse processes of the cervical vertebrae, the phenic nerve as it descends on the scalenus anterior, and the first part of the subclavian artery.

On the left side its also related to the thoracic duct.

140
Q

At the level of the wrist joint, which of the statements below best describes the relationship of the ulnar artery to the ulnar nerve?

	It lies on its radial side
	It lies deep to it
	It lies superficial to it
	It lies on its ulnar side
	None of the above
A

In the middle of the forearm, the artery is overlapped by the flexor carpi ulnaris and on the flexor retinaculum it is covered by a superficial layer from that structure. In its distal two-thirds, flexor digitorum superficialis lies on its radial side, and the ulnar nerve is situated on its ulnar side.

Ulnar artery

Path
Starts: middle of antecubital fossa
Passes obliquely downward, reaching the ulnar side of the forearm at a point about midway between the elbow and the wrist. It follows the ulnar border to the wrist, crossing over the flexor retinaculum. It then divides into the superficial and deep volar arches.

Relations
Deep to- Pronator teres, Flexor carpi radialis, Palmaris longus
Lies on- Brachialis and Flexor digitorum profundus
Superficial to the flexor retinaculum at the wrist

The median nerve is in relation with the medial side of the artery for about 2.5 cm. And then crosses the vessel, being separated from it by the ulnar head of the Pronator teres

The ulnar nerve lies medially to the lower two-thirds of the artery

Branch
Anterior interosseous artery

141
Q

Which of the following anatomical structures lies within the spiral groove of the humerus?

	Median nerve
	Radial nerve
	Tendon of triceps
	Musculocutaneous nerve
	Axillary nerve
A

The radial nerve lies in this groove and may be compromised by fractures involving the shaft.

Humerus

The humerus extends from the scapula to the elbow joint. It has a body and two ends. It is almost completely covered with muscle but can usually be palpated throughout its length. The smooth rounded surface of the head articulates with the shallow glenoid cavity. The head is connected to the body of the humerus by the anatomical neck. The surgical neck is the region below the head and tubercles and where they join the shaft and is the commonest site of fracture. The capsule of the shoulder joint is attached to the anatomical neck superiorly but extends down to 1.5cm on the surgical neck.

The greater tubercle is the prominence on the lateral side of the upper end of the bone. It merges with the body below and can be felt through the deltoid inferior to the acromion. The tendons of the supraspinatus and infraspinatus are inserted into impressions on its superior aspect. The lesser tubercle is a distinct prominence on the front of the upper end of the bone. It can be palpated through the deltoid just lateral to the tip of the coracoid process.

The intertubercular groove passes on the body between the greater and lesser tubercles, continuing down from the anterior borders of the tubercles to form the edges of the groove. The tendon of biceps within its synovial sheath passes through this groove, held within it by a transverse ligament.

The posterior surface of the body is marked by a spiral groove for the radial nerve which runs obliquely across the upper half of the body to reach the lateral border below the deltoid tuberosity. Within this groove lie the radial nerve and brachial vessels and both may be affected by fractures involving the shaft of the humerus.

The lower end of the humerus is wide and flattened anteroposteriorly, and inclined anteriorly. The middle third of the distal edge forms the trochlea. Superior to this are indentations for the coronoid fossa anteriorly and olecranon fossa posteriorly. Lateral to the trochlea is a rounded capitulum which articulates with the radius.

The medial epicondyle is very prominent with a smooth posterior surface which contains a sulcus for the ulnar nerve and collateral vessels. It’s distal margin gives attachment for the ulnar collateral ligament and, in front of this, the anterior surface has an impression for the common flexor tendon.